Anda di halaman 1dari 372

CENITA M.

CARIAGA,
Petitioner,

G.R. No. 180010


Present:

- versus -

PEOPLE OF THE PHILIPPINES,


Respondent.

CARPIO MORALES,
BRION,
BERSAMIN,
ABAD,* and
VILLARAMA, JR., JJ.
Promulgated:
July 30, 2010

x--------------------------------------------------x
DECISION

CARPIO MORALES, J.:


In issue in the present petition for review is one of jurisdiction.
By Resolutions of May 28, 2007 and September 27, 2007, the Court of Appeals, in CAG.R. CR No. 29514, People of the Philippines v. Cenita Cariaga, dismissed the appeal of Cenita
Cariaga (petitioner) for lack of jurisdiction over the subject matter.
Petitioner, as the municipal treasurer of Cabatuan, Isabela with a Salary Grade of 24, was
charged before the Regional Trial Court (RTC) of Cauayan City in Isabela with three counts of
malversation of public funds, defined under Article 217 of the Revised Penal Code.
The Information in the first case, Criminal Case No. 1293, reads:
That on or about the year 1993 or sometime prior or subsequent thereto in
the Municipality of Cabatuan, Province of Isabela, and within the jurisdiction of
this Honorable Court, the above-named accused, [C]ENITA M. CARIAGA, a
public officer, being the Municipal Treasurer of Cabatuan, Isabela, and as such is
accountable for taxes, fees and monies collected and/or received by her by reason
of her position, acting in relation to her office and taking advantage of the same,
did then and there, willfully, unlawfully and feloniously take, misappropriate and
convert to her personal use the amount of TWO THOUSAND SEVEN HUNDRED
EIGHTY FIVE PESOS (P2,785.00) representing the remittance of the Municipality
of Cabatuan to the Provincial Government of Isabela as the latters share in the real

property taxes collected, which amount was not received by the Provincial
Government of Isabela, to the damage and prejudice of the government in the
amount aforestated.
CONTRARY TO LAW.[1] (underscoring supplied)

The two other Informations in the second and third criminal cases, Nos. 1294 and 1295,
contain the same allegations except the malversed amounts which are P25,627.38 and P20,735.13,
respectively.[2]
Branch 20 of the Cauayan RTC, by Joint Decision of June 22, 2004,[3] convicted petitioner
in the three cases, disposing as follows:
WHEREFORE, finding the accused CENITA M. CARIAGA, GUILTY
beyond reasonable doubt of the crime of MALVERSATION for which she is
charged in the three (3) separate informations and in the absence of any mitigating
circumstance, hereby sentences her to suffer:
1. In Crim. Case No. Br.20-1293, an indeterminate penalty of from FOUR
(4) YEARS and ONE (1) DAY of PRISION CORRECCIONAL as minimum to
SEVEN (7) YEARS, FOUR (4) MONTHS and ONE (1) DAY of PRISION
MAYOR as maximum and its accessory penalty of perpetual special
disqualification and a fine of Two Thousand Seven Hundred Eighty Five
(P2,785.00) Pesos, without subsidiary imprisonment in case of insolvency. Cost
against the accused.
2. In Crim. Case No. Br. 20-1294, an indeterminate penalty of from TEN
(10) YEARS and ONE (1) DAY of PRISION MAYOR as minimum to EIGHTEEN
(18) YEARS, EIGHT (8) MONTHS and ONE (1) DAY of RECLUSION
TEMPORAL as maximum and to suffer the accessory penalty of perpetual special
disqualification and to pay a fine of Twenty Five Thousand Six Hundred Twenty
Seven (P25,627.00) Pesos. She is ordered to indemnify the Provincial Government
of Isabela Twenty Five Thousand Six Hundred Twenty Seven (P25,627.00) Pesos,
without subsidiary imprisonment in case of insolvency. Cost against the accused.
3. In Crim. Case No. Br. 20-1295, an indeterminate penalty of from TEN
(10) YEARS and ONE (1) DAY of PRISION MAYOR as minimum to
FOURTEEN (14) YEARS, EIGHT (8) MONTHS and ONE (1) DAY of
RECLUSION TEMPORAL as maximum, and to suffer the accessory penalty of
perpetual special disqualification and a fine of Twenty Thousand Seven Hundred
Thirty (P20,730.00) Pesos, without subsidiary imprisonment in case of
insolvency. The bailbonds are cancelled. Costs against the accused.
SO ORDERED.

Petitioner, through counsel, in time filed a Notice of Appeal, stating that he intended to
appeal the trial courts decision to the Court of Appeals.
By Resolution of May 28, 2007,[4] the Court of Appeals dismissed petitioners appeal for
lack of jurisdiction, holding that it is the Sandiganbayan which has exclusive appellate jurisdiction
thereon. Held the appellate court:
Concomitantly, jurisdiction over the offense is vested with the Regional
Trial Court considering that the position of Municipal Treasurer corresponds to
a salary grade below 27.Pursuant to Section 4 of [Presidential Decree No. 1606, as
amended by Republic Act No. 8249], it is the Sandiganbayan, to the exclusion
of all others, which enjoys appellate jurisdiction over the offense. Evidently, the
appeal to this Court of the conviction for malversation of public funds was
improperly and improvidently made. (emphasis and underscoring supplied)

Petitioners Motion for Reconsideration was denied by Resolution of September 27,


2007. Hence, the present petition for review, petitioner defining the issues as follows:
[5]

I.

WHETHER . . ., CONSIDERING THE CLEAR AND GRAVE ERROR


COMMITTED BY COUNSEL OF [PETITIONER] AND OTHER EXTRAORDINARY CIRCUMSTANCES, THE APPEAL OF [PETITIONER]
WRONGFULLY DIRECTED TO THE COURT OF APPEALS BE
DISMISSED OUTRIGHTOR BE ENDORSED AND TRANSMITTED TO
THE SANDIGANBAYAN WHERE THE APPEAL SHALL THEN
PROCEED IN DUE COURSE.

II.

WHETHER . . ., IN CONSIDERATION OF SUBSTANTIAL


JUSTICE IN A CRIMINAL CASE, NEW TRIAL BE GRANTED TO THE
PETITIONER TO BE UNDERTAKEN IN THE SANDIGANBAYAN
(ALTERNATIVELY IN THE REGIONAL TRIAL COURT) SO THAT
CRUCIAL EVIDENCE OF PETITIONERBE ADMITTED.[6]

Petitioner, now admitting the procedural error committed by her former counsel, implores
the Court to relax the Rules to afford her an opportunity to fully ventilate her appeal on the merits
and requests the Court to endorse and transmit the records of the cases to the Sandiganbayan in
the interest of substantial justice.

Section 2 of Rule 50 of the Rules of Court provides:


SEC. 2. Dismissal of improper appeal to the Court of Appeals. x x x.
An appeal erroneously taken to the Court of Appeals shall not be
transferred to the appropriate court but shall be dismissed outright. (emphasis
and underscoring supplied)

That appellate jurisdiction in this case pertains to the Sandiganbayan is clear. Section 4 of
Presidential Decree No. 1606,[7] as amended by Republic Act No. 8249, so directs:[8]
Sec. 4. Jurisdiction. The Sandiganbayan shall exercise exclusive original
jurisdiction in all cases involving:

xxxx
In cases where none of the accused are occupying positions
corresponding to Salary Grade 27 or higher, as prescribed in the said Republic
Act No. 6758, or military and PNP officers mentioned above, exclusive original
jurisdiction thereof shall be vested in the proper regional trial court,
metropolitan trial court, municipal trial court, and municipal circuit trial
court, as the case may be, pursuant to their respective jurisdictions as provided
in Batas Pambansa Blg. 129, as amended.
The Sandiganbayan shall exercise exclusive appellate jurisdiction over
final judgments, resolutions or orders of regional trial courts whether in the
exercise of their own original jurisdiction or of their appellate jurisdiction as
herein provided. x x x (emphasis, italics and underscoring supplied).

Since the appeal involves criminal cases, and the possibility of a person being deprived of
liberty due to a procedural lapse militates against the Courts dispensation of justice, the Court
grants petitioners plea for a relaxation of the Rules.
For rules of procedure must be viewed as tools to facilitate the attainment of justice, such
that any rigid and strict application thereof which results in technicalities tending to frustrate
substantial justice must always be avoided.[9]
In Ulep v. People,[10] the Court remanded the case to the Sandiganbayan when it found that

x x x petitioners failure to designate the proper forum for her appeal was
inadvertent. The omission did not appear to be a dilatory tactic on her part.
Indeed, petitioner had more to lose had that been the case as her appeal could
be dismissed outright for lack of jurisdiction which was exactly what happened
in the CA.
The trial court, on the other hand, was duty bound to forward the
records of the case to the proper forum, the Sandiganbayan. It is unfortunate
that the RTC judge concerned ordered the pertinent records to be forwarded to the
wrong court, to the great prejudice of petitioner. Cases involving government
employees with a salary grade lower than 27 are fairly common, albeit regrettably
so. The judge was expected to know and should have known the law and the rules
of procedure. He should have known when appeals are to be taken to the CA
and when they should be forwarded to the Sandiganbayan. He should have
conscientiously and carefully observed this responsibility specially in cases such as
this where a persons liberty was at stake. (emphasis and underscoring supplied)

The slapdash work of petitioners former counsel and the trial courts apparent ignorance of
the law effectively conspired to deny petitioner the remedial measures to question her
conviction.[11]
While the negligence of counsel generally binds the client, the Court has made exceptions
thereto, especially in criminal cases where reckless or gross negligence of counsel deprives the
client of due process of law; when its application will result in outright deprivation of the clients
liberty or property; or where the interests of justice so require. [12] It can not be gainsaid that the
case of petitioner can fall under any of these exceptions.
Moreover, a more thorough review and appreciation of the evidence for the prosecution
and defense as well as a proper application of the imposable penalties in the present case by the
Sandiganbayan would do well to assuage petitioner that her appeal is decided scrupulously.
WHEREFORE, the assailed Resolutions of the Court of Appeals in CA-G.R. CR No.
29514 are SET ASIDE. Let the records of the cases be FORWARDED to the Sandiganbayan for
proper disposition.

The Presiding Judge of Branch 20, Henedino P. Eduarte, of the Cauayan City Regional
Trial Court is WARNED against committing the same procedural error, under pain of
administrative sanction.

G.R. No. 180993, January 27, 2016


REPUBLIC OF THE PHILIPPINES, REPRESENTED BY THE LAND
REGISTRATION AUTHORITY,Petitioner, v. RAYMUNDO VIAJE, ET
AL., Respondents.
DECISION
REYES, J.:
The Republic of the Philippines (Republic) filed the present Petition for
Review on Certiorari1 under Rule 45 of the Rules of Court assailing the Court
of Appeals' (CA) Decision2 dated November 28, 2007 in CA-G.R. SP No.
90102, dismissing its petition for certiorari.
Facts
The Office of the Solicitor General (OSG), on behalf of the Republic and as
represented by the Land Registration Authority (LRA), filed on July 10, 2000
a complaint3 for Cancellation of Title and Reconveyance with the Regional
Trial Court (RTC) of Trece Martires City, docketed as Civil Case No. TM-1001
and raffled to Branch 23. The action mainly sought the nullity of the transfer
certificate of title (TCT) individually issued in the name of the defendants
therein, for having been issued in violation of law and for having dubious
origins. The titles were allegedly derived from TCT No. T-39046 issued on
October 1, 1969. TCT No. T-39046, in turn, was derived from Original
Certificate of Title (OCT) No. 114 issued on March 9, 1910 covering 342,842
square meters. The Republic alleged, among others, that OCT No. 114 and
the documents of transfer of TCT No. T-39046 do not exist in the records of
the Registers of Deeds of Cavite and Trece Martires City.4
The OSG entered its appearance on August 7, 2001 and deputized Atty.
Artemio C. Legaspi and the members of the LRA legal staff to appear in Civil
Case No. TM-1001, with the OSG exercising supervision and control over its
deputized counsel.5 The OSG also requested that notices of hearings, orders,
decisions and other processes be served on both the OSG and the deputized
counsel.6 The Notice of Appearance, however, stated that "only notices of
orders, resolutions, and decisions served on him will bind the party
represented."7 Subsequently, Atty. Alexander N.V. Acosta (Atty. Acosta) of
the LRA entered his appearance as deputized LRA lawyer, pursuant to the
OSG Letter8 dated August 7, 2001.9 The letter also contained the statement,
"only notices of orders, resolutions and decisions served on him will bind the
[Republic], the entity, agency and/or official represented."10

Thereafter, several re-settings of the pre-trial date were made due to the
absence of either the counsel for the Republic or the counsel of one of the
defendants, until finally, on April 11, 2003, the RTC dismissed the complaint
due to the non-appearance of the counsel for the Republic.11
The OSG filed a motion for reconsideration,12 which was granted by the RTC
in its Order dated July 22, 2003.13 Pre-trial was again set and re-set, and on
January 23, 2004, the RTC finally dismissed Civil Case No. TM-1001 with
prejudice.14 The order reads, in part:
WHEREFORE, in view of the above, and upon motion of the defendants
through counsel, Atty. Eufracio C. Fortuno, let this case be, as it is hereby,
DISMISSED with prejudice.
SO ORDERED.15ChanRoblesVirtualawlibrary
Having been informed of this, the OSG forthwith filed a Manifestation and
Motion,16 informing the RTC that Atty. Acosta was not given notice of the
pre-trial schedule. The OSG also stated that such lack of notice was pursuant
to a verbal court order that notice to the OSG is sufficient notice to the
deputized counsel, it being the lead counsel, and that they were not formally
notified of such order. The OSG argued that its deputized counsel should
have been notified of the settings made by the trial court as it is not merely
a collaborating counsel who appears with an OSG lawyer during hearing;
rather, its deputized counsel appears in behalf of the OSG and should be
separately notified. Aside from this, the OSG pointed out that it particularly
requested for a separate notice for the deputy counsel.17
The RTC denied the OSG's Manifestation and Motion in its Order18 dated May
31, 2004, from which the OSG filed a Notice of Appeal,19 which was given
due course by the RTC.20 Subsequently, the RTC, on motion of the
defendants, issued Order21 dated October 4, 2004 recalling its previous
order that gave due course to the OSG's appeal. The ground for the recall
was the OSG's failure to indicate in its notice of appeal the court to which
the appeal was being directed.22 The OSG moved for the reconsideration23 of
the order but it was denied by the RTC on March 16, 2005.24
Thus, the OSG filed a special civil action for certiorari with the CA. On
November 28, 2007, the CA rendered the assailed decision dismissing the
OSG's petition on the grounds that the petition was filed one day late and
the RTC did not commit any grave abuse of discretion when it dismissed Civil
Case No. TM-1001 and the OSG's notice of appeal. It ruled that the OSG's
failure to indicate in its notice of appeal the court to which the appeal is
being taken violated Section 5, Rule 41 of the Rules of Civil Procedure, which
provides, among others, that "[t]he notice of appeal shall x x x specify the
court to which the appeal is being taken x x x." The CA also ruled that the

OSG cannot claim lack of due process when its deputized counsel was not
served a notice of the pre-trial schedule. The CA disagreed with the OSG's
contention that its deputized counsel should have been notified. According to
the CA, the OSG remains the principal counsel of the Republic and it is
service on them that is decisive, and having received the notice of pre-trial,
it should have informed its deputized counsel of the date. Aside from this,
the authority given by the OSG to its deputized counsel did not include the
authority to enter into a compromise agreement, settle or stipulate on facts
and admissions, which is a part of the pre-trial; hence, even if the deputized
counsel was present, the case would still be dismissed.25cralawred
The OSG is now before the Court arguing that:
THE APPELLATE COURT ERRED IN NOT HOLDING THAT RESPONDENT JUDGE
COMMITTED GRAVE ABUSE OF DISCRETION IN DISMISSING THE
COMPLAINT DESPITE THE JUSTIFIED FAILURE OF THE DEPUTIZED COUNSEL
TO ATTEND THE PRE-TRIAL.
THE APPELLATE COURT ERRED IN NOT HOLDING THAT RESPONDENT JUDGE
COMMITTED GRAVE ABUSE OF DISCRETION IN DISMISSING THE NOTICE OF
APPEAL.26ChanRoblesVirtualawlibrary
The OSG contends that the rule that notice to the OSG is sufficient notice to
its deputized counsel applies only to collaborating counsels who appear with
the lead counsel. In case of deputized counsels, a separate notice is
necessary since they appear in behalf of the OSG. Also, the OSG pointed out
that it specifically requested that separate notices be furnished to its
deputized counsel.27
The OSG also argues that the RTC committed grave abuse of discretion
when it dismissed the notice of appeal despite the fact that the defendants
did not ask for its recall and merely sought clarification as to which court the
case was being appealed to. Moreover, the OSG maintains that its
inadvertence is not fatal as it did not create any ambiguity as to which court
the appeal shall be made, and that the interest of due process should prevail
over an inadvertent violation of procedural rules.28
Ruling of the Court
The power of the OSG to deputize legal officers of government departments,
bureaus, agencies and offices to assist it in representing the government is
well settled. The Administrative Code of 1987 explicitly states that the OSG
shall have the power to "deputize legal officers of government departments,
bureaus, agencies and offices to assist the Solicitor General and appear or
represent the Government in cases involving their respective offices, brought
before the courts and exercise supervision and control over such legal

officers with respect to such cases."29 But it is likewise settled that the
OSG's deputized counsel is "no more than the 'surrogate' of the
Solicitor General in any particular proceeding" and the latter
remains the principal counsel entitled to be furnished copies of all
court orders, notices, and decisions.30 In this case, records show that it
was the OSG that first entered an appearance in behalf of the Republic;
hence, it remains the principal counsel of record. The appearance of the
deputized counsel did not divest the OSG of control over the case and did
not make the deputized special attorney the counsel of record.31 Thus, the
RTC properly acted within bounds when it relied on the rule that it is the
notice to the OSG that is binding.32
Nonetheless, the OSG also pointed out that it specifically requested the RTC
to likewise furnish its deputized counsel with a copy of its notices. Records
show that the deputized counsel also requested that copies of notices and
pleadings be furnished to him.33 Despite these requests, it was only the OSG
that the RTC furnished with copies of its notices. It would have been more
prudent for the RTC to have furnished the deputized counsel of its notices.
All the same, doing so does not necessarily clear the OSG from its obligation
to oversee the efficient handling of the case. And even if the deputized
counsel was served with copies of the court's notices, orders and decisions,
these will not be binding until they are actually received by the OSG. More
so in this case where the OSG's Notice of Appearance and its Letter
deputizing the LRA even contained the caveat that it is only notices of
orders, resolutions and decisions served on the OSG that will bind
the Republic, the entity, agency and/or official represented.34 In fact,
the proper basis for computing a reglementary period and for determining
whether a decision had attained finality is service on the OSG.35 As was
stated in National Power Corporation v. National Labor Relations
Commission:36
The underlying justification for compelling service of pleadings, orders,
notices and decisions on the OSG as principal counsel is one and the same.
As the lawyer for the government or the government corporation
involved, the OSG is entitled to the service of said pleadings and
decisions, whether the case is before the courts or before a quasijudicial agency such as respondent commission. Needless to say, a
uniform rule for all cases handled by the OSG simplifies procedure,
prevents confusion and thus facilitates the orderly administration of
justice.37 (Emphasis ours)
The CA, therefore, cannot be limited for upholding the RTC's dismissal of
Civil Case No. TM-1001 due to the failure of the counsel for the Republic to
appear during pre-trial despite due notice.
The Court, likewise, cannot attribute error to the CA when it affirmed the

RTC's recall of its order granting the OSG's notice of appeal. The RTC simply
applied the clear provisions of Section 5, Rule 41 of the Rules of Court,
which mandated that a "notice of appeal shall x x x specify the court to
which the appeal is being taken x x x."
Nevertheless, under the circumstances obtaining in this case, the Court
resolves to relax the stringent application of the rules, both on the matter of
service of notices to the OSG and its deputized counsel, and on the notice of
appeal. Such relaxation of the rules is not unprecedented.
In Cariaga v. People of the Philippines,38 the Court ruled that rules of
procedure must be viewed as tools to facilitate the attainment of justice such
that its rigid and strict application which results in technicalities tending to
frustrate substantial justice must always be avoided.39 In Ulep v. People of
the Philippines,40 meanwhile, the Court ordered the remand of the case to
the proper appellate court, stating that the "petitioner's failure to designate
the proper forum for her appeal was inadvertent," and that "[t]he omission
did not appear to be a dilatory tactic on her part."41
Similarly in this case, the OSG's omission should not work against the
Republic. For one, the OSG availed of the proper remedy in seeking a review
of the RTC's order of dismissal by pursuing an ordinary appeal and filing a
notice of appeal, albeit without stating where the appeal will be taken. For
another, it is quite elementary that an ordinary appeal from a final
decision/order of the RTC rendered in the exercise of its original jurisdiction
can only be elevated to the CA under Rule 41 of the Rules of
Court.42 Moreover, as in Ulep, the OSG's failure to designate where the
appeal will be taken was a case of inadvertence and does not appear to be a
dilatory tactic on its part. More importantly, the OSG's omission should not
redound to the Republic's disadvantage for it is a well-settled principle that
the Republic is never estopped by the mistakes or error committed by its
officials or agents.43
Finally, the subject matter of the case before the RTC - the recovery by the
Republic of a 342,842-sq m property in Cavite covered by an allegedly nonexistent title - necessitates a full-blown trial. To sustain the peremptory
dismissal of Civil Case No. TM-1001 due to the erroneous appreciation by the
Republic's counsel of the applicable rules of procedure is an abdication of the
State's authority over lands of the public domain.44 Under the Regalian
doctrine, "all lands of the public domain belong to the State, and the State is
the source of any asserted right to ownership in land and charged with the
conservation of such patrimony." The Court, therefore, must exercise its
equity jurisdiction and relax the rigid application of the rules where strong
considerations of substantial justice are manifest.45

WHEREFORE, the petition is GRANTED. The Decision dated November 28,


2007 of the Court of Appeals in CA-G.R. SP No. 90102
is REVERSED and SET ASIDE. Civil Case No. TM-1001 and all its records
are REMANDED to the Regional Trial Court of Trece Martires City, Branch
23, for further disposition on the merits.
The Office of the Solicitor General and its deputized counsel/s are advised to
be more circumspect in the performance of their duties as counsels for the
Republic of the Philippines.
SO ORDERED.chanroblesvirtuallawlibrary

G.R. No. 90625

May 23, 1991

PEOPLE OF THE PHILIPPINES, plaintiff-appellee,


vs.
BENEDICTO DAPITAN y MARTIN, @ "Benny" and FRED DE GUZMAN, accused.
BENEDICTO DAPITAN y MARTIN @ "Benny", accused-appellant.
The Solicitor General for plaintiff-appellee.
Public Attorney's Office for accused-appellant.
DAVIDE, JR., J.:
This is an appeal from the Decision of the Regional Trial Court of Rizal (Branch 75, San
Mateo) 4th Judicial Region, finding the accused-appellant guilty of the crime of Robbery
with Homicide and sentencing him to:
. . . suffer the penalty of RECLUSION PERPETUA, and to pay the heirs of the
victim Rolando Amil in the amount of Thirty Thousand (P30,000.00) Pesos,
without subsidiary imprisonment in case of
insolvency. 1
Only the accused-appellant was tried. His co-accused, Fred de Guzman, remained at
large and the court ordered the archival of the case as against him, to be revived upon
his arrest.
The information filed with the court a quo on 7 August 1986 against accused-appellant
and his co-accused reads in part as follows:
That on or about the 16th day of May, 1986, in Barangay San Rafael,
Municipality of Rodriguez (formerly Montalban), Province of Rizal, Philippines, a
place within the jurisdiction of this Honorable Court, the above-named accused
conspiring and confederating together and mutually helping and aiding one
another, with intent to gain, armed with deadly weapon and by means of force
and violence, then and there willfully, unlawfully and feloniously took,
robbed/stole and carried (sic) away two (2) pieces of men's watches worth One
Thousand One Hundred Eighty Eight Pesos (P1,188.00), one (1) pair of long
pants worth Two Hundred Fifty Pesos (P250.00) and cash money in the amount
of Seventy Five Pesos (P75.00) belonging to Orencia E. Amil, without the
knowledge and consent of said owner and to her damage and prejudice in the
total amount of One Thousand Five Hundred Thirteen Pesos (P1,513.00),
Philippine Currency; that on the occasion of the said robbery and for the purpose
of enabling them to take, steal and carry away the above-mentioned articles, the
herein accused in pursuance of their conspiracy, did then and there willfully,
unlawfully and feloniously, with evident premeditation and taking advantage of
their superior strength and with intent to kill, treacherously attack, assault and
employ personal violence upon the person of Rolando Amil (an eight year old

child) by stabbing him on the neck and hitting him several times on the head with
a piece of wood, to prevent him from making an outcry, thereby inflicting upon
him physical injuries which directly caused his death. 2
When arraigned on 25 November 1986 with the assistance of counsel de oficio, Atty.
Magsanoc, accused entered a plea of not guilty. 3
At the scheduled hearing on 10 February 1987, new counsel de oficio for the accused,
Atty. Gabriel Alberto of the Citizens Legal Assistance Office (CLAO) of San Mateo,
Rizal, manifested that the accused had expressed to him the desire to enter a plea of
guilty to a lesser offense. The court forthwith issued an order reading as follows:
Atty. Alberto of CLAO and de oficio counsel for the accused manifested that the
accused has manifested his desire to make a plea of guilty to a lesser offense
but the circumstances are yet to be made in details. It appears that there are two
mitigating circumstances that maybe applied. The Prosecuting Fiscal made no
objection but also manifested that he has to look into the penalty applicable. The
counsel for the accused and the Prosecuting Fiscal jointly moved that the hearing
of this case be reset to another date.
WHEREFORE, reset the hearing of this case for March 9, 1987 at 9:30 A.M. . . .
.4
The scheduled hearing of 9 March 1987 was cancelled and reset to April 13, 1987 in
view of the required vacation leave of absence of the judge.
On 13 April 1987, upon motion of the prosecution and the defense in view of the
projected settlement of the civil liability of this case, the hearing was reset to 19 May
1987. 5 On that date, however, counsel de oficio for the accused did not appear, hence
"a report on the projected settlement of the civil aspect of the case cannot be made"
and the hearing was reset again to 15 June 1987 6 which schedule was later on
cancelled due to the compulsory retirement of the presiding judge (Judge Conrado
Beltran) which took effect on 7 June 1987. 7
In the meantime, Judge Francisco C. Rodriguez, Jr. presided over the trial court

The initial reception of evidence took place on 24 August 1987 with the accusedappellant represented by Atty. Benjamin Pozon, also of the CLAO.
On various dates thereafter, hearings were had until the parties completed the
presentation of their evidence. Witnesses Orencia Amil and Cpl. Rodolfo Rivera for the
prosecution testified during the incumbency of Judge Rodriguez. The rest testified
before Judge Edilberto H. Noblejas who succeeded Judge Rodriguez.
On 5 May 1989, the trial court promulgated its Decision 9 the dispositive portion of which
reads:

WHEREFORE, premises considered, after appraising the evidence presented by


the prosecution and the evidence of the defense, the Court finds the accused
BENEDICTO DAPITAN y MARTIN GUILTY BEYOND REASONABLE DOUBT of
the crime of ROBBERY WITH HOMICIDE, punishable under Article 294, par. 1 of
the Revised Penal Code and sentences him to suffer the penalty of RECLUSION
PERPETUA, and to pay the heirs of the victim Rolando Amil in the amount of
Thirty Thousand (P30,000.00) Pesos, without subsidiary imprisonment in case of
insolvency.
With respect to the case against FRED DE GUZMAN, the records of the case
insofar as he is concerned is hereby ordered ARCHIVED to be revived upon his
arrest when he may be heard to answer for the offense charged.
On 11 May 1 989, accused-appellant filed his Notice of Appeal, manifesting therein that
he was appealing the decision to this Court. 10 However, in the Order of 11 May 1989,
Judge Cipriano de Roma erroneously directed the transmittal of the records of the case
to the Court of Appeals. 11 The Court of Appeals transmitted to this Court on 4 March
1989 the records which were erroneously transmitted to it. 12
In this appeal accused-appellant assigns only one error:
THE TRIAL COURT ERRED IN NOT APPLYING THE INDETERMINATE
SENTENCE LAW THAT FAVORS THE ACCUSED APPELLANT. 12
He is thus deemed to be in complete agreement with the findings and conclusion of
facts by the trial court which We quote:
The evidence adduced by the prosecution more than prove with moral certainty
the guilt of the accused Benedicto Dapitan for the crime of ROBBERY WITH
HOMICIDE. While there may be no direct evidence linking the accused to said
crime, the witnesses who testified more than fully satisfy the requirements for
conviction on the basis of circumstancial evidence, because it affords enough
basis for a reasonable inference of the existence of the fact thereby sought to be
proved, that the accused performed the criminal act.
Orencia Amil, principal witness for the prosecution testified that at around 8:30 in
the morning of May 16, 1986, she left for her farm which was about 50 meters
away, leaving behind in her house his adopted son Rolando (the victim) very
much alive.(TSN, page 5, hearing of August 24, 1987) who refused to go with her
because he chose to play in the house instead; and that because she heard the
barking of her dog which aroused her suspicion, she immediately returned and
saw the accused Benedicto Dapitan and his co-accused Fred de Guzman
passing through her fence (TSN, pp. 5-6, hearing of August 24, 1987); and that
when she entered her house calling her child's name, and seeing the backdoor
open, she entered and saw Rolando's body sprawled on the floor and his brain
"scattered". Near his body was a piece of wood, also bloodied. Thinking her son

to be still alive she took her in her arms, placed him on the table and that was the
time she realized he was dead. (TSN pages 6-7, hearing of Aug. 24, 1987).
She likewise testified that she lost two watches worth P1,180; pants at P250.00
and cash amounting to P75.00; and after her son's burial she further found that
her child's toy worth P500.00, a flashlight and a bolo worth P45.00 and P120.00,
respectively, were missing. (TSN, pages 8-9, hearing of August 24, 1987).
Orencia Amil's testimony is likewise corroborated on its material points by the
testimony of Celo Nilo, another prosecution witness. He testified that between the
hours of 8:00 to 9:00 in the morning of May 16, 1986, he saw two persons
entering the house of Mrs. Orencia Amil, one of whom he identified as Benedicto
Dapitan, (TSN, pages 4-5, hearing of October 26, 1987). He positively identified
Benedicto Dapitan who was in Court (TSN, pages 5-6, hearing of October 26,
1987). He likewise testified that when the two suspects entered the house of Mrs.
Amil, he heard the voice of a child. In the statement he gave the police
investigators (Exhibit B) which he confirmed when he testified, pertinent portions
of which are herein quoted, he said:
xxx

xxx

xxx

T Noong May 16, 1986, sa pagitan ng ika 8:00 ng umaga, natatandaan


mo ba noon kung saan ka naroroon?
S Ako po ay galing sa aming bahay at ako po ay patungo sa bundok para
magtanim po ng punong saging.
xxx

xxx

xxx

T Noong ikaw ay papadaan sa malapit sa bahay ni Orencia Amil, wala ka


bang napansin na tao na nagtungo doon sa kanilang bahay.?
S Mayroon po.
T Nakilala mo ba naman kung sinong tao ang iyong nakita na dumaan
doon sa bahay nina Mrs. Orencia Amil?
S Iyon lang pong isang tao ang aking kilala na dumaan doon sa bahay
nina Mrs. Orencia Amil na siBenny Dapitan na ang tirahan po ay doon po
rin sa Sitio Tabak, Brgy. San Rafael, R/R, pero iyon pong isa na kasama
ni Benny Dapitan ay hindi ko po kilala sa kanyang tunay na pangalan.
T Ilan bang tao ang iyong nakita na nagpunta doon sa bahay ni Mrs.
Orencia Amil?
S Dalawang tao po.

T Mayroon ka ba gaano kalayo doon sa dalawang tao na ang isa ay


si Benny Dapitan ng sila ay makita mo na pumunta doon sa bahay ni Mrs.
Orencia Amil?
S Mayroon po lamang na mga 10 metro ang aking layo sa kanila.
T Matapos na makita mo si na si Benny Dapitan at iyong isa niyang
kasama ay pumasok doon sa bahay, ano pa ang sunod na pangyayari?
S Akin pong nakita na matapos na sila ay makapasok sa loob ng bahay ni
Mrs. Amil ay kanila pong isinara iyong pintuan noong bahay, at hindi ko po
naman sila pinansin at ako po ay nagpatuloy na sa aking pupuntahan.
xxx

xxx

xxx

T Matapos na makapasok iyong sina Benny Dapitan doon sa bahay, wala


ka ba namang narinig na sigaw ng isang bata?
S Mayroon po pero hindi ko po pinansin. (Emphasis supplied).
xxx

xxx

xxx

The testimonies of these two witnesses, evaluated together, on what transpired


in the morning of May 16, 1986, between the hours of 8:00-9:00 a.m. attest to the
existence of the following facts:
1. That the victim, Rolando Amil, was alive when her mother left her as testified
to by Orencia Amil and witness Celo Nilo, who cry out when the two suspects
entered the house. (Testimony of Orencia Amil)
2. That the accused Benedicto Dapitan and an unidentified companion entered
the house at a time when Mrs. Amil had already left, and that the victim, at the
time, was still alive. (Testimonies of Celo Nilo & Orencia Amil)
3. That when Mrs. Amil returned at quarter to nine she saw Benedicto Dapitan
and Fred de Guzman leaving the premises.(Testimony of Orencia Amil)
4. And that when Mrs. Amil entered her house, the victim, Rolando Amil, was
already dead. (Testimony of Orencia Amil)
As gleaned from the records, witness Orencia Amil was straightforward in her
testimony. She remained steadfast even on cross-examination, and there is
nothing on record concerning her testimony which would leave the court in doubt
as to the truth of what she testified to. Her testimony therefore, relative to the
circumstances transpiring at the time she left the house at 8:30 a.m. up to the
time she returned at quarter to nine engenders belief.

Celo Nilo's testimony was likewise made in the same vein as that of Orencia
Amil. This witness was not shown to have cause to perjure himself on a serious
crime against the accused. As the Court observed during the trial, his testimony,
based on his demeanor when he testified, is impressed with a ring of veracity.
The Court did not give credit to the testimony of Patrolman Rodolfo Rivera except
on the fact that he conducted an investigation. No value whatsoever was given to
the sworn statement of Benedicto Dapitan, even as to the portion in said
testimony, where Benedicto Dapitan admitted being present when Fred de
Guzman allegedly hit the victim on the head and that the stolen articles were in
the possession of Fred de Guzman, because as wisely put by defense counsel,
the sworn statement was taken in violation of the constitutional rights of the
accused.
In sum, therefore, there can be no other inference from the evidence presented
by the prosecution considering the short span of time the victim Rolando Amil
was left alive by his mother, and her return fifteen (15) minutes later to find him
dead and the testimony that the accused was seen entering and leaving the
premises during this intervening period, except the inevitable conclusion that the
accused is responsible for the death of Rolando Amil.
For his part, the accused Benedicto Dapitan interposes the defense of "alibi".
This, he sought to establish through the testimony of witness Ismael Anacio.
Pertinent portion of the witness' testimony, is herein quoted, to wit:
xxx

xxx

xxx

Q Now, do you remember, Mr. Witness, if this Benedicto Dapitan was


present in the said house on the period from May 16 to May 19, 1986?
A He was there, sir.
Q Was there any occasion when this Benedicto Dapitan left your house
during that period?
A None, sir.
(TSN, pages 3-5, hearing of September 12, 1988).
The testimony of witness Ismael Anacio, a salesman by occupation, that
defendant Benedicto Dapitan, from May 16 to May 19, 1986, was in his house all
the time, and that there was no occasion that he left the place during this period
does not spark belief. In the first place, the witness wants the Court to believe
that he was in his house during all the time so that he could during all the days
alluded to, be in a position to be positive as to the whereabouts of the accused.
This circumstance alone generates doubt on his testimony, because it was not

explained why the witness, a salesman by occupation, would be in his house


from the period beginning May 16-19, 1986 (TSN, pages 2-3, hearing of
September 12, 1988).
Assuming though, for the sake of argument, that the witness actually monitored
the whereabouts of the accused during all the time, his testimony sustaining
Benedicto Dapitan's defense of "alibi" cannot defeat the positive identification
made of Benedicto Dapitan and of his presence in Montalban on May 16, 1986,
by witness Orencia Amil and Celo Nilo. Even on this score alone, without taking
into consideration that Sampaloc District where he allegedly was, is
geographically not so far from Montalban, from where he could have commuted
through the ordinary means of transportation present in the area, his defense of
"alibi" naturally falls, so that his conviction is reasonably called for. 14
In support of the assigned error accused-appellant argues that the imposition over him
of the penalty of reclusion temporal by the trial court is "tantamount to deprivation of life
or liberty without due process of law or is tantamount to a cruel, degrading or inhuman
punishment prohibited by the Constitution" and he submits that "the righteous and
humane punishment that should have been meted out should be indeterminate
sentence" with "all mitigating circumstances as well as the legal provisions favorable to
the accused-appellant . . . appreciated or . . . taken advantage for constructive and
humanitarian reasons." He stresses that since mitigating circumstances are based on,
among others, the lesser perversity of the offender, such should be appreciated in his
favor since he had "a companion then when he entered Mrs. Orencia Amil's house and
perpetrated the offense. 15 And it was his companion or mate by the name of Fred de
Guzman who took the personal belongings of Mrs. Amil as the men's watch worth
P1,188.00. It was Fred de Guzman who is still at large who stabbed and hit the head of
Rolando Amil. 16 These facts or circumstances reveal that accused-appellant had a
"lesser perversity than his companion Fred de Guzman." As evidence of such lesser
perversity, "he did not flee or hide himself from the authorities. . . . within two (2) days'
time he surrendered voluntarily to the police authorities . . . ." Thus, the "mitigating
circumstance of voluntary surrender must be considered" in his favor. 17
He prays that he be sentenced to an indeterminate penalty ranging from twelve (12)
years and one (1) day ofreclusion temporal, as minimum, to reclusion perpetua as
maximum. 18
Meeting squarely the points raised by the accused-appellant, the People, in the Brief for
Plantiff-Appellee submitted by the Solicitor General on 9 June 1990, asserts that the
same are without merit for the accused was not deprived of due process as he was, as
admitted by him, afforded full opportunity to be heard; for a penalty to be cruel,
degrading or inhuman, "it must take more than merely being harsh, excessive, out of
proportion, or severe. . . . ; it must be flagrantly and plainly oppressive, disproportionate
to the nature of the offense as to shock the moral sense of the community 19 or when
they involve torture or lingering death" 20 and since the penalty ofreclusion
perpetua imposed on him is sanctioned by law, Act No. 3815 as amended, otherwise

known as the Revised Penal Code, said penalty is not cruel, degrading or inhuman. It
further argues that the special complex crime of robbery with homicide defined under
Article 294, par. 1, of the Revised Penal Code is punishable withreclusion perpetua to
death; with the abolition of the death penalty by the 1987 Constitution, the only penalty
imposable upon a person found to have committed such complex crime is the single
penalty of reclusion perpetua, which is an indivisible penalty. Under Article 63 of the
Revised Penal Code it should be applied regardless of the presence of any mitigating or
aggravating circumstances.
As regards the Indeterminate Sentence Law, the People submits that the accusedappellant cannot avail of it since Section 2 of the law (Act No. 4103) specifically
provides that it shall not apply to, among others, persons convicted of offenses
punished with death penalty or life imprisonment.
We find the instant appeal to be totally bereft of merit.
There was no denial of due process.
Due process is satisfied if the following conditions are present: (1) there must be a court
or tribunal clothed with judicial power to hear and determine the matter before it; (2)
jurisdiction must be lawfully acquired by it over the person of the defendant or over the
property which is the subject of the proceeding; (3) the defendant must be given an
opportunity to be heard; and (4) judgment must be rendered upon lawful hearing. 21
In People vs. Castillo, et al., 22 We ruled that if an accused has been heard in a court of
competent jurisdiction, and proceeded against under the orderly processes of law, and
only punished after inquiry and investigation, upon notice to him, with opportunity to be
heard, and a judgment awarded within the authority of the constitutional law, then he
has had due process .23
We reiterated the above doctrine in People vs. Muit. 24
All the requisites or conditions of due process are present in this case. The records
further disclose that accused-appellant was given the fullest and unhampered
opportunity not only to reflect dispassionately on his expressed desire to plead guilty to
a lesser offense which prompted the court to cancel the hearing of 10 February 1987,
but also to confront the witnesses presented against him and to present his own
evidence.
If indeed accused-appellant had been deprived of due process, he would have faulted
the trial court not just for failure to apply the Indeterminate Sentence Law, but definitely
for more. Yet, he found it futile to go any farther.
Neither is the penalty of reclusion perpetua cruel, degrading, and inhuman.1wphi1 To
make that claim is to assail the constitutionality of Article 294, par. 1 of the Revised
Penal Code, or of any other provisions therein and of special laws imposing the said

penalty for specific crimes or offenses. The proposition cannot find any support. Article
294, par. 1 of the Revised Penal Code has survived four Constitutions of the
Philippines, namely: the 1935 Constitution, the 1973 Constitution, the Freedom
Constitution of 1986 and the 1987 Constitution. All of these documents mention life
imprisonment or reclusion perpetua as a penalty which may be imposed in appropriate
cases.25 As a matter of fact, the same paragraph of the section of Article III (Bill of
Rights) of the 1987 Constitution which prohibits the imposition of cruel, degrading and
inhuman punishment expressly recognizes reclusion perpetua. Thus:
Sec. 19(l). Excessive fines shall not be imposed, nor cruel, degrading or inhuman
punishment inflicted. Neither shall the death penalty be imposed, unless, for
compelling reasons involving heinous crimes, the Congress hereafter provides it.
Any death penalty already imposed shall be reduced to reclusion perpetua.
As to the appreciation of mitigating circumstances, We also agree with the Solicitor
General that since robbery with homicide under paragraph 1 of Article 294 of the
Revised Penal Code is now punishable by the single and indivisible penalty of reclusion
perpetua in view of the abolition of the death penalty, it follows that the rule prescribed
in the first paragraph of Article 63 of the Revised Penal Code shall
apply. 26 Consequently, reclusion perpetua must be imposed in this case regardless of
the presence of mitigating or aggravating circumstances.
The trial court correctly imposed on the accused the penalty of reclusion perpetua.
The civil indemnity awarded by the trial court should, in line with Our decision in People
vs. Sison, G.R. No. 86455, 14 September 1990, and People vs. Sazon, G.R. No. 89684,
18 September 1970, be increased from P30,000.00 to P50,000.00.
WHEREFORE, except as modified above in respect to the civil indemnity, the decision
appealed from is AFFIRMED in toto, with costs against accused-appellant.
SO ORDERED.

G.R. No. 131652 March 9, 1998


BAYANI M. ALONTE, petitioner,
vs.
HON. MAXIMO A. SAVELLANO JR., NATIONAL BUREAU OF INVESTIGATION and
PEOPLE OF THE PHILIPPINES, respondents.
G.R. No. 131728 March 9, 1998
BUENAVENTURA CONCEPCION, petitioner,
vs.
JUDGE MAXIMO SAVELLANO, JR., THE PEOPLE OF THE PHILIPPINES, and
JUVIELYN Y. PUNONGBAYAN,respondents.

VITUG, J.:
Pending before this Court are two separate petitions, one filed by petitioner Bayani M.
Alonte, docketed G.R. No. 131652, and the other by petitioner Buenaventura
Concepcion, docketed G.R. No. 131728, that assail the decision of respondent Judge
Maximo A. Savellano, Jr., of the Regional Trial Court ("RTC"), Branch 53, of Manila
finding both petitioners guilty beyond reasonable doubt of the crime of rape. The two
petitions were consolidated.
On 05 December 1996, an information for rape was filed against petitioners Bayani M.
Alonte, an incumbent Mayor of Bian, Laguna, and Buenaventura Concepcion
predicated on a complaint filed by Juvie-lyn Punongbayan. The information contained
the following averments; thus:
That on or about September 12, 1996, in Sto. Tomas, Bian, Laguna, and within
the jurisdiction of this Honorable court, the above named accused, who is the
incumbent mayor of Bian, Laguna after giving complainant-child drinking water
which made her dizzy and weak, did then and there willfully, unlawfully and
feloniously have carnal knowledge with said JUVIELYN PUNONGBAYAN
against her will and consent, to her damage and prejudice.
That accused Buenaventura "Wella" Concepcion without having participated as
principal or accessory assisted in the commission of the offense by bringing said
complainant child to the rest house of accused Bayani "Arthur" Alonte at Sto.
Tomas, Bian, Laguna and after receiving the amount of P1,000.00 left her alone
with Bayani Alonte who subsequently raped her.
Contrary to Law.1

The case was docketed Criminal Case No. 9619-B and assigned by raffle to Branch 25
of the RTC of Bian, Laguna, presided over by Judge Pablo B. Francisco.
On 13 December 1996, Juvie-lyn Punongbayan, through her counsel Attorney
Remedios C. Balbin, and Assistant Chief State Prosecutor ("ACSP") Leonardo Guiyab,
Jr., filed with the Office of the Court Administrator a Petition for a Change of Venue
(docketed Administrative Matter No. 97-1-12-RTC) to have the case transferred and
tried by any of the Regional Trial Courts in Metro Manila.
During the pendency of the petition for change of venue, or on 25 June 1997, Juvie-lyn
Punongbayan, assisted by her parents and counsel, executed an affidavit of desistance,
quoted herein in full, as follows:
AFFIDAVIT OF DESISTANCE
I, JUVIE-LYN YAMBAO PUNONGBAYAN, 17 years of age, a resident of No. 5 Uranus
Street, Congressional Avenue Subdivision, Quezon City, duly assisted by private legal
counsel and my parents, after having duly sworn in accordance with law, depose and
say:
1. That I am the Complainant in the rape case filed against Mayor Bayani
"Arthur" Alonte of Bian, Laguna, with the RTC-Branch 25 of Bian,
Laguna;
2. That the case has been pending for some time, on preliminary issues,
specifically, (a) change of venue, filed with the Supreme Court; (b)
propriety of the appeal to the Court of Appeals, and after its denial by said
court, brought to the Office of the President, on the veracity of the findings
of the Five-Man Investigating Panel of the State Prosecutor's Office, and
the Secretary of Justice, and (c) a hold-departure order filed with the
Bian Court.
3. That the legal process moves ever so slowly, and meanwhile, I have
already lost two (2) semesters of my college residence. And when the
actual trial is held after all the preliminary issues are finally resolved, I
anticipate a still indefinite suspension of my schooling to attend the
hearings;
4. That during the entire period since I filed the case, my family has lived a
most abnormal life: my father and mother had to give up their jobs; my
younger brother, who is in fourth grade, had to stop his schooling, like
myself;
5 That I do not blame anyone for the long, judicial process, I simply wish
to stop and live elsewhere with my family, where we can start life anew,
and live normally once again;

6. That I pray that I be allowed to withdraw my complaint for rape and the
other charge for child abuse wherein the Five-Man Investigating Panel of
the Office of the State Prosecutor found a prima facie case although the
information has not been filed, and that I will not at any time revive this,
and related cases or file new cases, whether, criminal, civil, and/or
administrative, here or anywhere in the Philippines;
7 That I likewise realize that the execution of this Affidavit will put to doubt
my credibility as a witness-complainant;
8. That this is my final decision reached without fear or favor, premised on
a corresponding commitment that there will be no reprisals in whatever
form, against members of the police force or any other official of officer,
my relatives and friends who extended assistance to me in whatever way,
in my search for justice.
WHEREOF, I affix my signature this 25 day of June, 1997, in Quezon City.
(Sgd) JUVIE-LYN Y. PUNONGBAYAN
Complainant
Assisted by:
(Sgd) ATTY. REMEDIOS C. BALBIN
Private Prosecutor
In the presence of:
(Sgd) PABLO PUNONGBAYAN
Father
(Sgd) JULIE Y. PUNONGBAYAN
Mother
SUBSCRIBED AND SWORN to before me this 25 day of June, 1997, in Quezon City.
(Sgd) Illegible
Administering Officer2
On 28 June 1997, Atty. Ramon C. Casino, on behalf of petitioners, moved to have the
petition for change of venue dismissed on the ground that it had become moot in view of
complainant's affidavit of desistance. On 22 August 1997, ACSP Guiyab filed his
comment on the motion to dismiss. Guiyab asserted that he was not aware of the
desistance of private complainant and opined that the desistance, in any case, would
not produce any legal effect since it was the public prosecutor who had direction and

control of the prosecution of the criminal action. He prayed for the denial of the motion
to dismiss.
On 02 September 1997, this Court issued a Resolution (Administrative Matter No. 97-112-RTC), granting the petition for change of venue. The Court said:
These affidavits give specific names, dates, and methods being used to abort, by
coercion or corruption, the prosecution of Criminal Case No. 9619-B. It is thus
incorrect for oppositors Alonte and Concepcion to contend that the fear of the
petitioner, her private counsel and her witnesses are too generalized if not
fabricated. Indeed, the probability that in desisting from pursuing her complaint
for rape, petitioner, a minor, may have succumbed to some illicit influence and
undue pressure. To prevent possible miscarriage of justice is a good excuse to
grant the petition to transfer the venue of Criminal Case No. 9619-B from Bian,
Laguna to the City of Manila.
IN VIEW WHEREOF, the Petition for Change of Venue from Bian, Laguna to
the City of Manila is granted. The Executive Judge of RTC Manila is ordered to
raffle Crim. Case No. 9619-B to any of its branches. The judge to whom Crim.
Case No. 9619-B shall be raffled shall resolve the petitioner's Motion to Resume
Proceedings filed in Br. XXV of the RTC of Bian, Laguna and determine the
voluntariness and validity of petitioner's desistance in light of the opposition of the
public prosecutor, Asst. Chief State Prosecutor Leonardo Guiyab. The branch
clerk of court of Br. XXV of the RTC of Bian, Laguna is ordered to personally
deliver to the Executive Judge of Manila the complete records of Crim. Case No.
9619-B upon receipt of this Resolution.3
On 17 September 1997, the case, now re-docketed Criminal Case No. 97-159955 by
the Clerk of Court of Manila, was assigned by raffle to Branch 53, RTC Manila, with
respondent Judge Maximo A. Savellano, Jr., presiding.
On 07 October 1997, Juvie-lyn Punongbayan, through Attorney Balbin, submitted to the
Manila court a "compliance" where she reiterated "her decision to abide by her Affidavit
of Desistance."
In an Order, dated 09 October 1997, Judge Savellano found probable cause for the
issuance of warrants for the arrest of petitioners Alonte and Concepcion "without
prejudice to, and independent of, this Court's separate determination as the trier of
facts, of the voluntariness and validity of the [private complainant's] desistance in the
light of the opposition of the public prosecutor, Asst. Chief State Prosecutor Leonardo
Guiyab."
On 02 November 1997, Alonte voluntarily surrendered himself to Director Santiago
Toledo of the National Bureau of Investigation ("NBI"), while Concepcion, in his case,
posted the recommended bail of P150,000.00.

On 07 November 1997, petitioners were arraigned and both pleaded "not guilty" to the
charge. The parties manifested that they were waiving pre-trial. The proceedings
forthwith went on. Per Judge Savellano, both parties agreed to proceed with the trial of
the case on the merits.4 According to Alonte, however, Judge Savellano allowed the
prosecution to present evidence relative only to the question of the voluntariness and
validity of the affidavit of desistance.5
It would appear that immediately following the arraignment, the prosecution presented
private complainant Juvielyn Punongbayan followed by her parents. During this hearing,
Punongbayan affirmed the validity and voluntariness of her affidavit of desistance. She
stated that she had no intention of giving positive testimony in support of the charges
against Alonte and had no interest in further prosecuting the action. Punongbayan
confirmed: (i) That she was compelled to desist because of the harassment she was
experiencing from the media, (ii) that no pressures nor influence were exerted upon her
to sign the affidavit of desistance, and (iii) that neither she nor her parents received a
single centavo from anybody to secure the affidavit of desistance.
Assistant State Prosecutor Marilyn Campomanes then presented, in sequence: (i)
Punongbayan's parents, who affirmed their signatures on the affidavit of desistance and
their consent to their daughter's decision to desist from the case, and (ii) Assistant
Provincial Prosecutor Alberto Nofuente, who attested that the affidavit of desistance
was signed by Punongbayan and her parents in his presence and that he was satisfied
that the same was executed freely and voluntarily. Finally, Campomanes manifested
that in light of the decision of private complainant and her parents not to pursue the
case, the State had no further evidence against the accused to prove the guilt of the
accused. She, then, moved for the "dismissal of the case" against both Alonte and
Concepcion.
Thereupon, respondent judge said that "the case was submitted for decision." 6
On 10 November 1997, petitioner Alonte filed an "Urgent Motion to Admit to Bail."
Assistant State Prosecutor Campomanes, in a Comment filed on the same date, stated
that the State interposed "no objection to the granting of bail and in fact Justice and
Equity dictates that it joins the accused in his prayer for the granting of bail."
Respondent judge did not act on the application for bail.
On 17 November 1997, Alonte filed anew an Urgent Plea to Resolve the Motion for Bail.
On even date, ASP Campomanes filed a Manifestation deeming "it proper and in accord
with justice and fair play to join the aforestated motion."
Again, the respondent judge did not act on the urgent motion.
The records would indicate that on the 25th November 1997, 1st December 1997, 8th
December 1997 and 10th December 1997, petitioner Alonte filed a Second, Third,

Fourth and Fifth Motion for Early Resolution, respectively, in respect of his application
for bail. None of these motions were acted upon by Judge Savellano.
On 17 December 1997, Attorney Philip Sigfrid A. Fortun, the lead counsel for petitioner
Alonte received a notice from the RTC Manila. Branch 53, notifying him of the schedule
of promulgation, on 18 December 1997, of the decision on the case. The counsel for
accused Concepcion denied having received any notice of the scheduled promulgation.
On 18 December 1997, after the case was called, Atty. Sigrid Fortun and Atty. Jose
Flaminiano manifested that Alonte could not attend the promulgation of the decision
because he was suffering from mild hypertension and was confined at the NBI clinic and
that, upon the other hand, petitioner Concepcion and his counsel would appear not to
have been notified of the proceedings. The promulgation, nevertheless, of the decision
proceeded in absentia; the reading concluded:
WHEREFORE, judgment is hereby rendered finding the two (2) accused Mayor
Bayani Alonte and Buenaventura "Wella" Concepcion guilty beyond reasonable
doubt of the heinous crime of RAPE, as defined and penalized under Article
335(2) in relation to Article 27 of the Revised Penal Code, as amended by
Republic Act No. 7659, for which each one of the them is hereby sentenced to
suffer the indivisible penalty of RECLUSION PERPETUA or imprisonment for
twenty (20) years and one (1) day to forty (40) years.
In view thereof, the bail bond put up by the accused Buenaventura "Wella'"
Concepcion for his provisional liberty is hereby cancelled and rendered without
any further force and effect.
SO ORDERED.7
On the same day of 18th December 1997, petitioner Alonte filed a motion for
reconsideration. Without waiting for its resolution, Alonte filed the instant "Ex Abundante
Ad Cautelam" for "Certiorari, Prohibition, Habeas Corpus, Bail, Recusation of
respondent Judge, and for Disciplinary Action against an RTC Judge." Petitioner
Concepcion later filed his own petition for certiorari and mandamus with the Court.
Alonte submits the following grounds in support of his petition seeking to have the
decision nullified and the case remanded for new trial; thus:
The respondent Judge committed grave abuse of discretion amounting to lack or
excess of jurisdiction when he rendered a Decision in the case a quo (Annex A)
without affording the petitioner his Constitutional right to due process of law
(Article III, 1, Constitution).
The respondent Judge committed grave abuse of discretion amounting to lack or
excess of jurisdiction when he rendered a Decision in the case a quo in violation
of the mandatory provisions of the Rules on Criminal Procedure, specifically, in

the conduct and order of trial (Rule 119) prior to the promulgation of a judgment
(Rule 120; Annex A).
The respondent Judge committed grave abuse of discretion amounting to lack or
excess of jurisdiction when, in total disregard of the Revised Rules on Evidence
and existing doctrinal jurisprudence, he rendered a Decision in the case a
quo (Annex A) on the basis of two (2) affidavits (Punongbayan's and Balbin's)
which were neither marked nor offered into evidence by the prosecution, nor
without giving the petitioner anopportunity to cross-examine the affiants thereof,
again in violation of petitioner's right to due process (Article III, 1, Constitution).
The respondent Judge committed grave abuse of discretion amounting to lack or
excess of jurisdiction when he rendered a Decision in the case a quo without
conducting a trial on the facts which would establish that complainant was raped
by petitioner (Rule 119, Article III, 1, Constitution), thereby setting a dangerous
precedent where heinous offenses can result in conviction without trial (then with
more reason that simpler offenses could end up with the same result). 8
On the other hand, Concepcion relies on the following grounds in support of his own
petition; thus:
1. The decision of the respondent Judge rendered in the course of resolving the
prosecution's motion to dismiss the case is a patent nullity for having been
rendered without jurisdiction, without the benefit of a trial and in total violation of
the petitioner's right to due process of law.
2. There had been no valid promulgation of judgment at least as far as petitioner
is concerned.
3. The decision had been rendered in gross violation of the right of the accused
to a fair trial by an impartial and neutral judge whose actuations and outlook of
the case had been motivated by a sinister desire to ride on the crest of media
hype that surrounded this case and use this case as a tool for his ambition for
promotion to a higher court.
4. The decision is patently contrary to law and the jurisprudence in so far as it
convicts the petitioner as a principal even though he has been charged only as
an accomplice in the information.9
The petitions deserve some merit; the Court will disregard, in view of the case milieu,
the prematurity of petitioners' invocation, i.e., even before the trial court could resolve
Alonte's motion for reconsideration.
The Court must admit that it is puzzled by the somewhat strange way the case has
proceeded below. Per Judge Savellano, after the waiver by the parties of the pre-trial
stage, the trial of the case did proceed on the merits but that

The two (2) accused did not present any countervailing evidence during the trial.
They did not take the witness stand to refute or deny under oath the truth of the
contents of the private complainant's aforementioned affidavit which she
expressly affirmed and confirmed in Court, but, instead, thru their respective
lawyers, they rested and submitted the case for decision merely on the basis of
the private complainant's so called "desistance" which, to them, was sufficient
enough for their purposes. They left everything to the so-called "desistance" of
the private complainant.10
According to petitioners, however, there was no such trial for what was conducted on 07
November 1997, aside from the arraignment of the accused, was merely a proceeding
in conformity with the resolution of this Court in Administrative Case No. 97-1-12-RTC to
determine the validity and voluntariness of the affidavit of desistance executed by
Punongbayan.
It does seem to the Court that there has been undue precipitancy in the conduct of the
proceedings. Perhaps the problem could have well been avoided had not the basic
procedures been, to the Court's perception, taken lightly. And in this shortcoming,
looking at the records of the case, the trial court certainly is not alone to blame.
Section 14, paragraphs (1) and (2), of Article III, of the Constitution provides the
fundamentals.
(1) No person shall be held to answer for a criminal offense without due process
of law.
(2) In all criminal prosecutions, the accused shall be presumed innocent until the
contrary is proved, and shall enjoy the right to be heard by himself and counsel,
to be informed of the nature and cause of the accusation against him, to have a
speedy, impartial, and public trial, to meet the witnesses face to face, and to
have compulsory process to secure the attendance of witnesses and the
production of evidence in his behalf. However, after arraignment, trial may
proceed notwithstanding the absence of the accused provided that he has been
duly notified and his failure to appear is unjustifiable.
Jurisprudence11 acknowledges that due process in criminal proceedings, in particular,
require (a) that the court or tribunal trying the case is properly clothed with judicial
power to hear and determine the matter before it; (b) that jurisdiction is lawfully acquired
by it over the person of the accused; (c) that the accused is given an opportunity to be
heard; and (d) that judgment is rendered only upon lawful hearing. 12
The above constitutional and jurisprudential postulates, by now elementary and deeply
imbedded in our own criminal justice system, are mandatory and indispensable. The
principles find universal acceptance and are tersely expressed in the oft-quoted
statement that procedural due process cannot possibly be met without a "law which

hears before it condemns, which proceeds upon inquiry and renders judgment only after
trial." 13
The order of trial in criminal cases is clearly spelled out in Section 3, Rule 119, of the
Rules of Court; viz:
Sec. 3. Order of trial. The trial shall proceed in the following order:
(a) The prosecution shall present evidence to prove the charge and, in the proper
case, the civil liability.
(b) The accused may present evidence to prove his defense, and damages, if
any, arising from the issuance of any provisional remedy in the case.
(c) The parties may then respectively present rebutting evidence only, unless the
court, in furtherance of justice, permits them to present additional evidence
bearing upon the main issue.
(d) Upon admission of the evidence, the case shall be deemed submitted for
decision unless the court directs the parties to argue orally or to submit
memoranda.
(e) However, when the accused admits the act or omission charged in the
complaint or information but interposes a lawful defense, the order of trial may be
modified accordingly.
In Tabao vs. Espina,14 the Court has underscored the need to adhere strictly to the
above rules. It reminds that
. . . each step in the trial process serves a specific purpose. In the trial of criminal
cases, the constitutional presumption of innocence in favor of an accused
requires that an accused be given sufficient opportunity to present his defense.
So, with the prosecution as to its evidence.
Hence, any deviation from the regular course of trial should always take into
consideration the rights of all the parties to the case, whether in the prosecution
or defense. In the exercise of their discretion, judges are sworn not only to
uphold the law but also to do what is fair and just. The judicial gavel should not
be wielded by one who has an unsound and distorted sense of justice and
fairness.15
While Judge Savellano has claimed in his Comment that
Petitioners-accused were each represented during the hearing on 07 November
1997 with their respective counsel of choice. None of their counsel interposed an
intention to cross-examine rape victim Juvielyn Punongbayan, even after she

attested, in answer to respondent judge's clarificatory questions, the


voluntariness and truth of her two affidavits one detailing the rape and the
other detailing the attempts to buy her desistance; the opportunity was
missed/not used, hence waived. The rule of case law is that the right to confront
and cross-examine a witness "is a personal one and may be waived." (emphasis
supplied)
it should be pointed out, however, that the existence of the waiver must be positively
demonstrated. The standard of waiver requires that it "not only must be voluntary, but
must be knowing, intelligent, and done with sufficient awareness of the relevant
circumstances and likely consequences."16 Mere silence of the holder of the right should
not be so construed as a waiver of right, and the courts must indulge every reasonable
presumption against waiver.17 The Solicitor General has aptly discerned a few of the
deviations from what otherwise should have been the regular course of trial: (1)
Petitioners have not been directed to present evidence to prove their defenses nor have
dates therefor been scheduled for the purpose;18 (2) the parties have not been given the
opportunity to present rebutting evidence nor have dates been set by respondent Judge
for the purpose;19 and (3) petitioners have not admitted the act charged in the
Information so as to justify any modification in the order of trial.20 There can be no shortcut to the legal process, and there can be no excuse for not affording an accused his full
day in court. Due process, rightly occupying the first and foremost place of honor in our
Bill of Rights, is an enshrined and invaluable right that cannot be denied even to the
most undeserving.
This case, in fine, must be remanded for further proceedings. And, since the case would
have to be sent back to the court a quo, this ponencia has carefully avoided making any
statement or reference that might be misconstrued as prejudgment or as pre-empting
the trial court in the proper disposition of the case. The Court likewise deems it
appropriate that all related proceedings therein, including the petition for bail, should be
subject to the proper disposition of the trial court.
Nevertheless, it is needful to stress a few observations on the affidavit of desistance
executed by the complainant.
Firstly, the affidavit of desistance of Juvie-Lyn Punongbayan, hereinbefore quoted, does
not contain any statement that disavows the veracity of her complaint against petitioners
but merely seeks to "be allowed to withdraw" her complaint and to discontinue with the
case for varied other reasons. On this subject, the case ofPeople vs. Junio,21 should be
instructive. The Court has there explained:
The appellant's submission that the execution of an Affidavit of Desistance by
complainant who was assisted by her mother supported the "inherent incredibility
of prosecution's evidence" is specious. We have said in so many cases that
retractions are generally unreliable and are looked upon with considerable
disfavor by the courts. The unreliable character of this document is shown by the
fact that it is quite incredible that after going through the process of having

accused-appellant arrested by the police, positively identifying him as the person


who raped her, enduring the humiliation of a physical examination of her private
parts, and then repeating her accusations in open court by recounting her
anguish, Maryjane would suddenly turn around and declare that "[a]fter a careful
deliberation over the case, (she) find(s) that the same does not merit or warrant
criminal prosecution.
Thus, we have declared that at most the retraction is an afterthought which
should not be given probative value. It would be a dangerous rule to reject the
testimony taken before the court of justice simply because the witness who has
given it later on changed his mind for one reason or another. Such a rule will
make a solemn trial a mockery and place the investigation at the mercy of
unscrupulous witnesses. Because affidavits of retraction can easily be secured
from poor and ignorant witnesses, usually for monetary consideration, the Court
has invariably regarded such affidavits as exceedingly unreliable [Flores vs.
People, 211 SCRA 622, citing De Guzman vs. Intermediate Appellate Court, 184
SCRA 128; People vs. Galicia, 123 SCRA 550.]22
The Junio rule is no different from ordinary criminal cases. For instance, in People
vs. Ballabare,23 a murder case, the Court has ruled:
The contention has no merit. To begin with, the Affidavit executed by eyewitness
Tessie Asenita is not a recantation. To recant a prior statement is to renounce
and withdraw it formally and publicly. [36 WORDS AND PHRASES 683, citing
Pradlik vs. State, 41-A 2nd, 906, 907.] In her affidavit, Tessie Asenita did not
really recant what she had said during the trial. She only said she wanted to
withdraw her testimony because her father, Leonardo Tacadao, Sr., was no
longer interested in prosecuting the case against accused-appellant. Thus, her
affidavit stated:
3. That inasmuch as my father, Leonardo Tacadao, Sr., the complainant therein,
was no longer interested to prosecute the case as manifested in the Sworn
Affidavit of Desistance before the Provincial Prosecutor, I do hereby WITHDRAW
and/or REVOKE my testimony of record to confirm (sic) with my father's desire;
It is absurd to disregard a testimony that has undergone trial and scrutiny by the
court and the parties simply because an affidavit withdrawing the testimony is
subsequently presented by the defense. In the first place, any recantation must
be tested in a public trial with sufficient opportunity given to the party adversely
affected by it to cross-examine the recanting witness. In this case, Tessie Asenita
was not recalled to the witness stand to testify on her affidavit. Her affidavit is
thus hearsay. It was her husband, Roque Asenita, who was presented and the
matters he testified to did not even bear on the substance of Tessie's affidavit.
He testified that accused-appellant was not involved in the perpetration of the
crime.

In the second place, to accept the new evidence uncritically would be to make a
solemn trial a mockery and place the investigation at the mercy of unscrupulous
witnesses. [De Guzman vs. Intermediate Appellate Court, 184 SCRA 128, 134,
citing People vs. Morales, 113 SCRA 683.] For even assuming that Tessie
Asenita had made a retraction, this circumstance alone does not require the
court to disregard her original testimony. A retraction does not necessarily negate
an earlier declaration. [People vs. Davatos, 229 SCRA 647.] For this reason,
courts look with disfavor upon retractions because they can easily be obtained
from witnesses usually through intimidation or for monetary considerations.
[People vs. Clamor, 198 SCRA 642.] Hence, when confronted with a situation
where a witness recants his testimony, courts must not automatically exclude the
original testimony solely on the basis of the recantation. They should determine
which testimony should be given credence through a comparison of the original
testimony and the new testimony, applying the general rules of evidence. [Reano
vs. Court of Appeals, 165 SCRA 525.] In this case we think the trial court
correctly ruled.24
It may not be amiss to state that courts have the inherent power to compel the
attendance of any person to testify in a case pending before it, and a party is not
precluded from invoking that authority.25
Secondly, an affidavit of desistance by itself, even when construed as a pardon in the
so-called "private crimes," is not a ground for the dismissal of the criminal case once the
action has been instituted. The affidavit, nevertheless, may, as so earlier intimated,
possibly constitute evidence whose weight or probative value, like any other piece of
evidence, would be up to the court for proper evaluation. The decision in Junio went on
to hold
While "[t]he offenses of seduction, abduction, rape or acts of lasciviousness, shall
not be prosecuted except upon a complaint flied by the offended party or her
parents, grandparents, or guardian, nor in any case, if the offender has been
expressly pardoned by the above named persons, as the case may be," [Third
par. of Art. 344, The Revised Penal Code.] the pardon to justify the dismissal of
the complaint should have been made prior to the institution of the criminal
action. [People vs. Entes, 103 SCRA 162, cited by People vs. Soliao, 194 SCRA
250, which in turn is cited in People vs. Villorente, 210 SCRA 647.] Here, the
motion to dismiss to which the affidavit of desistance is attached was filed after
the institution of the criminal case. And, affiant did not appear to be serious in
"signifying (her) intention to refrain from testifying" since she still completed her
testimony notwithstanding her earlier affidavit of desistance. More, the affidavit is
suspect considering that while it was dated "April 1992," it was only submitted
sometime in August 1992, four (4) months after the Information was filed before
the court a quo on 6 April 1992, perhaps dated as such to coincide with the
actual filing of the case.26

In People vs. Miranda,27 applying the pertinent provisions of Article 344 of the Revised
Penal Code which, in full, states
Art. 344. Prosecution of the crimes of adultery, concubinage, seduction,
abduction, rape, and acts of lasciviousness. The crimes of adultery and
concubinage shall not be prosecuted except upon a complaint filed by the
offended spouse.
The offended party cannot institute criminal prosecution without including both
the guilty parties, if they are both alive, nor, in any case, if he shall have
consented or pardoned the offenders.
The offenses of seduction, abduction, rape or acts of lasciviousness, shall not be
prosecuted except upon a complaint filed by the offended party or her parents,
grandparents, or guardian, nor, in any case, if the offender has been expressly
pardoned by the above named persons, as the case may be.
In cases of seduction, abduction, acts of lasciviousness and rape, the marriage
of the offender with the offended party shall extinguish the criminal action or remit
the penalty already imposed upon him. The provisions of this paragraph shall
also be applicable to the coprincipals, accomplices and accessories after the fact
of the above-mentioned crimes.
the Court said:
Paragraph 3 of the legal provision above quoted prohibits a prosecution for
seduction, abduction, rape, or acts of lasciviousness, except upon a complaint
made by the offended party or her parents, grandparents, or guardian, nor, in any
case, if the offender has been expressly pardoned by the above-named persons,
as the case may be. It does not prohibit the continuance of a prosecution if the
offended party pardons the offender after the cause has been instituted, nor does
it order the dismissal of said cause. The only act that according to article 344
extinguishes the penal action and the penalty that may have been imposed is the
marriage between the offended and the offended party. 28
In People vs. Infante, 29 decided just a little over a month before Miranda, the Court
similarly held:
In this court, after the case had been submitted, a motion to dismiss was filed on
behalf of the appellant predicated on an affidavit executed by Manuel Artigas, Jr.,
in which he pardoned his guilty spouse for her infidelity. But this attempted
pardon cannot prosper for two reasons. The second paragraph of article 344 of
the Revised Penal Code which is in question reads: "The offended party cannot
institute criminal prosecution without including both the guilty parties, if they are
both alive, nor, in any case, if he shall have consented or pardoned the
offenders." This provision means that the pardon afforded the offenders must

come before the institution of the criminal prosecution, and means, further, that
both the offenders must be pardoned by the offended party. To elucidate further,
article 435 of the old Penal Code provided: "The husband may at any time remit
the penalty imposed upon his wife. In such case the penalty imposed upon the
wife's paramour shall also be deemed to be remitted." These provisions of the
old Penal Code became inoperative after the passage of Act No. 1773, section 2,
which had the effect of repealing the same. The Revised Penal Code thereafter
expressly repealed the old Penal Code, and in so doing did not have the effect of
reviving any of its provisions which were not in force. But with the incorporation of
the second paragraph of article 344, the pardon given by the offended party
again constitutes a bar to the prosecution for adultery. Once more, however, it
must be emphasized that this pardon must come before the institution of the
criminal prosecution and must be for both offenders to be effective
circumstances which do not concur in this case.30
The decisions speak well for themselves, and the Court need not say more than what it
has heretofore already held.
Relative to the prayer for the disqualification of Judge Savellano from further hearing the
case, the Court is convinced that Judge Savellano should, given the circumstances, the
best excused from the case. Possible animosity between the personalities here involved
may not all be that unlikely. The pronouncement of this Court in the old case of Luque
vs. Kayanan31 could again be said: All suitors are entitled to nothing short of the cold
neutrality of an independent, wholly-free, disinterested and unbiased tribunal. Second
only to the duty of rendering a just decision is the duty of doing it in a manner that will
not arouse any suspicion as to the fairness and integrity of the Judge.32 It is not enough
that a court is impartial, it must also be perceived as impartial.
The Court cannot end this ponencia without a simple reminder on the use of proper
language before the courts. While the lawyer in promoting the cause of his client or
defending his rights might do so with fervor, simple courtesy demands that it be done
within the bounds of propriety and decency. The use of intemperate language and
unkind ascriptions hardly can be justified nor can have a place in the dignity of judicial
forum. Civility among members of the legal profession is a treasured tradition that must
at no time be lost to it.
Finally, it may be opportune to say, once again, that prosecutors are expected not
merely to discharge their duties with the highest degree or excellence, professionalism
and skill but also to act each time with utmost devotion and dedication to duty. 33 The
Court is hopeful that the zeal which has been exhibited many times in the past, although
regrettably a disappointment on few occasions, will not be wanting in the proceedings
yet to follow.
WHEREFORE, conformably with all the foregoing, the Court hereby RULES that

(a) The submission of the "Affidavit of Desistance," executed by Juvie-Lyn Y.


Punongbayan on 25 June 1997, having been filed AFTER the institution of
Criminal Case No. 97-159935, DOES NOT WARRANT THE DISMISSAL of said
criminal case;
(b) For FAILURE OF DUE PROCESS, the assailed judgment, dated 12
December 1997, convicting petitioners is declared NULL AND VOID and thereby
SET ASIDE; accordingly, the case is REMANDED to the trial court for further
proceedings; and
(c) Judge Maximo A. Savellano, Jr., presiding Judge of Branch 53 of the
Regional Trial Court of Manila, is ENJOINED from further hearing Criminal Case
No. 97-159935; instead, the case shall immediately be scheduled for raffle
among the other branches of that court for proper disposition.
No special pronouncement on costs.
SO ORDERED.

G.R. No. L-56158-64 March 17, 1981


PEOPLE OF THE PHILIPPINES, petitioner,
vs.
MAYOR PABLO SOLA, SANGGUNIANG BAYAN MEMBER FRANCISCO (ECOT)
GARCIA, RICARDO (CADOY) GARCIA, JOSE BETHOVEN (ATSONG) CABRAL,
CAPTAIN FLORENDO BALISCAO, JOHN, PETER, OSCAR, OMAR, JACK,
RICHARD, JAMES, DONALD, WILLIAM, ROBERT, HOMER, JESSIE, ANDY, PAUL,
all surnamed DOES respondents.

FERNANDO, C.J.:
The power of this Tribunal, constitutionally mandated, 1 to order a change of venue to
avoid any miscarriage of justice as well as the procedure ordained in the
implementation of the right to bail 2 are involved in this petition which, even if not so
denominated, partakes of the nature of a certiorari. It must have been the zeal of private
prosecutors Francisco Cruz and Renecio Espiritu, 3 no doubt under the conviction that
there was no time to lose, that must have led them to devote less than that full measure
of attention to certain fundamentals. They ignored the principle that the responsibility for
the conduct of the prosecution is with the public officials concerned. Nonetheless, the
importance of the questions raised, the need for a change of venue and the cancellation
of the bail bonds, necessitated that further action be taken. Accordingly, in a resolution
dated February 12, 1981, one day after the filing of the petition, the Court required the
comment of the Solicitor General as well as of the private respondents, 4 the accused in
six pending criminal cases before the Court of First Instance of Negros Occidental.
On March 4, 1981, the Comment was submitted by Solicitor General Estelito P.
Mendoza. 5 It opened with this preliminary statement: "The present petition was filed by
the private prosecutors in Criminal Cases Nos. 1700-1706, People v. Pablo Sola, et al.,
pending trial before the Court of First Instance of Negros Occidental. Rightly, any
petition before this Honorable Court on behalf of the People of the Philippines can,
under the law, be instituted only by the Solicitor General. The assertion of the petitioner
private prosecutors that they are instituting the action 'subject to the control and
supervision of the Fiscal' will not, therefore, improve their legal standing." 6 Nonetheless,
it did not press the legal point but instead adopted "the two-pronged trusts of the
petition: 1. the setting aside, by certiorari, of the order of the Municipal Court of
Kabankalan, presided over by Judge Rafael Gasataya, granting bail to the accused in
the criminal cases mentioned above, and 2. the petition for a change of venue or place
of trial of the same criminal cases to avoid a miscarriage of justice. 7
The facts were therein narrated thus: "On September 15, 1980, acting on the evidence
presented by the Philippine Constabulary commander at Hinigaran, Negros Occidental,
the Court of First Instance of that province issued a search warrant for the search and
seizure of tile deceased bodies of seven persons believed in the possession of the

accused Pablo Sola in his hacienda at Sta. Isabel, Kabankalan, Negros Occidental. * * *
On September 16, 1980 armed with the above warrant, elements of the of the 332nd
PC/INP Company proceeded to the place of Sola. Diggings made in a canefield yielded
two common graves containing the bodies of Fernando Fernandez, Mateo Olimpos,
Alfredo Perez, Custodio Juanica, Arsolo Juanica, Rollie Callet and Bienvenido
Emperado. On September 23 and October 1, 1980, the PC provincial commander of
Negros Occidental filed seven (7) separate complaints for murder against the accused
Pablo Sola, Francisco Garcia, Ricardo Garcia, Jose Bethoven Cabral, Florendo
Baliscao and fourteen (14) other persons of unknown names. The cases were docketed
as Criminal Cases No. 4129, 4130, 4131, 4137, 4138, 4139 and 4140 of the Municipal
Court of Kabankalan. After due preliminary examination of the complainant's witnesses
and his other evidence, the municipal court found probable cause against the accused.
It thus issued an order for their a. rest. However, without giving the prosecution the
opportunity to prove that the evidence of guilt of the accused is strong, the court granted
them the right to post bail for their temporary release. The accused Pablo Sola,
Francisco Garcia, and Jose Bethoven Cabral availed themselves of this right and have
since been released from detention. In a parallel development. the witnesses in the
murder cases informed the prosecution of their fears that if the trial is held at the Court
of First Instance branch in Himamaylan which is but 10 kilometers from Kabankalan,
their safety could be jeopardized. At least two of the accused are officials with power
and influence in Kabankalan and they have been released on bail. In addition, most of
the accused remained at large. Indeed, there have been reports made to police
authorities of threats made on the families of the witnesses." 8 The facts alleged argue
strongly for the remedies sought, namely a change of venue and the cancellation of the
bail bonds.
On the very next day, March 15, 1981, this Court issued the following resolution: "The
Court Resolved to: (a) [Note] the comment of the Solicitor General on the urgent petition
for change of venue and cancellation of bail bonds, adopting the plea of the petition,
namely, (1) the setting aside, by certiorari, of the order of the Municipal Court of
Kabankalan, presided over by Judge Rafael Gasataya, granting bail to the accused in
Criminal Cases Nos. 4129, 4130, 4131, 4137, 4138, 4139 and 4140, all entitled "People
of the Philippines v. Mayor Pablo Sola. et al."; (2) the petition for a change of venue or
place of trial of the same criminal cases to avoid a miscarriage of Justice; (b) [Transfer]
the venue of the aforesaid criminal cases to Branch V of the Court of First Instance of
Negros Occidental at Bacolod City, presided by Executive Judge Alfonso Baguio,
considering that District Judge Ostervaldo Emilia of the Court of First Instance, Negros
Occidental, Branch VI at Himamaylan has an approved leave of absence covering the
period from January 12 to March 12, 1981 due to a mild attack of cerebral thrombosis
and that the said Branch V is the nearest court station to Himamaylan: and (c) [Await]
the comment of respondents on the petition to cancel bail, without prejudice to the
public officials concerned taking the necessary measures to assure the safety of the
witnesses of the prosecution." 9 Thus, the issue of a change of venue has become moot
and academic. The comments respectively submitted by respondent Florendo Baliscao
on March 5, 1981, respondent Francisco Garcia on March 11, 1981 and respondent
Pablo Sola on March 16, 1981, dealt solely with the question of the cancellation of the

bail bonds. Such comments were considered as answers, with the case thereafter
deemed submitted for decision.
The sole remaining issue of the cancellation of the bail bonds of respondents, there
being a failure to abide by the basic requirement that the prosecution be heard in a case
where the accused is charged with a capital offense, prior to bail being granted, must be
decided in favor of petitioner. The bail bonds must be cancelled and the case remanded
to the sala of Executive Judge Alfonso Baguio for such hearing. So we rule.
1. It may not be amiss to say a few words on the question of transferring the place of
trial, in this case, from Himamaylan to Bacolod City. The Constitution is quite explicit.
The Supreme Court could order "a change of venue or place of trial to avoid a
miscarriage of justice." 10 The Constitutional Convention of 1971 wisely incorporated the
ruling in the landmark decision of People v. Gutierrez, 11 where Justice J. B. L. Reyes
as ponente vigorously and categorically affirmed: "In the particular case before Us, to
compel the prosecution to proceed to trial in a locality where its witnesses will not be at
liberty to reveal what they know is to make a mockery of the judicial process, and to
betray the very purpose for which courts have been established." 12 Why a change of
venue is imperative was made clear in the Comment of the Solicitor General. Thus:
"The exercise by this Honorable Court of its above constitutional power in this case will
be appropriate. The witnesses in the case are fearful for their lives. They are afraid they
would be killed on their way to or from Himamaylan during any of the days of trial.
Because of qqqts fear, they may either refuse to testify or testimony falsely to save their
lives. 13 Respondent Florendo Baliscao was not averse to such transfer, but his
preference is for a court anywhere in Metro Manila. 14 Respondent Francisco Garcia
confined his comment to the question of the cancellation of the bail bonds. Respondent
Pablo Sola made clear that he had "no objection to the transfer. 15 It may be added that
there may be cases where the fear, objectively viewed, may, to some individuals, be
less than terrifying, but the question must always be the effect it has on the witnesses
who will testify. The primordial aim and intent of the Constitution must ever be kept in
mind. In case of doubt, it should be resolved in favor of a change of venue. As a matter
of fact, there need not be a petition of this character filed before this Court. Such a plea
could have been done administratively. In this particular case, however, there is
justification for the procedure followed in view of the fact that along with the change of
venue, the cancellation of the bail bonds was also sought.
2. Equally so the cancellation of the bail bonds is more than justified. Bail was granted
to the accused in the Order of the Municipal Court without hearing the prosecution That
is to disregard the authoritative doctrine enunciated in People v. San Diego. 16 As
pointed out by Justice Capistrano, speaking for the Court: "The question presented
before us is, whether the prosecution was deprived of procedural due process. The
answer is in the affirmative. We are of the considered opinion that whether the motion
for bail of a defendant who is in custody for a capital offense be resolved in a summary
proceeding or in the course of a regular trial, the prosecution must be given an
opportunity to present, within a reasonable time, all the evidence that it may desire to
introduce before the court should resolve the motion for bail. If, as in the criminal case

involved in the instant special civil action, the prosecution should be denied such an
opportunity, there would be a violation of procedural due process, and the order of the
court granting bail should be considered void on that ground." 17 These words of Justice
Cardozo come to mind: "The law, as we have seen, is sedulous in maintaining for a
defendant charged with crime whatever forms of procedure are of the essence of an
opportunity to defend. Privileges so fundamental as to be inherent in every concept of a
fair trial that could be acceptable to the thought of reasonable men will be kept inviolate
and inviolable, however crushing may be the pressure of incriminating proof. But justice,
though due to the accused, is due to the accuser also. The concept of fairness must not
be strained till it is narrowed to a filament. We are to keep the balance true." 18 This
norm which is of the very essence of due process as the embodiment of justice requires
that the prosecution be given the opportunity to prove that there is strong evidence of
guilt. It does not suffice, as asserted herein, that the questions asked by the municipal
judge before bail was granted could be characterized as searching. That fact did not
cure an infirmity of a jurisdictional character. 19
WHEREFORE, the assailed order of Judge Rafael Gasataya granting bail to private
respondents is nullified, set aside, and declared to be without force and effect.
Executive Judge Alfonso Baguio of the Court of First Instance of Negros Occidental, to
whose sala the cases had been transferred by virtue of the resolution of this Court of
March 5, 1981, is directed forthwith to hear the petitions for bail of private respondents,
with the prosecution being duly heard on the question of whether or not the evidence of
guilt against the respondents is strong. This decision is immediately executory. No
costs.

[G.R. No. 133289. December 23, 1999]

LICERIO A. ANTIPORDA, JR., ELITERIO RUBIACO, VICTOR GASCON and CAESAR


TALIA petitioners, vs. HON. FRANCIS E. GARCHITORENA, HON. EDILBERTO
G. SANDOVAL, HON. CATALINO CASTAEDA, JR. in their capacity as Presiding
Justice and Associate Justices of the Sandiganbayanrespondents.
DECISION
BUENA, J.:
This is a Petition for Certiorari and Prohibition with Preliminary Injunction and/or
Temporary Restraining Order to restrain the respondent Justices of the First Division of the
Sandiganbayan from further proceeding with Crim. Case No. 24339 and from enforcing the
warrants for the arrest of the accused named therein (herein petitioners) or to maintain
the status quo until further orders from this Court.
The antecedent facts of the case are as follows:
Accused Licerio A. Antiporda, Jr., Eliterio Rubiaco, Victor Gascon, and Caesar Talla were
charged with the crime of kidnapping one Elmer Ramos in an Information dated September 18,
1997. It was filed with the First Division of the Sandiganbayan comprised of the Honorable Francis
E. Garchitorena, Edilberto E. Sandoval, and Catalino Castaeda, Jr. The Information reads as
follows:
That on or about September 1, 1995, in the Municipality of Sanchez Mira, Province of Cagayan
and within the jurisdiction of this Honorable Court, the said accused Eliterio Rubiaco, Caesar
Talla, Vicente Gascon and Licerio Antiporda, Jr., armed with guns, conspiring together and
helping one another, by means of force, violence and intimidation and without legal grounds or
any authority of law, did then and there willfully, unlawfully and feloniously kidnap and carry
away one Elmer Ramos from his residence in Marzan, Sanchez Mira, Cagayan against his will
with the use of a Maroon Tamaraw FX motor vehicle.
CONTRARY TO LAW[1]
On November 10, 1997, the Court issued an order giving the prosecution represented by
Prosecutor Evelyn T. Lucero Agcaoili thirty (30) days within which to submit the amendment to
the Information.The said order is quoted in full as follows:
ORDER
This morning, the prosecution represented by Prosecutor Evelyn T. Lucero Agcaoili appeared in
response to this Courts Order of clarification on the propriety of proceeding with the Information
as it stands.

On her own, Prosecutor Agcaoili informed the Court that there were inadequacies in the
allegations in the Information for which reason she would beg leave to amend the same. The
Court for its part expressed anxiety as to the Courts jurisdiction over the case considering that it
was not clear whether or not the subject matter of the accusation was office related.
For this purpose, Prosecutor Agcaoili is given thirty (30) days within which to submit the
amendment embodying whatever changes she believes are appropriate or necessary in order for
the Information to effectively describe the offense herein charged. Within the same period,
Prosecutor Agcaoili shall submit an expansion of the recommendation to file the instant
Information against the accused before this Court indicating thereon the office related character
of the accusation herein so that the Court might effectively exercise its jurisdiction over the
same.
SO ORDERED.[2]
The prosecution on even date complied with the said order and filed an Amended Information,
which was admitted by the Sandiganbayan in a resolution dated November 24, 1997.[3] The
Amended Information thus reads:
That on or about September 10, 1997, at Sanchez Mira, Cagayan and within the jurisdiction of
this Honorable Court, the accused Licerio Antiporda, Jr., being the Municipal Mayor of Buguey,
Cagayan in the exercise of his official duties as such and taking advantage of his position,
ordered, confederated and conspired with Juan Gallardo, Barangay Captain of San Lorenzo,
Buguey, Cagayan (now deceased) and accused Eliterio Rubiaco, barangay councilman of San
Lorenzo, Buguey, Cagayan, Vicente Gascon and Caesar Talla with the use of firearms, force,
violence and intimidation, did then and there willfully, unlawfully and feloniously kidnap and
abduct the victim Elmer Ramos without any authority of law from his residence at Marzan,
Sanchez Mira, Cagayan against his will, with the use of a Maroon Tamaraw FX motor
vehicle and subsequently bring and detain him illegally at the residence of accused Mayor
Licerio Antiporda, Jr. for more than five (5) days.
CONTRARY TO LAW.[4]
Accused then filed an Urgent Omnibus Motion dated November 16, 1997 praying that a
reinvestigation of the case be conducted and the issuance of warrants of arrest be deferred.[5]
An order dated November 26, 1997 was penned by Prosecutor Evelyn T. Lucero-Agcaoili
recommending the denial of the accuseds Urgent Omnibus Motion[6] was approved by
Ombudsman Aniano A. Desierto on January 9, 1998.[7]
The accused thereafter filed on March 5, 1998 a Motion for New Preliminary Investigation
and to Hold in Abeyance and/or Recall Warrant of Arrest Issued.[8] The same was denied in an
order given in open court dated March 12, 1998 "on the ground that there was nothing in the
Amended Information that was added to the original Information so that the accused could not
claim a right to be heard separately in an investigation in the Amended Information. Additionally,
the Court ruled that 'since none of the accused have submitted themselves to the jurisdiction of the
Court, the accused are not in a position to be heard on this matter at this time' (p. 245, Record)."[9]

Subsequently, the accused filed on March 24, 1998 a Motion to Quash the Amended
Information for lack of jurisdiction over the offense charged.[10]
On March 27, 1998, the Sandiganbayan issued an Order, to wit:
"The Motion to Quash filed in behalf of the accused by Atty. Orlando B. Consigna is ignored, it
appearing that the accused have continually refused or otherwise failed to submit themselves to
the jurisdiction of this Court. At all events there is an Amended Information here which makes
an adequate description of the position of the accused thus vesting this Court with the office
related character of the offense of the accused.
"SO ORDERED."[11]
A motion for reconsideration was filed on April 3, 1998 by the accused wherein it was alleged
that the filing of the Motion to Quash and the appearance of their counsel during the scheduled
hearing thereof amounted to their voluntary appearance and invested the court with jurisdiction
over their persons.[12]
The Sandiganbayan denied the motion for reconsideration filed by the accused in its resolution
dated April 24, 1998.[13]
Hence, this petition filed by Licerio A. Antiporda, Jr., Eliterio Rubiaco, Victor Gascon, and
Caesar Talla.
The petitioners pose the following questions for the resolution of this Court.
a) CAN THE SANDIGANBAYAN, WHICH HAS NO JURISDICTION OVER THE
OFFENSE CHARGED IN THE ORIGINAL INFORMATION, SUBSEQUENTLY
ACQUIRE SUCH JURISDICTION BY THE SIMPLE EXPEDIENT OF
AMENDING THE INFORMATION TO SUPPLY, FOR THE FIRST TIME,
JURISDICTIONAL FACTS NOT PREVIOUSLY AVERRED IN THE ORIGINAL
INFORMATION? and
b) COROLLARILY, CAN THE AMENDED INFORMATION BE ALLOWED
WITHOUT CONDUCTING ANEW A PRELIMINARY INVESTIGATION FOR
THE GRAVER OFFENSE CHARGED THEREIN?
The petition is devoid of merit.
Jurisdiction is the power with which courts are invested for administering justice, that is, for
hearing and deciding cases. In order for the court to have authority to dispose of the case on the
merits, it must acquire jurisdiction over the subject matter and the parties.[14]
Section 4, paragraph (a) of P.D. No. 1606, as amended by P.D. No. 1861 provides for the
jurisdiction of the Sandiganbayan:
Sec. 4. Jurisdiction. -- The Sandiganbayan shall exercise:
(a) Exclusive original jurisdiction in all cases involving:
xxx

(2) Other offenses or felonies committed by public officers and employees in relation to their
office, including those employed in government-owned or controlled corporations, whether
simple or complexed with other crimes, where the penalty prescribed by law is higher than
prision correccional or imprisonment for six (6) years, or a fine of P6,000.00. Provided, however,
That offenses or felonies mentioned in this paragraph where the penalty prescribed by law does
not exceed prision correccional or imprisonment for six (6) years or a fine of P6,000.00 shall be
tried by the proper Regional Trial Court, Metropolitan Trial Court, Municipal Trial Court and
Municipal Circuit Trial Court.
The Sandiganbayan exercises not only civil but also criminal jurisdiction. Criminal
jurisdiction, as defined in the case of People vs. Mariano[15], is necessarily the authority to hear
and try a particular offense and impose the punishment for it.
The case of Arula vs. Espino[16]enumerates the requirements wherein a court acquires
jurisdiction to try a criminal case, to wit:
To paraphrase: beyond the pale of disagreement is the legal tenet that a court acquires
jurisdiction to try a criminal case only when the following requisites concur: (1) the offense is
one which the court is by law authorized to take cognizance of, (2) the offense must have been
committed within its territorial jurisdiction, and (3) the person charged with the offense must
have been brought in to its forum for trial, forcibly by warrant of arrest or upon his voluntary
submission to the court.
The petitioners argue that the Sandiganbayan had no jurisdiction to take cognizance of the
case because the original information did not allege that one of the petitioners, Licerio A.
Antiporda, Jr., took advantage of his position as mayor of Buguey, Cagayan to order the
kidnapping of Elmer Ramos. They likewise assert that lacking jurisdiction a court can not order
the amendment of the information. In the same breath, they contend however that the
Sandiganbayan had jurisdiction over the persons of the accused.
They question the assumption of jurisdiction by the Sandiganbayan over their case yet they
insist that said court acquired jurisdiction over their motion to quash. The petitioner can not have
their cake and eat it too.
In the aforementioned case of Arula vs. Espino[17]it was quite clear that all three
requisites, i.e., jurisdiction over the offense, territory and person, must concur before a court can
acquire jurisdiction to try a case.
It is undisputed that the Sandiganbayan had territorial jurisdiction over the case.
And we are in accord with the petitioners when they contended that when they filed a motion
to quash it was tantamount to a voluntary submission to the Courts authority. They cite the case
of Layosa vs. Rodriguez[18] in support of their contention. For therein, it was ruled that the
voluntary appearance of the accused at the pre-suspension hearing amounted to his submission to
the courts jurisdiction even if no warrant of arrest has yet been issued.
To counter this contention of the petitioners the prosecution adverted to case of de los SantosReyes vs. Montesa, Jr.[19] which was decided some 28 years after the Layosa case. In this more
recent case, it was held that:

xxx the accused xxx have no right to invoke the processes of the court since they have not been
placed in the custody of the law or otherwise deprived of their liberty by reason or as a
consequence of the filling of the information. For the same reason, the court had no authority to
act on the petition.
We find that the case of Layosa and de los Santos-Reyes are not inconsistent with each other
since both these cases discussed the rules on when a court acquires jurisdiction over the persons
of the accused, i.e., either through the enforcement of warrants of arrest or their voluntary
submission to the court.
The only difference, we find, is that the de los Santos-Reyes case harped mainly on the warrant
of arrest angle while the Layosa case dealt more on the issue of voluntary submission ruling, that
the appearance at the hearing through a lawyer was a submission to the courts jurisdiction.
Having discussed the third requirement we now come to the question of whether or not the
Sandiganbayan had jurisdiction over the offense charged.
We answer in the negative. The original Information filed with the Sandiganbayan did not
mention that the offense committed by the accused is office-related. It was only after the same was
filed that the prosecution belatedly remembered that a jurisdictional fact was omitted therein.
However, we hold that the petitioners are estopped from assailing the jurisdiction of the
Sandiganbayan for in the supplemental arguments to motion for reconsideration and/or
reinvestigation dated June 10, 1997[20] filed with the same court, it was they who challenged the
jurisdiction of the Regional Trial Court over the case and clearly stated in their Motion for
Reconsideration that the said crime is work connected, which is hereunder quoted, as follows:
Respondents (petitioners herein) have thoroughly scanned the entire records of the instant case
and no where is there any evidence to show that the Honorable Prosecution Office of the
Province of Cagayan have been authorized by the Office of the Honorable Ombudsman to
conduct the Preliminary Investigation much less had the former office been authorized to file the
corresponding Information as the said case, if evidence warrants, fall exclusively with the
jurisdiction of the Honorable Sandiganbayan notwithstanding the presence of other public
officers whose salary range is below 27 and notwithstanding the presence of persons who are not
public officers.
It is a well-settled rule that a party cannot invoke the jurisdiction of a court to secure
affirmative relief against his opponent, and after obtaining or failing to obtain such relief, repudiate
or question that same jurisdiction.[21]
We therefore hold that the Sandiganbayan has jurisdiction over the case because of estoppel
and it was thus vested with the authority to order the amendment of the Information.
Rule 110, Section 14 of the Rules of Court provides thus:
Section 14. Amendment. The information or complaint may be amended, in substance or form,
without leave of court, at any time before the accused pleads; and thereafter and during the trial
as to all matters of form, by leave and at the discretion of the court, when the same can be done
without prejudice to the rights of the accused.

xxx xxx xxx


Petitioner prayed that a reinvestigation be made in view of the Amended Information.
We hold that the reinvestigation is not necessary anymore. A reinvestigation is proper only if
the accuseds substantial rights would be impaired. In the case at bar, we do not find that their rights
would be unduly prejudiced if the Amended Information is filed without a reinvestigation taking
place. The amendments made to the Information merely describe the public positions held by the
accused/petitioners and stated where the victim was brought when he was kidnapped.
It must here be stressed that a preliminary investigation is essentially inquisitorial, and it is
often the only means of discovering the persons who may be reasonably charged with a crime, to
enable the prosecutor to prepare his complaint or information. It is not a trial of the case on the
merits and has no purpose except that of determining whether a crime has been committed and
whether there is probable cause to believe that the accused is guilty thereof, and it does not place
the persons accused in jeopardy. It is not the occasion for the full and exhaustive display of the
parties evidence; it is for the presentation of such evidence only as may engender a well-grounded
belief that an offense has been committed and that the accused is probably guilty thereof.[22]
The purpose of a preliminary investigation has been achieved already and we see no cogent
nor compelling reason why a reinvestigation should still be conducted.
As an aside, an offense is considered committed in relation to office when it is intimately
connected with their respective offices and was perpetrated while they were in the performance,
though improper or irregular, of their official functions.[23]
In the case of Cunanan vs. Arceo, it was held that:
... the absence in the information filed on 5 April 1991 before Branch 46 of the RTC of San
Fernando, Pampanga, of an allegation that petitioner had committed the offense charged in
relation to his office is immaterial and easily remedied. Respondent RTC judges had forwarded
petitioners case to the Sandiganbayan, and the complete records transmitted thereto in
accordance with the directions of this Court set out in the Asuncion case: x x x As if it was
originally filed with [the Sandiganbayan]. That Information may be amended at any time before
arraignment before the Sandiganbayan, and indeed, by leave of court at any time before
judgment is rendered by the Sandiganbayan, considering that such an amendment would not
affect the juridical nature of the offense charged (i.e., murder), the qualifying circumstances
alleged in the information, or the defenses that petitioner may assert before
the Sandiganbayan. In other words, the amendment may be made before
the Sandiganbayan without surprising the petitioner or prejudicing his substantive
rights.[24] (Underscoring Supplied)
WHEREFORE, IN VIEW OF THE FOREGOING, the petition is hereby DISMISSED.
SO ORDERED.

[G.R. No. 123340. August 29, 2002]

LUTGARDA CRUZ, petitioner, vs. THE COURT OF APPEALS, PEOPLE OF THE


PHILIPPINES and the HEIRS OF ESTANISLAWA C. REYES, represented by
MIGUEL C. REYES, respondents.
DECISION
CARPIO, J.:

The Case
This is a petition for review on certiorari under Rule 45 of the Rules of Court to reverse
the Decision of the Court of Appeals dated March 31, 1995[1] and its Resolution dated
December 1, 1995.[2] The Court of Appeals dismissed for being insufficient in substance
the Petition for Certiorari and Mandamus, which sought to nullify two orders of the
Regional Trial Court of Manila, Branch 53, dated April 18, 1994 and May 6, 1994.

The Antecedent Facts


The City Prosecutor of Manila charged petitioner with the crime of Estafa thru
Falsification of Public Document before the Manila Regional Trial Court.[3] Petitioner
executed before a Notary Public in the City of Manila an Affidavit of Self-Adjudication of
a parcel of land stating that she was the sole surviving heir of the registered owner when
in fact she knew there were other surviving heirs. Since the offended party did not reserve
the right to file a separate civil action arising from the criminal offense, the civil action was
deemed instituted in the criminal case.
After trial on the merits, the trial court rendered its decision dated January 17, 1994
acquitting petitioner on the ground of reasonable doubt. In the same decision, the trial
court rendered judgment on the civil aspect of the case, ordering the return to the
surviving heirs of the parcel of land located in Bulacan.[4]
On January 28, 1994, petitioner received a copy of the decision.
On February 10, 1994, petitioner filed by registered mail a motion for reconsideration
dated February 7, 1994, assailing the trial courts ruling on the civil aspect of the criminal
case. Petitioner furnished the City Prosecutor a copy of the motion by registered mail.
On April 18, 1994, the trial court denied petitioners motion for reconsideration stating:
Acting on the Motion for Reconsideration dated February 7, 1994, filed by the accused through
counsel and considering that there is nothing to show that the Office of the City Prosecutor was
actually furnished or served with a copy of the said Motion for Reconsideration within the

reglementary period of fifteen (15) days from receipt by the accused on January 28, 1994 of a
copy of the Courts decision dated January 17, 1994, so that the same is already final and
executory, let the Motion for Reconsideration be Denied for lack of merit.[5]
Petitioner moved for a reconsideration of the trial courts order of April 18, 1994. The
trial court denied the same in an order dated May 6, 1994, to wit:
Under the Interim Rules, no party shall be allowed a second motion for reconsideration of a final
order or judgment (Sec. 4). The motion of accused dated 22 April 1994 is a violation of this rule.
WHEREFORE, said motion is DENIED.[6]
Left with no recourse, petitioner filed a petition for certiorari and mandamus with the
Court of Appeals to nullify the two assailed orders of the trial court. Petitioner also asked
the Court of Appeals to compel the trial court to resolve her motion for reconsideration of
the decision dated February 7, 1994.

The Ruling of the Court of Appeals


On March 31, 1995, the Court of Appeals denied due course to the petition and
dismissed the case for being insufficient in substance.
The Court of Appeals sustained the trial courts order of April 18, 1994 denying
petitioners motion for reconsideration. The Court of Appeals declared in part:
Section 10, Rule 13, Rules of Court, provides as follows:
SEC. 10. Proof of Service. Proof of personal service shall consist of a written admission of the
party served, or the affidavit of the party serving, containing a full statement of the date, place
and manner of service. If the service is by ordinary mail, proof thereof shall consist of an
affidavit of the person mailing of facts showing compliance with Section 5 of this rule. If service
is made by registered mail, proof shall be made by such affidavit and the registry receipt issued
by the mailing office. The registry return card shall be filed immediately upon receipt thereof by
the sender, or in lieu thereof the letter unclaimed together with the certified or sworn copy of the
notice given by the postmaster to the addressee.
Patent from the language of the said section is that in case service is made by registered mail,
proof of service shall be made by (a) affidavit of the person mailing and (b) the registry receipt
issued by the mailing office. Both must concur. In the case at bench, there was no such affidavit
or registry receipt when the motion was considered. Thus, respondent Judge cannot be said to
have acted with grave abuse of discretion amounting to lack of jurisdiction, in ruling in the
manner he did.[7]
The Court of Appeals also affirmed the trial courts order of May 6, 1994 denying the
subsequent motion for reconsideration, as follows:

xxx, while there is merit in petitioners submission that the motion for reconsideration dated April
22, 1994 was not a second motion for reconsideration of a final order or judgment, as
contemplated in the Interim Rules because the motion sought to impugn the order dated 18 April
1994 not on the basis of the issues raised in the motion for reconsideration dated 07 February
1994 but on the erroneous legal conclusion of the order dated May 6, 1994,[8] this is already
academic. The decision dated January 7, 1994 had long become final when the second motion
for reconsideration was filed on 03 May 1994. Hence, the pairing Judge who issued the order on
06 May 1994 had no more legal competence to promulgate the same.[9]
Finally, the Court of Appeals upheld the assailed decision of the trial court on the civil
aspect of the case, to wit:
x x x, the institution of a criminal action carries with it the civil action for the recovery of the
civil liability arising from the offense charged. There was neither reservation nor waiver of the
right to file the civil action separately nor has one been instituted to the criminal action. Hence,
the civil action for the civil liability has been impliedly instituted with the filing of the criminal
case before respondent Judge. This is the law on the matter. The proposition submitted by
petitioner that the court presided by respondent Judge had no jurisdiction over the property
because it is located in Bulacan - outside the territorial jurisdiction of said court -does not hold
water. Being a civil liability arising from the offense charged, the governing law is the Rules of
Criminal Procedure, not the civil procedure rules which pertain to civil action arising from the
initiatory pleading that gives rise to the suit.[10]
In the dispositive portion of its assailed decision, the Court of Appeals declared:
WHEREFORE, the instant petition not being sufficient in substance is hereby DENIED DUE
COURSE and the case DISMISSED.[11]
In a resolution dated December 1, 1995, the Court of Appeals denied petitioners
motion for reconsideration.[12]
Hence, this petition.

The Issues
In her Memorandum, petitioner raises the following issues:
1. WHETHER THE COURT OF APPEALS ERRED IN NOT FINDING THAT THE
PROSECUTION WAS DULY FURNISHED WITH COPY OF THE
PETITIONERS MOTION FOR RECONSIDERATION WITH RESPECT TO
THE DECISION ON THE CIVIL ASPECT OF CRIMINAL CASE NO. 87-54773
(SIC) OF THE REGIONAL TRIAL COURT OF MANILA, BRANCH 53.
2. WHETHER THE COURT OF APPEALS ERRED IN FINDING THAT THE
REGIONAL TRIAL COURT OF MANILA HAD JURISDICTION TO RENDER
JUDGMENT ON THE CIVIL ASPECT OF CRIMINAL CASE NO. 87-57743

FOR FALSIFICATION OF PUBLIC DOCUMENT, INVOLVING A PROPERTY


LOCATED IN BULACAN.
3. WHETHER THE COURT OF APPEALS ERRED IN NOT FINDING THAT THE
PETITIONER WAS DENIED DUE PROCESS WHEN THE REGIONAL TRIAL
COURT OF MANILA, BRANCH 53, RENDERED DECISION ON THE CIVIL
ASPECT OF CRIMINAL CASE NO. 87-57743.[13]

The Ruling of the Court


We grant the petition.
When the accused is acquitted on reasonable doubt but is adjudged civilly liable, his
motion for reconsideration of the civil aspect must be served not only on the prosecution,
also on the offended party if the latter is not represented by a private counsel. Moreover,
if the trial court has jurisdiction over the subject matter and over the accused, and the
crime was committed within its territorial jurisdiction, it necessarily exercises jurisdiction
over all matters that the law requires the court to resolve. This includes the power to order
the restitution to the offended party of real property located in another province.

Absence of Proof of Service


The first issue is whether petitioners motion for reconsideration dated February 7,
1994 complied with the mandatory requirements of Section 6, Rule 15 on proof of service.
Petitioner submits that the Court of Appeals erred in sustaining the trial courts finding that
the City Prosecutor was not duly and timely furnished with petitioners motion for
reconsideration of February 7, 1994.
Petitioner asserts that both copies of the motion for reconsideration were sent to the
trial court and the City Prosecutor by registered mail on February 10, 1994. Petitioner
relies on jurisprudence that the date of mailing is the date of filing, arguing that the date
of mailing of both motions was on February 10, 1994. Petitioner maintains that the motion
was properly filed within the 15-day period, citing the registry return card which shows
actual receipt on February 22, 1994 by the City Prosecutor of a copy of the motion.
The Court of Appeals, noting that petitioner received a copy of the decision on
January 28, 1994, stated that petitioner had until February 12, 1994 to appeal the decision
or file a motion for reconsideration. The Court of Appeals ruled that petitioner, by filing a
motion for reconsideration without any proof of service, merely filed a scrap of paper and
not a motion for reconsideration. Hence, the reglementary period of petitioner to appeal
continued to run and lapsed after the 15-day period, making the trial courts decision final
and executory.
We agree with the Court of Appeals that petitioner patently failed to comply with the
mandatory requirements on proof of service insofar as the public prosecutor is concerned.
The Court has stressed time and again that non-compliance with Sections 4, 5 and 6 of

Rule 15 is a fatal defect. The well-settled rule is that a motion which fails to comply with
Sections 4, 5, and 6 of Rule 15 is a useless piece of paper. If filed, such motion is not
entitled to judicial cognizance and does not stop the running of the reglementary period
for filing the requisite pleading.[14]
Section 6 of Rule 15 reads:
SEC. 6. - Proof of service to be filed with motions. No motion shall be acted upon by the
court, without proof of service of the notice thereof.[15] (Emphasis supplied)
From the language of the rule, proof of service is mandatory. Without such proof of
service to the adverse party, a motion is nothing but an empty formality deserving no
judicial cognizance.
Section 13 of Rule 13 further requires that:
SEC. 13. Proof of Service. x x x. If service is made by registered mail, proof shall be made by
such affidavit and the registry receipt issued by the mailing office. The registry return card shall
be filed immediately upon its receipt by the sender, or in lieu thereof the unclaimed letter
together with the certified or sworn copy of the notice given by the postmaster to the
addressee.[16] (Emphasis supplied)
If service is by registered mail, proof of service consists of the affidavit of the person
mailing and the registry receipt, both of which must be appended to the motion. Absent
one or the other, or worse both, there is no proof of service.
In the instant case, an examination of the record shows that petitioner received a
copy of the trial courts decision of January 17, 1994 on January 28, 1994. Within the
reglementary period to appeal, petitioner filed on February 10, 1994, by registered mail,
a motion for reconsideration. However, petitioner failed to attach both the affidavit and
the registry receipt to the motion for reconsideration as required by the Rules.
The defect of the motion is apparent on its face. Petitioners motion for reconsideration
was a mere scrap of paper as it did not contain the required proof of service.
However, petitioner is contesting that part of the decision of the trial court finding him
civilly liable even as he is acquitted from the criminal charge on reasonable doubt. This
raises the issue of whether the public prosecutor is the only proper party to be served
with petitioners motion for reconsideration. The present Rules do not require the accused
to serve a copy of his motion for reconsideration on the offended party who may not be
represented by a private counsel. The Rules require service only on the public prosecutor
if the offended party is not represented by a private counsel.
A judgment of acquittal is immediately final and executory and the prosecution cannot
appeal the acquittal because of the constitutional prohibition against double jeopardy.
However, either the offended party or the accused may appeal the civil aspect of the
judgment despite the acquittal of the accused. The public prosecutor has generally no
interest in appealing the civil aspect of a decision acquitting the accused.

The acquittal ends the work of the public prosecutor and the case is terminated as far
as he is concerned.
The real parties in interest in the civil aspect of a decision are the offended party and
the accused. Thus, any appeal or motion for reconsideration of the civil aspect of a
decision in a criminal case must be served on the other real party in interest. If the
offended party appeals or moves for reconsideration, the accused is necessarily served
a copy of the pleading through his counsel.
If the accused appeals or moves for reconsideration, a lacuna arises if the offended
party is not represented by a private counsel. In such a situation, under the present Rules
only the public prosecutor is served the notice of appeal or a copy of the motion for
reconsideration. To fill in this lacuna in the present Rules, we require that henceforth if
the accused appeals or moves for reconsideration, he should serve a copy of his pleading
on the offended party himself if the latter is not represented by a private counsel. This is
in addition to service on the public prosecutor who is the counsel of record of the State.
In the instant case, the Court notes that petitioner did not serve a copy of her motion
for reconsideration on the offended party who was not represented by a private counsel
in the trial court. In the interest of justice, and considering that the present Rules are silent
on the matter, it is only fair to give petitioner a period of five days from receipt of this
decision within which to serve a copy of her motion for reconsideration on the offended
party.

Trial courts jurisdiction over the civil aspect.


Petitioner maintains that the Court of Appeals erred in finding that the trial court had
jurisdiction to render judgment on the civil aspect of the criminal case. Petitioner asserts
that the Manila trial court had no jurisdiction over the parcel of land in Bulacan which is
outside the trial courts territorial jurisdiction.
In upholding the trial courts jurisdiction, the Court of Appeals held:
Being a civil liability arising from the offense charged, the governing law is the Rules of
Criminal Procedure, not the civil procedure rules which pertain to civil action arising from the
initiatory pleading that gives rise to the suit.[17]
We agree with the ruling of the Court of Appeals.
Petitioner asserts that the location of the subject property outside the courts territorial
jurisdiction deprived the trial court of jurisdiction over the civil aspect of the criminal case.
This argument is contrary to the law and the rules.
There are three important requisites which must be present before a court can acquire
criminal jurisdiction. First, the court must have jurisdiction over the subject matter.
Second, the court must have jurisdiction over the territory where the offense was
committed. Third, the court must have jurisdiction over the person of the accused. [18] In
the instant case, the trial court had jurisdiction over the subject matter as the law has

conferred on the court the power to hear and decide cases involving estafa through
falsification of a public document. The trial court also had jurisdiction over the offense
charged since the crime was committed within its territorial jurisdiction. The trial court also
acquired jurisdiction over the person of accused-petitioner because she voluntarily
submitted to the courts authority.
Where the court has jurisdiction over the subject matter and over the person of the
accused, and the crime was committed within its territorial jurisdiction, the court
necessarily exercises jurisdiction over all issues that the law requires the court to resolve.
One of the issues in a criminal case is the civil liability of the accused arising from the
crime. Article 100 of the Revised Penal Code provides that [E]very person criminally liable
for a felony is also civilly liable. Article 104 of the same Code states that civil liability x x x
includes restitution.
The action for recovery of civil liability is deemed instituted in the criminal action
unless reserved by the offended party.[19] In the instant case, the offended party did not
reserve the civil action and the civil action was deemed instituted in the criminal action.
Although the trial court acquitted petitioner of the crime charged, the acquittal, grounded
on reasonable doubt, did not extinguish the civil liability.[20] Thus, the Manila trial court
had jurisdiction to decide the civil aspect of the instant case - ordering restitution even if
the parcel of land is located in Bulacan.
Consequently, while we find no reversible error in the decision of the Court of Appeals
as to proof of service and the trial courts jurisdiction on the civil aspect, we remand this
case for further proceedings in the interest of justice.
WHEREFORE, petitioner is given five (5) days from receipt of this decision within
which to serve a copy of her motion for reconsideration on the offended party. Let this
case be remanded to the trial court for further proceedings.
SO ORDERED.

G.R. No. 168539

March 25, 2014

PEOPLE OF THE PHILIPPINES, Petitioner,


vs.
HENRY T. GO, Respondent.
DECISION
PERALTA, J.:
Before the Court is a petition for review on certiorari assailing the Resolution 1 of the
Third Division2 of the Sandiganbayan (SB) dated June 2, 2005 which quashed the
Information filed against herein respondent for alleged violation of Section 3 (g) of
Republic Act No. 3019 (R.A. 3019), otherwise known as the Anti-Graft and Corrupt
Practices Act.
The Information filed against respondent is an offshoot of this Court's Decision3 in Agan,
Jr. v. Philippine International Air Terminals Co., Inc. which nullified the various contracts
awarded by the Government, through the Department of Transportation and
Communications (DOTC), to Philippine Air Terminals, Co., Inc. (PIATCO) for the
construction, operation and maintenance of the Ninoy Aquino International Airport
International Passenger Terminal III (NAIA IPT III). Subsequent to the above Decision, a
certain Ma. Cecilia L. Pesayco filed a complaint with the Office of the Ombudsman
against several individuals for alleged violation of R.A. 3019. Among those charged was
herein respondent, who was then the Chairman and President of PIATCO, for having
supposedly conspired with then DOTC Secretary Arturo Enrile (Secretary Enrile) in
entering into a contract which is grossly and manifestly disadvantageous to the
government.
On September 16, 2004, the Office of the Deputy Ombudsman for Luzon found
probable cause to indict, among others, herein respondent for violation of Section 3(g)
of R.A. 3019. While there was likewise a finding of probable cause against Secretary
Enrile, he was no longer indicted because he died prior to the issuance of the resolution
finding probable cause.
Thus, in an Information dated January 13, 2005, respondent was charged before the SB
as follows:
On or about July 12, 1997, or sometime prior or subsequent thereto, in Pasay City,
Metro Manila, Philippines and within the jurisdiction of this Honorable Court, the late
ARTURO ENRILE, then Secretary of the Department of Transportation and
Communications (DOTC), committing the offense in relation to his office and taking
advantage of the same, in conspiracy with accused, HENRY T. GO, Chairman and
President of the Philippine International Air Terminals, Co., Inc. (PIATCO), did then and
there, willfully, unlawfully and criminally enter into a Concession Agreement, after the
project for the construction of the Ninoy Aquino International Airport International

Passenger Terminal III (NAIA IPT III) was awarded to Paircargo Consortium/PIATCO,
which Concession Agreement substantially amended the draft Concession Agreement
covering the construction of the NAIA IPT III under Republic Act 6957, as amended by
Republic Act 7718 (BOT law), specifically the provision on Public Utility Revenues, as
well as the assumption by the government of the liabilities of PIATCO in the event of the
latter's default under Article IV, Section 4.04 (b) and (c) in relation to Article 1.06 of the
Concession Agreement, which terms are more beneficial to PIATCO while manifestly
and grossly disadvantageous to the government of the Republic of the Philippines. 4
The case was docketed as Criminal Case No. 28090.
On March 10, 2005, the SB issued an Order, to wit:
The prosecution is given a period of ten (10) days from today within which to show
cause why this case should not be dismissed for lack of jurisdiction over the person of
the accused considering that the accused is a private person and the public official
Arturo Enrile, his alleged co-conspirator, is already deceased, and not an accused in
this case.5
The prosecution complied with the above Order contending that the SB has already
acquired jurisdiction over the person of respondent by reason of his voluntary
appearance, when he filed a motion for consolidation and when he posted bail. The
prosecution also argued that the SB has exclusive jurisdiction over respondent's case,
even if he is a private person, because he was alleged to have conspired with a public
officer.6
On April 28, 2005, respondent filed a Motion to Quash 7 the Information filed against him
on the ground that the operative facts adduced therein do not constitute an offense
under Section 3(g) of R.A. 3019. Respondent, citing the show cause order of the SB,
also contended that, independently of the deceased Secretary Enrile, the public officer
with whom he was alleged to have conspired, respondent, who is not a public officer nor
was capacitated by any official authority as a government agent, may not be prosecuted
for violation of Section 3(g) of R.A. 3019.
The prosecution filed its Opposition.8
On June 2, 2005, the SB issued its assailed Resolution, pertinent portions of which read
thus:
Acting on the Motion to Quash filed by accused Henry T. Go dated April 22, 2005, and it
appearing that Henry T. Go, the lone accused in this case is a private person and his
alleged co-conspirator-public official was already deceased long before this case was
filed in court, for lack of jurisdiction over the person of the accused, the Court grants the
Motion to Quash and the Information filed in this case is hereby ordered quashed and
dismissed.9

Hence, the instant petition raising the following issues, to wit:


I
WHETHER OR NOT THE COURT A QUO GRAVELY ERRED AND DECIDED A
QUESTION OF SUBSTANCE IN A MANNER NOT IN ACCORD WITH LAW OR
APPLICABLE JURISPRUDENCE IN GRANTING THE DEMURRER TO EVIDENCE
AND IN DISMISSING CRIMINAL CASE NO. 28090 ON THE GROUND THAT IT HAS
NO JURISDICTION OVER THE PERSON OF RESPONDENT GO.
II
WHETHER OR NOT THE COURT A QUO GRAVELY ERRED AND DECIDED A
QUESTION OF SUBSTANCE IN A MANNER NOT IN ACCORD WITH LAW OR
APPLICABLE JURISPRUDENCE, IN RULING THAT IT HAS NO JURISDICTION
OVER THE PERSON OF RESPONDENT GO DESPITE THE IRREFUTABLE FACT
THAT HE HAS ALREADY POSTED BAIL FOR HIS PROVISIONAL LIBERTY
III
WHETHER OR NOT THE COURT A QUO GRAVELY ERRED WHEN, IN COMPLETE
DISREGARD OF THE EQUAL PROTECTION CLAUSE OF THE CONSTITUTION, IT
QUASHED THE INFORMATION AND DISMISSED CRIMINAL CASE NO. 2809010
The Court finds the petition meritorious.
Section 3 (g) of R.A. 3019 provides:
Sec. 3. Corrupt practices of public officers. In addition to acts or omissions of public
officers already penalized by existing law, the following shall constitute corrupt practices
of any public officer and are hereby declared to be unlawful:
xxxx
(g) Entering, on behalf of the Government, into any contract or transaction manifestly
and grossly disadvantageous to the same, whether or not the public officer profited or
will profit thereby.
The elements of the above provision are:
(1) that the accused is a public officer;
(2) that he entered into a contract or transaction on behalf of the government;
and

(3) that such contract or transaction is grossly and manifestly disadvantageous to


the government.11
At the outset, it bears to reiterate the settled rule that private persons, when acting in
conspiracy with public officers, may be indicted and, if found guilty, held liable for the
pertinent offenses under Section 3 of R.A. 3019, in consonance with the avowed policy
of the anti-graft law to repress certain acts of public officers and private persons alike
constituting graft or corrupt practices act or which may lead thereto.12 This is the
controlling doctrine as enunciated by this Court in previous cases, among which is a
case involving herein private respondent.13
The only question that needs to be settled in the present petition is whether herein
respondent, a private person, may be indicted for conspiracy in violating Section 3(g) of
R.A. 3019 even if the public officer, with whom he was alleged to have conspired, has
died prior to the filing of the Information.
Respondent contends that by reason of the death of Secretary Enrile, there is no public
officer who was charged in the Information and, as such, prosecution against
respondent may not prosper.
The Court is not persuaded.
It is true that by reason of Secretary Enrile's death, there is no longer any public officer
with whom respondent can be charged for violation of R.A. 3019. It does not mean,
however, that the allegation of conspiracy between them can no longer be proved or
that their alleged conspiracy is already expunged. The only thing extinguished by the
death of Secretary Enrile is his criminal liability. His death did not extinguish the crime
nor did it remove the basis of the charge of conspiracy between him and private
respondent. Stated differently, the death of Secretary Enrile does not mean that there
was no public officer who allegedly violated Section 3 (g) of R.A. 3019. In fact, the
Office of the Deputy Ombudsman for Luzon found probable cause to indict Secretary
Enrile for infringement of Sections 3 (e) and (g) of R.A. 3019.14 Were it not for his death,
he should have been charged.
The requirement before a private person may be indicted for violation of Section 3(g) of
R.A. 3019, among others, is that such private person must be alleged to have acted in
conspiracy with a public officer. The law, however, does not require that such person
must, in all instances, be indicted together with the public officer. If circumstances exist
where the public officer may no longer be charged in court, as in the present case
where the public officer has already died, the private person may be indicted alone.
Indeed, it is not necessary to join all alleged co-conspirators in an indictment for
conspiracy.15 If two or more persons enter into a conspiracy, any act done by any of
them pursuant to the agreement is, in contemplation of law, the act of each of them and
they are jointly responsible therefor.16 This means that everything said, written or done
by any of the conspirators in execution or furtherance of the common purpose is

deemed to have been said, done, or written by each of them and it makes no difference
whether the actual actor is alive or dead, sane or insane at the time of trial. 17 The death
of one of two or more conspirators does not prevent the conviction of the survivor or
survivors.18 Thus, this Court held that:
x x x [a] conspiracy is in its nature a joint offense. One person cannot conspire alone.
The crime depends upon the joint act or intent of two or more persons. Yet, it does not
follow that one person cannot be convicted of conspiracy. So long as the acquittal or
death of a co-conspirator does not remove the bases of a charge for conspiracy, one
defendant may be found guilty of the offense.19
The Court agrees with petitioner's contention that, as alleged in the Information filed
against respondent, which is deemed hypothetically admitted in the latter's Motion to
Quash, he (respondent) conspired with Secretary Enrile in violating Section 3 (g) of R.A.
3019 and that in conspiracy, the act of one is the act of all. Hence, the criminal liability
incurred by a co-conspirator is also incurred by the other co-conspirators.
Moreover, the Court agrees with petitioner that the avowed policy of the State and the
legislative intent to repress "acts of public officers and private persons alike, which
constitute graft or corrupt practices,"20 would be frustrated if the death of a public officer
would bar the prosecution of a private person who conspired with such public officer in
violating the Anti-Graft Law.
In this regard, this Court's disquisition in the early case of People v. Peralta 21 as to the
nature of and the principles governing conspiracy, as construed under Philippine
jurisdiction, is instructive, to wit:
x x x A conspiracy exists when two or more persons come to an agreement concerning
the commission of a felony and decide to commit it. Generally, conspiracy is not a crime
except when the law specifically provides a penalty therefor as in treason, rebellion and
sedition. The crime of conspiracy known to the common law is not an indictable offense
in the Philippines. An agreement to commit a crime is a reprehensible act from the viewpoint of morality, but as long as the conspirators do not perform overt acts in
furtherance of their malevolent design, the sovereignty of the State is not outraged and
the tranquility of the public remains undisturbed.
However, when in resolute execution of a common scheme, a felony is committed by
two or more malefactors, the existence of a conspiracy assumes pivotal importance in
the determination of the liability of the perpetrators. In stressing the significance of
conspiracy in criminal law, this Court in U.S. vs. Infante and Barreto opined that
While it is true that the penalties cannot be imposed for the mere act of conspiring to
commit a crime unless the statute specifically prescribes a penalty therefor,
nevertheless the existence of a conspiracy to commit a crime is in many cases a fact of
vital importance, when considered together with the other evidence of record, in

establishing the existence, of the consummated crime and its commission by the
conspirators.
Once an express or implied conspiracy is proved, all of the conspirators are liable as coprincipals regardless of the extent and character of their respective active participation
in the commission of the crime or crimes perpetrated in furtherance of the conspiracy
because in contemplation of law the act of one is the act of all. The foregoing rule is
anchored on the sound principle that "when two or more persons unite to accomplish a
criminal object, whether through the physical volition of one, or all, proceeding severally
or collectively, each individual whose evil will actively contributes to the wrong-doing is
in law responsible for the whole, the same as though performed by himself alone."
Although it is axiomatic that no one is liable for acts other than his own, "when two or
more persons agree or conspire to commit a crime, each is responsible for all the acts
of the others, done in furtherance of the agreement or conspiracy." The imposition of
collective liability upon the conspirators is clearly explained in one case where this Court
held that x x x it is impossible to graduate the separate liability of each (conspirator)
without taking into consideration the close and inseparable relation of each of them with
the criminal act, for the commission of which they all acted by common agreement x x x.
The crime must therefore in view of the solidarity of the act and intent which existed
between the x x x accused, be regarded as the act of the band or party created by
them, and they are all equally responsible x x x
Verily, the moment it is established that the malefactors conspired and confederated in
the commission of the felony proved, collective liability of the accused conspirators
attaches by reason of the conspiracy, and the court shall not speculate nor even
investigate as to the actual degree of participation of each of the perpetrators present at
the scene of the crime. Of course, as to any conspirator who was remote from the situs
of aggression, he could be drawn within the enveloping ambit of the conspiracy if it be
proved that through his moral ascendancy over the rest of the conspirators the latter
were moved or impelled to carry out the conspiracy.
In fine, the convergence of the wills of the conspirators in the scheming and execution
of the crime amply justifies the imputation to all of them the act of any one of them. It is
in this light that conspiracy is generally viewed not as a separate indictable offense, but
a rule for collectivizing criminal liability.
xxxx
x x x A time-honored rule in the corpus of our jurisprudence is that once conspiracy is
proved, all of the conspirators who acted in furtherance of the common design are liable
as co-principals. This rule of collective criminal liability emanates from the ensnaring
nature of conspiracy. The concerted action of the conspirators in consummating their
common purpose is a patent display of their evil partnership, and for the consequences
of such criminal enterprise they must be held solidarily liable. 22

This is not to say, however, that private respondent should be found guilty of conspiring
with Secretary Enrile. It is settled that the absence or presence of conspiracy is factual
in nature and involves evidentiary matters.23 Hence, the allegation of conspiracy against
respondent is better left ventilated before the trial court during trial, where respondent
can adduce evidence to prove or disprove its presence.
Respondent claims in his Manifestation and Motion24 as well as in his Urgent Motion to
Resolve25 that in a different case, he was likewise indicted before the SB for conspiracy
with the late Secretary Enrile in violating the same Section 3 (g) of R.A. 3019 by
allegedly entering into another agreement (Side Agreement) which is separate from the
Concession Agreement subject of the present case. The case was docketed as Criminal
Case No. 28091. Here, the SB, through a Resolution, granted respondent's motion to
quash the Information on the ground that the SB has no jurisdiction over the person of
respondent. The prosecution questioned the said SB Resolution before this Court via a
petition for review on certiorari. The petition was docketed as G.R. No. 168919. In a
minute resolution dated August 31, 2005, this Court denied the petition finding no
reversible error on the part of the SB. This Resolution became final and executory on
January 11, 2006. Respondent now argues that this Court's resolution in G.R. No.
168919 should be applied in the instant case.
The Court does not agree. Respondent should be reminded that prior to this Court's
ruling in G.R. No. 168919, he already posted bail for his provisional liberty. In fact, he
even filed a Motion for Consolidation26 in Criminal Case No. 28091. The Court agrees
with petitioner's contention that private respondent's act of posting bail and filing his
Motion for Consolidation vests the SB with jurisdiction over his person. The rule is well
settled that the act of an accused in posting bail or in filing motions seeking affirmative
relief is tantamount to submission of his person to the jurisdiction of the court. 27
Thus, it has been held that:
When a defendant in a criminal case is brought before a competent court by virtue of a
warrant of arrest or otherwise, in order to avoid the submission of his body to the
jurisdiction of the court he must raise the question of the courts jurisdiction over his
person at the very earliest opportunity. If he gives bail, demurs to the complaint or files
any dilatory plea or pleads to the merits, he thereby gives the court jurisdiction over his
person. (State ex rel. John Brown vs. Fitzgerald, 51 Minn., 534)
xxxx
As ruled in La Naval Drug vs. CA [236 SCRA 78, 86]:
"[L]ack of jurisdiction over the person of the defendant may be waived either expressly
or impliedly. When a defendant voluntarily appears, he is deemed to have submitted
himself to the jurisdiction of the court. If he so wishes not to waive this defense, he must
do so seasonably by motion for the purpose of objecting to the jurisdiction of the court;
otherwise, he shall be deemed to have submitted himself to that jurisdiction."

Moreover, "[w]here the appearance is by motion for the purpose of objecting to the
jurisdiction of the court over the person, it must be for the sole and separate purpose of
objecting to said jurisdiction. If the appearance is for any other purpose, the defendant
is deemed to have submitted himself to the jurisdiction of the court. Such an
appearance gives the court jurisdiction over the person."
Verily, petitioners participation in the proceedings before the Sandiganbayan was not
confined to his opposition to the issuance of a warrant of arrest but also covered other
matters which called for respondent courts exercise of its jurisdiction. Petitioner may
not be heard now to deny said courts jurisdiction over him. x x x.28
In the instant case, respondent did not make any special appearance to question the
jurisdiction of the SB over his person prior to his posting of bail and filing his Motion for
Consolidation. In fact, his Motion to Quash the Information in Criminal Case No. 28090
only came after the SB issued an Order requiring the prosecution to show cause why
the case should not be dismissed for lack of jurisdiction over his person.
As a recapitulation, it would not be amiss to point out that the instant case involves a
contract entered into by public officers representing the government. More importantly,
the SB is a special criminal court which has exclusive original jurisdiction in all cases
involving violations of R.A. 3019 committed by certain public officers, as enumerated in
P.D. 1606 as amended by R.A. 8249. This includes private individuals who are charged
as co-principals, accomplices or accessories with the said public officers. In the instant
case, respondent is being charged for violation of Section 3(g) of R.A. 3019, in
conspiracy with then Secretary Enrile. Ideally, under the law, both respondent and
Secretary Enrile should have been charged before and tried jointly by the
Sandiganbayan. However, by reason of the death of the latter, this can no longer be
done. Nonetheless, for reasons already discussed, it does not follow that the SB is
already divested of its jurisdiction over the person of and the case involving herein
respondent. To rule otherwise would mean that the power of a court to decide a case
would no longer be based on the law defining its jurisdiction but on other factors, such
as the death of one of the alleged offenders.
Lastly, the issues raised in the present petition involve matters which are mere incidents
in the main case and the main case has already been pending for over nine (9) years.
Thus, a referral of the case to the Regional Trial Court would further delay the resolution
of the main case and it would, by no means, promote respondent's right to a speedy trial
and a speedy disposition of his case.
WHEREFORE, the petition is GRANTED. The Resolution of the Sandiganbayan dated
June 2, 2005, granting respondent's Motion to Quash, is hereby REVERSED and SET
ASIDE. The Sandiganbayan is forthwith DIRECTED to proceed with deliberate dispatch
in the disposition of Criminal Case No. 28090.
SO ORDERED.

G.R. No. 134307 December 21, 1998


EDUARDO M. COJUANGCO, JR., petitioner,
vs.
SANDIGANBAYAN and PEOPLE OF THE PHILIPPINES, respondent.

QUISUMBING, J.:
This petition for prohibition under Section 2 of Rule 65 of the Rules of Court seeks to
dismiss Criminal Case No. 22018 entitled "People of the Philippines vs. Eduardo M.
Cojuangco, Jr., et al." now pending before respondent Sandiganbayan (First Division),
and to prohibit said court from further proceeding with the case. Petitioner invokes his
constitutional right to due process, a speedy trial, and a speedy determination of his
cases before all judicial, quasi-judicial and administrative bodies. Further, he prays for
the issuance of a Temporary Restraining Order and/or Writ of Preliminary Injunction
enjoining respondent Sandiganbayan (First Division) from further enforcing and or
implementing its order dated February 20, 1995 which bans petitioner from leaving the
country except upon prior approval by said court. 1
Criminal Case No. 22018 is an offshoot of a complaint filed on January 12, 1990, by the
Office of the Solicitor General before the Presidential Commission on Good
Government (PCGG), docketed as I.S. No. 74, against the former Administrator of the
Philippine Coconut Authority (PCA) and the former members of the PCA Governing
Board, petitioner among them, for violation of Republic Act No. 3019, the Anti-Graft and
Corrupt Practices Act, as amended. In said complaint, the respondents were charged
"for having conspired and confederated together and taking undue advantage of their
public positions and/or using their powers; authority, influence, connections or
relationship with the former President Ferdinand E. Marcos and former First Lady,
Imelda Romualdez-Marcos without authority, granted a donation in the amount of Two
Million Pesos (P2,000,000.00) to the Philippine Coconut Producers Federation
(COCOFED), a private entity, using PCA special fund, thereby giving COCOFED
unwarranted benefits, advantage and preference through manifest partiality, evident
bad faith and gross inexcusable negligence to the grave (sic) and prejudice of the
Filipino people and to the Republic of the Philippines. 2
Subsequently, however, this Court ruled that all proceedings in the preliminary
investigation conducted by the PCGG were null and void and the PCGG was directed to
transmit the complaints and records of the case to the Office of the Ombudsman for
appropriate action. 3
In a Resolution dated June 2, 1992, the panel of investigators recommended the filling
of an Information for violation of Section 3(e) of R.A. No. 3019, as amended, against
herein petitioner and five other respondent.

As set out in the Memorandum of the Office of the Special Prosecutor, subsequently,
the following relevant incidents took place:
The above Resolution dated June 2, 1992 was referred by Assistant
Ombudsman Abelardo L. Aportadera, Jr. to the Office of the Special
Prosecutor for review and if warranted, for the preparation of the criminal
information.
In a memorandum dated July l5, 1992 the Office of the Special Prosecutor
affirmed the recommendation as contained in the Resolution dated June
2, 1992.
However, on August 19, 1992 then Ombudsman Conrado M. Vasquez
ordered the panel of investigators to discuss the merits of the prejudicial
question posed by respondent Lobregat.
In a Memorandum dated November 18, 1992, the panel of investigators
found that Civil Case No. 0033 does not pose a prejudicial question which
will warrant the suspension of the filing of the criminal case.
The aforesaid Memorandum was received by Assistant Ombudsman
Abelardo L. Aportadera on December 1, 1992 who submitted his comment
thereto on December 16, 1992 to then Ombudsman Vasquez.
On December 23, 1992, then Ombudsman Vasquez ordered the panel of
investigators to go to the specifics and not the general averments on issue
of prejudicial question.
In a Memorandum dated December 1, 1993 the panel of investigators
recommended that the motion to suspend proceedings be granted.
On December 3, 1993 then Ombudsman Vasquez referred for comment to
the Office of the Special Prosecutors the Memorandum dated December
1, 1993 of the panel of investigators on the issue of the existence of
prejudicial question.
In a Memorandum dated January 16, 1995, Special Prosecution Officer
Daniel B. Jovacon, Jr. resolved that no prejudicial question exists to
warrant the suspension of the criminal proceedings which
recommendation was approved by then Ombudsman Vasquez on January
26, 1995. The Information, together with the case record of OMB-0-902806, was forwarded to the Office of the Ombudsman on February 10,
1995.
On February 16, 1995 Criminal Case No. 22018 was filed with the
Sandiganbayan and thereafter raffled to the First Division.

On February 17, 1995, an order for the arrest of petitioner was issued by
the respondent Sandiganbayan.
On February l9, 1995 petitioner filed with respondent court an Opposition
to Issuance of Warrant of Arrest with Motion For Leave To File Motion For
Reconsideration of Ombudsman Resolutions. In his Opposition, petitioner
alleged that since the only documents attached to the Information and
submitted to respondent Sandiganbayan were the Resolution dated June
2, 1992 of the panel of investigators and the Memorandum dated January
16, 1995 of the Office of the Special Prosecutor, the same were not
adequate for the determination of probable cause for the issuance of a
warrant of arrest by respondent Sandiganbayan. Hence, petitioner claims
the respondent Sandiganbayan should recall the warrant of arrest already
issued or desist from issuing a warrant of arrest. Petitioner, avers,
furthermore that the filing of the information was premature considering
that he was not furnished a copy of the Ombudsman's Resolution in
violation of Section 27 of R.A No. 6770 and prays that he be given leave
to file a motion for reconsideration of the Ombudsman's Resolution dated
June 2, 1992 and the Office of the Special Prosecutor's Memorandum
dated January 16, 1995.
On February 22, 1995, petitioner posted bail. On the same day he likewise
filed, through counsel, a Manifestation stating that he was posting bail
without prejudice to the Opposition To Issuance of Warrant of Arrest with
Motion For Leave To File a Motion For Reconsideration of the
Ombudsman's Resolution which he filed.
In a Resolution dated February 20, 1995, the respondent Sandiganbayan
barred petitioner from leaving the country except upon approval of the
court.
In an Order dated February 22, 1995, the respondent Sandiganbayan
gave petitioner and the other accused twenty (20) days to file their
respective motions for reconsideration of the Ombudsman's Resolution
with the Office of the Ombudsman. PCGG was likewise given a similar
period within which to file its comments to the motions for reconsideration.
Furthermore, the respondent Sandiganbayan ordered petitioner to
supplement or amplify his existing motion on the issue of the propriety of
the issuance of an Order of Arrest based merely on the resolution of the
Ombudsman in support of the filing of the Information, among others.
On March 9, 1995, petitioner filed a Memorandum in Amplification of
Oppositon To Issuance of Warrant of Arrest.

In a Resolution dated March 14, 1995, petitioner was granted additional


fifteen (15) days or until March 29, 1995, within which to file his motion for
reconsideration with the Office of the Ombudsman.
Petitioner filed his motion for reconsideration on March 28, 1995.
In a Resolution dated, April 3, 1995, the respondent Sandiganbayan
denied petitioner's motion seeking the recall of the issuance of the warrant
for his arrest.
On April 7, 1995, petitioner filed a motion for reconsideration of the
Resolution dated April 3, 1995 of the respondent Sandiganbayan.
On May 25, 1995, petitioner was conditionally arraigned pleading not
guilty to the Information. The arraignment was undertaken solely to
accommodate the petitioner in his request to travel pending the
determination of probable cause against him at the reinvestigation stage.
The conditional arraignment is subject to the condition that if petitioner is
exonerated at the preliminary investigation, the arraignment is set aside.
On the other hand, should there be cause against the petitioner either as
already charged or a separate charge which might be related to the case
pending, the arraignment will not serve as basis for the invocation of the
right against double jeopardy.
In the meantime, in a Memorandum dated October 22, 1995, Special
Prosecution Officer Victorio U. Tabanguil found no probable cause to
warrant the filing against petitioner and the other accused in Criminal
Case No. 22018 and recommended the dismissal of the case. The
recommendation for dismissal was approved by the Honorable
Ombudsman on November 15, 1996.
On December 6, 1996, Special Prosecutors Officer Victorio U. Tabanguil
filed a Manifestation attaching a copy of the Memorandum dated October
22, 1995 with the respondent Sandiganbayan for its consideration.
On December 13, 1996 petitioner filed an Urgent Motion To Dismiss
alleging that with the reversal of the earlier findings of the Ombudsman of
probable cause, there was therefore nothing on record before the
respondent Sandiganbayan which would warrant the issuance of a
warrant of arrest and the assumption of jurisdiction over the instant case.
On December 23, 1996 the Office of the Solicitor General, in
representation of the OCGG, filed with the Office of the Special Prosecutor
a motion for reconsideration of the Memorandum dated October 22, 1996
recommending the dismissal of the case against petitioner and the other
accused in Criminal Case No. 22018.

In an Order dated January 6, 1997, Special Prosecution Officer Victorio U.


Tabanguil merely noted the motion for reconsideration dated December
23, 1996 oft he Office of the Solicitor General.
On January 13, 1997, petitioner filed a Motion To Strike Out Alternatively,
Opposition To Complaint's Motion For Reconsideration dated December
23, 1996 alleging that the motion was filed out of time.
In an Order dated January 9, 1997, the respondent Sandiganbayan
ordered the prosecution to justify the relationship that may be established
with respect to the COCOFED on one hand and the Philippine Coconut
Authority on the other, as a basis for justifying the position of the
prosecution in the case. Furthermore, upon information provided by
Prosecutors Tabanguil that the Office of the Solicitor General has sought a
reconsideration on the desire of the prosecution to withdraw the
information, the Office of the Solicitor General was given fifteen (15) days
to submit its comment to the Motion to Withdraw Information. The
petitioner and the other accused were given the same period to reply to
the comment if they so desire. After which the matter will be deemed
submitted for resolution.
On January 17, 1997, the prosecution filed its compliance to the Order
dated January 9, 1997. On the other hand, the Office of the Solicitor
General filed its comment on January 24, 1997.
In an Order dated February 4, 1997, the respondent Sandiganbayan
ordered the PCGG lawyers to "present themeselves before the
respondent court and respond to the claim of the OSG that the exhibits
necessary are with the PCGG so that the Republic might effectively
substantiate its position that probable cause exists. Furthermore, it is as
much the function of the court to determine the existence of probable
cause and the propriety of the withdrawal of the Information to be assured
that the evidence for the complainant has been properly presented or the
accused is properly protected at preliminary investigation.
In an Order dated February 17, 1997, the respondent Sandiganbayan,
with the agreement of the parties, gave the Office of the Solicitor General
ten (10) days within which to submit some form of cataloging and
explanation of the documents on record to the prosecution. On the other
hand, the prosecution was given fifteen (15) days from receipt of the
submission within which to review the matter once more and to respond
thereat.
On June 13, 1997, the PCGG filed its Entry of Appearance dated June 3,
1997.

On June 19, 1997, petitioner filed a Second Motion To Resolve the Urgent
Motion To Dismiss dated December 12, 1996.
On July 3, 1997, petitioner filed a Motion to Strike Out (Re PCGG's Entry
of Appearane) dated June 30, 1997.
On July 16, 1997, the PCGG filed a Opposition to the Motion to Strike Out
(Re: PCGG's Entry of Appearance).
On July 18, 1997, petitioner filed a Reply to the Oppositions to Strike Out.
On July 31, 1997, the PCGG filed a Rejoinder to the Reply of petitioner.
On July 23, 1998, petitioner filed a Third Motion To Resolve the Urgent
Motion To Dismiss dated December 12, 1996.
In an Order dated January 26, 1998, respondent Sandiganbayan duly
noted petitioner's Motion to Dismiss. 4
Hence, the present petition.
On July 22, 1998, the Court issued a resolution requiring respondents to file their
respective comments to the petition. 5
On August 5, 1998, petitioner filed a motion reiterating his application for temporary
restraining order and/or writ of prelimary injunction with urgent motion for hearing
thereon 6 citing the urgency of lifting the travel restriction on him in view of the various
problems involving the investments of San Miguel Corporation (SMC) abroad which
must be immediately attended to by petitioner as duly elected Chairman and Chief
Executive Officer of SMC. Petitioner asserts that quite often, it becomes necessary for
him to attend meetings and conferences abroad where attendance must be confirmed
promptly. Considering that he must first secure the permission of respondent
Sandiganbayan before he can travel abroad and abide by the conditions imposed by
said court upon the grant of such permission, petitioner contends that it becomes
impossible for him to immediately attend to the aforecited tasks.
On September 2, 1998, the Court noted the respective comments to the petition filed by
the Office of the Special Prosecutor and the Solicitor General and required petitioner to
file a consolidated reply within ten (10) days from notice. 7
On September 3, 1998, petitioner filed a Second Motion Reiterating Application for
Temporary Restraining Order and/or Writ of Preliminary Injunction with Urgent Motion
for Hearing, 8 arguing among others that the continued maintenance of the holddeparture order against him has deleterious consequence not only on him personally
but also on San Miguel Corporation, a publicly listed stock company, of which he is now
Chairman and Executive Officer. 9

On September 7, 1998, the Court resolved to defer action on the aforementioned


second motion reiterating the application for the issuance of a temporary restraining
order and/or a writ of preliminary injunction until the filing of petitioner's Consolidated
Reply and required the Sandiganbayan to file its own Comment on the petition in view
of the Comment filed by the Office of the Special Prosecutor divergent from the position
taken by respondent Sandiganbayan. 10
On September 10, 1998, petitioner filed a Consolidated Reply 11 and prayed that his
Second Application for a Tempory Restraining Order and/or Writ of Preliminary
Injunction with Urgent Motion for hearing dated September 2, 1998 be now acted upon.
On September 17, 1998, respondent Sandiganbayan filed a motion for extension of time
to file its Comment to the petition. Subsequently, petitioner filed his Third Motion
Reiterating Application for Temporary Restraining Order and/or Writ of Preliminary
Injunction with Urgent Motion for Hearing 12 in view of the urgency of lifting the ban on
foreign travel imposed on him by respondent Sandiganbayan.
After respondent Sandiganbayan filed its comment on October 5, 1998, the Court in its
Resolution dated October 7, 1998, noted the aforesaid comment and resolved to set the
case for oral argument on October 21, 1998. 13
During the oral argument, the Court suggested that the parties take up in their
arguments the following issues:
(1) whether the warrant of arrest issued by respondent
Sandiganbayan is null and void, or should now be lifted if
initially valid;
(2) whether petitioner's basic rights to due process, speedy
trial and speedy disposition of the case have been violated
as to warrant dismissal of Criminal Case No. 22018; and
(3) whether the ban on foreign travel imposed on petitioner
per Order of February 20, 1995 should be vacated to enable
petitioner to go abroad without prior permission of, and other
restrictions imposed by the respondent Sandiganbayan. 14
After hearing the arguments of the parties, the Court resolved to require them to submit
their respective memoranda on the related issues taken up on the hearing including the
merits of the case within twenty (20) days. The motion of counsel for petitioner that the
issue of lifting the ban on foreign travel imposed on petitioner be resolved first, was held
under advisement. 15
On November 6, 1998, petitioner filed another Motion to Resolve Petitioner's "Motion for
Issuance of a Temporary Restraining Order or Writ of Preliminary Injunction" Enjoining
Enforcement of Respondent Sandiganbayan's Order dated February 20, 1995 (Hold

Departure Order) with an alternative prayer to travel abroad within a period of six (6)
months. 16
In its Resolution dated November 9, 1998, the Court noted the aforesaid motion and
directed petitioner that in the meanwhile, he may address his request for permission to
travel abroad to the Sandiganbayan. 17
On November 12, 1998, petitioner filed a Motion for Reconsideration of the Court's
resolution dated November 9, 1998 and argued that:
xxx xxx xxx
(6) While the petitioner may indeed obtain some relief by addressing his
"prayer for permission to travel abroad to the Sandiganbayan" to a large
extent, this defeats the purpose of the petition because petitioner has
precisely come to the Supreme Court to obtain relief from an oppressive
regime of authorization to travel abroad that the Order of the
Sandiganbayan of February 20, 1995 (Annex 'E', Petition) has imposed.
Significantly, not any of the respondents have opposed petitioner's
application for the issuance of temporary restraining order, and/or writ of
preliminary injunction or for permission to travel abroad. 18
On November 20, 1998, petitioner filed a Manifestation 19 in support of his motion for
reconsideration, setting forth the urgency of lifting the ban on foreign travel imposed on
him in view of the need to oversee the critical stages in the international operations of
SMC as its Chairman and Chief Executive Officer.
On November 20, 1998, the Office of the Solicitor General filed a Manifestation
indicating that it is not interposing any objection to petitioner's prayer that he be allowed
to travel abroad.
With the submission of the parties' respective memoranda, the Court now proceeds to
resolve the petition.
As postulated during the oral argument, three main issues confront us in this petition, to
wit:
(1) whether the warrant of arrest issued by respondent
Sandiganbayan is null and void, or should now be lifted if
initially valid;
(2) whether petitioner's basic rights to due process, speedy
trial and speedy disposition of the case have been violated
as to warrant dismissal of Criminal Case No. 22018; and

(3) whether the ban on foreign travel imposed on petitioner


per Order of February 20, 1995 should be vacated to enable
petitioner to go abroad without prior permission of and other
restrictions imposed by the respondent Sandiganbayan. 20
On the first issue, petitioner and the Office of the Special Prosecutor both argue that the
warrant of arrest issued by respondent Sandiganbayan is null and void for lack of
sufficient basis upon which it could have "personally" determined the existence of
probable cause to issue the warrant of arrest against him. They contend that there was
a violation of Section 2, Article III of the Constitution because the Information in Criminal
Case No. 22018 was accompanied only by the Resolution dated June 2, 1992 of the
Panel of Graft Investigators of the Office of the Ombudsman recommending the filing of
the information and the Memorandum dated January 16, 1995 of the Office of the
Special Prosecutor denying the existence of a prejudicial question which will warrant the
suspension of the filing of the criminal case. Their argument is principally anchored on
the pronouncements made in the case of Ho vs.
People 21 that reliance on the prosecutor's report alone is not sufficient in determining
whether there is probable cause for the issuance of a warrant of arrest. Consequent to
the nullity of the warrant of arrest, petitioner further argues that the Sandiganbayan has
not acquired jurisdiction over him and is without power to exercise the same.
However, the Office of the Special Prosecutor and the Office of the Solicitor General
maintain that any infirmity that may have attended the issuance of the warrant of arrest
was cured by petitioner's voluntary submission to the jurisdiction of the respondent
Sandiganbayan when petitioner posted bail and subsequently invoked the jurisdiction of
the Sandiganbayan by filing numerous motions wherein he sought affirmative reliefs.
Now, pertinent to the issue at hand is the second clause of Section 2, Article III of the
1987 Constitution, which provides that:
Sec. 2. . . . no search warrant or warrant of arrest shall issue except upon
a probable cause to be determined personally by the judge after
examination under oath or affirmation of the complainant and the
witnesses he may produce, and particularly describing the place to be
searched and the persons or things to be seized. (Emphasis supplied)
In Ho vs. People, 22 the Court had the opportunity to elucidate on the matter of
determining of probable cause to merit the issuance of a warrant of arrest:
First, . . . the determination of probable cause by the prosecutor is for a
purpose different from that which is to be made by the judge. Whether
there is reasonable ground to believe that the accused is guilty of the
offense charged and should be held for trial is what the prosecutor passes
upon. The judge, on the other hand, determines whether a warrant of
arrest should be issued against the accused, i.e., whether there is a
necessity for placing him under immediate custody in order not to frustrate

the ends of justice. Thus, even if both should base their findings on one
and the same proceeding or evidence, there should be no confusion as to
their distinct objectives.
Second, since their objectives are different, the judge cannot rely solely on
the report of the prosecutor in finding probable cause to justify the
issuance of a warrant of arrest. Obviously and understandably, the
contents of the prosecutor's report will support his own conclusion that
there is reason to charge the accused of an offense and hold him for trial.
However, the judge must decide independently. Hence, he must have
supporting evidence, other than the prosecutor's bare report, upon which
to legally sustain his own findings on the existence (or nonexistence) of a
probable cause to issue an arrest order. This responsibility of determining
personally and independently the existence or nonexistence of probable
cause is lodged in him by no less than the most basic law of the land.
Parenthetically, the prosecutor could ease the burden of the judge and
speed up the litigation process by forwarding to the latter not only the
information and his bare resolution finding probable cause, but also so
much of the records and the evidence on hand as to enable His Honor to
make his personal and separate judicial finding on whether to issue a
warrant of arrest.
Lastly, it is not required that the complete or entire records of the case
during the preliminary investigation be submitted to and examined by the
judge. We do not intend to unduly burden trial courts by obliging them to
examine the complete records of every case all the time simply for the
purpose of ordering the arrest of an accused. What is required, rather, is
that the judge must have sufficient supporting documents (such as the
complaint, affidavits, counter-affidavits, sworn statements of witnesses or
transcripts of stenographic notes, if any) upon which to make his
independent judgment or, at the very least, upon which to verify the
findings of the prosecutor as to the existence of probable cause. The point
is: he cannot rely solely and entirely on the prosecutor's recommendation,
as Respondent Court did in this case. Although the prosecutor enjoys the
legal presumption of regularity in the performance of his official duties and
functions, which in turn gives his report the presumption of accuracy, the
Constitution, we repeat, commands the judge topersonally determine
probable cause in the issuance of warrants of arrest. This Court has
consistently held that a judge fails in his bounden duty if he relies merely
on the certification or the report of the investigating officer. 23
As alleged by petitioner, in the case at bar, the Sandiganbayan had two pieces of
documents to consider when it resolved to issue the warrant of arrest against the
accused: (1) the Resolution dated June 2, 1992 of the Panel of Investigators of the
Office of the Ombudsman recommending the filing of the Information and (2) the
Memorandum dated June 16, 1995 of the Office of the Special Prosecutor denying the

existence of a prejudicial question which will warrant the suspension of the criminal
case. The Sandiganbayan had nothing more to support its resolution.
In Roberts vs. Court of Appeals, 24 we struck down as invalid an order for the issuance
of a warrant of arrest which were based only on "the information, amended information
and Joint Resolution", without the benefit of the records or evidence supporting the
prosecutor's finding of probable cause. And in Ho vs. People, 25 we declared that
respondent "palpably committed grave abuse of discretion in ipso facto issuing the
challenged warrant of arrest on the sole basis of the prosecutor's findings and
recommendation, and without determining on its own the issue of probable cause based
on evidence other than such bare findings and recommendation. 26
Similarly, we are now constrained to rule that herein respondent court failed to abide by
the constitutional mandate of personally determining the existence of probable cause
before issuing a warrant of arrest. For the two cited documents were the product of
somebody else's determination, insufficient to support a finding of probable cause by
the Sandiganbayan. Hence, the warrant of arrest issued by respondent court on
February 17, 1995 against herein petitioner is palpably invalid.
Consequent to the nullity of the warrant of arrest, the crucial issue now posed is
whether or not respondent Sandiganbayan could still exercise jurisdiction over the
petitioner and proceed with the trial of the case.
As already adverted to, the Office of the Special Prosecutor and the Office of the
Solicitor General are in agreement, that whatever infirmity might have attended the
issuance of the warrant of arrest against petitioner, it was cured by petitioner's
subsequent act of voluntarily submitting to respondent court's jurisdiction by posting his
bail and filing the following pleadings which sought affirmative relief, to writ: (1)
Opposition to Issuance of Warrant of Arrest with Motion for Leave to File Motion for
Reconsideration; (2) Motion for extension of time to file Motion for Reconsideration. (3)
seven Motions to Travel Abroad and two Motions for Extension of time to stay
abroad. 27Hence, they contend that respondent court's jurisdiction over petitioner has
remained in effect.
Petitioner objects to this contention, and asserts that "since the warrant of arrest issued
by respondent Sandiganbayan is null and void, it never acquired jurisdiction over the
person of the petitioner; as a consequence, it never acquired jurisdiction to take
cognizance of the offense charged and to issue any order adverse to the rights of
petitioner, including an Order restricting his right to travel. 28 According to petitioner, the
submission of both the Office of the Special Prosecutor and the Office of the Solicitor
General is not only absurd but also oppressive and offensive to the Bill Rights since it
would mean that to preserve his right against the issuance of a warrant of arrest without
probable cause determined in accordance with Sec. 2, Article III of the Constitution,
petitioner should have allowed himself to be incarcerated or imprisoned from the time
the warrant of arrest was issued on February 20, 1995 up to the present, or for more

than three (3) years now, and continue to be imprisoned until the Supreme Court
decides to declare the arrest void.29
On this score, the rule is well-settled that the giving or posting of bail by the accused is
tantamount to submission of his person to the jurisdiction of the court. 30 Thus, it has
been held that:
When a defendant in a criminal case is brought before a competent court
by virtue of a warrant of arrest or otherwise, in order to avoid the
submission of his body to the jurisdiction of the court he must raise the
question of the court's jurisdiction over his person at the very earliest
opportunity. If he gives bail, demurs to the complaint or files any dilatory
plea or pleads to the merits, he thereby gives the court jurisdiction over his
person. (State ex rel. John Brown vs. Fitzgerald, 51 Minn., 534)
xxx xxx xxx
Conceding again that the warrant issued in this case was void for the
reason that no probable cause was found by the court before issuing it,
the defendant waived all his rights to object to the same by appearing and
giving bond. 31
By posting bail, herein petitioner cannot claim exemption effect of being subject to the
jurisdiction of respondent court. While petitioner has exerted efforts to continue
disputing the validity of the issuance of the warrant of arrest despite his posting bail, his
claim has been negated when he himself invoked the jurisdiction of respondent court
through the filing of various motions that sought other affirmative reliefs.
As ruled in La Naval Drug vs. CA 32.
[L]ack of jurisdiction over the person of the defendant may be waived
either expressly or impliedly. When a defendant voluntarily appears, he is
deemed to have submitted himself to the jurisdiction of the court. If he so
wishes not to waive this defense, he must do so seasonably by motion for
the purpose of objecting to the jurisdiction of the court, otherwise, he shall
be deemed to have submitted himself to that jurisdiction.
Moreover, "[w]here the appearance is by motion for the purpose of objecting to the
jurisdiction of the court over the person, it must be for the sole and separate purpose of
objecting to said jurisdiction. If the appearance is for any other purpose, the defendant
is deemed to have submitted himself to the jurisdiction of the court. Such an
appearance gives the court jurisdiction over the person. 33
Verily, petitioner's participation in the proceedings before the Sandiganbayan was not
confined to his opposition to the issuance of a warrant of arrest but also covered other
matters which called for respondent court's exercise of its jurisdiction. Petitioner may not

be heard now to deny said court's jurisdiction over him. Nor can we ignore the long line
of precedents declaring that where the accused had posted bail, as required, to obtain
his provisional liberty, "it becomes futile to assail the validity of the issuance of the
warrants of arrest. 34
As to petitioner's contention that he should have just allowed himself to stay in jail
pending the resolution of his opposition to the issuance of the warrant of arrest against
him, if only to avoid waiving his right to question the jurisdiction of respondent court, the
Office of the Special Prosecutor has pointed out that petitioner is not without a remedy.
Petitioner could have filed a petition for certiorari and prohibition with prayer for the
issuance of a temporary restraining order, rather than actively participate in the
proceedings before the Sandiganbayan. And as exemplified by the case of Allado vs.
Diokno, 35 this remedy has already proved to be effective.
Against the continued exercise of jurisdiction by respondent Sandiganbayan in Criminal
Case No. 22018, petitioner also invokes the Memorandum of the Office of the Special
Prosecutor dated October 22, 1995 recommending the dismissal of the case against
him due to the absence of probable cause, which was later on approved by the
Ombudsman on November 15, 1996. Citing the case of Torralba vs.
Sandiganbayan, 36 petitioner argues that this Memorandum is an integral part of the
preliminary investigation and should take precedence notwithstanding the fact that the
same was made after the filing of the Information before the Sandiganbayan, for to deny
any efficacy to the finding of the Office of the Special Prosecutor would negate the right
of the petitioner to a preliminary investigation.
The well-entrenched rule however, as laid down by the case of Crespo vs. Mogul 37 is
that:
. . . once a complaint or information is filed in Court any disposition of the
case as its dismissal or the conviction or acquittal of the accused rests in
the sound discretion of the Court. Although the fiscal retains the direction
and control of the prosecution of criminal cases even while the case is
already in Court he cannot impose his opinion on the trial court. The Court
is the best and sole judge on what to do with the case before it. The
determination of the case is within its exclusive jurisdiction and
competence. A motion to dismiss the case filed by the fiscal should be
addressed to the Court who has the option to grant or deny the same. It
does nor matter if this is done before or after the arraignment of the
accused or that the motion was filed after a reinvestigation or upon
instructions of the Secretary of Justice who reviewed the records of the
investigation.
Nevertheless, petitioner claims exception to this rule by making this distinction:
b. The preliminary investigation in Crespo vs. Mogul, supra, was
conducted by the Office of the Provincial Fiscal and, following established

procedure with respect to such preliminary investigations, the preliminary


investigation conducted by the fiscal, in the language of Crespo, is
"terminated upon the filing of the information in the proper court" (at p.
470). On the other hand, the instant case involves a preliminary
investigation conducted by the Office of the Special Prosecutor pursuant
to Sec. 11[4](a), and under Sec. 27 of R.A. No. 6770. In preliminary
investigations conducted by the Office of the Special Prosecutor, the
respondent has the right to file a motion for reconsideration of any
resolution within five (5) days from receipt of written notice, and pursuant
to Sec. 7, Rule II of Administrative Order No. 7 (Rules of Procedure of the
Ombudsman), the respondent has the right to file a motion for
reconsideration within fifteen (15) days from notice of the Resolution of the
Ombudsman. Until the motion for reconsideration is resolved, preliminary
investigation is not terminated notwithstanding filing of information in court.
In the instant case, no copy of the Resolution of the Office of the Special
Prosecutor which brought about the filing of the Information, was served
on the petitioner; consequently, when the Information was filed, the
preliminary investigation had not yet been terminated. It follows that the
Resolution of the Office of the Special Prosecutor (approved by the
Ombudsman) resolving in petitioner's favor the "Motion for
Reconsideration" he had filed, now finding no probable cause, was an
integral part of the preliminary investigation, not subject to review by the
Sandiganbayan (see Torralba vs. Sandiganbayan, 230 SCRA 33 [1994]. 38
Petitioner's reliance on Torralba vs. Sandiganbayan is not, in our view, persuasive. In
that case the petitioners were not given any chance at all to seek reconsideration from
the Ombudsman's final resolution because they were not furnished with a copy of the
final resolution of the Ombudsman that could have enabled them to file a motion for
reconsideration. As a result, the Court declared that "petitioners were not only
effectively denied the opportunity to file a motion for reconsideration of the
Ombudsman's final resolution but also deprived of their right to a full preliminary
investigation preparatory to the filing of the information against them. 39
In the case at bar, however, notwithstanding the filing of the Information before the
Sandiganbayan, petitioner was able to file a motion for reconsideration of the
Ombudsman's Resolution with leave of court, and in fact his two motions for extensions
to file the same were granted by the respondent court. 40 This eventually paved the way
for the filing of subsequent Memorandum of the Office of the Special Prosecutor, which
was later on approved by the Ombudsman, recommending the dismissal of the case
against him. However, since the Information has already been filed before the
Sandigabayan, the resolution of the aforesaid recommendation now lies within the
jurisdiction and discretion of respondent court. Parenthetically, in the Torralba case, we
did not altogether deprive the Sandiganbayan of its jurisdiction to proceed with the case,
despite the defect in the conduct of the preliminary investigation, since we declared that:

The incomplete preliminary investigation in this case, however, does not


warrant the quashal of the information, nor should it obliterate the
proceedings already had. Neither is the court's jurisdiction nor validity of
an information adversely affected by deficiencies in the preliminary
investigation. Instead,the Sandiganbayan is to hold in abeyance any
further proceedings therein and to remand the case to the Office of the
Ombudsman for the completion of the preliminary investigation, the
outcome of which shall then be indorsed to Sandiganbayan for its
appropriate action. 41 (Emphasis supplied)
Clearly, consistent with the rule in Crespo vs. Mogul, after the filing of the information in
court, "any disposition of the case as to its dismissal or the conviction or acquittal of the
accused rests in the sound discretion of the
Court. 42
Proceeding now to the second issue, petitioner maintains that the long delay that
characterized the proceedings in Criminal Case No. 22018 before respondent
Sandiganbayan has resulted in the violation of his Constitutional right to a speedy trial
and a speedy determination of his case. Thus, petitioner submits that:
409. It has been more than three (3) years since the Information in
Criminal Case No. 22018 was filed with respondent Sandiganbayan. More
than one and a half (1/2) years have elapsed since the Office of the
Special Prosecutor filed its Manifestation seeking the dismissal of the
case. Based on the Office of the Special Prosecutor's finding of the
absence of probable cause, petitioner filed on December 13, 1996, an
"Urgent Motion To Dismiss". Three times, on March 24, 1997, June 18,
1997 and January 23, 1998, petitioner has sought resolution of his "Urgent
Motion To Dismiss." These notwithstanding, the dismissal of the
information as to petitioner remains pending and petitioner continues to be
under criminal indictment constrained to suffer without justification in
law and the Constitution, the humiliation, the restraints to liberty and the
tormenting anxieties of an accused. 43
Respondents concede that there has indeed been some delay but deny that it
amounted to a violation of petitioner's right of speedy disposition of his case. They cite
as justification the reorganization of the Sandiganbayan on September 23, 1997
wherein it was reconstituted into five (5) Divisions; 44 (2) the filing of motions by
petitioner seeking affirmative reliefs from the Sandiganbayan; (3) the failure of petitioner
himself to invoke his right to speedy resolution of his pending motions prior to the filing
of this petition; 45 (4) the heavy caseload of respondent court. 46
The right to a speedy disposition of a case, like the right to speedy trial, is deemed
violated only when the proceeding is attended by vexatious, capricious, and oppressive
delays. 47 It should be emphasized that the factors that must be taken into account in
determining whether this constitutional rights has been violated are as follows: (1) the

length of delay, (2) the reason for such delay and (3) the assertion or failure to assert
such right by the accused, and the prejudice caused by the delay. 48
As in previous occasions, the Court takes judicial cognizance of the fact that structural
reorganizations 49 and the ever increasing case load of courts have adversely affected
the speedy disposition of the cases pending before them.
In the instant case, however, the Court finds that delay concerns the resolution of
petitioner's "Urgent Motion to Dismiss", which is an offshoot of the Memorandum of the
Office of the Special Prosecutor recommending the dismissal of the case. Such delay is
now far from excusable. Petitioner's Motion to Dismiss has been filed as early as
December 13, 1996 and, on three occasions, petitioner has moved for the urgent
resolution of this motion. 50What further militates against further delay in resolving this
case is the fact that the government prosecutors themeselves concede that this case is
of paramount importance, involving as it does "the recovery of the ill-gotten wealth or
government funds, unlawfully used or misused by persons close or percieved to be
close to the Marcoses. 51 Respondent court declared in its Order dated February 17,
1997 that the matter would be deemed submitted for resolution upon compliance with
the Office of the Special Prosecutor as to whether there is indeed no probable cause
against petitioner, 52 which compliance was submitted by the Office of the Special
Prosecutor on March 17, 1997. 53 Under these circumstances, the Court does find the
period of more than one year that elapsed for resolving petitioner's motion to dismiss
quite long, considering that all pertinent pleadings required by the Sandiganbayan were
already submitted.
Even if petitioner himself might have contributed to said delay, as contended by
respondent, in our view it is best that the case be resolved on the merits by the
Sandiganbayan with due regard to petitioner's right to due process, speedy trial and
speedy disposition of the case against him and his co-accused.
Finally, with respect to the issue of whether or not the ban on foreign travel should be
continued, as imposed on petitioner by respondent Sandiganbayan per its Order dated
February 20, 1995 with accompanying restrictions in effect, we resolve to rule in the
negative. The travel ban should be lifted, considering all the circumstances now
prevailing.
The rule laid down by this Court is that a person facing a criminal indictment and
provisionally released on bail does not have an unrestricted right to travel, the reason
being that a person's right to travel is subject to the usual constraints imposed by the
very necessity of safeguarding the system of
justice. 54 But, significantly, the Office of the Solicitor General in its Manifestation dated
November 20, 1998 indicated that it is not interposing any objection to petitioner's
prayer that he be allowed to travel abroad based on the following considerations:
. . . (1) that it is well within the power of this Court to supend its own rules,
including the second paragraph, Section 23, Rule 114 of the Rules of

Court; (2) that it has been shown in the past that the petitioner has always
returned to the Philippines after the expiration of the period of his allowed
travel; and (3) that petitioner, now Chairman of the Board of San Miguel
Corporation, may be constrained to leave the country for business
purposes, more often than he had done in the past, . . . . 55
It however recommended that the period of travel should be reduced to three (3)
months instead of six (6) months as requested by petitioner and that the latter should be
required to post an additional cash bond equivalent to the present cash bond posted by
him. 56
Moreover, prescinding from our initial declaration that the issuance of warrant of arrest
against petitioner by respondent court is invalid, it now becomes necessary that there
be strong and compelling reasons to justify the continued restriction on petitioner's right
to travel abroad. Admittedly, all of petitioner's previous requests to travel abroad has
been granted and that, as confirmed by the Office of the Solicitor General, that
petitioner has always returned to the Philippines and complied with the restrictions
imposed on him. The necessity of further denying petitioner's right to travel abroad, with
attendant restrictions, appears less than clear. The risk of flights is further diminished in
view of petitioner's recent reinstatement as Chairman and Chief Executive Officer of
San Miguel Corporation, though he has now more justification to travel so as to oversee
the entire operations of that company. In this regard, it has to be conceded that this
assumption of such vital post has come at a time when the current economic crisis has
adversely affected the international operations of many companies, including San
Miguel. The need to travel abroad frequently on the part of petitioner, to formulate and
implement the necessary corporate strategies and decisions, could not be forestalled.
These considerations affecting the petitioner's duties to a publicly held company,
militate against imposing further restrictions on petitioner's right to travel abroad.
WHEREFORE, the Court hereby resolves to DISMISS the petition insofar as the
dismissal of Criminal Case No. 22018 against the petitioner is concerned. Respondent
Sandiganbayan (First Division) is hereby ordered to proceed with the resolution of the
pending motions and incidents in Criminal Case No. 22018 with utmost dispatch.
Meanwhile, the Resolution of the Sandiganbayan (First Division), dated February 20,
1995, imposing a ban on petitioner's travel abroad without its prior approval pending the
resolution of Criminal Case No. 22018 is, for the reasons heretofore advanced, hereby
LIFTED for a period of three (3) months counted from the finality of this decision. Any
similar request during the pendency of said case before the Sandiganbayan shall be
addressed to that court.
No pronouncement as to cost.
SO ORDERED.

G.R. No. 158763

March 31, 2006

JOSE C. MIRANDA, ALBERTO P. DALMACIO, and ROMEO B. OCON, Petitioners,


vs.
VIRGILIO M. TULIAO, Respondent.
DECISION
CHICO-NAZARIO, J.:
This is a petition for review on certiorari under Rule 45 of the Rules of Court, assailing
the 18 December 2002 Decision 1 of the Court of Appeals in CA-G.R. SP No. 67770
and its 12 June 2003 Resolution denying petitioners Motion for Reconsideration. The
dispositive portion of the assailed decision reads as follows:
WHEREFORE, finding public respondent Judge Anastacio D. Anghad to have acted
with grave abuse of discretion amounting to lack or excess of jurisdiction in issuing the
assailed Orders, the instant petition for certiorari, mandamus and prohibition is hereby
GRANTED and GIVEN DUE COURSE, and it is hereby ordered:
1. The assailed Joint Order dated August 17, 2001, Order dated September 21,
2001, Joint Order dated October 16, 2001 and Joint Order dated November 14,
2001 dismissing the two (2) Informations for Murder, all issued by public
respondent Judge Anastacio D. Anghad in Criminal Cases Nos. 36-3523 and 363524 are hereby REVERSED and SET ASIDE for having been issued with grave
abuse of discretion amounting to lack or excess of jurisdiction, and another
entered UPHOLDING, AFFIRMING[,] and REINSTATING the Order dated June
25, 2001 and Joint Order dated July 6, 2001 issued by the then acting Presiding
Judge Wilfredo Tumaliuan;
2. Criminal Cases Nos. 36-3523 and 36-3524 are hereby ordered REINSTATED
in the docket of active criminal cases of Branch 36 of the Regional Trial Court of
Santiago City, Isabela; and
3. Public respondent Judge Anastacio D. Anghad is DIRECTED to ISSUE
forthwith Warrants of Arrest for the apprehension of private respondents Jose
"Pempe" Miranda, SPO3 Alberto P. Dalmacio, PO3 Romeo B. Ocon and
accused Rodel T. Maderal in said Criminal Cases Nos. 36-3523 and 36-3524. 2
The factual and procedural antecedents of the case are as follows:
On 8 March 1996, two burnt cadavers were discovered in Purok Nibulan, Ramon,
Isabela, which were later identified as the dead bodies of Vicente Bauzon and Elizer
Tuliao, son of private respondent Virgilio Tuliao who is now under the witness protection
program.

Two informations for murder were filed against SPO1 Wilfredo Leao, SPO1 Ferdinand
Marzan, SPO1 Ruben B. Agustin, SPO2 Alexander Micu, SPO2 Rodel Maderal, and
SPO4 Emilio Ramirez in the Regional Trial Court (RTC) of Santiago City.
The venue was later transferred to Manila. On 22 April 1999, the RTC of Manila
convicted all of the accused and sentenced them to two counts of reclusion perpetua
except SPO2 Maderal who was yet to be arraigned at that time, being at large. The
case was appealed to this Court on automatic review where we, on 9 October 2001,
acquitted the accused therein on the ground of reasonable doubt.
Sometime in September 1999, SPO2 Maderal was arrested. On 27 April 2001, he
executed a sworn confession and identified petitioners Jose C. Miranda, PO3 Romeo B.
Ocon, and SPO3 Alberto P. Dalmacio, a certain Boyet dela Cruz and Amado Doe, as
the persons responsible for the deaths of Vicente Bauzon and Elizer Tuliao.
Respondent Tuliao filed a criminal complaint for murder against petitioners, Boyet dela
Cruz, and Amado Doe, and submitted the sworn confession of SPO2 Maderal. On 25
June 2001, Acting Presiding Judge Wilfredo Tumaliuan issued warrants of arrest
against petitioners and SPO2 Maderal.
On 29 June 2001, petitioners filed an urgent motion to complete preliminary
investigation, to reinvestigate, and to recall and/or quash the warrants of arrest.
In the hearing of the urgent motion on 6 July 2001, Judge Tumaliuan noted the absence
of petitioners and issued a Joint Order denying said urgent motion on the ground that,
since the court did not acquire jurisdiction over their persons, the motion cannot be
properly heard by the court. In the meantime, petitioners appealed the resolution of
State Prosecutor Leo T. Reyes to the Department of Justice.
On 17 August 2001, the new Presiding Judge Anastacio D. Anghad took over the case
and issued a Joint Order reversing the Joint Order of Judge Tumaliuan. Consequently,
he ordered the cancellation of the warrant of arrest issued against petitioner Miranda.
He likewise applied this Order to petitioners Ocon and Dalmacio in an Order dated 21
September 2001. State Prosecutor Leo S. Reyes and respondent Tuliao moved for the
reconsideration of the said Joint Order and prayed for the inhibition of Judge Anghad,
but the motion for reconsideration was denied in a Joint Order dated 16 October 2001
and the prayer for inhibition was denied in a Joint Order dated 22 October 2001.
On 25 October 2001, respondent Tuliao filed a petition for certiorari, mandamus and
prohibition with this Court, with prayer for a Temporary Restraining Order, seeking to
enjoin Judge Anghad from further proceeding with the case, and seeking to nullify the
Orders and Joint Orders of Judge Anghad dated 17 August 2001, 21 September 2001,
16 October 2001, and 22 October 2001.
On 12 November 2001, this Court issued a Resolution resolving to grant the prayer for a
temporary restraining order against Judge Anghad from further proceeding with the

criminal cases. Shortly after the aforesaid resolution, Judge Anghad issued a Joint
Order dated 14 November 2001 dismissing the two Informations for murder against
petitioners. On 19 November 2001, this Court took note of respondents cash bond
evidenced by O.R. No. 15924532 dated 15 November 2001, and issued the temporary
restraining order while referring the petition to the Court of Appeals for adjudication on
the merits.
Respondent Tuliao filed with this Court a Motion to Cite Public Respondent in
Contempt, alleging that Judge Anghad "deliberately and willfully committed contempt of
court when he issued on 15 November 2001 the Order dated 14 November 2001
dismissing the informations for murder." On 21 November 2001, we referred said motion
to the Court of Appeals in view of the previous referral to it of respondents petition for
certiorari, prohibition and mandamus.
On 18 December 2002, the Court of Appeals rendered the assailed decision granting
the petition and ordering the reinstatement of the criminal cases in the RTC of Santiago
City, as well as the issuance of warrants of arrest against petitioners and SPO2
Maderal. Petitioners moved for a reconsideration of this Decision, but the same was
denied in a Resolution dated 12 June 2003.
Hence, this petition.
The facts of the case being undisputed, petitioners bring forth to this Court the following
assignments of error:
FIRST ASSIGNMENT OF ERROR
With all due respect, the Honorable Court of Appeals gravely erred in reversing and
setting aside the Joint Order of Judge Anastacio D. Anghad dated August 17, 2001,
September 21, 2001, October 16, 2001 and November 14, 2001 issued in criminal
cases numbered 36-3523 and 36-3524; and, erred in upholding, affirming and
reinstating the Order dated July 6, 2001 issued by then Acting Presiding Judge Wilfredo
Tumaliuan, on the alleged rule that an accused cannot seek any judicial relief if he does
not submit his person to the jurisdiction of the court.
SECOND ASSIGNMENT OF ERROR
With all due respect, the Honorable Court of Appeals gravely erred in directing the
reinstatement of Criminal Cases No. 36-3523 and 36-3524 in the docket of Active
Criminal Cases of Branch 36 of the Regional Trial Court of Santiago City, Philippines,
and in ordering the public respondent to re-issue the warrants of arrest against herein
petitioners.
THIRD ASSIGNMENT OF ERROR

Wit all due respect, the Honorable Court of Appeals committed a reversible error in
ordering the reinstatement of Criminal Cases No. 36-3523 and No. 36-3524 in the
docket of active criminal cases of Branch 36 of the regional trial court of Santiago City,
Philippines, and in ordering the public respondent to issue warrants of arrest against
herein petitioners, the order of dismissal issued therein having become final and
executory.
Adjudication of a motion to quash a warrant of arrest requires neither jurisdiction over
the person of the accused, nor custody of law over the body of the accused.
The first assignment of error brought forth by the petitioner deals with the Court of
Appeals ruling that:
[A]n accused cannot seek any judicial relief if he does not submit his person to the
jurisdiction of the court. Jurisdiction over the person of the accused may be acquired
either through compulsory process, such as warrant of arrest, or through his voluntary
appearance, such as when he surrenders to the police or to the court. It is only when
the court has already acquired jurisdiction over his person that an accused may invoke
the processes of the court (Pete M. Pico vs. Alfonso V. Combing, Jr., A.M. No. RTJ-91764, November 6, 1992). Thus, an accused must first be placed in the custody of the
law before the court may validly act on his petition for judicial reliefs.3
Proceeding from this premise, the Court of Appeals ruled that petitioners Miranda, Ocon
and Dalmacio cannot seek any judicial relief since they were not yet arrested or
otherwise deprived of their liberty at the time they filed their "Urgent Motion to complete
preliminary investigation; to reinvestigate; to recall and/or quash warrants of arrest." 4
Petitioners counter the finding of the Court of Appeals by arguing that jurisdiction over
the person of the accused is required only in applications for bail. Furthermore,
petitioners argue, assuming that such jurisdiction over their person is required before
the court can act on their motion to quash the warrant for their arrest, such jurisdiction
over their person was already acquired by the court by their filing of the above Urgent
Motion.
In arguing that jurisdiction over the person is required only in the adjudication of
applications for bail, petitioners quote Retired Court of Appeals Justice Oscar Herrera:
Except in applications for bail, it is not necessary for the court to first acquire jurisdiction
over the person of the accused to dismiss the case or grant other relief. The outright
dismissal of the case even before the court acquires jurisdiction over the person of the
accused is authorized under Section 6(a), Rule 112 of the Revised Rules of Criminal
Procedure and the Revised Rules on Summary Procedure (Sec. 12a). In Allado vs.
Diokno (232 SCRA 192), the case was dismissed on motion of the accused for lack of
probable cause without the accused having been arrested. In Paul Roberts vs. Court of
Appeals (254 SCRA 307), the Court was ordered to hold the issuance of a warrant of
arrest in abeyance pending review by the Secretary of Justice. And in Lacson vs.

Executive Secretary (301 SCRA 1025), the Court ordered the case transferred from the
Sandiganbayan to the RTC which eventually ordered the dismissal of the case for lack
of probable cause.6
In arguing, on the other hand, that jurisdiction over their person was already acquired by
their filing of the above Urgent Motion, petitioners invoke our pronouncement, through
Justice Florenz D. Regalado, in Santiago v. Vasquez7:
The voluntary appearance of the accused, whereby the court acquires jurisdiction over
his person, is accomplished either by his pleading to the merits (such as by filing a
motion to quash or other pleadings requiring the exercise of the courts jurisdiction
thereover, appearing for arraignment, entering trial) or by filing bail. On the matter of
bail, since the same is intended to obtain the provisional liberty of the accused, as a rule
the same cannot be posted before custody of the accused has been acquired by the
judicial authorities either by his arrest or voluntary surrender.
Our pronouncement in Santiago shows a distinction between custody of the law and
jurisdiction over the person. Custody of the law is required before the court can act upon
the application for bail, but is not required for the adjudication of other reliefs sought by
the defendant where the mere application therefor constitutes a waiver of the defense of
lack of jurisdiction over the person of the accused.8 Custody of the law is accomplished
either by arrest or voluntary surrender,9 while jurisdiction over the person of the accused
is acquired upon his arrest or voluntary appearance. 10 One can be under the custody of
the law but not yet subject to the jurisdiction of the court over his person, such as when
a person arrested by virtue of a warrant files a motion before arraignment to quash the
warrant. On the other hand, one can be subject to the jurisdiction of the court over his
person, and yet not be in the custody of the law, such as when an accused escapes
custody after his trial has commenced. 11Being in the custody of the law signifies
restraint on the person, who is thereby deprived of his own will and liberty, binding him
to become obedient to the will of the law. 12 Custody of the law is literally custody over
the body of the accused. It includes, but is not limited to, detention.
The statement in Pico v. Judge Combong, Jr., 13 cited by the Court of Appeals should
not have been separated from the issue in that case, which is the application for
admission to bail of someone not yet in the custody of the law. The entire paragraph of
our pronouncement in Pico reads:
A person applying for admission to bail must be in the custody of the law or otherwise
deprived of his liberty. A person who has not submitted himself to the jurisdiction of the
court has no right to invoke the processes of that court. Respondent Judge should have
diligently ascertained the whereabouts of the applicant and that he indeed had
jurisdiction over the body of the accused before considering the application for bail. 13
While we stand by our above pronouncement in Pico insofar as it concerns bail, we
clarify that, as a general rule, one who seeks an affirmative relief is deemed to have
submitted to the jurisdiction of the court. 15 As we held in the aforecited case of

Santiago, seeking an affirmative relief in court, whether in civil or criminal proceedings,


constitutes voluntary appearance.
Pico deals with an application for bail, where there is the special requirement of the
applicant being in the custody of the law. In Feliciano v. Pasicolan, 16 we held that "[t]he
purpose of bail is to secure ones release and it would be incongruous to grant bail to
one who is free. Thus, bail is the security required and given for the release of a person
who is in the custody of law." The rationale behind this special rule on bail is that it
discourages and prevents resort to the former pernicious practice wherein the accused
could just send another in his stead to post his bail, without recognizing the jurisdiction
of the court by his personal appearance therein and compliance with the requirements
therefor. 17
There is, however, an exception to the rule that filing pleadings seeking affirmative relief
constitutes voluntary appearance, and the consequent submission of ones person to
the jurisdiction of the court. This is in the case of pleadings whose prayer is precisely for
the avoidance of the jurisdiction of the court, which only leads to a special appearance.
These pleadings are: (1) in civil cases, motions to dismiss on the ground of lack of
jurisdiction over the person of the defendant, whether or not other grounds for dismissal
are included; 18 (2) in criminal cases, motions to quash a complaint on the ground of
lack of jurisdiction over the person of the accused; and (3) motions to quash a warrant
of arrest. The first two are consequences of the fact that failure to file them would
constitute a waiver of the defense of lack of jurisdiction over the person. The third is a
consequence of the fact that it is the very legality of the court process forcing the
submission of the person of the accused that is the very issue in a motion to quash a
warrant of arrest.
To recapitulate what we have discussed so far, in criminal cases, jurisdiction over the
person of the accused is deemed waived by the accused when he files any pleading
seeking an affirmative relief, except in cases when he invokes the special jurisdiction of
the court by impugning such jurisdiction over his person. Therefore, in narrow cases
involving special appearances, an accused can invoke the processes of the court even
though there is neither jurisdiction over the person nor custody of the law. However, if a
person invoking the special jurisdiction of the court applies for bail, he must first submit
himself to the custody of the law.
In cases not involving the so-called special appearance, the general rule applies, i.e.,
the accused is deemed to have submitted himself to the jurisdiction of the court upon
seeking affirmative relief. Notwithstanding this, there is no requirement for him to be in
the custody of the law. The following cases best illustrate this point, where we granted
various reliefs to accused who were not in the custody of the law, but were deemed to
have placed their persons under the jurisdiction of the court. Note that none of these
cases involve the application for bail, nor a motion to quash an information due to lack
of jurisdiction over the person, nor a motion to quash a warrant of arrest:

1. In Allado v. Diokno, 19 on the prayer of the accused in a petition for certiorari on the
ground of lack of probable cause, we issued a temporary restraining order enjoining
PACC from enforcing the warrant of arrest and the respondent judge therein from
further proceeding with the case and, instead, to elevate the records to us.
2. In Roberts, Jr. v. Court of Appeals,20 upon the accuseds Motion to Suspend
Proceedings and to Hold in Abeyance Issuance of Warrants of Arrest on the ground that
they filed a Petition for Review with the Department of Justice, we directed respondent
judge therein to cease and desist from further proceeding with the criminal case and to
defer the issuance of warrants of arrests against the accused.
3. In Lacson v. Executive Secretary,21 on the prayer of the accused in a petition for
certiorari on the ground of lack of jurisdiction on the part of the Sandiganbayan, we
directed the Sandiganbayan to transfer the criminal cases to the Regional Trial Court
even before the issuance of the warrants of arrest.
We hold that the circumstances forcing us to require custody of the law in applications
for bail are not present in motions to quash the warrant of arrest. If we allow the granting
of bail to persons not in the custody of the law, it is foreseeable that many persons who
can afford the bail will remain at large, and could elude being held to answer for the
commission of the offense if ever he is proven guilty. On the other hand, if we allow the
quashal of warrants of arrest to persons not in the custody of the law, it would be very
rare that a person not genuinely entitled to liberty would remain scot-free. This is
because it is the same judge who issued the warrant of arrest who will decide whether
or not he followed the Constitution in his determination of probable cause, and he can
easily deny the motion to quash if he really did find probable cause after personally
examining the records of the case.
Moreover, pursuant to the presumption of regularity of official functions, the warrant
continues in force and effect until it is quashed and therefore can still be enforced on
any day and at any time of the day and night.22Furthermore, the continued absence of
the accused can be taken against him in the determination of probable cause, since
flight is indicative of guilt.
In fine, as much as it is incongruous to grant bail to one who is free, it is likewise
incongruous to require one to surrender his freedom before asserting it. Human rights
enjoy a higher preference in the hierarchy of rights than property rights,23 demanding
that due process in the deprivation of liberty must come before its taking and not after.
Quashing a warrant of arrest based on a subsequently filed petition for review with the
Secretary of Justice and based on doubts engendered by the political climate
constitutes grave abuse of discretion.
We nevertheless find grave abuse of discretion in the assailed actions of Judge
Anghad. Judge Anghad seemed a little too eager of dismissing the criminal cases
against the petitioners. First, he quashed the standing warrant of arrest issued by his

predecessor because of a subsequently filed appeal to the Secretary of Justice, and


because of his doubts on the existence of probable cause due to the political climate in
the city. Second, after the Secretary of Justice affirmed the prosecutors resolution, he
dismissed the criminal cases on the basis of a decision of this Court in another case
with different accused, doing so two days after this Court resolved to issue a temporary
restraining order against further proceeding with the case.
After Judge Tumaliuan issued warrants for the arrest of petitioners, petitioner Miranda
appealed the assistant prosecutors resolution before the Secretary of Justice. Judge
Anghad, shortly after assuming office, quashed the warrant of arrest on the basis of said
appeal. According to Judge Anghad, "x x x prudence dictates (that) and because of
comity, a deferment of the proceedings is but proper."24
Quashal on this basis is grave abuse of discretion. It is inconceivable to charge Judge
Tumaliuan as lacking in prudence and oblivious to comity when he issued the warrants
of arrest against petitioners just because the petitioners might, in the future, appeal the
assistant prosecutors resolution to the Secretary of Justice. But even if the petition for
review was filed before the issuance of the warrants of arrest, the fact remains that the
pendency of a petition for the review of the prosecutors resolution is not a ground to
quash the warrants of arrest.
In Webb v. de Leon,25 we held that the petitioners therein cannot assail as premature
the filing of the information in court against them on the ground that they still have the
right to appeal the adverse resolution of the DOJ Panel to the Secretary of Justice.
Similarly, the issuance of warrants of arrest against petitioners herein should not have
been quashed as premature on the same ground.
The other ground invoked by Judge Anghad for the quashal of the warrant of arrest is in
order if true: violation of the Constitution. Hence, Judge Anghad asked and resolved the
question:
In these double murder cases, did this Court comply or adhere to the above-quoted
constitutional proscription, which is Sec. 2, Article III Bill of Rights; to Sec. 6(a), Rule
112, Rules of Criminal Procedure and to the above-cited decisional cases? To this
query or issue, after a deep perusal of the arguments raised, this Court, through [its]
regular Presiding Judge, finds merit in the contention of herein accused-movant, Jose
"Pempe" Miranda.26
Judge Anghad is referring to the following provision of the Constitution as having been
violated by Judge Tumaliuan:
Sec. 2. The right of the people to be secure in their persons, houses, papers and effects
against unreasonable searches and seizures of whatever nature and for any purpose
shall be inviolable, and no search warrant or warrant of arrest shall issue except upon
probable cause to be determined personally by the judge after examination under oath

or affirmation of the complainant and the witnesses he may produce, and particularly
describing the place to be searched and the persons or things to be seized. 27
However, after a careful scrutiny of the records of the case, including the supporting
evidence to the resolution of the prosecutor in his determination of probable cause, we
find that Judge Anghad gravely abused his discretion.
According to petitioners:
In this case, the nullity of the order of Judge Tumaliuan, for the arrest of the petitioners
is apparent from the face of the order itself, which clearly stated that the determination
of probable cause was based on the certification, under oath, of the fiscal and not on a
separate determination personally made by the Judge. No presumption of regularity
could be drawn from the order since it expressly and clearly showed that it was based
only on the fiscals certification.28
Petitioners claim is untrue. Judge Tumaliuans Joint Order contains no such indication
that he relied solely on the prosecutors certification. The Joint Order even indicated the
contrary:
Upon receipt of the information and resolution of the prosecutor, the Court proceeded to
determine the existence of a probable cause by personally evaluating the records x x
x.[29]
The records of the case show that the prosecutors certification was accompanied by
supporting documents, following the requirement under Lim, Sr. v. Felix30 and People v.
Inting.31 The supporting documents are the following:
1. Resolution dated 21 June 2001 of State Prosecutor Leo S. Reyes;
2. Affidavit dated 22 May 2001 of Modesto Gutierrez;
3. Affidavit dated 19 May 2001 of Romeo B. Ocon;
4. Joint Counter Affidavit dated 23 May 2001 of Mayor Jose C. Miranda and
Reynaldo de la Cruz;
5. Affidavit dated 19 May 2001 of Alberto Dalmacio;
6. Decision dated 22 April 1999 of the Regional Trial Court of Manila, Branch 41
in Criminal Case No. 97-160355;
7. Sworn statement dated 27 April 2001 of Rodel Maderal;
8. Information dated 22 June 2001;

9. Affidavit-complaint of Virgilio Tuliao; and


10. Medico-legal Reports of the cadavers of Elezer Tuliao and Vicente Buazon.
Hence, procedurally, we can conclude that there was no violation on the part of Judge
Tumaliuan of Article III, Section 2, of the Constitution. Judge Anghad, however, focused
on the substantive part of said section, i.e., the existence of probable cause. In failing to
find probable cause, Judge Anghad ruled that the confession of SPO2 Maderal is
incredible for the following reasons: (1) it was given after almost two years in the
custody of the National Bureau of Investigation; (2) it was given by someone who
rendered himself untrustworthy for being a fugitive for five years; (3) it was given in
exchange for an obvious reward of discharge from the information; and (4) it was given
during the election period amidst a "politically charged scenario where "Santiago City
voters were pitted against each other along the lines of the Miranda camp on one side
and former City Mayor Amelita S. Navarro, and allegedly that of DENR Secretary
Heherson Alvarez on the other."32
We painstakingly went through the records of the case and found no reason to disturb
the findings of probable cause of Judge Tumaliuan.
It is important to note that an exhaustive debate on the credibility of a witness is not
within the province of the determination of probable cause. As we held in Webb 33:
A finding of probable cause needs only to rest on evidence showing that more likely
than not a crime has been committed and was committed by the suspects. Probable
cause need not be based on clear and convincing evidence of guilt, neither on evidence
establishing guilt beyond reasonable doubt and definitely, not on evidence establishing
absolute certainty of guilt. As well put in Brinegar v. United States, while probable cause
demands more than "bare suspicion," it requires "less than evidence which would justify
x x x conviction." A finding of probable cause merely binds over the suspect to stand
trial. It is not a pronouncement of guilt.
x x x Probable cause merely implies probability of guilt and should be determined in a
summary manner. Preliminary investigation is not a part of trial x x x.
Dismissing a criminal case on the basis of a decision of this Court in another case with
different accused constitutes grave abuse of discretion.
Judge Anghad had quashed the warrant of arrest on the ground, among other things,
that there was a petition for review of the assistant prosecutors resolution before the
Secretary of Justice. However, after the Secretary of Justice affirmed the prosecutors
resolution, Judge Anghad summarily dismissed the two criminal cases against the
petitioners on the basis of the following explanation:
Rodel Maderal was one of the accused in People vs. Wilfredo Leano, et al., RTC,
Branch 41, Manila, and based from his sworn statements, he pinpointed to Mr. Miranda

the mastermind and with him and the other police officers as the direct perpetrators,
the October 9, 2001 Decision of the Supreme Court absolving the five cops of murder,
certainly makes his sworn Statements a "narration of falsehood and lies" and that
because of the decision acquitting said officers "who were likewise falsely linked by said
Rodel Maderal in his April 27, 2001 statements, it is now beyond doubt that Rodel
Maderal made untruthful, fabricated and perjured statements and therefore the same is
without probable value." This Court agrees with the defenses views. Indeed, of what
use is Maderals statements when the Supreme Court rejected the prosecutions
evidence presented and adduced in Criminal Case No. 97-160355. Rodel Maderal is
supposed to turn state witness in these two (2) cases but with the Supreme Court
decision adverted to, the probative value of his statements is practically nil.
xxxx
This Court finds merit to the manifestation of the accused Miranda dated October 18,
2001, praying for the summary dismissal of the two (2) murder charges in view of the
latest decision of the Supreme Court in People of the Philippines vs. Wilfredo Leao, et
al., G.R. No. 13886, acquitting the accused therein and in effect disregarding all the
evidence presented by the prosecution in that case. Accordingly, the two (2)
informations [for] murder filed against Jose Miranda are ordered dismissed. 34
This is a clear case of abuse of discretion. Judge Anghad had no right to twist our
decision and interpret it to the discredit of SPO2 Maderal, who was still at large when
the evidence of the prosecution in the Leao case was presented. A decision, even of
this Court, acquitting the accused therein of a crime cannot be the basis of the dismissal
of criminal case against different accused for the same crime. The blunder of Judge
Anghad is even more pronounced by the fact that our decision in Leao was based on
reasonable doubt. We never ruled in Leao that the crime did not happen; we just found
that there was reasonable doubt as to the guilt of the accused therein, since the
prosecution in that case relied on circumstantial evidence, which interestingly is not
even the situation in the criminal cases of the petitioners in the case at bar as there is
here an eyewitness: Rodel Maderal. The accused in Leao furthermore had no motive
to kill respondent Tuliaos son, whereas petitioners herein had been implicated in the
testimony of respondent Tuliao before the Senate Blue Ribbon Committee.
It is preposterous to conclude that because of our finding of reasonable doubt in Leao,
"it is now beyond doubt that Rodel Maderal made untruthful, fabricated and perjured
statements and therefore the same is without probable value." 35 On the contrary, if we
are to permit the use of our decision in Leao, an acquittal on the ground of reasonable
doubt actually points to the probability of the prosecutions version of the facts therein.
Such probability of guilt certainly meets the criteria of probable cause.
We cannot let unnoticed, too, Judge Anghads dismissal of the informations two days
after we resolved to issue, upon the filing of a bond, a temporary restraining order
prohibiting him from further proceeding with the case. The bond was filed the day after
the informations were dismissed. While the dismissal of the case was able to beat the

effectivity date of the temporary restraining order, such abrupt dismissal of the
informations (days after this Courts resolve to issue a TRO against Judge Anghad)
creates wild suspicions about the motives of Judge Anghad.
Nullification of a proceeding necessarily carries with it the reinstatement of the orders
set aside by the nullified proceeding.
In their second assignment of error, petitioners claim that the Court of Appeals did not
recall or reinstate the warrants of arrest issued by Judge Tumaliuan, but instead
directed Judge Anghad to issue apparently new warrants of arrest. 36 According to the
petitioners, it was an error for the Court of Appeals to have done so, without a personal
determination of probable cause.
We disagree. Whether the Court of Appeals ordered the issuance of new warrants of
arrest or merely ordered the reinstatement of the warrants of arrest issued by Judge
Tumaliuan is merely a matter of scrupulous semantics, the slight inaccuracy whereof
should not be allowed to affect the dispositions on the merits, especially in this case
where the other dispositions of the Court of Appeals point to the other direction. Firstly,
the Court of Appeals had reinstated the 25 June 2001 Order of Judge
Tumaliuan,37 which issued the warrants of arrest. Secondly, the Court of Appeals
likewise declared the proceedings conducted by Judge Anghad void. Certainly, the
declaration of nullity of proceedings should be deemed to carry with it the reinstatement
of the orders set aside by the nullified proceedings. Judge Anghads order quashing the
warrants of arrest had been nullified; therefore those warrants of arrest are henceforth
deemed unquashed.
Even if, however, the Court of Appeals had directed the issuance of new warrants of
arrest based on a determination of probable cause, it would have been legally
permissible for them to do so. The records of the preliminary investigation had been
available to the Court of Appeals, and are also available to this Court, allowing both the
Court of Appeals and this Court to personally examine the records of the case and not
merely rely on the certification of the prosecutor. As we have ruled in Allado v. Diokno
and Roberts v. Court of Appeals, the determination of probable cause does not rest on
a subjective criteria. As we had resolved in those cases to overrule the finding of
probable cause of the judges therein on the ground of grave abuse of discretion, in the
same vein, we can also overrule the decision of a judge reversing a finding of probable
cause, also on the ground of grave abuse of discretion.
There is no double jeopardy in the reinstatement of a criminal case dismissed before
arraignment
In their third assignment of error, petitioners claim that the Court of Appeals committed
a reversible error in ordering the reinstatement of Criminal Cases No. 36-3523 and No.
36-3524, alleging that the order of dismissal issued therein had become final and
executory. According to petitioners:

It is also worthy to point out at this juncture that the Joint Order of Judge Anghad dated
November 14, 2001 is NOT ONE of those Orders which were assailed in the private
respondent Tuliaos Petition for Certiorari, Mandamus and Prohibition filed by the
private respondent before the Court of Appeals. As carefully enumerated in the first
page of the assailed Decision, only the following Orders issued by Judge Anghad were
questioned by private respondent, to wit:
1.) Joint Order dated August 17, 2001;
2.) Order dated September 21, 2001;
3.) Joint Order dated October 16, 2001; and
4.) Joint Order dated October 22, 2001.
Obviously, the Joint Order dated November 14, 2001 of Judge Anghad, which ultimately
dismissed Criminal Cases Nos. 36-3523 AND 36-3524 is NOT included in the list of the
assailed Order/Joint Orders. Hence, the Court of Appeals should not have passed upon
the validity or nullity of the Joint Order of November 14, 2001.38
Petitioners must have forgotten that respondent Tuliaos Petition for Certiorari,
Prohibition and Mandamus was filed not with the Court of Appeals, but with this Court.
The Court of Appeals decided the case because we referred the same to them in our 19
November 2001 Resolution. Such petition was filed on 25 October 2001, around three
weeks before the 14 November 2001 Order. Upon receipt of the 14 November 2001
Order, however, respondent Tuliao lost no time in filing with this Court a Motion to Cite
Public Respondent in Contempt, alleging that Judge Anghad "deliberately and willfully
committed contempt of court when he issued on 15 November 2001 the Order dated 14
November 2001 dismissing the informations for murder." On 21 November 2001, we
referred said motion to the Court of Appeals, in view of the previous referral of
respondent Tuliaos petition for certiorari, prohibition and mandamus.
Our referral to the Court of Appeals of the Motion to Cite Public Repondent in Contempt
places the 14 November 2001 Order within the issues of the case decided by the Court
of Appeals. In claiming that Judge Anghad committed contempt of this Court in issuing
the 14 November 2001 Order, respondent Tuliao had ascribed to Judge Anghad an act
much more serious than grave abuse of discretion.
Respondent Tuliao claims that Judge Anghad issued the 14 November 2001 Order on
15 November 2001, antedating it so as to avoid the effects of our 12 November 2001
Resolution. In said 12 November 2001 Resolution, we resolved to issue a temporary
restraining order enjoining Judge Anghad from further proceeding with the criminal
cases upon the respondent Tuliaos filing of a bond in the amount of P20,000.00.
Respondent Tuliao had filed the bond on 15 November 2005.

While we cannot immediately pronounce Judge Anghad in contempt, seeing as


disobedience to lawful orders of a court and abuse of court processes are cases of
indirect contempt which require the granting of opportunity to be heard on the part of
respondent,39 the prayer to cite public respondent in contempt and for other reliefs just
and equitable under the premises should be construed to include a prayer for the
nullification of said 14 November 2001 Order.
In any case, the reinstatement of a criminal case dismissed before arraignment does
not constitute double jeopardy. Double jeopardy cannot be invoked where the accused
has not been arraigned and it was upon his express motion that the case was
dismissed.40
As to respondent Tuliaos prayer (in both the original petition for certiorari as well as in
his motion to cite for contempt) to disqualify Judge Anghad from further proceeding with
the case, we hold that the number of instances of abuse of discretion in this case are
enough to convince us of an apparent bias on the part of Judge Anghad. We further
resolve to follow the case of People v. SPO1 Leao,41 by transferring the venue of
Criminal Cases No. 36-3523 and No. 36-3524 to the City of Manila, pursuant to Article
VIII, Section 4, of the Constitution.
WHEREFORE, the petition is DENIED. The Decision dated 18 December 2002 and the
Resolution dated 12 June 2003 of the Court of Appeals are hereby AFFIRMED, with the
modification that Criminal Cases No. 36-3523 and No. 36-3524 be transferred to and
raffled in the Regional Trial Court of the City of Manila. In this connection,
1) Let a copy of this decision be furnished the Executive Judge of the RTC of the
City of Santiago, Isabela, who is directed to effect the transfer of the cases within
ten (10) days after receipt hereof;
2) The Executive Judge of the RTC of the City of Santiago, Isabela, is likewise
directed to report to this Court compliance hereto within ten (10) days from
transfer of these cases;
3) The Executive Judge of the City of Manila shall proceed to raffle the criminal
cases within ten (10) days from the transfer;
4) The Executive Judge of the City of Manila is likewise directed to report to this
Court compliance with the order to raffle within ten (10) days from said
compliance; and
5) The RTC Judge to whom the criminal cases are raffled is directed to act on
said cases with reasonable dispatch.
6) Finally, Judge Anastacio D. Anghad is directed to issue forthwith warrants of
arrest for the apprehension of petitioners Jose C. Miranda, Alberto P. Dalmacio,

Romeo B. Ocon, and accused Rodel T. Maderal, conformably with the decision
of the Court of Appeals dated 18 December 2002.
The Temporary Restraining Order issued by this Court dated 4 August 2003 is hereby
LIFTED. Costs against Petitioners.
SO ORDERED.

ARNOLD ALVA,
P e t i t i o n e r,

G.R. No. 157331

Present:
- versus -

PANGANIBAN, CJ,
Chairman,
YNARES -SANTIAGO
AUSTRIA-MARTINEZ,
CALLEJO, SR., and
CHICO-NAZARIO, JJ.

Promulgated:
HON. COURT OF APPEALS
R e s p o n d e n t.
April 12, 2006
x - - - - - - - - - - - - - - - - - - - - - - - - - - - - - - - - - - - - - - - - x

D E C I S I O N

CHICO-NAZARIO, J.:
Before us is a petition for review on certiorari under Rule 45 of the Rules
of Court, as amended, assailing the twin Resolutions of the Court of Appeals
(CA), dated 18 October 2002 [ 1 ] and 19 February 2003, [ 2 ] respectivel y, in CA -G.R.
CR No. 24077, entitled People of the Philippines v. Arnold Alva.

The CA, in the assailed resolutions, dismissed petitioners appeal of the trial
courts judgment of conviction for failing to post a new bail bond to secure his
provisional libert y on appeal.

The Facts

The present petition stemmed from an Information [ 3 ] charging petitioner


with having committed the crime of estafa defined under Article 315, Paragraph
2(a) of the Revised Penal Code, alleging as follows:
The undersigned accuses ARNOLD ALVA of the crime of
ESTAFA, committed as follows:
That in or about and during the period covered bet ween
October 18, 1993 up to December 18, 1993, inclusive, in the Cit y of
Manila, Philippines, the said accused, did then and there willfull y
(sic), unlawfull y and feloniousl y defraud YUMI VERANGA y
HERVERA in the following manner, to wit: the said accused, by
means of false manifestation and fraudulent representation which he
made to said YUMI VERANGA y HERVERA to the effect that he
could process the latters application for U.S. Visa provided she would
give the amount of P120,000.00, and by means of other s imilar
deceit, induced and succeeded in inducing said YUMI VERANGA y
HERVERA to give and deliver, as in fact she gave and delivered to
said accused the amount of P120,000.00 on the strength of said
manifestation and representation said accused well knowing that the
same were false and untrue for the reason that the U.S. Visa is not
genuine and were made solel y to obtain, as in fact he did obtain the
amount of P120,000.00 which amount once in his possession with
intent to defraud, he wilfull y (sic), unlawful l y and feloniousl y
misappropriated, misapplied and converted the said amount to his
own personal use and benefit, to the damage and prejudice of the said
YUMI VERANGA y HERVERA in the aforesaid amount
of P120,000.00, Philippine Currency.
CONTRARY TO LAW.

The resultant criminal case was filed and docketed as Criminal Case No. 95 143803 and raffled to the Regional Trial Court (RTC) of Manila, Branch 54,
presided by Judge Manuel T. Muro.
On 5 September 1995, the RTC issued a Recall Order [ 4 ] of the Warrant of
Arrest issued on 18 July 1995 against petitioner in view of the approval of his
bail bond by Hon. William Bayhon, then Executive Judge of the RTC of Manila.

Upon arraignment on 7 December 1995, petitioner, dul y assisted by


counsel, [ 5 ] pleaded not guilt y to the crime charged.
After the trial on the merits, in

an Order [ 6 ] dated 6 April

1998,

the RTC considered the case submitted for decision.


On 4 May 1999, petitioners counsel filed an Urgent Motion to Cancel
Promulgation [ 7 ] praying for the resetting of the 5 May 1999 schedule of
promulgation of the RTCs decision to 17 June 1999 in view of the fact that said
counsel already had a prior commitment on subject date. The RTCgranted the
motion. The promulgation, however, was deferred onl y until 19 May 1999.
A day before the rescheduled date of promul gation, or on 18 May 1999,
petitioners counsel again moved for the deferment of the promulgation, due to
prior undertakings of similar importance. [ 8 ]
On 19 May 1999, petitioner and counsel both failed to appear in court
despite due notice. In his stead, claiming to be petitioners representative, a
certain Joey Perez personall y delivered to the RTC a hand written medical
certificate [ 9 ] expressing petitioners inabilit y to attend the days hearing due to
hypertension.
In response to the aforestated acts of petitioner and counsel, the RTC issued
an Order [ 1 0 ] directing the promulgation of its decision in absentiaand the issuance
of a bench warrant of arrest against petitioner for his failure to appear before it
despite due notice.
In its decision dated 25 March 1999, [ 1 1 ] the RTC found petitioner guilt y of
the crime of estafa under Article 315, paragraph 2(a) of the Revised Penal Code,
the decretal part of which reads:
WHEREFORE, judgment is hereby rendered: finding the
accused guilt y beyond reasonable doubt of the crime of estafa under
Article 315, No. 2(a) of the RPC and sentences him t o an
indeterminate term of imprisonment of nine (9) years and one (1) day
as minimum of prision mayor to seventeen (17) years as maximum

of reclusion temporal in accordance with the provisions of Article


315, first, and the Indeterminate Sentence Law, and further for the
accused to return the P120,000.00 to the complainant with an interest
at the rate of twelve percent (12%) compounded annuall y from
January 1, 1994 (the amount has been given to the accused in October
and December 1993).

Meanwhile, as appearing in the records of the RTC, immediately following


an original duplicate copy of the aforequoted decision, a document
entitled Personal Bail Bond [ 1 2 ] dated 21 May 1999 issued by Mega Pacific
Insurance Corporation, seemed to have been filed before and approved by
the RTC as evidenced by the signature of Judge Muro on the face of said bail
bond. [ 1 3 ] For such reason, petitioner appeared to have been admitted to bail anew
after his conviction.
Incongruous to the above inference, however, in an Order [ 1 4 ] dated 25 May
1999, judgment was rendered against Eastern Insurance and Suret y Corporation,
the bonding company that issued petitioners or iginal bail bond, in the amount
of P17,000.00, for failure to produce the person of petitioner within the 10 day
period earlier provided and to explain why the amount of its undertaking should
not be forfeited.
In the interregnum, Police Superintendent R amon Flores De Jesus, Chief of
Warrant and Subpoena Section, [ 1 5 ] manifested to the RTC the return of the
unexecuted Warrant of Arrest issued o n 19 May 1999 for the reason that the
address of the accused (petitioner) is not within our area of responsibilit y. x x x
Nevertheless, De Jesus reassured the RTC that the name of the accused will be
included in our list of wanted persons for our future re ference. Examination of
the records of the case revealed that petitioner already moved out of his address
on record without informing the RTC.

On 15 Jul y 1999, hand delivered by a certain Remedios Caneda, petitioner


wrote [ 1 6 ] the RTC requesting for a certified photocopy of his exhibits submitted
to it during trial.

On 21 Jul y 1999, a Termination of Legal Services was filed by petitioner


before the RTC informing it of his decision to terminate the services of his
counsel and that he was currentl y in the process of hiring a new one.
On 26 Jul y 1999, [ 1 7 ] petitioner filed a Motion for Reconsideration before
the RTC.
In an Order [ 1 8 ] dated 30 August 1999, the RTC decl ined to give due course
to said motion for failure to set it for hearing; thus, treating it as a mere scrap of
paper.
On 2 September 1999, petitioner received the above Order. The next day,
or on 3 September 1999, petitioner filed a Notice of Appeal [ 1 9 ] before the RTC.
In an Order [ 2 0 ] dated 20 September 1999, the RTC again declined to give
due course to the Notice of Appeal, ratiocinating thus:
The Notice of Appeal filed by accused cannot be given due
course as it was filed out of time. Although accused filed a Motion
for Reconsideration dated 23 Jul y 1999, the Court considered it as a
mere scrap of paper and was not acted upon as the same was not set
for hearing, hence, it did not stop the reglementary period to file
appeal.

On 25 November 1999, petitioner filed anew a motion praying for


the RTCs categorical resolution of his 23 July 1999 Motion for Reconsideration .
In an Order dated 7 December 1999, the RTC granted the abovestated
motion, the full text of which states:
The Motion to Resolve the Motion for Reconsider ation of the
accused, dated November 20, 1999 is granted in the interest of
justice, considering that the one who prepared the Motion for
Reconsideration appears to be the accused himself, who may not

appear to be a lawyer and may not be conversant with th e rules,


among others, governing motions.
Acting on the said Motion for Reconsideration itself, same is
denied for lack of merit. The Decision has examined and discussed
the evidence presented and the merits of the case.
Because of the pendency of the Motion for Reconsideration,
the appeal is deemed filed on time, and the appeal is given due
course.
Let the records of the case, together with three (3) copies of
the transcripts of stenographic notes be transmitted to the Hon. Court
of Appeals.
On appeal before the Court of Appeals, in a Resolution [ 2 1 ] dated 16 October
2001, the appellate court required petitioner to show cause why his appe al should
not be dismissed it appearing that no new bail bond for his provisional libert y on
appeal had been posted, to wit:
Considering the arrest warrant issued by the trial court against
the accused who failed to appear at the promulgation of the judg ment,
and it appearing from the record that no new bond for his provisional
libert y on appeal has been posted, appellant is ORDERED to SHOW
CAUSE within ten (10) days from notice why his appeal should not
be dismissed outright.

On 29 October 2001, peti tioner, through


a Compliance [ 2 2 ] essentiall y stating therein that:

new

counsel,

x x x x
3. Upon learning of the course of action taken by the presiding judge,
and for purposes of appealing the decision subject of the instant case,
on May 21, 1999, accused immediatel y posted a new bond for his
provisional libert y. The presiding judge of the lower court, which
issued the questioned de cision, dul y approved the new bond.
Certified true copy of the bond is hereto attached as Annex 3 and
made an integral part hereof;
x x x x.

filed

In a Resolution [ 2 3 ] dated 18 October 2002, the Court of Appeals,


nonetheless dismissed the appeal filed by petitioner for appellants failure to post
a new bond for his provisional libert y on appeal despite our directive as contained
in our Resolution dated October 16, 2001, and in view of the fact that his personal
bail bond posted in the lower court had already expired, x x x.
Undaunted, petitioner filed a Motion for Reconsideration [ 2 4 ] thereto
seeking its reversal. According to petitioners counsel, he was of the understanding
that the Show Cause Resolution of 16 October 2001 merel y sought an
explanation vis--vis the absence of a bail bond guaranteeing petitioners
provisional libert y while his conviction was on appeal. All the same, petitioners
counsel manifested that Mega Pacific Insurance Corporation, had already
extended the period covered by its 21 May 1999 bail bond. Attached to said
motion was a Bond Endorsement [ 2 5 ] extending the coverage of the bail bond
from 21 May 1999 to 21 May 2003.
Asked to comment on the Motion for Reconsideration , respondent People
of the Philippines (People), through the Office of the Solicitor General (OSG),
interposed objections. In its Comment, [ 2 6 ] respondent People raised two
arguments: 1) that an application for bail can onl y be availed of by a person who
is in the custody of the l aw or otherwise deprived of his liberty; and 2) that bail
on appeal is a matter of discretion when the penalt y imposed by the trial court is
imprisonment exceeding six (6) years.

On 19 February 2003, the Court of Appeals issued the second assailed


Resolution, [ 2 7 ] disposing of petitioners motion as follows:
Finding no merit in appellants motion for reconsideration
(citation omitted) filed on November 12, 2002, the same is hereby
DENIED. We agree with the appellee that appellant has failed to
submit himself under the jurisdiction of the court or under the
custody of the law since his convi ction in 1999 and that there was
no valid bail bond in place when appellant took his appeal .
WHEREFORE, appellants
DENIED. [Emphasis supplied.]

motion

for

reconsideration

is

Hence, this petition.

The Issues
Petitioner now comes to this Court via a petition for review
on certiorari under Rule 45 of the Rules of Court alleging the following errors: [ 2 8 ]

I.
THE HONORABLE COURT OF APPEALS HAS DECIDED
QUESTIONS OF SUBSTANCE IN A WAY NOT IN ACCORD WITH
LAW OR WITH APPLICABLE DEC ISIONS OF THIS HONORABLE
SUPREME COURT;

II.
THE HONORABLE COURT OF APPEALS ACTED WITH GRAVE
ABUSE OF DISCRETION AMOUNTING TO LACK OR EXCESS OF
JURISDICTION
WHEN
IT
DISMISSED
THE
PETITION
DOCKETED AS CA G.R. CR NO. 24077 ON THE GROUND OF
ALLEGED FAILURE TO POST A NEW BOND FOR PETITIONERS
PROVISIONAL LIBERTY AND THAT THE PERSONAL BAIL
BOND POSTED IN THE LOWER COURT HAD ALLEGEDLY
ALREADY EXP IRED;

III.
THE HONORABLE COURT OF APPEALS GRAVELY ERRED OR
ACTED WITH GRAVE ABUSE OF DISCRETION WHEN IT DID
NOT CONS IDER AS SUBSTANTIAL, THE COMPLAINCE FILED
BY THE PETITIONER WHICH SHOWED THE FACT THAT
INDEED THERE WAS A BAIL BOND FILED FOR THE
PROVISIONAL LIBERTY OF THE ACCUSED DUR ING THE
PENDENCY OF THE APPEAL;
IV.
THE HONORABLE COURT OF APPEALS GRAVELY ERRED OR
ACTED WITH GRAVE ABUSE OF DISCRETION WHEN IT

IGNORED THE RECENT BAIL BOND EXTENSION ATTACHED


TO THE MOTION FOR RECONS IDERATION FILED BY THE
PETITIONER;
V.
THE HONORABLE COURT OF APPEALS GRAVELY ERRED OR
ACTED W ITH GRAVE ABUSE OF DISCRETION WHEN IT RULED
THAT THE PETITIONER FAILED TO SUBMIT TO THE
JURISDICTION OF THE COURT OR TO THE CUSTODY OF LAW
DESP ITE THE BAIL BOND POSTED ON MAY 21, 1999; and
VI.
THE HONORABLE COURT OF APPEALS GRAVELY ERRED OR
ACTED W ITH GRAVE ABUSE OF DISCRETION WHEN IT RULED
THAT THERE WAS NO VALID BAIL BOND IN P LACE WHEN
THE PETITIONER TOOK HIS APPEAL.

The bombardment of errors notwithstanding, onl y two issues are raised in


this petition: 1) with the exception of the fifth assignment of error, all six can be
encapsulated in one solitary question, that is, whether or not the Court of Appeals
committed reversible error in dismissing the appeal in view of petitioners alleged
failure to post a valid bail bond to secure his provisional libert y on appeal; and
2) whether or not petitioner failed to submit himself to the jurisdiction of the
court or to the custody of the law despite the posting of the subject bail bond.
The Courts Ruling
Petitioner faults the appellate court for expressing x x x in its questioned
resolutions that herein petitioner did not submit to the jurisdiction of the court or
custody of the law, or that there was no valid bail bond when the appeal was taken
when the records of the case would readily prove the contrary. [ 2 9 ] In issuing said
resolution, petitioner concludes that the Court of Appeals made x x x no careful
examination of the records x x x. Petitioner rationalizes his deduction in the
following manner:

x x x [T]he records of the case readil y reveals (sic) that several


pleadings were filed by the petitioner before the lower court even
after the promulgation of judgment was made. Right after the
promulgation of the decision in the lower court, herein petitioner
went to the court and posted a bail bond. If the posting of the bond
which was approved by the same Regional Trial Court who ren dered
the decision subject of appeal is not yet a submission to the
jurisdiction of the court, then the respondent Hon. Court of Appeals
must have been thinking of another matter beyond the comprehension
of the petitioner and obviousl y outside the matters being
contemplated by law and the Rules of Court.

Equall y, petitioner further posits that:


x x x Although it is respectfull y submitted that an accused shall be
denied bail or his bail shall be cancelled if sentenced to an
imprisonment exceeding six (6) years as provided in Section 5, Rule
114 of the Rules of Court, just the same, there must be a showing
by the prosecution with notice to the accused of the fact that, the
accused is a recidivist, has previousl y escaped from confinement,
evaded sentence, has committed an offense while under probation,
there are circumstances indicating the probabilit y of flight if released
on bail, etc. But there was none of the said instances that may be
attributable to herein petitioner. [ 3 0 ]

Respondent People, in contrast, counters that x x x [a]lthough a personal


bail bond dated May 21, 1999 was executed in favor of petitioner by Mega Pacific
Insurance Corporation two days after the promulgation of the Decision, there is
nothing

on

record

which

shows

that

petitioner

had surrendered, was

arrested or otherwise deprived of his libert y after the promulgation of the


judgment of his conviction in his absence. x x x. To illustrate its point,
respondent People cites the following facts: 1) the return of the Warrant of Arrest
issued on May 19, 1999 signed by P/Superintendent Ramon Flores De Jesus, Chief
of Warrant and Subpoena Section, which states in ful l:

Respectfull y returned this unexecuted Warrant of Arrest for the


reason that the address of the accused is not within our area of
responsibilit y. Further request that the warrant of Arrest be
forwarded to the Police Station which has Jurisdiction over the
address of the accused.
However, the name of the accused will be included in our list
of wanted persons for our future reference.

2) the fact that six days after the decision of the RTC was promulgated, or on 25
May 1999, said court rendered judg ment against the bail bond issued by Eastern
Assurance and Surety Corporation executed to secure petitioners provisional
libert y during the trial, for the bondsmans failure to produce petitioner before the
court, to wit:
In view of the failure of Eastern Insurance & Suret y
Corporation, bondsman of herein accused, to produce the herein
accused within the period granted it by this Court, judgment is hereby
rendered against said bond in the amount of Seventeen Thousand
(P17,000.00) Pesos. [ 3 1 ]

Respondent People explains that the first two facts make it improbable to
conclude that there existed a valid bail bond securing petitioners provisional
libert y even after conviction. Stated in another way, petitioners admission to bail
presumes that the latter surrendered, was arrested or he had otherwise submitted
himself under the custody of the law.

And, 3) that petitioner belatedl y attached a bond e ndorsement to his motion for
reconsideration dated November 7, 2002 submitted before the Court of Appeals,
purportedl y to extend the expired personal bond dated May 21, 1999 x x x, did
not automaticall y confer on petitioner the benefits of an effective bai l bond, [ 3 2 ] as
petitioner made no extension of the previous personal bond before the same
expired.

We disagree in petitioners assertions; hence, the petition must fail.


A definitive disposition of the issue relating to the existence and validit y
of petitioners bail bond on appeal presupposes that the latter was allowed by law
to post bail notwithstanding the RTCs judgment of conviction and the imposition
of the penalt y of imprisonment for an indeterminate period of nine (9) years and
one (1) day as minimum of prision mayor to seventeen (17) years as maximum
of reclusion temporal .
Section 5 of Rule 114 of the 1994 Rules of Court, as amende d, intrinsicall y
addresses the foregoing prefatory matter viz:
SEC. 5. Bail, when discretionary. Upon conviction by the
Regional Trial Court of an offense not punishable by death, reclusion
perpetua or life imprisonment, the court, on application, may admit
the accused to bail.
The court, in its discretion, may allow the accused to continue
on provisional liberty under the same bail bond during the period to
appeal subject to the consent of the bondsman.
If the court imposed a penalty of imprisonment exceeding six
(6) years, but not more than twenty (20) years, the accused shall be
denied bail, or his bail previously granted shall be cancelled, upon
a showing by the prosecution, with notice to the accused, of the
following or other similar circumstance s:
(a) That the accused is a recidivist, quasi -recidivist, or
habitual delinquent, or has committed the crime aggravated by the
circumstances of reiteration;
(b) That the accused is found to have previousl y escaped from
legal confinement, evaded sentence, or has violated the conditions of
his bail without valid justification;
(c) That the accused committed the offense while on probation,
parole, or under conditional pardon;
(d) That the circumstances of the accused or his case indicate
the probabilit y of flight if released on bail; or

(e) That there is undue risk that during the pendency of the
appeal, the accused may commit another crime.
The appellate court may review the resolution of the Regional
Trial Court, on motion and with notice t o the adverse part y.
[Emphasis supplied.]
From the preceding quoted provision, the RTC is given the discretion to
admit to bail an accused even after the latter has been convicted to suffer the
penalt y of imprisonment for a term of more than six (6) year s but less than twent y
(20) years. However, the same also provides for the cancellation of bail bonds
already granted or the denial of a bail bond application upon the concurrence of
two points: 1) if the judgment of the Regional Trial Court exceeds six (6 ) years
but not more than twent y (20) years; and 2) upon a showing by the prosecution,
with notice to the accused, of the presence of any of the five circumstances therein
enumerated or other similar circumstances.

In the case at bar, petitioner was conv icted by the RTC to suffer the penalt y
of imprisonment for an indeterminate term of nine (9) years and one (1) day as
minimum of prision mayor to seventeen (17) years as maximum of reclusion
temporal. Quite clearl y, the approval of petitioners application for bail was
discretionary upon the RTC.

It is incongruous, to say the least, that the posting of a bail presupposes that the
accused and/ or accused -appellant is detained or in the custody of the law. [ 3 3 ] In
the case at bar, the bench warrant issued by the RTC on 19 May 1999 still
remains unserved. Nothing in the records of the case, neither in the RTC nor the
Court of Appeals, demonstrates that petitioner was ever arrested, as there has
been no related Order of Release issued by any court, or that he voluntaril y
surrendered or at the very least placed himself under the custody of the law.

Basic is the principle that that the right to bail can onl y be availed of by a person
who is in custody of the law or otherwise deprived of his liberty and it would be

premature, x x x, to file a petition for bail for someone whose freedom has yet to
be curtailed. [ 3 4 ]

All told, no bail should have been granted petitioner. It is beyond dispute
that the subject bail bond issued by Mega Pacific Insurance Corporation was
irregularl y approved. Wort h noting is the fact that nowhere in the records of the
case is it shown that petitioner applied for bail through a motion dul y filed for
such purpose nor is there showing that the RTC issued an Order of Approval or
any other court process acknowledging su ch document.Be that as it may, even
granting for the sake of argument that it was indeed approved by Judge Muro,
such approval did not render the subject bail bond valid and binding for it has
been established that petitioner was not entitled to bail on ap peal.

That the prosecution appears not to have been given the chance to object,
as evidentl y required under the quoted rule, to the application or approval of the
subject bail bond (with notice to the accused), fortifies the declaration as to its
invalidit y. Nowhere in the original records of the RTC does it even show that the
prosecution was informed of petitioners application for bail, much less the
approval of such application.

Noting that the raison d'tre for such requirement is the discretionary nature
of the admission to bail of an accused after conviction, though discretionary, such
assessment must be exercised in accordance with applicable legal principles. As
when there is a concurrence of the enumerated circumstances and the range of
penalt y imposed, t he prosecution must first be accorded an opportunit y to object
and present evidence, if necessary, with notice to the accused. It is on this basis
that judicial discretion is balanced in determining whether or not an accused appellant should be admitted to bail pending appeal of his conviction vis--vis the
increased possibilit y or likelihood of flight.

Approval of an application for bail on appeal, absent the knowledge of the


prosecution of such application or, at the very least, failing to allow it to ob ject,
is not the product of sound judicial discretion but of impulse and arbitrariness,
not to mention violative of respondent Peoples right of procedural due process.

This is especiall y true in this case as a close scrutiny of the original records
of the case at bar reveals that petitioner violated the conditions of his bail without
valid justification his failure to appear before the RTC, despite due notice, on the
day of the promulgation of the latters judgment, absent any justifiable reason. His
absence was a clear contravention of the conditions of his bail bond to say the
least. As evidenced by the undertaking printed on the face of the bond issued b y
Eastern Insurance and Suret y Corporation and likewise required under Section
6 [ 3 5 ] of Rule 120 of the Rules of Court, petitioner must present himself before the
court for the reading of the judgment of the RTC in order to render himself to th e
execution thereof.

While, indeed, a medical certificate was hand delivered and filed by a


certain Joey Perez, allegedl y a representative of petitioner, stating therein the
reason for the latters absence, the RTC found insubstantial the explanation
proffered. Appropriatel y, it ordered the promulgation of its judgment in absentia .
It also issued a bench warrant of arrest against petitioner.
Upon examination, the subject medical certificate [ 3 6 ] merely states that
petitioner was diagnosed to be suffering from hypertension. It failed to elucidate
further any concomitant conditions necessitating petitioners physical incapabilit y
to present himself before the court even for an hour or two; thus, it considered
the absence of petitioner unjustified. What's more, though notarized, the subject
document failed to indicate evidence of affiants [ 3 7 ] identity making its due
execution doubtful.

Further, it should be recalled as well, that as earl y as 4 May 1999, petitioner


and counsel had already been notified of the 19 May 1999 schedule of
promulgation. The first having been postponed in view of the Urgent Motion to
Cancel Promulgation (on 5 May 1999) filed by petitioners counsel.

Another telling evidence of the violation of petitioners original bail bond


is revealed by the Process Servers Return, [ 3 8 ] indicated at the dorsal portion of
the RTCs Produce Order, indicating petitioners change of address without prior
notice to the RT C, it states:
PROCESS SERVERS RETURN
This certifies that on the 17 t h day of May, (sic) 1999,
undersigned
return (sic) again
to Fersal Apartelle located
at
130 Kalayaan Ave. (sic) Diliman,Quezon Cit y for confirmation and
indeed the addressee, Arnold Alva, had no (sic) longer been residing
nor holding office at the aforementioned address.

By failing to inform the RTC of his change of address, petitioner failed to hold
himself amenable to the orders and processes of the RTC. It was an unmistakable
arrant breach of the conditions of his bail bond.

Prescinding from the above discussion, the conviction of petitioner to a


period beyond six (6) years but less than twent y (20) years in tandem with
attendant circumstances effectivel y violating his bail w ithout valid justification
should have effectivel y precluded him from being admitted to bail on appeal.

The issue of the validit y of petitioners bail bond on appeal having been laid
to rest by Section 5 of Rule 114 of the 1994 Rules of Court, as amended,
petitioners alleged failure to post a bail bond on appeal is, therefore,
inconsequential as, under the circumstances, he is disallowed by law to be
admitted to bail on appeal. Thus, for all legal intents and purposes, there can be

no other conclusion than that at the time petitioner filed his notice of appeal and
during the pendency of his appeal even until now he remains at large, placing
himself beyond the pale, and protection of the law.

Inexorabl y, having jumped bail and eluded arrest until the prese nt, the issue of
whether or not petitioner has lost his right to appeal his conviction now ensues.

The manner of review of petitioners conviction is governed by the Rules of


Court. Appropriatel y, Rule 124 of the Rules of Court presents the procedural
requirements regarding appeals taken to the Court of Appeals. Section 8 of said
Rule finds application to the case at bar, viz:
SEC. 8. Dismissal of appeal for abandonment or failure to
prosecute. The appellate court may, upon motion of the appellee or
its own motion and notice to the appellant, dismiss the appeal if the
appellant fails to file his brief within the time prescribed by this
Rule, except in case the appellant is represented by a counsel de
oficio.
The court may also, upon motion of the appellee or on its own
motion, dismiss the appeal if the appellant escapes from prison or
confinement or jumps bail or flees to a foreign coun try during the
pendency of the appeal . [Emphasis supplied.]

By virtue of the second paragraph of the abovequoted provision, the act of


jumping bail, among otherthings, will result in the outright dismissal of
petitioners appeal. As pointed out by the C ourt in the case of People v.
Mapalao, [ 3 9 ] the reason for said rule is that:
[O]nce an accused escapes from prison or confinement or jumps bail
or flees to a foreign country, he losses his standing in court and
unless he surrenders or submits to the jurisdiction of the court he is
deemed to have waived any right to seek relief from the court.

Thus, the Court of Appeals committed no revers ible error in dismissing


petitioners appeal. Within the meaning of the principles governing the prevailing
criminal procedure, petitioner impliedl y withdrew his appeal by jumping bail and
thereby made the judgment of the RTC final and executory. [ 4 0 ]

By putting himself beyond the reach and application of the legal processes
of the land, petitioner revealed his contempt of the law and placed him self in a
position to speculate at his pleasure his chances for a reversal. This, we cannot
condone. Once more, by jumping bail, petitioner has waived his right to appeal.
In the case of People v. Ang Gioc, [ 4 1 ] we enunciated that:
There are certain fundamental rights which cannot be waived
even by the accused himself, but the right of appeal is not one of
them. This right is granted solel y for the benefit of the accused. He
may avail of it or not, as he pleases. He may waive it either expressl y
or by implication. When the accused flees after the case has been
submitted to the court for decision, he will be deemed to have waived
his right to app eal from the judgment rendered against him x x x.

Coming now to the second issue of whether or not petitioner failed to


submit himself to the jurisdiction of the court or to the custody of the law, despite
the posting of the subject bail bond, petitioner argues that his act of filing several
pleadings after the promulgation of the RTCs judgment plus his filing of the
application for his admission to bail should be considered a submission to the
courts jurisdiction. He rationalizes that:
[T]he records of the case readil y reveals that several pleadings were
filed by the petitioner before the lower court even after the
promulgation of judgment was made. Right after the promulgation of
the decision in the lower court, herein petitioner went to the court
and posted a bail bond. If the posting of the bond which was approved
by the same Regional Trial Court who rendered the decision subject
of appeal is not yet a submission to the jurisdiction of the court, then
the respondent Hon. Court of Appeals must have bee n thinking of
another matter beyond the comprehension of the petitioner and

obviousl y outside the matters being contemplated by law and the


Rules of Court.

For the resolution of the second issue, it should have been sufficient to
state that for reasons s tated in the foregoing discussion, the question posed has
now become academic. However, to diminish the confusion brought about b y
ostensibl y equating the term jurisdiction of the court (over the person of the
accused) with that of custody of the law, it is fundamental to differentiate the two.
The term:
Custody of the law is accomplished either by arrest or voluntary
surrender (citation omitted); while (the term) jurisdiction over the
person of the accused is acquired upon his arrest or voluntary
appearance (citation omitted). One can be under the custody of the
law but not yet subject to the jurisdiction of the court over his person,
such as when a person arrested by virtue of a warrant files a motion
before arraignment to quash the warrant. On the other hand, one can
be subject to the jurisdiction of the court over his person, and yet not
be in the custody of the law, such as when an accused escapes custody
after his trial has commenced (citation omitted). [ 4 2 ]

Moreover, jurisdiction, once acquired, is not lost at the instance of parties, as


when an accused escapes from the custody of the law, but continues until the case
is terminated. [ 4 3 ] Evidentl y, petitioner is correct in that there is no doubt that the
RTC already acquired jurisdiction over the person of the accused petitioner when
he appeared at the arraignment and pleaded not guilt y to the crime charged
notwithstanding the fact that he jumped bail and is now considered a fugitive.

As to whether or not petitioner has placed himself under the custody of the CA,
alas, we cannot say the same for [b]eing in the custody of the law signifies
restraint on the person, who is thereby deprived of his own will and libert y,
binding him to become obedient to the will of the law (citation omitted). Custody
of the law is literall y custody over the body of the accused. It includes, but is not
limited to, detention. [ 4 4 ] In the case at bar, petitioner, being a fugitive, un til and

unless he submits himself to the custody of the law, in the manner of being under
the jurisdiction of the courts, he cannot be granted any relief by the CA.

Parentheticall y, we cannot end this ponencia without calling attention to a


very disturbi ng fact that petitioner admits of being the author of a falsified public
document was treated nonchalantl y by authorities.

In fine, the petitioner has remained at large even as he hopes that his appeal,
and consequentl y, this petition, will succeed and h e can then appear before the
Court to claim his victory. He hopes in vain.
WHEREFORE, the instant petition is DENIED for lack of merit. The
assailed Resolutions of the Court of Appeals, in CA -G.R. CR No. 24077, which
dismissed petitioners appeal, are hereby AFFIRMED. In this connection, Judge
Manuel Muro is DIRECTED to issue forthwith a warrant of arrest for the
apprehension of Petitioner Arnold Alva and for proper disposition of the case in
line with the foregoing discussion.
Costs against the petitio ner.
SO ORDERED.
RENATO M. DAVID, Petitioner, - versus - EDITHA A. AGBAY and PEOPLE OF THE
PHILIPPINES, G.R. No. 199113 Present: VELASCO, JR, J., Chairperson, PERALTA,
VILLARAMA, JR., REYES, and PERLAS-BERNABE,* JJ. Promulgated: Respondents. ____ ~ ___ _ x- - - - - - - - - - - - - - - - - - - - - - - - - - - - - - - - - ~ti-~- ----x March 18, 2015 DECISION
VILLARAMA, JR., J.:
This is a petition for review under Rule 45 seeking to reverse the Order1 dated October 8, 2011
of the Regional Trial Court (RTC) of Pinamalayan, Oriental Mindoro, which denied the petition
for certiorari filed by Renato M. David (petitioner). Petitioner assailed the Order2 dated March
22, 2011 of the Municipal Trial Court (MTC) of Socorro, Oriental Mindoro denying his motion
for redetermination of probable cause. The factual antecedents: In 1974, petitioner migrated to
Canada where he became a Canadian citizen by naturalization. Upon their retirement, petitioner
and his wife returned to the Philippines. Sometime in 2000, they purchased a 600-square meter
lot along the beach in Tambong, Gloria, Oriental Mindoro where they constructed a residential
house. However, in the year 2004, they came to know that the portion where they built their
house is public land and part of the salvage zone.

On April 12, 2007, petitioner filed a Miscellaneous Lease Application3 (MLA) over the subject
land with the Department of Environment and Natural Resources (DENR) at the Community
Environment and Natural Resources Office (CENRO) in Socorro. In the said application,
petitioner indicated that he is a Filipino citizen. Private respondent Editha A. Agbay opposed the
application on the ground that petitioner, a Canadian citizen, is disqualified to own land. She also
filed a criminal complaint for falsification of public documents under Article 172 of the Revised
Penal Code (RPC) (I.S. No. 08-6463) against the petitioner. Meanwhile, petitioner re-acquired
his Filipino citizenship under the provisions of Republic Act No. 9225,4 (R.A. 9225) as
evidenced by Identification Certificate No. 266-10-075 issued by the Consulate General of the
Philippines (Toronto) on October 11, 2007. In his defense, petitioner averred that at the time he
filed his application, he had intended to re-acquire Philippine citizenship and that he had been
assured by a CENRO officer that he could declare himself as a Filipino. He further alleged that
he bought the property from the Agbays who misrepresented to him that the subject property was
titled land and they have the right and authority to convey the same. The dispute had in fact led
to the institution of civil and criminal suits between him and private respondents family. On
January 8, 2008,6 the Office of the Provincial Prosecutor issued its Resolution7 finding probable
cause to indict petitioner for violation of Article 172 of the RPC and recommending the filing of
the corresponding information in court. Petitioner challenged the said resolution in a petition for
review he filed before the Department of Justice (DOJ). On June 3, 2008, the CENRO issued an
order rejecting petitioners MLA. It ruled that petitioners subsequent re-acquisition of Philippine
citizenship did not cure the defect in his MLA which was void ab initio. 8 In the meantime, on
July 26, 2010, the petition for review filed by petitioner was denied by the DOJ which held that
the presence of the elements of the crime of falsification of public document suffices to warrant
indictment of the petitioner notwithstanding the absence of any proof that he gained or intended
to injure a third person in committing the act of falsification.9 Consequently, an information for
Falsification of Public Document was filed before the MTC (Criminal Case No. 2012) and a
warrant of arrest was issued against the petitioner. On February 11, 2011, after the filing of the
Information and before his arrest, petitioner filed an Urgent Motion for Re-Determination of
Probable Cause10 in the MTC. Interpreting the provisions of the law relied upon by petitioner,
the said court denied the motion, holding that R.A. 9225 makes a distinction between those who
became foreign citizens during its effectivity, and those who lost their Philippine citizenship
before its enactment when the governing law was Commonwealth Act No. 6311 (CA 63). Since
the crime for which petitioner was charged was alleged and admitted to have been committed on
April 12, 2007 before he had reacquired his Philippine citizenship, the MTC concluded that
petitioner was at that time still a Canadian citizen. Thus, the MTC ordered: WHEREFORE, for
lack of jurisdiction over the person of the accused, and for lack of merit, the motion is DENIED.
SO ORDERED. 12 In his motion for reconsideration,13 petitioner questioned the foregoing
order denying him relief on the ground of lack of jurisdiction and insisted that the issue raised is
purely legal. He argued that since his application had yet to receive final evaluation and action
by the DENR Region IV-B office in Manila, it is academic to ask the citizenship of the applicant
(petitioner) who had re-acquired Philippine citizenship six months after he applied for lease of
public land. The MTC denied the motion for reconsideration.14 Dissatisfied, petitioner elevated
the case to the RTC via a petition15 for certiorari under Rule 65, alleging grave abuse of
discretion on the part of the MTC. He asserted that first, jurisdiction over the person of an
accused cannot be a pre-condition for the re-determination of probable cause by the court that
issues a warrant of arrest; and second, the March 22, 2011 Order disregarded the legal fiction

that once a natural-born Filipino citizen who had been naturalized in another country re-acquires
his citizenship under R.A. 9225, his Filipino citizenship is thus deemed not to have been lost on
account of said naturalization. In his Comment and Opposition,16 the prosecutor emphasized
that the act of falsification was already consummated as petitioner has not yet re-acquired his
Philippine citizenship, and his subsequent oath to re-acquire Philippine citizenship will only
affect his citizenship status and not his criminal act which was long consummated prior to said
oath of allegiance. On October 8, 2011, the RTC issued the assailed Order denying the petition
for certiorari after finding no grave abuse of discretion committed by the lower court, thus:
ACCORDINGLY, the petition is hereby DENIED. At any rate petitioner is not left without any
remedy or recourse because he can proceed to trial where he can make use of his claim to be a
Filipino citizen as his defense to be adjudicated in a full blown trial, and in case of conviction, to
appeal such conviction. SO ORDERED. 17 Petitioner is now before us arguing that A. By
supporting the prosecution of the petitioner for falsification, the lower court has disregarded the
undisputed fact that petitioner is a natural-born Filipino citizen, and that by re-acquiring the same
status under R.A. No. 9225 he was by legal fiction deemed not to have lost it at the time of his
naturalization in Canada and through the time when he was said to have falsely claimed
Philippine citizenship. B. By compelling petitioner to first return from his legal residence in
Canada and to surrender or allow himself to be arrested under a warrant for his alleged false
claim to Philippine citizenship, the lower court has pre-empted the right of petitioner through his
wife and counsel to question the validity of the said warrant of arrest against him before the same
is implemented, which is tantamount to a denial of due process.18 In his Comment, the Solicitor
General contends that petitioners argument regarding the retroactivity of R.A. 9225 is without
merit. It is contended that this Courts rulings in Frivaldo v. Commission on Elections19 and
Altarejos v. Commission on Elections20 on the retroactivity of ones reacquisition of Philippine
citizenship to the date of filing his application therefor cannot be applied to the case of herein
petitioner. Even assuming for the sake of argument that such doctrine applies in the present
situation, it will still not work for petitioners cause for the simple reason that he had not alleged,
much less proved, that he had already applied for reacquisition of Philippine citizenship before
he made the declaration in the Public Land Application that he is a Filipino. Moreover, it is
stressed that in falsification of public document, it is not necessary that the idea of gain or intent
to injure a third person be present. As to petitioners defense of good faith, such remains to be a
defense which may be properly raised and proved in a fullblown trial. On the issue of jurisdiction
over the person of accused (petitioner), the Solicitor General opines that in seeking an
affirmative relief from the MTC when he filed his Urgent Motion for Re-determination of
Probable Cause, petitioner is deemed to have submitted his person to the said courts jurisdiction
by his voluntary appearance. Nonetheless, the RTC correctly ruled that the lower court
committed no grave abuse of discretion in denying the petitioners motion after a judicious,
thorough and personal evaluation of the parties arguments contained in their respective
pleadings, and the evidence submitted before the court. In sum, the Court is asked to resolve
whether (1) petitioner may be indicted for falsification for representing himself as a Filipino in
his Public Land Application despite his subsequent re-acquisition of Philippine citizenship under
the provisions of R.A. 9225; and (2) the MTC properly denied petitioners motion for redetermination of probable cause on the ground of lack of jurisdiction over the person of the
accused (petitioner). R.A. 9225, otherwise known as the Citizenship Retention and
Reacquisition Act of 2003, was signed into law by President Gloria Macapagal-Arroyo on
August 29, 2003. Sections 2 and 3 of said law read: SEC. 2. Declaration of Policy.It is hereby

declared the policy of the State that all Philippine citizens who become citizens of another
country shall be deemed not to have lost their Philippine citizenship under the conditions of this
Act. SEC. 3. Retention of Philippine Citizenship.Any provision of law to the contrary
notwithstanding, natural-born citizens of the Philippines who have lost their Philippine
citizenship by reason of their naturalization as citizens of a foreign country are hereby deemed to
have reacquired Philippine citizenship upon taking the following oath of allegiance to the
Republic: I ______________________, solemnly swear (or affirm) that I will support and
defend the Constitution of the Republic of the Philippines and obey the laws and legal orders
promulgated by the duly constituted authorities of the Philippines; and I hereby declare that I
recognize and accept the supreme authority of the Philippines and will maintain true faith and
allegiance thereto; and that I impose this obligation upon myself voluntarily without mental
reservation or purpose of evasion. Natural-born citizens of the Philippines who, after the
effectivity of this Act, become citizens of a foreign country shall retain their Philippine
citizenship upon taking the aforesaid oath. (Emphasis supplied) While Section 2 declares the
general policy that Filipinos who have become citizens of another country shall be deemed not
to have lost their Philippine citizenship, such is qualified by the phrase under the conditions of
this Act. Section 3 lays down such conditions for two categories of natural-born Filipinos
referred to in the first and second paragraphs. Under the first paragraph are those natural-born
Filipinos who have lost their citizenship by naturalization in a foreign country who shall reacquire their Philippine citizenship upon taking the oath of allegiance to the Republic of the
Philippines. The second paragraph covers those natural-born Filipinos who became foreign
citizens after R.A. 9225 took effect, who shall retain their Philippine citizenship upon taking the
same oath. The taking of oath of allegiance is required for both categories of natural-born
Filipino citizens who became citizens of a foreign country, but the terminology used is different,
re-acquired for the first group, and retain for the second group. The law thus makes a
distinction between those natural-born Filipinos who became foreign citizens before and after the
effectivity of R.A. 9225. Although the heading of Section 3 is Retention of Philippine
Citizenship, the authors of the law intentionally employed the terms re-acquire and retain to
describe the legal effect of taking the oath of allegiance to the Republic of the Philippines. This
is also evident from the title of the law using both re-acquisition and retention. In fine, for those
who were naturalized in a foreign country, they shall be deemed to have re-acquired their
Philippine citizenship which was lost pursuant to CA 63, under which naturalization in a foreign
country is one of the ways by which Philippine citizenship may be lost. As its title declares, R.A.
9225 amends CA 63 by doing away with the provision in the old law which takes away
Philippine citizenship from natural-born Filipinos who become naturalized citizens of other
countries and allowing dual citizenship,21 and also provides for the procedure for re-acquiring
and retaining Philippine citizenship. In the case of those who became foreign citizens after R.A.
9225 took effect, they shall retain Philippine citizenship despite having acquired foreign
citizenship provided they took the oath of allegiance under the new law. Petitioner insists we
should not distinguish between re-acquisition and retention in R.A. 9225. He asserts that in
criminal cases, that interpretation of the law which favors the accused is preferred because it is
consistent with the constitutional presumption of innocence, and in this case it becomes more
relevant when a seemingly difficult question of law is expected to have been understood by the
accused, who is a non-lawyer, at the time of the commission of the alleged offense. He further
cites the letter-reply dated January 31, 201122 of the Bureau of Immigration (BI) to his query,
stating that his status as a natural-born Filipino will be governed by Section 2 of R.A. 9225.

These contentions have no merit. That the law distinguishes between re-acquisition and retention
of Philippine citizenship was made clear in the discussion of the Bicameral Conference
Committee on the Disagreeing Provisions of House Bill No. 4720 and Senate Bill No. 2130 held
on August 18, 2003, where Senator Franklin Drilon was responding to the query of
Representative Exequiel Javier: REP. JAVIER. I have some questions in Section 3. Here, under
Section 3 of the Senate version, Any provision of law on the contrary notwithstanding, naturalborn citizens of the Philippines who, after the effectivity of this Act, shall and so forth, ano,
shall retain their Philippine citizenship. Now in the second paragraph, natural-born citizens who
have lost their citizenship by reason of their naturalization after the effectivity of this Act are
deemed to have reacquired THE CHAIRMAN (SEN. DRILON). Prior to the effectivity. REP.
JAVIER. Well, you have two kinds of natural-born citizens here. Natural-born citizens who
acquired foreign citizenship after the effectivity of this act are considered to have retained their
citizenship. But natural-born citizens who lost their Filipino citizenship before the effectivity of
this act are considered to have reacquired. May I know the distinction? Do you mean to say that
natural-born citizens who became, lets say, American citizens after the effectivity of this act are
considered natural-born? Now in the second paragraph are the natural-born citizens who lost
their citizenship before the effectivity of this act are no longer natural born citizens because they
have just reacquired their citizenship. I just want to know this distinction, Mr. Chairman. THE
CHAIRMAN (SEN. DRILON). The title of the Senate version is precisely retention and
reacquisition. The reacquisition will apply to those who lost their Philippine citizenship by virtue
of Commonwealth Act 63. Upon the effectivity -- assuming that we can agree on this, upon the
effectivity of this new measure amending Commonwealth Act 63, the Filipinos who lost their
citizenship is deemed to have reacquired their Philippine citizenship upon the effectivity of the
act. The second aspect is the retention of Philippine citizenship applying to future instances. So
thats the distinction. REP. JAVIER. Well, Im just asking this question because we are here
making distinctions between natural-born citizens. Because this is very important for certain
government positions, no, because natural-born citizens are only qualified for a specific THE
CHAIRMAN (SEN. DRILON). That is correct. REP. JAVIER. ...positions under the
Constitution and under the law. THE CHAIRMAN (SEN. DRILON). Yes. We can get to that
later on. Its one of the provisions, yes. But just for purposes of the explanation, Congressman
Javier, that is our conceptualization. Reacquired for those who previously lost [Filipino
citizenship] by virtue of Commonwealth Act 63, and retention for those in the future. (Emphasis
supplied) Considering that petitioner was naturalized as a Canadian citizen prior to the effectivity
of R.A. 9225, he belongs to the first category of naturalborn Filipinos under the first paragraph
of Section 3 who lost Philippine citizenship by naturalization in a foreign country. As the new
law allows dual citizenship, he was able to re-acquire his Philippine citizenship by taking the
required oath of allegiance. For the purpose of determining the citizenship of petitioner at the
time of filing his MLA, it is not necessary to discuss the rulings in Frivaldo and Altarejos on the
retroactivity of such reacquisition because R.A. 9225 itself treats those of his category as having
already lost Philippine citizenship, in contradistinction to those natural-born Filipinos who
became foreign citizens after R.A. 9225 came into force. In other words, Section 2 declaring the
policy that considers Filipinos who became foreign citizens as not to have lost their Philippine
citizenship, should be read together with Section 3, the second paragraph of which clarifies that
such policy governs all cases after the new laws effectivity. As to the letter-reply of BI, it
simply quoted Section 2 of R.A. 9225 without any reference to Section 3 on the particular
application of reacquisition and retention to Filipinos who became foreign citizens before and

after the effectivity of R.A. 9225. Petitioners plea to adopt the interpretation most favorable to
the accused is likewise misplaced. Courts adopt an interpretation more favorable to the accused
following the time-honored principle that penal statutes are construed strictly against the State
and liberally in favor of the accused.23 R.A. 9225, however, is not a penal law. Falsification of
documents under paragraph 1, Article 17224 in relation to Article 17125 of the RPC refers to
falsification by a private individual, or a public officer or employee who did not take advantage
of his official position, of public, private, or commercial documents. The elements of
falsification of documents under paragraph 1, Article 172 of the RPC are: (1) that the offender is
a private individual or a public officer or employee who did not take advantage of his official
position; (2) that he committed any of the acts of falsification enumerated in Article 171 of the
RPC; and (3) that the falsification was committed in a public, official or commercial
document.26 Petitioner made the untruthful statement in the MLA, a public document, that he is
a Filipino citizen at the time of the filing of said application, when in fact he was then still a
Canadian citizen. Under CA 63, the governing law at the time he was naturalized as Canadian
citizen, naturalization in a foreign country was among those ways by which a natural-born
citizen loses his Philippine citizenship. While he re-acquired Philippine citizenship under R.A.
9225 six months later, the falsification was already a consummated act, the said law having no
retroactive effect insofar as his dual citizenship status is concerned. The MTC therefore did not
err in finding probable cause for falsification of public document under Article 172, paragraph 1.
The MTC further cited lack of jurisdiction over the person of petitioner accused as ground for
denying petitioners motion for redetermination of probable cause, as the motion was filed prior
to his arrest. However, custody of the law is not required for the adjudication of reliefs other than
an application for bail.27 In Miranda v. Tuliao, 28 which involved a motion to quash warrant of
arrest, this Court discussed the distinction between custody of the law and jurisdiction over the
person, and held that jurisdiction over the person of the accused is deemed waived when he files
any pleading seeking an affirmative relief, except in cases when he invokes the special
jurisdiction of the court by impugning such jurisdiction over his person. Thus: In arguing, on the
other hand, that jurisdiction over their person was already acquired by their filing of the above
Urgent Motion, petitioners invoke our pronouncement, through Justice Florenz D. Regalado, in
Santiago v. Vasquez: The voluntary appearance of the accused, whereby the court acquires
jurisdiction over his person, is accomplished either by his pleading to the merits (such as by
filing a motion to quash or other pleadings requiring the exercise of the courts jurisdiction
thereover, appearing for arraignment, entering trial) or by filing bail. On the matter of bail, since
the same is intended to obtain the provisional liberty of the accused, as a rule the same cannot be
posted before custody of the accused has been acquired by the judicial authorities either by his
arrest or voluntary surrender. Our pronouncement in Santiago shows a distinction between
custody of the law and jurisdiction over the person. Custody of the law is required before the
court can act upon the application for bail, but is not required for the adjudication of other reliefs
sought by the defendant where the mere application therefor constitutes a waiver of the defense
of lack of jurisdiction over the person of the accused. Custody of the law is accomplished either
by arrest or voluntary surrender, while jurisdiction over the person of the accused is acquired
upon his arrest or voluntary appearance. One can be under the custody of the law but not yet
subject to the jurisdiction of the court over his person, such as when a person arrested by virtue
of a warrant files a motion before arraignment to quash the warrant. On the other hand, one can
be subject to the jurisdiction of the court over his person, and yet not be in the custody of the
law, such as when an accused escapes custody after his trial has commenced. Being in the

custody of the law signifies restraint on the person, who is thereby deprived of his own will and
liberty, binding him to become obedient to the will of the law. Custody of the law is literally
custody over the body of the accused. It includes, but is not limited to, detention. x x x x While
we stand by our above pronouncement in Pico insofar as it concerns bail, we clarify that, as a
general rule, one who seeks an affirmative relief is deemed to have submitted to the jurisdiction
of the court. As we held in the aforecited case of Santiago, seeking an affirmative relief in court,
whether in civil or criminal proceedings, constitutes voluntary appearance. x x x x To
recapitulate what we have discussed so far, in criminal cases, jurisdiction over the person of the
accused is deemed waived by the accused when he files any pleading seeking an affirmative
relief, except in cases when he invokes the special jurisdiction of the court by impugning such
jurisdiction over his person. Therefore, in narrow cases involving special appearances, an
accused can invoke the processes of the court even though there is neither jurisdiction over the
person nor custody of the law. However, if a person invoking the special jurisdiction of the court
applies for bail, he must first submit himself to the custody of the law. 29 (Emphasis supplied)
Considering that petitioner sought affirmative relief in filing his motion for re-determination of
probable cause, the MTC clearly erred in stating that it lacked jurisdiction over his person.
Notwithstanding such erroneous ground stated in the MTC's order, the RTC correctly ruled that
no grave abuse of discretion was committed by the MTC in denying the said motion for lack of
merit. WHEREFORE, the petition is DENIED. The Order dated October 8, 2011 of the Regional
Trial Court of Pinamalayan, Oriental Mindoro in Civil Case No. SCA-07-11 (Criminal Case No.
2012) is hereby AFFIRMED and UPHELD. With costs against the petitioner. SO ORDERED.

G.R. No. 169588

October 7, 2013

JADEWELL PARKING SYSTEMS CORPORATION represented by its manager and


authorized representative Norma Tan, Petitioner,
vs.
HON. JUDGE NELSON F. LIDUA SR., Presiding Judge of The Municipal Trial Court
Branch 3, Baguio City, BENEDICTO BALAJADIA, EDWIN ANG, "JOHN DOES" and
"PETER DOES" Respondents.
DECISION
LEONEN, J.:
We are asked to rule on this Petition for Review on Certiorari under Rule 45 of the
Rules of Court, praying that the assailed Decision of Branch 7 of the Regional Trial
Court of Baguio City and Order dated August 15, 2005 be reversed and that Criminal
Case Nos. 112934 and 112935 be ordered reinstated and prosecuted before the
Municipal Trial Court of Baguio City.
Petitioner Jadewell Parking Systems Corporation is a private parking operator duly
authorized to operate and manage the parking spaces in Baguio City pursuant to City
Ordinance 003-2000. It is also authorized under Section 13 of the City Ordinance to
render any motor vehicle immobile by placing its wheels in a clamp if the vehicle is
illegally parked.1
According to the Resolution of the Office of the Provincial Prosecutor, San Fernando
City, La Union, the facts leading to the filing of the Informations are the following:
Jadewell Parking Systems Corporation (Jadewell), thru [sic] its General Manager Norma
Tan and Jadewell personnel Januario S. Ulpindo and Renato B. Dulay alleged in their
affidavit-complaint that on May 17, 2003, the respondents in I.S No. 2003-1996 Edwin
Ang, Benedicto Balajadia and John Doe dismantled, took and carried away the clamp
attached to the left front wheel of a Mitsubishi Adventure with Plate No. WRK 624
owned by Edwin Ang. Accordingly, the car was then illegally parked and left unattended
at a Loading and Unloading Zone. The value of the clamp belonging to Jadewell which
was allegedly forcibly removed with a piece of metal isP26,250.00. The fines of P500.00
for illegal parking and the declamping fee of P500.00 were also not paid by the
respondents herein.
In I.S. No., 2003-1997, Jadewell thru [sic] its General Manager Norina C. Tan, Renato
B. Dulay and Ringo Sacliwan alleged in their affidavit-complaint that on May 7, 2003,
along Upper Mabini Street, Baguio City, herein respondents Benedicto Balajadia,
Jeffrey Walan and two (2) John Does forcibly removed the clamp on the wheel of a
Nissan Cefiro car with Plate No. UTD 933, belonging to Jeffrey Walan which was then
considered illegally parked for failure to pay the prescribed parking fee. Such car was
earlier rendered immobile by such clamp by Jadewell personnel. After forcibly removing

the clamp, respondents took and carried it away depriving its owner, Jadewell, its use
and value which is P26,250.00. According to complainants, the fine of P500.00 and the
declamping fee of P500.00 were not paid by the respondents.2
The incident resulted in two cases filed by petitioner and respondents against each
other. Petitioner Jadewell filed two cases against respondents: Robbery under I.S. Nos.
2003-1996 and 2003-1997. Petitioner filed an Affidavit-Complaint against respondents
Benedicto Balajadia, Jeffrey Walan, and three (3) John Does, one of whom was
eventually identified as respondent Ramon Ang. The Affidavit-Complaint was filed with
the Office of the City Prosecutor of Baguio City on May 23, 2003.3 A preliminary
investigation took place on May 28, 2003. Respondent Benedicto Balajadia likewise
filed a case charging Jadewell president, Rogelio Tan, and four (4) of Jadewell's
employees with Usurpation of Authority/Grave Coercion in I.S. No. 2003-1935.
In his Counter-affidavit for the two cases he filed for himself and on behalf of his corespondents, respondent Benedicto Balajadia denied that his car was parked illegally.
He admitted that he removed the clamp restricting the wheel of his car since he alleged
that the placing of a clamp on the wheel of the vehicle was an illegal act. He alleged
further that he removed the clamp not to steal it but to remove the vehicle from its clamp
so that he and his family could continue using the car. He also confirmed that he had
the clamp with him, and he intended to use it as a piece of evidence to support the
Complaint he filed against Jadewell.4
In the Resolution5 of the Office of the Provincial Prosecutor of San Fernando City, La
Union, Acting City Prosecutor Mario Anacleto Banez found probable cause to file a case
of Usurpation of Authority against the petitioner. Regarding the case of Robbery against
respondents, Prosecutor Banez stated that:
We find no probable cause to charge respondents in these two (2) cases for the felony
of Robbery. The elements of Robbery, specifically the intent to gain and force upon
things are absent in the instant cases, thereby negating the existence of the crime.
xxxx
We, however, respectfully submit that the acts of respondents in removing the wheel
clamps on the wheels of the cars involved in these cases and their failure to pay the
prescribed fees were in violation of Sec. 21 of Baguio City Ordinance No. 003-2000
which prescribes fines and penalties for violations of the provisions of such ordinance.
Certainly, they should not have put the law into their own hands. (Emphasis supplied)
WHEREFORE, premises considered, there is probable cause against all the
respondents, except Jeffrey Walan or Joseph Walan (who has been dragged into this
controversy only by virtue of the fact that he was still the registered owner of the Nissan
Cefiro car) for violation of Section 21 of City Ord. No. 003-2000 in both cases and we
hereby file the corresponding informations against them in Court.6

Prosecutor Banez issued this Resolution on July 25, 2003.


On October 2, 2003, two criminal Informations were filed with the Municipal Trial Court
of Baguio City dated July 25, 2003, stating:
That on May 17, 2003 at Baguio City and within the jurisdiction of this Honorable Court,
the above-named accused with unity of action and concerted design, did then and
there, with unity of action and concerted design, willfully, unlawfully and feloniously
forcibly dismantled [sic] and took [sic] an immobilizing clamp then attached to the left
front wheel of a Mitsubishi Adventure vehicle with Plate No. WRK 624 belonging to
Edwin Ang which was earlier rendered immobilized by such clamp by Jadewell
Personnel's for violation of the Baguio City ordinance No. 003-2600 to the damage and
prejudice of private complainant Jadewell Parking System Corporation (Jadewell) which
owns such clamp worth P26,250.00 and other consequential damages.
CONTRARY TO LAW,
San Fernando City, La Union for Baguio City, this 25th day of July 2003. 7
The cases were docketed as Criminal Case Nos. 112934 and 112935 with the
Municipal Trial Court of Baguio City, Branch 3. Respondent Benedicto Balajadia and the
other accused through their counsel Paterno Aquino filed a January 20, 2004 Motion to
Quash and/or Manifestation8 on February 2, 2004. The Motion to Quash and/or
Manifestation sought the quashal of the two Informations on the following grounds:
extinguishment of criminal action or liability due to prescription; failure of the Information
to state facts that charged an offense; and the imposition of charges on respondents
with more than one offense.
In their Motion to Quash, respondents argued that:
1. The accused in this case are charged with violation of Baguio City Ordinance
No. 003-2000.
2. Article 89 of the Revised Penal [sic] provides that criminal liability is totally
extinguished by prescription of the crime.
3. Act No. 3326, as amended by Act No. 3763, provides: "Section 1. x x x
Violations penalized by municipal ordinances shall prescribed [sic] after two
months."
4. As alleged in the Information, the offense charged in this case was committed
on May 7, 2003. 5. As can be seen from the right hand corner of the Information,
the latter was filed with this Honorable Court on October 2, 2003, almost five (5)
months after the alleged commission of the offense charged. Hence, criminal
liability of the accused in this case, if any, was already extinguished by
prescription when the Information was filed.9

In an Order10 dated February 10, 2004, respondent Judge Nelson F. Lidua, Sr.,
Presiding Judge of the Municipal Trial Court of Baguio City, Branch 3, granted the
accused's Motion to Quash and dismissed the cases.
Petitioner filed a Motion for Reconsideration on February 27, 2004 responding to the
February 10, 2004 Order11to argue among other points that:
6.b. For another, the offenses charged have not yet prescribed. Under the law, the
period of prescription of offenses shall be interrupted by the filing of the complaint or
information. While it may be true that the Informations in these cases have been filed
only on October 2, 2003, the private complainant has, however, filed its criminal
complaint on May 23, 2003, well within the prescribed period.12
Respondents filed their Opposition13 on March 24, 2004, and petitioner filed a
Reply14 on April 1, 2004.
The respondent judge released a Resolution15 dated April 16, 2004 upholding the Order
granting respondents' Motion to Quash. The Resolution held that:
For the guidance of the parties, the Court will make an extended resolution on one of
the ground [sic] for the motion to quash, which is that the criminal action has been
extinguished on grounds of prescription.
These offenses are covered by the Rules on Summary Procedure being alleged
violations of City Ordinances.
Under Section 9 of the Rule [sic] on Summary Procedure, the running of the prescriptive
period shall be halted on the date the case is filed in Court and not on any date before
that (Zaldivia vs. Reyes, Jr. G.R. No. 102342, July 3, 1992, En Banc).
In case of conflict, the Rule on Summary Procedure as the special law prevails over
Sec. 1 of Rule 110 of the Rules on Criminal Procedure and also Rule 110 of the Rules
of Criminal Procedure must yield to Act No. 3326 or "AN ACT TO ESTABLISH
PERIODS OF PRESCRIPTION FOR VIOLATIONS PENALIZED BY SPECIAL ACTS
AND MUNICIPAL ORDINANCES AND TO PROVIDE WHEN PRESCRIPTION SHALL
BEGIN TO RUN" (Ibid).
Petitioner then filed a Petition16 for Certiorari under Rule 65 with the Regional Trial
Court of Baguio City. The case was raffled to Branch 7 of the Regional Trial Court of
Baguio City. Petitioners contended that the respondent judge committed grave abuse of
discretion amounting to lack or excess of jurisdiction in dismissing Criminal Case Nos.
112934 and 112935 on the ground of prescription. Petitioners argued that the
respondent judge ruled erroneously saying that the prescriptive period for the offenses
charged against the private respondents was halted by the filing of the
Complaint/Information in court and not when the Affidavit-Complaints were filed with the

Office of the City Prosecutor of Baguio City. Petitioner cited Section 1 of Rule 110 of the
Rules on Criminal Procedure:
x x x "criminal actions shall be instituted x x x in x x x other chartered cities, the
complaint shall be filed with the office of the prosecutor unless otherwise provided in
their charter" and the last paragraph thereof states that "the institution of the criminal
action shall interrupt the running of the period of prescription of the offense charged
unless otherwise provided in special laws."17
Petitioner contended further that:
the filing of the criminal complaint with the Office of the City Prosecutor of Baguio City,
not the filing of the criminal information before this Honorable Court, is the reckoning
point in determining whether or not the criminal action in these cases had prescribed.
xxxx
The offenses charged in Criminal Case Nos. 112934 and 112935 are covered by the
Revised Rules on Summary Procedure, not by the old Rules on Summary Procedure.
Considering that the offenses charged are for violations of a City Ordinance, the
criminal cases can only be commenced by informations. Thus, it was only legally and
procedurally proper for the petitioner to file its complaint with the Office of the City
Prosecutor of Baguio City as required by Section 11 of the new Rules on Summary
Procedure, these criminal cases "shall be commenced only by information." These
criminal cases cannot be commenced in any other way.
Moreover, the ruling of the Supreme Court in Zaldivia vs. Reyes cited in the assailed
Resolution does not apply in this case. The offense charged in Zaldivia is a violation of
municipal ordinance in which case, the complaint should have been filed directly in court
as required by Section 9 of the old Rules on Summary Procedure. On the other hand,
Criminal Case Nos. 112934 and 112935 are for violations of a city ordinance and as
aforestated, "shall be commenced only by information." 18
Thus, petitioner contended that the filing of the criminal complaint with the Office of the
City Prosecutor stopped the running of the two-month prescriptive period. Hence, the
offenses charged have not prescribed.
In their Comment,19 respondents maintained that the respondent judge did not gravely
abuse his discretion. They held that Section 2 of Act No. 3326, as amended, provides
that:
Sec. 2. Prescription shall begin to run from the day of the commission of the violation of
the law, and if the same be not known at the time, from the discovery thereof and the
institution of judicial proceeding for its investigation and punishment.

The prescription shall be interrupted when proceedings are instituted against the guilty
person, and shall begin to run again if the proceedings are dismissed for reasons not
constituting jeopardy.20 (Emphasis supplied)
Respondents argued that Zaldivia v. Reyes21 held that the proceedings mentioned in
Section 2 of Act No. 3326, as amended, refer to judicial proceedings . Thus, this Court,
in Zaldivia, held that the filing of the Complaint with the Office of the Provincial
Prosecutor was not a judicial proceeding. The prescriptive period commenced from the
alleged date of the commission of the crime on May 7, 2003 and ended two months
after on July 7, 2003. Since the Informations were filed with the Municipal Trial Court on
October 2, 2003, the respondent judge did not abuse its discretion in dismissing
Criminal Case Nos. 112934 and 112935.
In a Decision dated April 20, 2005, the Regional Trial Court of Baguio City Branch 7,
through Judge Clarence F. Villanueva, dismissed the Petition for Certiorari. The
Regional Trial Court held that, since cases of city ordinance violations may only be
commenced by the filing of an Information, then the two-month prescription period may
only be interrupted by the filing of Informations (for violation of City Ordinance 0032000) against the respondents in court. The Regional Trial Court of Baguio City, Branch
7, ruled in favor of the respondents and upheld the respondent judges Order dated
February 10, 2004 and the Resolution dated April 16, 2004.
Petitioners then filed a May 17, 2005 Motion for Reconsideration which was denied by
the Regional Trial Court in an August 15, 2005 Order.
Hence, this Petition.
The principal question in this case is whether the filing of the Complaint with the Office
of the City Prosecutor on May 23, 2003 tolled the prescription period of the commission
of the offense charged against respondents Balajadia, Ang, "John Does," and "Peter
Does."
Petitioner contends that the prescription period of the offense in Act No. 3326, as
amended by Act No. 3763, does not apply because respondents were charged with the
violation of a city ordinance and not a municipal ordinance. In any case, assuming
arguendo that the prescriptive period is indeed two months, filing a Complaint with the
Office of the City Prosecutor tolled the prescription period of two months. This is
because Rule 110 of the Rules of Court provides that, in Manila and in other chartered
cities, the Complaint shall be filed with the Office of the Prosecutor unless otherwise
provided in their charters.
In their Comment,22 respondents maintain that respondent Judge Lidua did not err in
dismissing the cases based on prescription. Also, respondents raise that the other
grounds for dismissal they raised in their Motion to Quash, namely, that the facts
charged constituted no offense and that respondents were charged with more than one
offense, were sustained by the Metropolitan Trial Court. Also, respondents argue that

petitioner had no legal personality to assail the Orders, since Jadewell was not assailing
the civil liability of the case but the assailed Order and Resolution. This was contrary to
the ruling in People v. Judge Santiago23 which held that the private complainant may
only appeal the civil aspect of the criminal offense and not the crime itself.
In the Reply,24 petitioner argues that the respondent judge only dismissed the case on
the ground of prescription, since the Resolution dated April 16, 2004 only cited that
ground. The Order dated February 10, 2004 merely stated but did not specify the
grounds on which the cases were dismissed. Petitioner also maintains that the
proceedings contemplated in Section 2 of Act No. 3326 must include the preliminary
investigation proceedings before the National Prosecution Service in light of the Rules
on Criminal Procedure25 and Revised Rules on Summary Procedure.
Lastly, petitioner maintains that it did have legal personality, since in a Petition for
Certiorari, "persons aggrieved x x x may file a verified petition"26 before the court.
The Petition is denied.
The resolution of this case requires an examination of both the substantive law and the
procedural rules governing the prosecution of the offense. With regard to the
prescription period, Act No. 3326, as amended, is the only statute that provides for any
prescriptive period for the violation of special laws and municipal ordinances. No other
special law provides any other prescriptive period, and the law does not provide any
other distinction. Petitioner may not argue that Act No. 3326 as amended does not
apply.
In Romualdez v. Hon. Marcelo,27 this Court defined the parameters of prescription:
In resolving the issue of prescription of the offense charged, the following should be
considered: (1) the period of prescription for the offense charged; (2) the time the period
of prescription starts to run; and (3) the time the prescriptive period was
interrupted.28 (Citation omitted)
With regard to the period of prescription, it is now without question that it is two months
for the offense charged under City Ordinance 003-2000.
The commencement of the prescription period is also governed by statute. Article 91 of
the Revised Penal Code reads:
Art. 91. Computation of prescription of offenses. The period of prescription shall
commence to run from the day on which the crime is discovered by the offended party,
the authorities, or their agents, and shall be interrupted by the filing of the complaint or
information, and shall commence to run again when such proceedings terminate without
the accused being convicted or acquitted, or are unjustifiably stopped for any reason not
imputable to him.

The offense was committed on May 7, 2003 and was discovered by the attendants of
the petitioner on the same day. These actions effectively commenced the running of the
prescription period.
The procedural rules that govern this case are the 1991 Revised Rules on Summary
Procedure.
SECTION 1. Scope This rule shall govern the summary procedure in the Metropolitan
Trial Courts, the Municipal Trial Courts in Cities, the Municipal Trial Courts, and the
Municipal Circuit Trial Courts in the following cases falling within their jurisdiction:
xxxx
B. Criminal Cases:
(1) Violations of traffic laws, rules and regulations;
(2) Violations of the rental law;
(3) Violations of municipal or city ordinances (Emphasis supplied)
Section 11 of the Rules provides that:
Sec. 11. How commenced. The filing of criminal cases falling within the scope of this
Rule shall be either by complaint or by information: Provided, however, that in
Metropolitan Manila and in Chartered Cities, such cases shall be commenced only by
information, except when the offense cannot be prosecuted de officio.
The Local Government Code provides for the classification of cities. Section 451 reads:
SEC. 451. Cities, Classified. A city may either be component or highly urbanized:
Provided, however, that the criteria established in this Code shall not affect the
classification and corporate status of existing cities. Independent component cities are
those component cities whose charters prohibit their voters from voting for provincial
elective officials. Independent component cities shall be independent of the province.
Cities in the Philippines that were created by law can either be highly urbanized cities or
component cities. An independent component city has a charter that proscribes its
voters from voting for provincial elective officials. It stands that all cities as defined by
Congress are chartered cities. In cases as early as United States v. Pascual
Pacis,29 this Court recognized the validity of the Baguio Incorporation Act or Act No.
1963 of 1909, otherwise known as the charter of Baguio City.
As provided in the Revised Rules on Summary Procedure, only the filing of an
Information tolls the prescriptive period where the crime charged is involved in an

ordinance. The respondent judge was correct when he applied the rule in Zaldivia v.
Reyes.
In Zaldivia v. Reyes, the violation of a municipal ordinance in Rodriguez, Rizal also
featured similar facts and issues with the present case. In that case, the offense was
committed on May 11, 1990. The Complaint was received on May 30, 1990, and the
Information was filed with the Metropolitan Trial Court of Rodriguez on October 2, 1990.
This Court ruled that:
As it is clearly provided in the Rule on Summary Procedure that among the offenses it
covers are violations of municipal or city ordinances, it should follow that the charge
against the petitioner, which is for violation of a municipal ordinance of Rodriguez, is
governed by that rule and not Section 1 of Rule 110.
Where paragraph (b) of the section does speak of "offenses falling under the jurisdiction
of the Municipal Trial Courts and Municipal Circuit Trial Courts," the obvious reference is
to Section 32(2) of B.P. No. 129, vesting in such courts:
(2) Exclusive original jurisdiction over all offenses punishable with imprisonment of not
exceeding four years and two months, or a fine of not more than four thousand pesos,
or both such fine and imprisonment, regardless of other imposable accessory or other
penalties, including the civil liability arising from such offenses or predicated thereon,
irrespective of kind, nature, value, or amount thereof; Provided, however, That in
offenses involving damage to property through criminal negligence they shall have
exclusive original jurisdiction where the imposable fine does not exceed twenty
thousand pesos.
These offenses are not covered by the Rules on Summary Procedure.
Under Section 9 of the Rules on Summary Procedure, "the complaint or information
shall be filed directly in court without need of a prior preliminary examination or
preliminary investigation." Both parties agree that this provision does not prevent the
prosecutor from conducting a preliminary investigation if he wants to. However, the case
shall be deemed commenced only when it is filed in court, whether or not the
prosecution decides to conduct a preliminary investigation. This means that the running
of the prescriptive period shall be halted on the date the case is actually filed in court
and not on any date before that.
This interpretation is in consonance with the afore-quoted Act No. 3326 which says that
the period of prescription shall be suspended "when proceedings are instituted against
the guilty party." The proceedings referred to in Section 2 thereof are "judicial
proceedings," contrary to the submission of the Solicitor General that they include
administrative proceedings. His contention is that we must not distinguish as the law
does not distinguish. As a matter of fact, it does.

At any rate, the Court feels that if there be a conflict between the Rule on Summary
Procedure and Section 1 of Rule 110 of the Rules on Criminal Procedure, the former
should prevail as the special law. And if there be a conflict between Act No. 3326 and
Rule 110 of the Rules on Criminal Procedure, the latter must again yield because this
Court, in the exercise of its rule-making power, is not allowed to "diminish, increase or
modify substantive rights" under Article VIII, Section 5(5) of the Constitution.
Prescription in criminal cases is a substantive right.30
Jurisprudence exists showing that when the Complaint is filed with the Office of the
Prosecutor who then files the Information in court, this already has the effect of tolling
the prescription period. The recent People v. Pangilinan31categorically stated that
Zaldivia v. Reyes is not controlling as far as special laws are concerned. Pangilinan
referred to other cases that upheld this principle as well. However, the doctrine of
Pangilinan pertains to violations of special laws but not to ordinances.
There is no distinction between the filing of the Information contemplated in the Rules of
Criminal Procedure and in the Rules of Summary Procedure. When the representatives
of the petitioner filed the Complaint before the Provincial Prosecutor of Baguio, the
prescription period was running. It continued to run until the filing of the Information.
They had two months to file the Information and institute the judicial proceedings by
filing the Information with the Municipal Trial Court. The conduct of the preliminary
investigation, the original charge of Robbery, and the subsequent finding of the violation
of the ordinance did not alter the period within which to file the Information.
Respondents were correct in arguing that the petitioner only had two months from the
discovery and commission of the offense before it prescribed within which to file the
Information with the Municipal Trial Court.
Unfortunately, when the Office of the Prosecutor filed the Informations on October 5,
2003, the period had already prescribed. Thus, respondent Judge Nestor Lidua, Sr. did
not err when he ordered the dismissal of the case against respondents. According to the
Department of Justice National Prosecutors Service Manual for Prosecutors, an
Information is defined under Part I, Section 5 as:
SEC. 5. Information. - An information is the accusation in writing charging a person with
an offense, subscribed by the prosecutor, and filed with the court. The information need
not be placed under oath by the prosecutor signing the same.
The prosecutor must, however, certify under oath that
a) he has examined the complainant and his witnesses;
b) there is reasonable ground to believe that a crime has been committed and
that the accused is probably guilty thereof;
c) the accused was informed of the complaint and of the evidence submitted
against him; and

d) the accused was given an opportunity to submit controverting evidence.


As for the place of the filing of the Information, the Manual also provides that:
SEC. 12. Place of the commission of offense. - The complaint or information is sufficient
if it states that the crime charged was committed or some of the ingredients thereof
occurred at some place within the jurisdiction of the court, unless the particular place in
which the crime was committed is an essential element of the crime, e.g. in a
prosecution for violation of the provision of the Election Code which punishes the
carrying of a deadly weapon in a "polling place," or if it is necessary to identify the
offense charged, e.g., the domicile in the offense of "violation of domicile."
Finally, as for the prescription period, the Manual provides that:
SEC. 20. How Period of Prescription Computed and Interrupted. - For an offense
penalized under the Revised Penal Code, the period of prescription commences to run
from the day on which the crime is discovered by the offended party, the authorities, or
their agents, and shall be interrupted:
a) by the filing of the complaint with the Office of the City/Provincial Prosecutor;
or with the Office of the Ombudsman; or
b) by the filing of the complaint or information with the court even if it is merely for
purposes of preliminary examination or investigation, or even if the court where
the complaint or information is filed cannot try the case on its merits.
However, for an offense covered by the Rules on Summary Procedure, the period of
prescription is interrupted only by the filing of the complaint or information in court.
xxxx
For violation of a special law or ordinance, the period of prescription shall commence to
run from the day of the commission of the violation, and if the same is not known at the
time, from the discovery and the institution of judicial proceedings for its investigation
and punishment. The prescription shall be interrupted only by the filing of the complaint
or information in court and shall begin to run again if the proceedings are dismissed for
reasons not constituting double jeopardy. (Emphasis supplied).1wphi1
Presidential Decree No. 127532 reorganized the Department of Justices Prosecution
Staff and established Regional State Prosecution Offices. These Regional State
Prosecution Offices were assigned centers for particular regions where the Informations
will be filed. Section 6 provides that the area of responsibility of the Region 1 Center
located in San Fernando, La Union includes Abra, Benguet, Ilocos Norte, Ilocos Sur, La
Union, Mt. Province, Pangasinan, and the cities of Baguio, Dagupan, Laoag, and San
Carlos.

The Regional Prosecutor for Region 1 or his/her duly assigned prosecutor was
designated to file the Information within the two-month period provided for in Act No.
3326, as amended.1wphi1
The failure of the prosecutor to seasonably file the Information is unfortunate as it
resulted in the dismissal of the case against the private respondents. It stands that the
doctrine of Zaldivia is applicable to ordinances and their prescription period. It also
upholds the necessity of filing the Information in court in order to toll the period. Zaldivia
also has this to say concerning the effects of its ruling:
The Court realizes that under the above interpretation, a crime may prescribe even if
the complaint is filed seasonably with the prosecutor's office if, intentionally or not, he
delays the institution of the necessary judicial proceedings until it is too late. However,
that possibility should not justify a misreading of the applicable rules beyond their
obvious intent as reasonably deduced from their plain language.
The remedy is not a distortion of the meaning of the rules but a rewording thereof to
prevent the problem here sought to be corrected.33
WHEREFORE the Petition is DENIED.
SO ORDERED.

REPUBLIC OF THE PHILIPPINES, G.R. No. 139930


Petitioner,
Present:
CARPIO,
VELASCO, JR.,
LEONARDO-DE CASTRO,
BRION,
- versus - PERALTA,
BERSAMIN,
DEL CASTILLO,
ABAD,
VILLARAMA, JR.,
PEREZ,
MENDOZA,
SERENO,
REYES, and
PERLAS-BERNABE, JJ.
Respondents. Promulgated:
June 26, 2012
x --------------------------------------------------------------------------------------- x
DECISION
ABAD, J.:

This case, which involves another attempt of the government to recover ill-gotten wealth
acquired during the Marcos era, resolves the issue of prescription.
The Facts and the Case
On April 25, 1977 respondents Teodoro D. Regala, Victor P. Lazatin, Eleazar B. Reyes, Eduardo
U. Escueta and Leo J. Palma incorporated the United Coconut Oil Mills, Inc. (UNICOM)[1] with
an authorized capital stock of P100 million divided into one million shares with a par value of P100
per share. The incorporators subscribed to 200,000 shares worth P20 million and paid P5 million.
On September 26, 1978 UNICOM amended its capitalization by (1) increasing its authorized
capital stock to three million shares without par value; (2) converting the original subscription of
200,000 to one million shares without par value and deemed fully paid for and non-assessable by
applying the P5 million already paid; and (3) waiving and abandoning the subscription receivables
of P15 million.[2]
On August 29, 1979 the Board of Directors of the United Coconut Planters Bank (UCPB)
composed of respondents Eduardo M. Cojuangco, Jr., Juan Ponce Enrile, Maria Clara L. Lobregat,

Jose R. Eleazar, Jr., Jose C. Concepcion, Rolando P. Dela Cuesta, Emmanuel M. Almeda,
Hermenegildo C. Zayco, Narciso M. Pineda, Iaki R. Mendezona, and Danilo S. Ursua approved
Resolution 247-79 authorizing UCPB, the Administrator of the Coconut Industry Investment Fund
(CII Fund), to invest not more than P500 million from the fund in the equity of UNICOM for the
benefit of the coconut farmers.[3]
On September 4, 1979 UNICOM increased its authorized capital stock to 10 million shares without
par value. The Certificate of Increase of Capital Stock stated that the incorporators held one million
shares without par value and that UCPB subscribed to 4 million shares worth P495 million.[4]
On September 18, 1979 a new set of UNICOM directors, composed of respondents Eduardo M.
Cojuangco, Jr., Juan Ponce Enrile, Maria Clara L. Lobregat, Jose R. Eleazar, Jr., Jose Concepcion,
Emmanuel M. Almeda, Iaki R. Mendezona, Teodoro D. Regala, Douglas Lu Ym, Sigfredo Veloso,
and Jaime Gandiaga, approved another amendment to UNICOMs capitalization. This increased its
authorized capital stock to one billion shares divided into 500 million Class A voting common
shares, 400 million Class B voting common shares, and 100 million Class C non-voting common
shares, all with a par value of P1 per share. The paid-up subscriptions of 5 million shares without
par value (consisting of one million shares for the incorporators and 4 million shares for UCPB)
were then converted to 500 million Class A voting common shares at the ratio of 100 Class A
voting common shares for every one without par value share.[5]
About 10 years later or on March 1, 1990 the Office of the Solicitor General (OSG) filed a
complaint for violation of Section 3(e) of Republic Act (R.A.) 3019[6] against respondents, the
1979 members of the UCPB board of directors, before the Presidential Commission on Good
Government (PCGG). The OSG alleged that UCPBs investment in UNICOM was manifestly and
grossly disadvantageous to the government since UNICOM had a capitalization of only P5 million
and it had no track record of operation. In the process of conversion to voting common shares, the
governments P495 million investment was reduced by P95 million which was credited to
UNICOMs incorporators. The PCGG subsequently referred the complaint to the Office of the
Ombudsman in OMB-0-90-2810 in line with the ruling in Cojuangco, Jr. v. Presidential
Commission on Good Government,[7] which disqualified the PCGG from conducting the
preliminary investigation in the case.
About nine years later or on March 15, 1999 the Office of the Special Prosecutor (OSP) issued a
Memorandum,[8] stating that although it found sufficient basis to indict respondents for violation
of Section 3(e) of R.A. 3019, the action has already prescribed. Respondents amended UNICOMs
capitalization a third time on September 18, 1979, giving the incorporators unwarranted benefits
by increasing their 1 million shares to 100 million shares without cost to them. But, since
UNICOM filed its Certificate of Filing of Amended Articles of Incorporation with the Securities
and Exchange Commission (SEC) on February 8, 1980, making public respondents acts as board
of directors, the period of prescription began to run at that time and ended on February 8,
1990. Thus, the crime already prescribed when the OSG filed the complaint with the PCGG for
preliminary investigation on March 1, 1990.

In a Memorandum[9] dated May 14, 1999, the Office of the Ombudsman approved the OSPs
recommendation for dismissal of the complaint. It additionally ruled that UCPBs subscription to
the shares of stock of UNICOM on September 18, 1979 was the proper point at which the
prescription of the action began to run since respondents act of investing into UNICOM was
consummated on that date. It could not be said that the investment was a continuing act. The giving
of undue benefit to the incorporators prescribed 10 years later on September 18, 1989. Notably,
when the crime was committed in 1979 the prescriptive period for it had not yet been amended. The
original provision of Section 11 of R.A. 3019 provided for prescription of 10 years. Thus, the OSG
filed its complaint out of time.
The OSG filed a motion for reconsideration on the Office of the Ombudsmans action but the latter
denied the same;[10] hence, this petition.
Meanwhile, the Court ordered the dismissal of the case against respondent Maria Clara L. Lobregat
in view of her death on January 2, 2004.[11]
The Issue Presented
The pivotal issue in this case is whether or not respondents alleged violation of Section
3(e) of R.A. 3019 already prescribed.
The Courts Ruling
Preliminarily, the Court notes that what Republic of the Philippines (petitioner) filed in this
case is a petition for review on certiorari under Rule 45. But the remedy from an adverse
resolution of the Office of the Ombudsman in a preliminary investigation is a special civil action
of certiorari under Rule 65.[12] Still, the Court will treat this petition as one filed under Rule 65
since a reading of its contents reveals that petitioner imputes grave abuse of discretion and
reversible jurisdictional error to the Ombudsman for dismissing the complaint. The Court has
previously treated differently labeled actions as special civil actions for certiorari under Rule 65
for acceptable reasons such as justice, equity, and fair play.[13]
As to the main issue, petitioner maintains that, although the charge against respondents was
for violation of the Anti-Graft and Corrupt Practices Act, its prosecution relates to its efforts to
recover the ill-gotten wealth of former President Ferdinand Marcos and of his family and
cronies. Section 15, Article XI of the 1987 Constitution provides that the right of the State to
recover properties unlawfully acquired by public officials or employees is not barred by
prescription, laches, or estoppel.
But the Court has already settled in Presidential Ad Hoc Fact-Finding Committee on
Behest Loans v. Desierto[14] that Section 15, Article XI of the 1987 Constitution applies only to
civil actions for recovery of ill-gotten wealth, not to criminal cases such as the complaint against

respondents in OMB-0-90-2810. Thus, the prosecution of offenses arising from, relating or


incident to, or involving ill-gotten wealth contemplated in Section 15, Article XI of the 1987
Constitution may be barred by prescription.[15]
Notably, Section 11 of R.A. 3019 now provides that the offenses committed under that law
prescribes in 15 years. Prior to its amendment by Batas Pambansa (B.P.) Blg. 195 on March 16,
1982, however, the prescriptive period for offenses punishable under R.A. 3019 was only 10
years.[16] Since the acts complained of were committed before the enactment of B.P. 195, the
prescriptive period for such acts is 10 years as provided in Section 11 of R.A. 3019, as originally
enacted.[17]
Now R.A. 3019 being a special law, the 10-year prescriptive period should be computed in
accordance with Section 2 of Act 3326,[18] which provides:
Section 2. Prescription shall begin to run from the day of the
commission of the violation of the law, and if the same be not known at the
time, from the discovery thereof and the institution of judicial proceedings for
its investigation and punishment.
The above-mentioned section provides two rules for determining when the prescriptive
period shall begin to run: first, from the day of the commission of the violation of the law, if such
commission is known; and second, from its discovery, if not then known, and the institution of
judicial proceedings for its investigation and punishment.[19]
Petitioner points out that, assuming the offense charged is subject to prescription, the same
began to run only from the date it was discovered, namely, after the 1986 EDSA Revolution. Thus,
the charge could be filed as late as 1996.
In the prosecution of cases of behest loans, the Court reckoned the prescriptive period from
the discovery of such loans. The reason for this is that the government, as aggrieved party, could
not have known that those loans existed when they were made. Both parties to such loans
supposedly conspired to perpetrate fraud against the government.They could only have been
discovered after the 1986 EDSA Revolution when the people ousted President Marcos from
office. And, prior to that date, no person would have dared question the legality or propriety of the
loans.[20]
Those circumstances do not obtain in this case. For one thing, what is questioned here is
not the grant of behest loans that, by their nature, could be concealed from the public eye by the
simple expedient of suppressing their documentations. What is rather involved here is UCPBs
investment in UNICOM, which corporation is allegedly owned by respondent Cojuangco,
supposedly a Marcos crony. That investment does not, however, appear to have been withheld
from the curious or from those who were minded to know like banks or competing

businesses. Indeed, the OSG made no allegation that respondent members of the board of directors
of UCPB connived with UNICOM to suppress public knowledge of the investment.
Besides, the transaction left the confines of the UCPB and UNICOM board rooms when
UNICOM applied with the SEC, the publicly-accessible government clearing house for increases
in corporate capitalization, to accommodate UCPBs investment. Changes in shareholdings are
reflected in the General Information Sheets that corporations have been mandated to submit
annually to the SEC. These are available to anyone upon request.
The OSG makes no allegation that the SEC denied public access to UCPBs investment in
UNICOM during martial law at the Presidents or anyone elses instance. Indeed, no accusation of
this kind has ever been hurled at the SEC with reference to corporate transactions of whatever kind
during martial law since even that regime had a stake in keeping intact the integrity of the SEC as
an instrumentality of investments in the Philippines.
And, granted that the feint-hearted might not have the courage to question the UCPB
investment into UNICOM during martial law, the second elementthat the action could not have
been instituted during the 10-year period because of martial lawdoes not apply to this case. The
last day for filing the action was, at the latest, on February 8, 1990, about four years after martial
law ended. Petitioner had known of the investment it now questions for a sufficiently long time
yet it let those four years of the remaining period of prescription run its course before bringing the
proper action.
Prescription of actions is a valued rule in all civilized states from the beginning of
organized society. It is a rule of fairness since, without it, the plaintiff can postpone the filing of
his action to the point of depriving the defendant, through the passage of time, of access to defense
witnesses who would have died or left to live elsewhere, or to documents that would have been
discarded or could no longer be located. Moreover, the memories of witnesses are eroded by
time. There is an absolute need in the interest of fairness to bar actions that have taken the plaintiffs
too long to file in court.
Respondents claim that, in any event, the complaint against them failed to show probable
cause. They point out that, prior to the third amendment of UNICOMs capitalization, the stated
value of the one million shares without par value, which belonged to its incorporators, was P5
million. When these shares were converted to 5 million shares with par value, the total par value
of such shares remained at P5 million. But, the action having prescribed, there is no point in
discussing the existence of probable cause against the respondents for violation of Section 3(e) of
R.A. 3019.
WHEREFORE, the Court DENIES the petition and AFFIRMS the Memorandum dated
May 14, 1999 of the Office of the Ombudsman that dismissed on the ground of prescription the
subject charge of violation of Section 3(e) of R.A. 3019 against respondents Eduardo M.
Cojuangco, Jr., Juan Ponce Enrile, Jose R. Eleazar, Jr., Jose C. Concepcion, Rolando P. Dela

Cuesta, Emmanuel M. Almeda, Hermenegildo C. Zayco, Narciso M. Pineda, Iaki R. Mendezona,


Danilo S. Ursua, Teodoro D. Regala, Victor P. Lazatin, Eleazar B. Reyes, Eduardo U. Escueta,
Leo J. Palma, Douglas Lu Ym, Sigfredo Veloso, and Jaime Gandiaga.
SO ORDERED.

SANRIO COMPANY G.R. No. 168662


LIMITED,
Petitioner, Present:
PUNO, C.J., Chairperson,
SANDOVAL-GUTIERREZ,
- v e r s u s - CORONA,
AZCUNA and
LEONARDO-DE CASTRO, JJ.
EDGAR C. LIM, doing
business as ORIGNAMURA
TRADING, Promulgated:
Respondent.
February 19, 2008
x---------------------------------------------------x
DECISION
CORONA, J.:

This petition for review on certiorari[1] seeks to set aside the decision of the Court of Appeals (CA)
in CA-G.R. CV No. 74660[2] and its resolution[3] denying reconsideration.
Petitioner Sanrio Company Limited, a Japanese corporation, owns the copyright of various
animated characters such as Hello Kitty, Little Twin Stars, My Melody, Tuxedo Sam and
Zashikibuta among others.[4] While it is not engaged in business in the Philippines, its products are
sold locally by its exclusive distributor, Gift Gate Incorporated (GGI).[5]
As such exclusive distributor, GGI entered into licensing agreements with JC Lucas Creative
Products, Inc., Paper Line Graphics, Inc. and Melawares Manufacturing Corporation.[6] These
local entities were allowed to manufacture certain products (bearing petitioner's copyrighted
animated characters) for the local market.
Sometime in 2001, due to the deluge of counterfeit Sanrio products, GGI asked IP Manila
Associates (IPMA) to conduct a market research. The research's objective was to identify those

factories, department stores and retail outlets manufacturing and/or selling fake Sanrio
items.[7] After conducting several test-buys in various commercial areas, IPMA confirmed that
respondent's Orignamura Trading in Tutuban Center, Manila was selling imitations of petitioner's
products.[8]
Consequently, on May 29, 2000, IPMA agents Lea A. Carmona and Arnel P. Dausan executed a
joint affidavit attesting to the aforementioned facts.[9] IPMA forwarded the said affidavit to the
National Bureau of Investigation (NBI) which thereafter filed an application for the issuance of a
search warrant in the office of the Executive Judge of the Regional Trial Court of Manila.[10]
After conducting the requisite searching inquiry, the executive judge issued a search warrant on
May 30, 2000.[11] On the same day, agents of the NBI searched the premises of Orignamura
Trading. As a result thereof, they were able to seize various Sanrio products.[12]
On April 4, 2002, petitioner, through its attorney-in-fact Teodoro Y. Kalaw IV of the Quisumbing
Torres law firm, filed a complaint-affidavit[13] with the Task-Force on Anti-Intellectual Property
Piracy (TAPP) of the Department of Justice (DOJ) against respondent for violation of Section 217
(in relation to Sections 177[14] and 178[15]) of the Intellectual Property Code (IPC) which states:
Section 217. Criminal Penalties. 217.1. Any person infringing any right secured
by provisions of Part IV of this Act or aiding or abetting such infringement shall
be guilty of a crime punishable by:
(a) Imprisonment of one (1) year to three (3) years plus a fine ranging from Fifty
thousand pesos (P50,000) to One hundred fifty thousand pesos (P150,000) for the
first offense.
(b) Imprisonment of three (3) years and one (1) day to six (6) years plus a fine
ranging from One hundred fifty thousand pesos (P150,000) to Five hundred
thousand pesos (P500,000) for the second offense.
(c) Imprisonment of six (6) years and one (1) day to nine (9) years plus a fine
ranging from Five hundred thousand pesos (P500,000) to One million five
hundred thousand pesos (P1,500,000) for the third and subsequent offenses.
(d) In all cases, subsidiary imprisonment in cases of insolvency.

217.2. In determining the number of years of imprisonment and the amount of


fine, the court shall consider the value of the infringing materials that the defendant
has produced or manufactured and the damage that the copyright owner has
suffered by reason of infringement.
217.3. Any person who at the time when copyright subsists in a work has in
his possession an article which he knows, or ought to know, to be an infringing
copy of the work for the purpose of:
(a) Selling, letting for hire, or by way of trade offering or exposing for sale, or
hire, the article;
(b) Distributing the article for purpose of trade or any other purpose to an
extent that will prejudice the rights of the copyright of the owner in the work;
or
(c) Trade exhibit of the article in public, shall be guilty of an offense and shall be
liable on conviction to imprisonment and fine as above mentioned. (emphasis
supplied)

Respondent asserted in his counter-affidavit[16] that he committed no violation of the provisions of


the IPC because he was only a retailer.[17] Respondent neither reproduced nor manufactured any
of petitioner's copyrighted item; thus, he did not transgress the economic rights of
petitioner.[18] Moreover, he obtained his merchandise from authorized manufacturers of
petitioner's products.[19]
On September 25, 2002, the TAPP found that:
Evidence on record would show that respondent bought his merchandise from
legitimate sources, as shown by official receipts issued by JC Lucas Creative
Products, Inc., Paper Line Graphics, Inc. and Melawares Manufacturing
Corporation. In fact, in her letter dated May 23, 2002, Ms. Ma. Angela S. Garcia
certified that JC Lucas Creative Products, Inc., Paper Line Graphics, Inc. and
Melawares Manufacturing Corporation are authorized to produce certain Sanrio
products. While it appears that some of the items seized during the search are
not among those products which [GGI] authorized these establishments to
produce, the fact remains that respondent bought these from the abovecited
legitimate sources. At this juncture, it bears stressing that respondent relied on
the representations of these manufacturers and distributors that the items
they sold were genuine. As such, it is not incumbent upon respondent to verify
from these sources what items [GGI] only authorized them to produce. Thus, as

far as respondent is concerned, the items in his possession are not infringing
copies of the original [petitioner's] products. (emphasis supplied)[20]

Thus, in a resolution dated September 25, 2002, it dismissed the complaint due to insufficiency of
evidence.[21]
Petitioner moved for reconsideration but it was denied.[22] Hence, it filed a petition for review in
the Office of the Chief State Prosecutor of the DOJ.[23] In a resolution dated August 29,
2003,[24] the Office of the Chief State Prosecutor affirmed the TAPP resolution. The petition was
dismissed for lack of reversible error.
Aggrieved, petitioner filed a petition for certiorari in the CA. On May 3, 2005, the appellate
court dismissed the petition on the ground of prescription. It based its action on Act 3326 which
states:
Section 1. Violations penalized by special acts shall, unless otherwise
provided in such acts, prescribe in accordance with the following rules: (a) after a
year for offenses punished only by a fine or by imprisonment for not more than
one month, or both; (b) after four years for those punished by imprisonment
for more than one month, but less than two years; (c) after eight years for
those punished by imprisonment for two years or more, but less than six
years; and (d) after twelve years for any other offense punished by imprisonment
for six years or more, except the crime of treason, which shall prescribe after
twenty years; Provided, however, That all offenses against any law or part of law
administered by the Bureau of Internal Revenue shall prescribe after five years.
Violations penalized by municipal ordinances shall prescribe after two months.
Section 2. Prescription shall begin to run from the day of the commission of
the violation of the law, and if the same may not be known at the time, from the
discovery thereof and the institution of judicial proceedings for its investigation
and punishment.
The prescription shall be interrupted when proceedings are instituted against
the guilty person, and shall begin to run again if the proceedings are dismissed
for reasons not constituting jeopardy. (emphasis supplied)

According to the CA, because no complaint was filed in court within two years after the
commission of the alleged violation, the offense had already prescribed.[25]

On the merits of the case, the CA concluded that the DOJ did not commit grave abuse of discretion
in dismissing the petition for review.[26] To be criminally liable for violation of Section 217.3 of
the IPC, the following requisites must be present:
1.
possession of the infringing copy and
2.

knowledge or suspicion that the copy is an infringement of the genuine


article.

The CA agreed with the DOJ that petitioner failed to prove that respondent knew that the
merchandise he sold was counterfeit. Respondent, on the other hand, was able to show that he
obtained these goods from legitimate sources.[27]
Petitioner moved for reconsideration but it was denied. Hence, this petition.
Petitioner now essentially avers that the CA erred in concluding that the alleged violations of the
IPC had prescribed. Recent jurisprudence holds that the pendency of a preliminary investigation
suspends the running of the prescriptive period.[28] Moreover, the CA erred in finding that the DOJ
did not commit grave abuse of discretion in dismissing the complaint. Respondent is liable for
copyright infringement (even if he obtained his merchandise from legitimate sources) because he
sold counterfeit goods.[29]
Although we do not agree wholly with the CA, we deny the petition.

FILING OF THE COMPLAINT IN THE DOJ


TOLLED THE PRESCRIPTIVE PERIOD

Section 2 of Act 3326 provides that the prescriptive period for violation of special laws starts on
the day such offense was committed and is interrupted by the institution of proceedings against
respondent (i.e., the accused).

Petitioner in this instance filed its complaint-affidavit on April 4, 2002 or one year, ten
months and four days after the NBI searched respondent's premises and seized Sanrio merchandise
therefrom. Although no information was immediately filed in court, respondent's alleged violation
had not yet prescribed.[30]
In the recent case of Brillantes v. Court of Appeals,[31] we affirmed that the filing of the complaint
for purposes of preliminary investigation interrupts the period of prescription of criminal
responsibility.[32] Thus, the prescriptive period for the prosecution of the alleged violation of the
IPC was tolled by petitioner's timely filing of the complaint-affidavit before the TAPP.
IN THE ABSENCE OF GRAVE ABUSE OF
DISCRETION, THE FACTUAL FINDINGS OF
THE
DOJ
IN
PRELIMINARY
INVESTIGATIONS
WILL
NOT
BE
DISTURBED
In a preliminary investigation, a public prosecutor determines whether a crime has been committed
and whether there is probable cause that the accused is guilty thereof.[33]Probable cause is defined
as such facts and circumstances that will engender a well-founded belief that a crime has been
committed and that the respondent is probably guilty thereof and should be held for
trial.[34] Because a public prosecutor is the one conducting a preliminary investigation, he
determines the existence of probable cause.[35]Consequently, the decision to file a criminal
information in court or to dismiss a complaint depends on his sound discretion.[36]
As a general rule, a public prosecutor is afforded a wide latitude of discretion in the conduct of a
preliminary investigation. For this reason, courts generally do not interfere with the results of such
proceedings. A prosecutor alone determines the sufficiency of evidence that will establish probable
cause justifying the filing of a criminal information against the respondent.[37] By way of
exception, however, judicial review is allowed where respondent has clearly established that the
prosecutor committed grave abuse of discretion.[38] Otherwise stated, such review is appropriate

only when the prosecutor has exercised his discretion in an arbitrary, capricious, whimsical or
despotic manner by reason of passion or personal hostility, patent and gross enough to amount to
an evasion of a positive duty or virtual refusal to perform a duty enjoined by law.[39]
The prosecutors in this case consistently found that no probable cause existed against respondent
for violation of the IPC. They were in the best position to determine whether or not there was
probable cause. We find that they arrived at their findings after carefully evaluating the respective
evidence of petitioner and respondent. Their conclusion was not tainted with grave abuse of
discretion.
WHEREFORE, the petition is hereby DENIED.
Costs against petitioner.
SO ORDERED.

G.R. No. L-42925 January 31, 1977


PEOPLE OF THE PHILIPPINES, petitioner,
vs.
HON. RICARDO D. GALANO, Presiding Judge, Court of First Instance of Manila,
Branch XIII, and GREGORIO SANTOS, respondents.
Acting Solicitor General Hugo E. Gutierrez, Jr., Assistant Solicitor General Hugo
Nathanael P. de Pano, Jr. and Trial Attorney Blesila O. Quintillan for petitioner.
Juanito M. Romano for respondent.

TEEHANKEE, J:
The Court sets aside the respondent judge's orders dismissing the information for estafa
against respondent accused, since the offense charged clearly has not prescribed. The
complaint filed with the Batangas court which expressly alleged commission of the
offense within the municipality and which pended for twelve years (the accused having
jumped bail and evaded rearrest for nine years) and which was eventually dismissed by
said court for lack of territorial jurisdiction as a result of the proof adduced before it
properly interrupted and tolled the prescription period. Respondent judge failed, in ruling
otherwise, to apply the settled rule that the jurisdiction of a court is determined in
criminal cases by the allegations of the complaint or information and not by the result of
proof. The case is ordered remanded for determination with the utmost dispatch, since
this case has already been pending for fifteen years owing to respondent accused's
deplorable tactics. The undisputed factual background of the case is succinctly stated
by then Acting Solicitor General, now Associate Justice of the Court of Appeals, Hugo
E. Gutierrez, Jr., thus:
1. On October 2,1962, a criminal complaint for estafa was filed in the
municipal court of Batangas, Batangas (now City Court of Batangas City)
against the accused-respondent Gregorio Santos by complainant, Juanito
Limbo, ...
2. Gregorio Santos was arrested to answer for the above charge, and
upon his arrest, posted a bail bond for his provisional liberty. The accused
was thereafter arraigned and he pleaded not guilty to the charge. Then,
the case was heard on its merits. However, on September 16, 1964, the
accused jumped bail. As a result, his bail bond was forfeited and the case
against him archived by the municipal court of Batangas, Batangas.
3. It was not until September 14, 1973, about nine years later, when the
accused was re-arrested, and the trial of the said case resumed.

4. On October 21, 1974, while the said case was pending trial, private
respondent Gregorio Santos filed a motion to dismiss the case on the
ground that the Batangas court did not have territorial jurisdiction over the
case, the evidence showing that the crime was committed in Manila.
5. Finding the motion meritorious, the Batangas City Court issued an order
dated November 5, 1974, dismissing the case against Gregorio Santos for
lack of territorial jurisdiction over the crime charged ...
6. On November 14, 1974, the complainant Juanito B. Limbo refiled the
same case against Gregorio Santos in the Fiscal's Office of Manila. A
preliminary investigation was conducted. On July 29, 1975, the
corresponding information was filed with the Court of First Instance of
Manila, docketed as Criminal Case No. 22397, ...
7. On November 12, 1975 the accused Gregorio Santos filed a motion to
dismiss criminal Case No. 22397 on the grounds
of prescription and double jeopardy.
8. The prosecuting fiscal filed his opposition to said motion on December
2, 1975, to which the accused filed a rejoinder on December 5, 1975.
9. On December 8, 1975, the Court of First Instance of Manila, Branch
XIII, presided over by the Honorable Ricardo D. Galano, issued an order
dismissing Criminal Case No. 22397 on the ground that the offense
charged had already prescribed, ... The prosecution moved for the
reconsideration of said order but this was denied by the lower court by
order of January 7, 1976. ...
10. From the said Order of dismissal, the City Fiscal of Manila offenses
provides: interposed an appeal by certiorari to this Honorable Court on
January 24, 1976. On March 3, 1976, this honorable Court issued the
Resolution of March 3, 1976 requiring the Solicitor General to file the
petition for review within fifteen days from receipt thereof ...
The People avers in the petition 1 that respondent judge "dismissing criminal Case No.
22397 despite the provisions of Article l of the Revised Penal Code, which clearly
indicate that the offense charged has not prescribed" and "in not considering the
prevailing jurisprudence indicating non-prescription of the offense charged, and in
holding that the case ofPeople v. Olarte, 19 SCRA 494, does not apply to the case at
bar."
The petition is patently meritorious and must be granted.
I. The offense of estafa for which respondent accused stands charged clearly
has not prescribed.

Art. 91. Computation of prescription of offenses. The period of


prescription shall commence to run from the day on which the discovered
by the offended party, the authorities, or by their agents, and shall be
interrupted by the filing of the complaint or information and shall
commence to run again when the proceedings terminate without the
accused being convicted or acquitted or are unjustifiably stopped for any
reason not imputable to him. ...
The offense was committed on or about September 16, 1962 when respondent failed to
account for and instead misappropriated to his own use the sum of P8,704.00
representing the net proceeds (minus his commission) of 272 booklets of sweepstakes
tickets that had been entrusted to him be the complainant, who promptly filed on
October 2, 1962 plainly within the ten-year prescriptive period the criminal complaint
against respondent accused in the Municipal Court of Batangas, Batangas. The
prescriptive period was thereupon interrupted.
After his plea of not guilty and during the trial, respondent accused jumped bail in
September, 1964 and evaded rearrest for nine years until September, 1973 and the trial
was resumed. When the Batangas court in its Order of November 5, 1974 upon
respondent's motion dismissed the complaint "for lack of jurisdiction" since
the evidence(of both prosecution and accused) showed that all elements of the crime
were committed in Manila (and not in Batangas), 2 the proceedings therein terminated
without conviction or acquittal of respondent accused and it was only then that the
prescriptive period (which was interrupted during the pendency of the case in the
Batangas court) commenced to run again.
When the City Fiscal of Manila upon complainant's instance refiled on July 29, 1975 the
same case against respondent accused in the Manila court of first instance, (after
having conducted a preliminary investigation), it is clear that not even a year of the tenyear prescriptive period had been consumed.
Respondent accused intent on thwarting his prosecution filed anew a motion to dismiss
the information on grounds of prescription and double jeopardy. There is manifestly no
jeopardy, because he was not acquitted by the Batangas court which on the basis of the
evidence could neither convict him because it was thereby shown to have no jurisdiction
over the offense.
But respondent judge gravely erred in sustaining the ground of prescription, ruling that
there was no interruption of the prescriptive period during the pendency of the case in
the Batangas court because "(T)he proceedings contemplated by Article 91 are
proceedings which are valid and before a competent court. If they are void from the
beginning because the court has no territorial jurisdiction of the offense charged, it is as
if no proceedings were held thereat. If this is so, then the warrant or order of arrest as
well as the bail given by the accused for his provisional liberty is of no effect. Inevitably,
there can be no jumping bail to speak of and there are no proceedings to be
interrupted."

This is plain error for "Settled is the rule ... that the jurisdiction of a court is determined in
criminal cases by theallegations of the complaint or information and not by the result
of proof." 4
It follows clearly that the Batangas court was vested with lawful jurisdiction over the
criminal complaint filed with it which expressly alleged that the offense was committed
"in the Municipality of Batangas, province of Batangas" and that the proceedings therein
were valid and before a competent court, (including the arrest warrant, the grant of bail
and forfeiture thereof upon the accused's jumping of bail), until the same court issued its
November. 1974 order dismissing the Case and declaring itself
without territorial jurisdiction on the basis of the evidence presented to it by both
prosecution and the accused.
It follows just as clearly that the prescriptive period was interrupted and tolled during the
12-year pendency of the proceedings before the Batangas Court (for nine years of
which respondent accused had jumped bail and evaded re-arrest).
II. Respondent judge gravely erred in dismissing the information on the ground of
prescription and disregarding the controlling case of People vs. Olarte. 5
In the second People vs. Olarte case, 6 the Court clarified precisely for the guidance of
bench and bar that the true doctrine is that the filing of the compliant in the municipal
court, even if it be merely for purposes of preliminary investigation (where the offense
charged is beyond its jurisdiction to try the case on the merits) should, and does
interrupt the period of prescription, as follows:
Analysis of the precedents on the issue of prescription discloses that there
are two lines of decisions following differing criteria in determining whether
prescription of crimes has been interrupted. One line of precedents holds
that the filing of the complaint with the justice of the peace (or municipal
judge) does interrupt the course of the prescriptive term: (People vs.
Olarte, L-131027, June 30, 1960 and cases cited therein; People vs. Uba,
L-13106, October 16, 1959; People vs. Aquino, 68 Phil. 588, 590.)
Another series of decisions declares that to produce interruption the
complaint or information must have been filed in the proper court that has
jurisdiction to try the case on its merits:People vs. Del Rosario, L-15140,
December 29, 1960; People vs. Coquia, L-15456, June 29, 1963.
In view of this diversity of precedents, and in order to provide guidance for
Bench and Bar, this Court has reexamined the question and after mature
consideration has arrived at the conclusion that thetrue doctrine is, and
should be, the one established by the decision holding that the filing of the
complaint in the Municipal Court, even if it be merely for purposes of
preliminary examination or investigation, should and does, interrupt the
period of prescription of the criminal responsibility, even if the court where
the complaint or information is filed can not try the case on its merits.

Several reasons buttress this conclusion: first, the text of Article 91 of the
Revised Penal Code, in declaring that the period of prescription 'shall be
interrupted by the filing of the complaint or information' without
distinguishing whether the complaint is filed in the court for preliminary
examination or investigation merely, or for action on the merits. Second,
even if the court where the complaint or information is filed may only
proceed to investigate the case, its actuation already represents the initial
step of the proceedings against the offender. Third, it is unjust to deprive
the injured party of the right to obtain vindication on account of delays that
are not under his control. All that the victim of the offense may do on his
part to initiate the prosecution is to file the requisite complaint. 7
Respondent judge in his dismissal order correctly cited the rationale for statutory
prescriptions, inter alia, that "the delay in instituting the proceedings not only causes
expenses to the State, but exposes public justice to peril, for it weakens oral evidence
due to the lapse of the natural period of duration of memory if not to anything, else. And
it is the policy of the law that prosecutions should be prompt and that statutes enforcing
that promptitude should be maintained, they being not merely acts of grace, but checks
imposed by the State upon its subalterns, to exact vigilant activity and to secure for
criminal trials the best evidence that can be obtained. 8
But respondent judge fell into grave error in not applying the controlling case
of Olarte on his misconception that there had been no valid complaint filed with a
competent court in Batangas contrary to what has already been held hereinabove that
the express allegations of the complaint that the offense was committed in Batangas
vested the Batangas court with lawful jurisdiction until its dismissal order twelve years
later for lack of jurisdiction as a result of the proof presented before it during the tiral
(and in not taking into account that the delay was not at all due to the State but to
respondent accused himself who jumped bail and escaped tile law for nine [9] years and
who apparently has made no effort all this time to make good the amount the to
complainant or any part thereof).
Since the record with transcript of the testimonial evidence in the Batangas court is
complete (and shows that the trial was continued on August 2, 1974 to September 10,
1974 while respondent accused was testifying on the witness stand but that he instead
filed his motion to dismiss of October 14, 1974 which granted by the Batangas court for
lack of territorial jurisdiction) and this case had already been pending for almost 15
years, all the evidence already taken by the Batangas court as recorded in the minutes
and transcript shall be deemed reproduced upon remand of the case to the Manila court
which is hereby ordered to receive only the remaining evidence of the respondent
accused and such rebuttal evidence as the parties may have and thereafter resolve the
case with the utmost dispatch.
ACCORDINGLY, respondent judge's dismissal orders of December 8, 1975 and
January 7, 1976 are hereby set aside, and the case is remanded to respondent judge or
whoever presides Branch XIII of the Manila court of first instance for continuation of the

trial (with reproduction of the evidence in the Batangas city court in Criminal Case No.
532 thereof, entitled "People vs. Gregorio Santos") in line with the directives in the
preceding paragraph. Respondent judge or the judge presiding his court is further
ordered to report to this Court the action taken hereon within a period of ninety (90)
days from promulgation of this decision. In view of the many years that the criminal case
has been pending, this decision is declared immediately executory upon promulgation.
SO ORDERED.

II. JURISDICTION
G.R. No. L-47448 May 17, 1978
THE PEOPLE OF THE PHILIPPINES, petitioner,
vs.
HON. EMETERIO C. OCAYA, as District Judge, 15th Judicial District, Branch VI,
Province of Bukidnon, and ESTERLINA MARAPAO, LETICIA MARAPAO and
DIOSDADO MARAPAO, respondents.
Arcadio D. Fabria and Camilo E. Tamin, Office of the Provincial Fiscal of Malaybalay,
Bukidnon for petitioner.
Eusebio P. Aquino for private respondents.

TEEHANKEE, J.:
The Court declares the questioned orders of respondent judge dismissing the
information for supposed lack of jurisdiction as null and void. Respondent judge
wrongfully dismissed the case before him in disregard to the elemental rule that
jurisdiction is determined by the allegations of the information and that the offense of
serious physical injuries charged in the information had duly vested his court with
jurisdiction. The Court orders the transfer of the case below to another branch of the
Bukidnon court of-first instance, since it is doubtful that the State and offended party
may expect a fair and impartial hearing and determination of the case from respondent
judge who with his erroneous pre-conceptions and predilections has adversely
prejudged their case for serious physical injuries as one merely of slight or less serious
physical injuries.
The office of the provincial fiscal of Bukidnon, after preliminary investigation filed an
information dated October 13, 1977 in the court of respondent judge, charging the three
private respondents- accused (Esterlina Marapao, Leticia Marapao and Diosdado
Marapao) for serious physical injuries committed as follows:
That on or about the 23rd day of July, 1977, in Don Carlos, Bukidnon,
Philippines and within the jurisdiction of this Honorable Court, the abovenamed accused, conspiring, confederating and mutually helping each
other, did then and there willfully unlawfully and feloneously attack, assault
and use personal violence upon one Mrs. LOLITA ARES, a mother who
was then still on the twelfth (12th) day from her child delivery, by then and
there wrestling her to the ground and thereafter throwing and hitting her
with a fist-size stone at the face thereby inflicting upon said Mrs. LOLITA
ARES:-

lacerated wound, transverse right at about 2.5 cm. x 0.5 cm. in width at
the level of the m arch of the face, with contusion and swelling all around
the inflicted area
which injury considerably deforms her face, and further causing upon said Mrs. LOLITA
ARES to suffer a relapse (nabughat in the local dialect) arising from her weak
constitution due to her recent child delivery, which relapse incapacitated her from
performing her customary labor for a period of more than thirty days.
Contrary to and in violation of Article 263, paragraph 3 of the Revised
Penal Code.
The records do not show that arraignment or trial on the merits has been held, much
less that warrants for the arrest of the accused had been issued. Instead, after
"scanning the records of (the) case" and noting that the thereto attached medical
certificate stated that the injuries suffered by the victim Lolita Ares would require
medical attention from 7 to 10 days and, therefore, 4 "may either be slight or less
serious physical injuries only" contrary to victim's affidavit that she was incapacitated
from her customary labor for more than 30 days and the fiscal's findings as to the
prominent sear left on the victim's face as a result "which considerably deforms her
face" (as duly alleged in the information), respondent judge motu proprio ordered the
dismissal of the case "as the crime of slight or less physical injury is not within the
jurisdiction of the court" as per his Order of October 27, 1977, stating as his reason that
The Court is of the opinion that what governs in the filing of a physical
injury case is the certificate issued by the physician regarding the duration
of treatment, and not what the victim declares because the same is selfserving.
The fiscal's motion for reconsideration proved futile with respondent judge in his Order
of November 16, 1977 denying the same, evaluating the case without having heard the
parties or their witnesses (particularly the physician who issued the medical certificate)
nor having received their evidence and ruling against the deformity alleged in the
information on the basis of his perception from a reading of the medical certificate and
the fiscal's written resolution finding proper basis for the filing of the information, that
Now, does the finding of the fiscal to the effect that he observed a big scar
at the left cheek bone of Mrs. Lolita Ares justify the filing of the charge of
serious physical injuries, under Article 263 of the Revised Penal Code,
when the attending physician certified that what he found was a lacerated
wound on the right side of the face? Clearly, the scar found by the
investigating fiscal could not be the result of the acts imputed to the
accused but for some other cause, for how could the scar be found on
the left side when the injury inflicted was on the right side? (Emphasis
supplied)

Hence, the petition at bar as filed by the provincial fiscal for nullification of respondent
judge's orders.
The Solicitor General in his comment has noted that there is ample legal and factual
basis for the information charging serious physical injuries, stating that "(T)hat the
allegations in the Information that a fist-size stone hit the face of Lolita Ares causing
lacerated wound on the maxillary arch of the face which considerably deformed her face
(are) not only supported by the medical certificate, but also by the admission of accused
Diosdado Marapao during the pre investigation that he threw a fist-size stone which hit
the face of Lolita Ares and the personal finding of Fiscal Tamin during the preliminary in.
investigation that there is a prominent scar on her face," and that the offense as
charged falls under Article 263, paragraph 3 of the Revised Penal Code which imposes
thereon a penalty of prision correccional in its minimum and medium periods and is
therefore properly cognizable by respondent judge's court.
The Court finds that respondent judge committed a grave abuse of discretion in
precipitately dismissing the case for alleged lack of jurisdiction on the mere basis of his
totally wrong notion that what governs in the filing of a physical injury case is the
medical certificate regarding the duration of treatment and "not what the victim declares
because the same is self-serving."
It is elemental that the jurisdiction of a court in criminal cases is determined by the
allegations of the information or criminal complaint and not by the result of the evidence
presented at the trial,' much less by the trial judge's personal appraisal of the affidavits
and exhibits attached by the fiscal to the record of the case without hearing the parties
and their witnesses nor receiving their evidence at a proper trial.
It is equally elementary that the mere fact that evidence presented at the trial would
indicate that a lesser offense outside the trial, 1 court's jurisdiction was committed does
not deprive the trial court of its jurisdiction which had vested in it under the allegations of
the information as filed since "(once) the jurisdiction attaches to the person and subject
matter of the litigation, the subsequent happening of events, although they are of such a
character as would have prevented jurisdiction from attaching in the first instance, will
not operate to oust jurisdiction already attached. 2
Indeed, the Solicitor General has aptly commented that "the dismissal of the case had
only resulted in duplication of work and wasted time in the remand of records when
respondent trial judge dismissed the instant case for want of jurisdiction, when it could
have immediately proceeded to arraign the accused and try him. "
Once more the Court is constrained to admonish the trial courts to proceed with proper
study and circumspection before summarily dismissing cases duly filed within their
court's cognizance and needlessly burdening the appellate courts with cases such as
that at bar which should not have reached us at all in the first instance. Respondent
judge's disregard of the established rule that the information for serious physical injuries
properly vested his court with jurisdiction to try and hear the case, and that if from the

evidence submitted a lesser offense was established, that he equally had jurisdiction to
impose the sentence for such lesser offense, is difficult of comprehension. Besides, the
doctor who issued the medical certificate had yet to be presented at the trial and
conceivably could corroborate the victim's testimony that her injuries had taken longer to
heal than had at first been estimated by him as well as clearify the location of he victim's
facial scar.
Respondent judge's actions and premature and baseless declaration that the victim's
declaration as to the period of her incapacity is "self-serving" raise serious doubts as to
whether the State and the offended party may expect a fair and impartial hearing and
determination of the case from him, since seemingly with his erroneous pre-conceptions
and predilections, he has adversely prejudged their case as one merely of slight or less
serious physical injuries. The case below should therefore be transferred to another
court presided by another judge.
ACCORDINGLY, the questioned orders of respondent judge are declared null and void.
The case below for serious physical injuries is remanded and ordered transferred to
Branch V of the court of first instance below, and the judge presiding the same is
ordered to issue the corresponding warrants of arrest and to proceed with dispatch with
the arraignment of the respondents-accused and the trial and determination of the case
on the merits. Let copy of this decision be attached to the personal record of respondent
judge. No pronouncement as to costs.
SO ORDERED.

G.R. No. 75256 January 26, 1989


JOHN PHILIP GUEVARRA, petitioner,
vs.
HONORABLE IGNACIO ALMODOVAR, respondent.
Teresita Dy-Liacco and Roberto Madrid for petitioner.

PARAS, J.:
Presented before Us is a special civil action for certiorari against the Honorable Judge
Ignacio Almodovar of the City Court of Legaspi, Branch 1, Legaspi City, raising beautiful
questions of law which We are tasked to resolve. Considering the issues and arguments
raised by petitioner, We impleaded the People of the Philippines as party respondents
herein in a resolution dated 17 September 1986 (p. 41, Rollo).
The relevant facts gathered from the records are as follows:
Petitioner John Philip Guevarra, then 11 years old, was playing with his best friend
Teodoro Almine, Jr. and three other children in their backyard in the morning of 29
October 1984. They were target-shooting a bottle cap (tansan) placed around fifteen
(15) to twenty (20) meters away with an air rifle borrowed from a neighbor. In the course
of their game, Teodoro was hit by a pellet on his left collar bone which caused his
unfortunate death.
After conduct a preliminary investigation, the examining Fiscal exculpated petitioner due
to his age and because the unfortunate occurrence appeared to be an accident. The
victim's parents appealed to the Ministry of Justice, which ordered the Fiscal to file a
case against petitioner for Homicide through reckless Imprudence. The information
dated 9 October 1985 was consequently filed, which narrated in part:
. . . the above-named accused, who is over 9 years but below 15 years of
age and acting with discernment, did then and there, without taking the
necessary precautions to prevent and/or avoid accident or injuries to
persons, willfully, unlawfully and feloniously operate and cause to be fired,
in a reckless and imprudent manner, an air rifle with .22 caliber bore with
rifling, oxygen and bolt operated thereby hitting as a result of said
carelessness and imprudence one TEODORICO PABLO ALMINE at the
left side of the body with its pellet, causing injuries which directly caused
his untimely death; . . . (p. 8, Rollo)
On 25 October 1985, petitioner moved to quash the said information on the following
grounds:

I
THAT THE FACTS CHARGED DO NOT CONSTITUTE OFFENSE.
II
THAT THE INFORMATION CONTAINS AVERMENTS WHICH IF TRUE
WOULD CONSTITUTE A LEGAL EXCUSE OR JUSTIFICATION.
III
THAT THIS HONORABLE COURT HAS NO JURISDICTION OVER THE
OFFENSE CHARGED AND THE PERSON OF THE DEFENDANT. (p. 9,
Rollo)
This motion, in an Order dated 4 April 1986, was denied with respect to the first and
third grounds relied upon. However, the resolution of the second ground was deferred
until evidence shall have been presented during trial.
On 26 July 1986, this present petition for certiorari was filed, raising two (2) issues, to
wit:
I
WHETHER AN ELEVEN (11) YEAR OLD BOY COULD BE CHARGED
WITH THE CRIME OF HOMICIDE THRU RECKLESS IMPRUDENCE,
AND
II
WHETHER THE COURT HAD JURISDICTION OVER THE CASE
NOTWITHSTANDING THE FACT THAT IT DID NOT PASS THRU THE
BARANGAY LUPON. (Petition, p. 3, Rollo)
Going through the written arguments of the parties, the surfacing of a corollary
controversy with respect to the first issue raised is evident, that is, whether the term
"discernment", as used in Article 12(3) of the Revised Penal Code (RPC) is
synonymous with "intent." It is the position of the petitioner that "discernment" connotes
'intent' (p. 96, Rollo), invoking the unreported case of People vs. Nieto, G.R. No. 11965,
30 April 1958. In that case We held that the allegation of "with intent to kill . . ." amply
meets the requirement that discernment should be alleged when the accused is a minor
between 9 and 15 years old. Petitioner completes his syllogism in saying that:
If discernment is the equivalent of 'with intent', then the allegation in the
information that the accused acted with discernment and willfully
unlawfully, and feloniously, operate or cause to be fired in a reckless and

imprudent manner an air rifle .22 caliber' is an inherent contradiction


tantamount to failure of the information to allege a cause of action or
constitute a legal excuse or exception. (Memorandum for Petitioner, p. 97,
Rollo)
If petitioner's argument is correct, then no minor between the ages of 9 and 15 may be
convicted of a quasi-offense under Article 265 of the RPC.
On the contrary, the Solicitor General insists that discernment and intent are two
different concepts. We agree with the Solicitor General's view; the two terms should not
be confused.
The word "intent" has been defined as
(a) design; a determination to do a certain things; an aim; the purpose of
the mind, including such knowledge as is essential to such intent;. . .; the
design resolve, or determination with which a person acts.' (46 CJS Intent
p. 1103.)
It is this intent which comprises the third element of dolo as a means of committing a
felony, freedom and intelligence being the other two. On the other hand, We have
defined the term discernment, as used in Article 12(3) of the RPC, in the old case
of People vs. Doquena, 68 Phil. 580(1939), in this wise:
The discernment that constitutes an exception to the exemption from
criminal liability of a minor under fifteen years of age but over nine, who
commits an act prohibited by law, is his mental capacity to understand the
difference between right and wrong . . . (Emphasis supplied) p. 583
From the foregoing, it is clear that the terms "intent" and "discernment" convey two
distinct thoughts. While both are products of the mental processes within a person, the
former refers to the desired of one's act while the latter relates to the moral significance
that person ascribes to the said act. Hence a person may not intend to shoot another
but may be aware of the consequences of his negligent act which may cause injury to
the same person in negligently handling an air rifle. It is not connect, therefore, to argue,
as petitioner does, that since a minor above nine years of age but below fifteen acted
with discernment, then he intended such act to be done. He may negligently shoot his
friend, thus did not intend to shoot him, and at the same time recognize the undesirable
result of his negligence.
In further outlining the distinction between the words "intent" and "discernment," it is
worthy to note the basic reason behind the enactment of the exempting circumstances
embodied in Article 12 of the RPC; the complete absence of intelligence, freedom of
action, or intent, or on the absence of negligence on the part of the accused. 1 In
expounding on intelligence as the second element of dolus, Albert 2 has stated:

The second element of dolus is intelligence; without this power, necessary


to determine the morality of human acts to distinguish a licit from an illicit
act, no crime can exist, and because ... the infant 3(has) no intelligence,
the law exempts (him) from criminal liability. (Emphasis supplied)
lt is for this reason, therefore, why minors nine years of age and below are not capable
of performing a criminal act. On the other hand, minors above nine years of appeal but
below fifteen are not absolutely exempt. However, they are presumed to be without
criminal capacity, but which presumption may be rebutted if it could be proven that they
were "capable of appreciating the nature and criminality of the act, that is, that (they)
acted with discernment. " 4 The preceding discussion shows that "intelligence" as an
element of dolo actually embraces the concept of discernment as used in Article 12 of
the RPC and as defined in the aforecited case of People vs. Doquena, supra. It could
not therefore be argued that discernment is equivalent or connotes 'intent' for they refer
to two different concepts. Intelligence, which includes discernment, is a distinct element
of dolo as a means of committing an offense.
In evaluating felonies committed by means of culpa, three (3) elements are
indispensable, namely, intelligence, freedom of action, and negligence. Obviously,
intent is wanting in such felonies. However, intelligence remains as an essential
element, hence, it is necessary that a minor above nine but below fifteen years of age
be possessed with intelligence in committing a negligent act which results in a quasioffense. For him to be criminally liable, he must discern the rightness or wrongness of
the effects of his negligent act. Indeed, a minor over nine years of age but below fifteen
may be held liable for a quasi-offense under Article 365 of the RPC. A reading of the
said Article would reveal such fact as it starts off with the phrase "Any person. . ."
without any distinction or exception made. Ubi lex non distinquit nec nos distinguere
debemos.
In his last attempt to justify his position equating the words "intent" and "discernment"
used under the law, he cites the case of People vs. Nieto, supra. However, petitioner
failed to present the qualifying sentence preceding the ruling he now invokes, which
reads:
That requirement should be deemed amply met with the allegation in the
information that she. . ."with the intent to kill, did then and there wilfully,
criminally and feloniously push one Lolita Padilla . . ." into a deep place of
the Pearanda River and as a consequence thereof Lolita Padilla got
drowned and died right then and there.' This allegation clearly conveys the
Idea that she knew what would be the consequence of her unlawful act of
pushing her victim into deep water and that she knew it to be
wrong. (Emphasis supplied)
From the above, it is clear that We did not mean to equate the words "intent" and
"discernment." What We meant was that the combined effect of the words used in the
information is to express a knowledge, on the part of the accused Nieto, of the

wrongness or rightness of her act. Hence, petitioner may not validly contend that since
the information now in question alleged "discernment", it in effect alleged "intent." The
former may never embrace the Idea of the latter; the former expresses the thought of
passivity while the latter signifies activity.
Coming now to the second issue of jurisdiction, it is contended by the petitioner that the
case against him should have first been brought before the Lupong Tagapayapa
pursuant to Presidential Decree No. 1508, Section 2(3). He submits that, considering
his entitlement to a two-degree privileged mitigating circumstance due to his minority,
P.D. 1508 applies to his case because the penalty imposable is reduced to not higher
than arresto menor from an original arresto mayor maximum to prision
correccional medium as prescribed in Article 365 of the RPC. This is not correct. The
jurisdiction of a court over a criminal case is determined by the penalty imposable under
the law for the offense and not the penalty ultimately imposed (People vs. Caldito, 72
Phil. 263; People vs. Purisima, 69 SCRA 314; Dioquino vs. Cruz and People vs.
Savellano, 116 SCRA 451). The same principle applies in construing Section 2(3) of
P.D. 1508, which states:
xxx xxx xxx
(3) Offense punishable by imprisonment exceeding 30 day , or a fine
exceeding P 200.00; ... (emphasis supplied)
Expounding on the above provision, a member of the committee that drafted P.D. 1508
has said:
The law says 'punishable,' not 'punished.' One should therefore consider
the penalty provided for by law or ordinance as distinguished from the
penalty actually imposed in particular cases after considering the
attendant circumstances affecting criminal liability. 5
The foregoing finds support in our jurisprudence as above cited. We therefore rule that,
in construing Section 2(3) of P.D. 1508, the penalty which the law defining the offense
attaches to the latter should be considered. Hence, any circumstance which may affect
criminal liability must not be considered.
The petitioner, in his arguments, asserts that since P.D. 1508 has not been complied
with, the trial court has no jurisdiction over the case. This erroneous perception has
been corrected long before. As intimated in the case ofRoyales vs. IAC, 127 SCRA 470,
and categorically stated in Ebol vs. Amin, 135 SCRA 438, P.D. 1508 is not jurisdictional.
WHEREFORE, PREMISES CONSIDERED, this petition is hereby DISMISSED for lack
of merit and the Temporary Restraining Order effective 17 September 1986 is LIFTED.
Let this case be REMANDED to the lower court for trial on the merits. No cost.
SO ORDERED.

G.R. No. 164007 August 10, 2006


LT. (SG) EUGENE GONZALES, LT. (SG) ANDY TORRATO, LT. (SG) ANTONIO
TRILLANES IV, CPT. GARY ALEJANO, LT. (SG) JAMES LAYUG, CPT. GERARDO
GAMBALA, CPT. NICANOR FAELDON, LT. (SG) MANUEL CABOCHAN, ENS.
ARMAND PONTEJOS, LT. (JG) ARTURO PASCUA, and 1LT. JONNEL
SANGGALANG, Petitioners,
vs.
GEN. NARCISO ABAYA, in his capacity as Chief of Staff of the Armed Forces of
the Philippines, and B. GEN. MARIANO M. SARMIENTO, JR., in his capacity as the
Judge Advocate General of the Judge Advocate Generals Office
(JAGO), Respondents.
DECISION
SANDOVAL-GUTIERREZ, J.:
For our resolution is the Petition for Prohibition (with prayer for a temporary restraining
order) filed by the above-named members of the Armed Forces of the Philippines
(AFP), herein petitioners, against the AFP Chief of Staff and the Judge Advocate
General, respondents.
The facts are:
On July 26, 2003, President Gloria Macapagal Arroyo received intelligence reports that
some members of the AFP, with high-powered weapons, had abandoned their
designated places of assignment. Their aim was to destabilize the government. The
President then directed the AFP and the Philippine National Police (PNP) to track and
arrest them.
On July 27, 2003 at around 1:00 a.m., more than 300 heavily armed junior officers and
enlisted men of the AFP mostly from the elite units of the Armys Scout Rangers and
the Navys Special Warfare Group entered the premises of the Oakwood Premier
Luxury Apartments on Ayala Avenue, Makati City. They disarmed the security guards
and planted explosive devices around the building.
Led by Navy Lt. (SG) Antonio Trillanes IV, the troops sported red armbands
emblazoned with the emblem of the"Magdalo" faction of the Katipunan. 1 The troops
then, through broadcast media, announced their grievances against the administration
of President Gloria Macapagal Arroyo, such as the graft and corruption in the military,
the illegal sale of arms and ammunition to the "enemies" of the State, and the bombings
in Davao City intended to acquire more military assistance from the US government.
They declared their withdrawal of support from their Commander-in-Chief and
demanded that she resign as President of the Republic. They also called for the
resignation of her cabinet members and the top brass of the AFP and PNP.

About noontime of the same day, President Arroyo issued Proclamation No. 427
declaring a state of rebellion, followed by General Order No. 4 directing the AFP and
PNP to take all necessary measures to suppress the rebellion then taking place in
Makati City. She then called the soldiers to surrender their weapons at five oclock in the
afternoon of that same day.
In order to avoid a bloody confrontation, the government sent negotiators to dialogue
with the soldiers. The aim was to persuade them to peacefully return to the fold of the
law. After several hours of negotiation, the government panel succeeded in convincing
them to lay down their arms and defuse the explosives placed around the premises of
the Oakwood Apartments. Eventually, they returned to their barracks.
A total of 321 soldiers, including petitioners herein, surrendered to the authorities.
The National Bureau of Investigation (NBI) investigated the incident and recommended
that the military personnel involved be charged with coup detat defined and penalized
under Article 134-A of the Revised Penal Code, as amended. On July 31, 2003, the
Chief State Prosecutor of the Department of Justice (DOJ) recommended the filing of
the corresponding Information against them.
Meanwhile, on August 2, 2003, pursuant to Article 70 of the Articles of War, respondent
General Narciso Abaya, then AFP Chief of Staff, ordered the arrest and detention of the
soldiers involved in the Oakwood incident and directed the AFP to conduct its own
separate investigation.
On August 5, 2003, the DOJ filed with the Regional Trial Court (RTC), Makati City an
Information for coup detat 2against those soldiers, docketed as Criminal Case No. 032784 and eventually raffled off to Branch 61, presided by Judge Romeo F.
Barza. 3 Subsequently, this case was consolidated with Criminal Case No. 03-2678,
involving the other accused, pending before Branch 148 of the RTC, Makati City,
presided by Judge Oscar B. Pimentel.
On August 13, 2003, the RTC directed the DOJ to conduct a reinvestigation of Criminal
Case No. 03-2784.
On the same date, respondent Chief of Staff issued Letter Order No. 625 creating a
Pre-Trial Investigation Panel tasked to determine the propriety of filing with the military
tribunal charges for violations of the Articles of War under Commonwealth Act No.
408, 4 as amended, against the same military personnel. Specifically, the charges are:
(a) violation of Article 63 for disrespect toward the President, the Secretary of National
Defense, etc., (b) violation of Article 64 for disrespect toward a superior officer, (c)
violation of Article 67 for mutiny or sedition, (d) violation of Article 96 for conduct
unbecoming an officer and a gentleman, and (e) violation of Article 97 for conduct
prejudicial to good order and military discipline.

Of the original 321 accused in Criminal Case No. 03-2784, only 243 (including
petitioners herein) filed with the RTC, Branch 148 an Omnibus Motion praying that the
said trial court assume jurisdiction over all the charges filed with the military tribunal.
They invoked Republic Act (R.A.) No. 7055. 5
On September 15, 2003, petitioners filed with the Judge Advocate Generals Office
(JAGO) a motion praying for the suspension of its proceedings until after the RTC shall
have resolved their motion to assume jurisdiction.
On October 29, 2003, the Pre-Trial Investigation Panel submitted its Initial Report to the
AFP Chief of Staff recommending that the military personnel involved in the Oakwood
incident be charged before a general court martial with violations of Articles 63, 64, 67,
96, and 97 of the Articles of War.
Meanwhile, on November 11, 2003, the DOJ, after conducting a reinvestigation, found
probable cause against only 31 (petitioners included) of the 321 accused in Criminal
Case No. 03-2784. Accordingly, the prosecution filed with the RTC an Amended
Information. 6
In an Order dated November 14, 2003, the RTC admitted the Amended Information and
dropped the charge ofcoup detat against the 290 accused.
Subsequently, or on December 12, 2003, the Pre-Trial Investigation Panel submitted its
Final Pre-Trial Investigation Report 7 to the JAGO, recommending that, following the
"doctrine of absorption," those charged withcoup detat before the RTCshould not be
charged before the military tribunal for violation of the Articles of War.
For its part, the RTC, on February 11, 2004, issued an Order 8 stating that "all charges
before the court martial against the accusedare hereby declared not serviceconnected, but rather absorbed and in furtherance of the alleged crime of coup detat."
The trial court then proceeded to hear petitioners applications for bail.
In the meantime, Colonel Julius A. Magno, in his capacity as officer-in-charge of the
JAGO, reviewed the findings of the Pre-Trial Investigation Panel. He recommended that
29 of the officers involved in the Oakwood incident, including petitioners, be prosecuted
before a general court martial for violation of Article 96 (conduct unbecoming an officer
and a gentleman) of the Articles of War.
On June 17, 2004, Colonel Magnos recommendation was approved by the AFP top
brass. The AFP Judge Advocate General then directed petitioners to submit their
answer to the charge. Instead of complying, they filed with this Court the instant Petition
for Prohibition praying that respondents be ordered to desist from charging them with
violation of Article 96 of the Articles of War in relation to the Oakwood incident. 9
Petitioners maintain that since the RTC has made a determination in its Order of
February 11, 2004 that the offense for violation of Article 96 (conduct unbecoming an

officer and a gentleman) of the Articles of War is not service-connected, but is absorbed
in the crime of coup detat, the military tribunal cannot compel them to submit to its
jurisdiction.
The Solicitor General, representing the respondents, counters that R.A. No. 7055
specifies which offenses covered by the Articles of War areservice-connected. These
are violations of Articles 54 to 70, 72 to 92, and 95 to 97. The law provides that
violations of these Articles are properly cognizable by the court martial. As the charge
against petitioners is violation of Article 96 which, under R.A. No. 7055 is a serviceconnected offense, then it falls under the jurisdiction of the court martial.
Subsequently, petitioners filed with this Court a Supplemental Petition raising the
additional issue that the offense charged before the General Court Martial has
prescribed. Petitioners alleged therein that during the pendency of their original petition,
respondents proceeded with the Pre-Trial Investigation for purposes of charging them
with violation of Article 96 (conduct unbecoming an officer and a gentleman) of the
Articles of War; that the Pre-Trial Investigation Panel then referred the case to the
General Court Martial; that "almost two years since the Oakwood incident on July 27,
2003, only petitioner Lt. (SG) Antonio Trillanes was arraigned, and this was done under
questionable circumstances;" 10 that in the hearing of July 26, 2005, herein petitioners
moved for the dismissal of the case on the ground that they were not arraigned within
the prescribed period of two (2) years from the date of the commission of the alleged
offense, in violation of Article 38 of the Articles of War; 11 that "the offense charged
prescribed on July 25, 2005;" 12 that the General Court Martial ruled, however, that "the
prescriptive period shall end only at 12:00 midnight of July 26, 2005;" 13 that "(a)s
midnight of July 26, 2005 was approaching and it was becoming apparent that the
accused could not be arraigned, the prosecution suddenly changed its position and
asserted that 23 of the accused have already been arraigned;" 14 and that petitioners
moved for a reconsideration but it was denied by the general court martial in its Order
dated September 14, 2005. 15
In his Comment, the Solicitor General prays that the Supplemental Petition be denied
for lack of merit. He alleges that "contrary to petitioners pretensions, all the accused
were duly arraigned on July 13 and 18, 2005." 16 The "(r)ecords show that in the hearing
on July 13, 2005, all the 29 accused were present" and, "(o)n that day, Military
Prosecutor Captain Karen Ong Jags read the Charges and Specifications from the
Charge Sheet in open court (pp. 64, TSN, July 13, 2005)." 17
The sole question for our resolution is whether the petitioners are entitled to the writ of
prohibition.
There is no dispute that petitioners, being officers of the AFP, are subject to military law.
Pursuant to Article 1 (a) of Commonwealth Act No. 408, as amended, otherwise known
as the Articles of War, the term "officer" is "construed to refer to a commissioned
officer." Article 2 provides:

Art. 2. Persons Subject to Military Law. The following persons are subject to these
articles and shall be understood as included in the term "any person subject to military
law" or "persons subject to military law," whenever used in these articles:
(a) All officers and soldiers in the active service of the Armed Forces of the
Philippines or of the Philippine Constabulary, all members of the reserve force, from
the dates of their call to active duty and while on such active duty; all trainees
undergoing military instructions; and all other persons lawfully called, drafted, or ordered
into, or to duty or for training in the said service, from the dates they are required by the
terms of the call, draft, or order to obey the same.
Upon the other hand, Section 1 of R.A. No. 7055 reads:
SEC. 1. Members of the Armed Forces of the Philippines and other persons subject to
military law, including members of the Citizens Armed Forces Geographical Units, who
commit crimes or offenses penalized under the Revised Penal Code, other special
penal laws, or local government ordinances, regardless of whether or not civilians are
co-accused, victims, or offended parties, which may be natural or juridical persons, shall
be tried by the proper civil court, except when the offense, as determined before
arraignment by the civil court, is service-connected, in which case, the offense shall be
tried by court-martial, Provided, That the President of the Philippines may, in the interest
of justice, order or direct at any time before arraignment that any such crimes or
offenses be tried by the proper civil courts.
As used in this Section, service-connected crimes or offenses shall be limited to those
defined in Articles 54 to 70, Articles 72 to 92, and Articles 95 to 97 of Commonwealth
Act No. 408, as amended.
In imposing the penalty for such crimes or offenses, the court-martial may take into
consideration the penalty prescribed therefor in the Revised Penal Code, other special
laws, or local government ordinances.
Section 1 of R.A. No. 7055, quoted above, is clear and unambiguous. First, it lays down
the general rule that members of the AFP and other persons subject to military law,
including members of the Citizens Armed Forces Geographical Units, who commit
crimes or offenses penalized under the Revised Penal Code (like coup detat), other
special penal laws, or local ordinances shall be tried by the proper civil court. Next, it
provides the exception to the general rule, i.e., where the civil court, before arraignment,
has determined the offense to be service-connected, then the offending soldier shall be
tried by a court martial. Lastly, the law states an exception to the exception, i.e., where
the President of the Philippines, in the interest of justice, directs before arraignment that
any such crimes or offenses be tried by the proper civil court.
The second paragraph of the same provision further identifies the "service-connected
crimes or offenses" as "limited to those defined in Articles 54 to 70, Articles 72 to 92,
and Articles 95 to 97" of the Articles of War. Violations of these specified Articles

are triable by court martial. This delineates the jurisdiction between the civil courts
and the court martial over crimes or offenses committed by military personnel.
Such delineation of jurisdiction by R.A. No. 7055 is necessary to preserve the peculiar
nature of military justice system over military personnel charged with service-connected
offenses. The military justice system is disciplinary in nature, aimed at achieving the
highest form of discipline in order to ensure the highest degree of military
efficiency. 18 Military law is established not merely to enforce discipline in times of war,
but also to preserve the tranquility and security of the State in time of peace; for there is
nothing more dangerous to the public peace and safety than a licentious and
undisciplined military body. 19 The administration of military justice has been universally
practiced. Since time immemorial, all the armies in almost all countries of the world look
upon the power of military law and its administration as the most effective means of
enforcing discipline. For this reason, the court martial has become invariably an
indispensable part of any organized armed forces, it being the most potent agency in
enforcing discipline both in peace and in war. 20
Here, petitioners are charged for violation of Article 96 (conduct unbecoming an officer
and a gentleman) of the Articles of War before the court martial, thus:
All persons subject to military law, did on or about 27 July 2003 at Oakwood Hotel,
Makati City, Metro Manila, willfully, unlawfully and feloniously violate their solemn oath
as officers to defend the Constitution, the law and the duly-constituted authorities
and abused their constitutional duty to protect the people and the State by, among
others, attempting to oust the incumbent duly-elected and legitimate President by force
and violence, seriously disturbing the peace and tranquility of the people and the nation
they are sworn to protect,thereby causing dishonor and disrespect to the military
profession, conduct unbecoming an officer and a gentleman, in violation of AW 96
of the Articles of War.
CONTRARY TO LAW. (Underscoring ours)
Article 96 of the Articles of War 21 provides:
ART. 96. Conduct Unbecoming an Officer and Gentleman. Any officer, member of the
Nurse Corps, cadet, flying cadet, or probationary second lieutenant, who is convicted of
conduct unbecoming an officer and a gentleman shall be dismissed from the service.
(Underscoring ours)
We hold that the offense for violation of Article 96 of the Articles of War is serviceconnected. This is expressly provided in Section 1 (second paragraph) of R.A. No.
7055. It bears stressing that the charge against the petitioners concerns the
alleged violation of their solemn oath as officers to defend the Constitution and the
duly-constituted authorities.Such violation allegedly caused dishonor and disrespect
to the military profession. In short, the charge has a bearing on
their professional conduct or behavior as military officers. Equally indicative of the

"service-connected" nature of the offense is the penalty prescribed for the same
dismissal from the service imposable only by the military court.Such penalty
is purely disciplinary in character, evidently intended to cleanse the military profession
of misfits and to preserve the stringent standard of military discipline.
Obviously, there is no merit in petitioners argument that they can no longer be charged
before the court martial for violation of Article 96 of the Articles of War because the
same has been declared by the RTC in its Order of February 11, 2004 as "not serviceconnected, but rather absorbed and in furtherance of the alleged crime of coup detat,"
hence, triable by said court (RTC). The RTC, in making such declaration, practically
amended the law which expressly vests in the court martial the jurisdiction over
"service-connected crimes or offenses." What the law has conferred the court should
not take away. It is only the Constitution or the law that bestows jurisdiction on the court,
tribunal, body or officer over the subject matter or nature of an action which can do
so. 22 And it is only through a constitutional amendment or legislative enactment that
such act can be done. The first and fundamental duty of the courts is merely to apply
the law "as they find it, not as they like it to be." 23 Evidently, such declaration by the
RTC constitutes grave abuse of discretion tantamount to lack or excess of jurisdiction
and is, therefore, void.
In Navales v. Abaya., 24 this Court, through Mr. Justice Romeo J. Callejo, Sr., held:
We agree with the respondents that the sweeping declaration made by the RTC
(Branch 148) in the dispositive portion of its Order dated February 11, 2004 that all
charges before the court-martial against the accused were not service-connected, but
absorbed and in furtherance of the crime of coup detat, cannot be given effect. x x x,
such declaration was made without or in excess of jurisdiction; hence, a nullity.
The second paragraph of the above provision (referring to Section 1 of R.A. No. 7055)
explicitly specifies what are considered "service-connected crimes or offenses" under
Commonwealth Act No. 408, as amended, also known as the Articles of War, to wit:
Articles 54 to 70:
Art. 54. Fraudulent Enlistment.
Art. 55. Officer Making Unlawful Enlistment.
Art. 56. False Muster.
Art. 57. False Returns.
Art. 58. Certain Acts to Constitute Desertion.
Art. 59. Desertion.

Art. 60. Advising or Aiding Another to Desert.


Art. 61. Entertaining a Deserter.
Art. 62. Absence Without Leave.
Art. 63. Disrespect Toward the President, Vice-President,
Congress of the Philippines, or Secretary of National
Defense.
Art. 64. Disrespect Toward Superior Officer.
Art. 65. Assaulting or Willfully Disobeying Superior Officer.
Art. 66. Insubordinate Conduct Toward Non-Commissioned Officer.
Art. 67. Mutiny or Sedition.
Art. 68. Failure to Suppress Mutiny or Sedition.
Art. 69. Quarrels; Frays; Disorders.
Art. 70. Arrest or Confinement.
Articles 72 to 92:
Art. 72. Refusal to Receive and Keep Prisoners.
Art. 73. Report of Prisoners Received.
Art. 74. Releasing Prisoner Without Authority.
Art. 75. Delivery of Offenders to Civil Authorities.
Art. 76. Misbehavior Before the Enemy.
Art. 77. Subordinates Compelling Commander to Surrender.
Art. 78. Improper Use of Countersign.
Art. 79. Forcing a Safeguard.
Art. 80. Captured Property to be Secured for Public Service.

Art. 81. Dealing in Captured or Abandoned Property.


Art. 82. Relieving, Corresponding With, or Aiding the Enemy.
Art. 83. Spies.
Art. 84. Military Property.Willful or Negligent Loss, Damage
or wrongful Disposition.
Art. 85. Waste or Unlawful Disposition of Military Property
Issued to Soldiers.
Art. 86. Drunk on Duty.
Art. 87. Misbehavior of Sentinel.
Art. 88. Personal Interest in Sale of Provisions.
Art. 88-A. Unlawful Influencing Action of Court.
Art. 89. Intimidation of Persons Bringing Provisions.
Art. 90. Good Order to be Maintained and Wrongs Redressed.
Art. 91. Provoking Speeches or Gestures.
Art. 92. Dueling.
Articles 95 to 97:
Art. 95. Frauds Against the Government.
Art. 96. Conduct Unbecoming an Officer and Gentleman.
Art. 97. General Article.
Further, Section 1 of Rep. Act No. 7055 vests on the military courts the jurisdiction over
the foregoing offenses. x x x.
It is clear from the foregoing that Rep. Act No. 7055 did not divest the military courts of
jurisdiction to try cases involving violations of Articles 54 to 70, Articles 72 to 92, and
Articles 95 to 97 of the Articles of War as these are considered "service-connected
crimes or offenses." In fact, it mandates that these shall be tried by the court-martial.

Moreover, the observation made by Mr. Justice Antonio T. Carpio during the
deliberation of this case is worth quoting, thus:
The trial court aggravated its error when it justified its ruling by holding that the charge
of Conduct Unbecoming an Officer and a Gentleman is absorbed and in furtherance to
the alleged crime of coup detat. Firstly, the doctrine of absorption of crimes is peculiar
to criminal law and generally applies to crimes punished by the same statute, 25 unlike
here where different statutes are involved. Secondly, the doctrine applies only if the trial
court has jurisdiction over both offenses. Here, Section 1 of R.A. 7055 deprives civil
courts of jurisdiction over service-connected offenses, including Article 96 of the Articles
of War. Thus, the doctrine of absorption of crimes is not applicable to this case.
Military law is sui generis (Calley v. Callaway, 519 F.2d 184 [1975]), applicable only to
military personnel because the military constitutes an armed organization requiring a
system of discipline separate from that of civilians (see Orloff v. Willoughby, 345 U.S. 83
[1953]). Military personnel carry high-powered arms and other lethal weapons not
allowed to civilians. History, experience, and the nature of a military organization dictate
that military personnel must be subjected to a separate disciplinary system not
applicable to unarmed civilians or unarmed government personnel.
A civilian government employee reassigned to another place by his superior may
question his reassignment by asking a temporary restraining order or injunction from a
civil court. However, a soldier cannot go to a civil court and ask for a restraining or
injunction if his military commander reassigns him to another area of military operations.
If this is allowed, military discipline will collapse.
xxx
This Court has recognized that courts-martial are instrumentalities of the Executive to
enable the President, as Commander-in-Chief, to effectively command, control, and
discipline the armed forces (see Ruffy v. Chief of Staff, 75 Phil. 875 [1946], citing
Winthrops Military Law and Precedents, 2nd edition, p. 49). In short, courts-martial form
part of the disciplinary system that ensures the Presidents control, and thus civilian
supremacy, over the military. At the apex of this disciplinary system is the President
who exercises review powers over decisions of courts-martial (citing Article 50 of the
Articles of War; quoted provisions omitted).
xxx
While the Court had intervened before in courts-martial or similar proceedings, it did so
sparingly and only to release a military personnel illegally detained (Ognir v. Director of
Prisons, 80 Phil. 401 [1948] or to correct objectionable procedures (Yamashita v. Styer,
75 Phil. 563 [1945]). The Court has never suppressed court-martial proceedings on the
ground that the offense charged is absorbed and in furtherance of another criminal
charge pending with the civil courts. The Court may now do so only if the offense

charged is not one of the service-connected offenses specified in Section 1 of RA 7055.


Such is not the situation in the present case.
With respect to the issue of prescription raised by petitioners in their Supplemental
Petition, suffice it to say that we cannot entertain the same. The contending parties are
at loggerheads as to (a) who among the petitioners were actually arraigned, and (b) the
dates of their arraignment. These are matters involving questions of fact, not within our
power of review, as we are not a trier of facts. In a petition for prohibition, such as the
one at bar, only legal issues affecting the jurisdiction of the tribunal, board or officer
involved may be resolved on the basis of the undisputed facts. 26
Clearly, the instant petition for prohibition must fail. The office of prohibition is to prevent
the unlawful and oppressive exercise of authority and is directed against proceedings
that are done without or in excess of jurisdiction, or with grave abuse of discretion, there
being no appeal or other plain, speedy, and adequate remedy in the ordinary course of
law. 27 Stated differently, prohibition is the remedy to prevent inferior courts,
corporations, boards, or persons from usurping or exercising a jurisdiction or power with
which they have not been vested by law. 28
In fine, this Court holds that herein respondents have the authority in convening a court
martial and in charging petitioners with violation of Article 96 of the Articles of War.
WHEREFORE, the instant petition for prohibition is DISMISSED.
SO ORDERED.

PEOPLE OF THE PHILIPPINESand


PHOTOKINA MARKETING CORPORATION,

G.R. No. 154473

Petitioners,

- versus -

ALFREDO L. BENIPAYO,
Respondent.

X - - - - - - - - - - - - - - - - - - - - - - - - - - -X

PHOTOKINA MARKETING CORPORATION,


Petitioner,

G.R. No. 155573

Present:

- versus -

PUNO, C.J.,
QUISUMBING,*
YNARES-SANTIAGO,
CARPIO,
AUSTRIA-MARTINEZ,
CORONA,
CARPIO MORALES,
TINGA,
CHICO-NAZARIO,
VELASCO, JR.,
NACHURA,
LEONARDO-DE CASTRO,
BRION,
PERALTA, and
BERSAMIN, JJ.
Promulgated:

April 24, 2009

ALFREDO L. BENIPAYO,
Respondent.
x-----------------------------------------------------------------------------------------x

DECISION
NACHURA, J.:

Before the Court are two consolidated petitions for review on certiorari filed under Rules 45 and
122 of the Rules of Court: (1) G.R. No. 154473 assailing the June 18, 2002 [1]and the June 23,
2002[2] Orders of the Regional Trial Court (RTC) of Quezon City, Branch 102 in Criminal Case No.
Q-02-109407; and (2) G.R. No. 155573 challenging the June 25, 2002 [3] and the September 18,
2002[4] Orders of the RTC of Quezon City, Branch 101 in Criminal Case No. Q-02-109406.

The petitions, while involving the same issues, rest on different factual settings, thus:

G.R. No. 154473

On January 31, 2002, respondent Alfredo L. Benipayo, then Chairman of the Commission on
Elections (COMELEC), delivered a speech in the Forum on Electoral Problems: Roots and
Responses in the Philippines held at the Balay Kalinaw, University of the Philippines-Diliman
Campus, Quezon City.[5] The speech was subsequently published in the February 4 and 5, 2002
issues of the Manila Bulletin.[6]

Petitioner corporation, believing that it was the one alluded to by the respondent when he stated
in his speech that

Even worse, the Commission came right up to the brink of signing a 6.5
billion contract for a registration solution that could have been bought for 350
million pesos, and an ID solution that isnt even a requirement for voting. But
reason intervened and no contract was signed. Now, they are at it again, trying
to hoodwink us into contract that is so grossly disadvantageous to the
government that it offends common sense to say that it would be worth the 6.5
billion-peso price tag.[7]

filed, through its authorized representative, an Affidavit-Complaint[8] for libel.

Arguing that he was an impeachable officer, respondent questioned the jurisdiction of


the Office of the City Prosecutor of Quezon City (OCP-QC).[9] Despite the challenge, the City
Prosecutor filed an Information[10] for libel against the respondent, docketed as Criminal Case No.
Q-02-109407, with the RTC of Quezon City, Branch 102.

Petitioner later filed a Motion for Inhibition and Consolidation,[11] contending that Judge Jaime
N. Salazar of Branch 102 could not impartially preside over the case because his appointment to
the judiciary was made possible through the recommendation of respondents father-in-law.
Petitioner further moved that the case be ordered consolidated with the other libel case
[Criminal Case No. Q-02-103406, which is the subject of G.R. No. 155573] pending with Branch
101 of the RTC.

While the said motion remained unresolved, respondent, for his part, moved for the dismissal of
the case on the assertion that the trial court had no jurisdiction over his person for he was an

impeachable officer and thus, could not be criminally prosecuted before any court during his
incumbency; and that, assuming he can be criminally prosecuted, it was the Office of the
Ombudsman that should investigate him and the case should be filed with the Sandiganbayan.[12]

On June 18, 2002, the trial court issued the challenged Order[13] dismissing Criminal Case No. Q02-109407 and considering as moot and academic petitioners motion to inhibit. While the RTC
found that respondent was no longer an impeachable officer because his appointment was not
confirmed by Congress, it ruled that the case had to be dismissed for lack of jurisdiction
considering that the alleged libel was committed by respondent in relation to his officehe
delivered the speech in his official capacity as COMELEC Chair. Accordingly, it was the
Sandiganbayan that had jurisdiction over the case to the exclusion of all other courts.

On motion for reconsideration, the trial court adhered to its ruling that it was not vested
with jurisdiction to hear the libel case.[14]
Aggrieved, petitioners timely filed before the Court, on pure questions of law, the instant
Petition for Review on Certiorari[15] under Rule 122 in relation to Rule 45 of the Rules of Court
raising the following grounds:

I.

THE TRIAL COURT SHOULD HAVE FIRST RESOLVED THE MOTION TO


INHIBIT BEFORE RESOLVING THE MOTION TO DISMISS;

II.

THE TRIAL COURT ERRED IN RULING THAT THE CRIME OF LIBEL IN THIS
CASE WAS COMMITTED BY ACCUSED IN RELATION TO HIS OFFICE; AND

III.

THE TRIAL COURT ERRED IN RULING THAT IT HAD NO JURISDICTION IN


THIS CASE.[16]

G.R. No. 155573

On March 13, 2002, respondent, as COMELEC Chair, and COMELEC Commissioner Luzviminda
Tangcangco were guests of the talk show Point Blank, hosted by Ces Drilon and televised
nationwide on the ANC-23 channel. The television shows episode that day was entitled COMELEC
Wars.[17] In that episode, the following conversation transpired:

Drilon: Are you saying, Chairman, that COMELEC funds are being used for a PR
campaign against you? Is that what you are saying?

Benipayo: No, I think [its] not COMELEC funds, [its] Photokina funds. You know,
admittedly, according to [c]harg d[a]ffaires of the U.S. Embassy[,]
in a letter sent to me in July of 2001, it is whats been [so] happening
to the Photokina deal, they have already spent in excess of 2.4
[m]illion U.S. [d]ollars. At that time[,] thats about 120 [m]illion
pesos and I said, what for[?] [T]hey wouldnt tell me, you see. Now
you asked me, [who is] funding this? I think its pretty obvious.[18]

Petitioner considered respondents statement as defamatory, and, through its authorized


representative, filed a Complaint-Affidavit[19] for libel. Respondent similarly questioned the
jurisdiction of the OCP-QC.[20] The City Prosecutor, however, consequently instituted Criminal
Case No. Q-02-109406 by filing the corresponding Information[21] with the RTC of Quezon City,
Branch 101.

Respondent also moved for the dismissal of the information raising similar arguments that the
court had no jurisdiction over his person, he being an impeachable officer; and that, even if
criminal prosecution were possible, jurisdiction rested with the Sandiganbayan.[22]

On June 25, 2002, the trial court issued the assailed Order[23] dismissing Criminal Case No. Q-02109406 for lack of jurisdiction over the person of the respondent. The RTC, in the further assailed
September 18, 2002 Order,[24] denied petitioners Motion for Reconsideration.[25]

Displeased with the rulings of the trial court, petitioners seasonably filed before this
Court, on pure questions of law, another Petition for Review on Certiorari[26] under Rule 122 in
relation to Rule 45 of the Rules of Court raising the following grounds:

I.

THE TRIAL COURT ERRED IN RULING THAT THE CRIME OF LIBEL IN THIS
CASE WAS COMMITTED BY RESPONDENT IN RELATION TO HIS OFFICE; AND

II.

IN THE ABSENCE OF ANY ALLEGATION IN THE INFORMATION THAT THE


CRIME OF LIBEL WAS COMMITTED BY RESPONDENT IN RELATION TO HIS
OFFICE, THE TRIAL COURT ERRED IN RULING THAT IT HAD NO
JURISDICTION OVER THE CASE BELOW.

III.

EVEN ON THE ASSUMPTION THAT THE SANDIGANBAYAN HAS


JURISDICTION OVER THE CASE, THE TRIAL COURT SHOULD HAVE
ENDORSED THE CASE TO THE SANDIGANBAYAN INSTEAD OF DISMISSING
IT OUTRIGHT.[27]

Considering that the two petitions, as aforesaid, involve the same issues and the same
parties, the Court, upon the recommendation of the Clerk of Court,[28] consolidated the cases.[29]

The core issue for the resolution of the Court in these twin cases is whether the RTC has
jurisdiction over libel cases to the exclusion of all other courts.

The Ruling of the Court

The Court observes that the parties have argued at length in their pleadings on the issue
of whether the alleged criminal acts of respondent are committed in relation to his office. They
are of the conviction that the resolution of the said question will ultimately determine which
courtthe RTC or the Sandiganbayanhas jurisdiction over the criminal cases filed. The Court,
however, notes that both parties are working on a wrong premise. The foremost concern, which

the parties, and even the trial court, failed to identify, is whether, under our current laws,
jurisdiction over libel cases, or written defamations to be more specific, is shared by the RTC with
the Sandiganbayan. Indeed, if the said courts do not have concurrent jurisdiction to try the
offense, it would be pointless to still determine whether the crime is committed in relation to
office.

Uniformly applied is the familiar rule that the jurisdiction of the court to hear and decide
a case is conferred by the law in force at the time of the institution of the action, unless a latter
statute provides for a retroactive application thereof.[30] Article 360 of the Revised Penal Code
(RPC),[31] as amended by Republic Act No. 4363,[32] is explicit on which court has jurisdiction to
try cases of written defamations, thus:

The criminal and civil action for damages in cases of written


defamations as provided for in this chapter, shall be filed simultaneously or
separately with the court of first instance [now, the Regional Trial Court] of the
province or city where the libelous article is printed and first published or where
any of the offended parties actually resides at the time of the commission of the
offense xxx.[33] [Underscoring and italics ours.]

More than three decades ago, the Court, in Jalandoni v. Endaya,[34] acknowledged the
unmistakable import of the said provision:

There is no need to make mention again that it is a court of first instance [now,
the Regional Trial Court] that is specifically designated to try a libel case. Its
language is categorical; its meaning is free from doubt. This is one of those
statutory provisions that leave no room for interpretation. All that is required is
application. What the law ordains must then be followed.[35]

This exclusive and original jurisdiction of the RTC over written defamations is echoed in Bocobo
v. Estanislao,[36] where the Court further declared that jurisdiction remains with the trial court

even if the libelous act is committed by similar means,[37] and despite the fact that the phrase by
similar means is not repeated in the latter portion of Article 360.[38] In these cases, and in those
that followed, the Court had been unwavering in its pronouncement that the expanded
jurisdiction of the municipal trial courts cannot be exercised over libel cases. Thus, in Manzano
v. Hon. Valera,[39] we explained at length that:

The applicable law is still Article 360 of the Revised Penal Code, which
categorically provides that jurisdiction over libel cases [is] lodged with the Courts
of First Instance (now Regional Trial Courts).

This Court already had the opportunity to rule on the matter in G.R. No.
123263, People vs. MTC of Quezon City, Branch 32 and Isah v. Red wherein a
similar question of jurisdiction over libel was raised. In that case, the MTC judge
opined that it was the first level courts which had jurisdiction due to the
enactment of RA 7691. Upon elevation of the matter to us, respondent judges
orders were nullified for lack of jurisdiction, as follows:

WHEREFORE, the petition is granted: the respondent


Courts Orders dated August 14, 1995, September 7, 1995, and
October 18, 1995 are declared null and void for having been issued
without jurisdiction; and said Court is enjoined from further taking
cognizance of and proceeding with Criminal Case No. 43-00548,
which it is commanded to remand to the Executive Judge of the
Regional Trial Court of Quezon City for proper disposition.

Another case involving the same question was cited as resolving the
matter:

Anent the question of jurisdiction, we ** find no reversible


error committed by public respondent Court of Appeals in denying
petitioners motion to dismiss for lack of jurisdiction. The
contention ** that R.A. 7691 divested the Regional Trial Courts of
jurisdiction to try libel cases cannot be sustained. While libel is
punishable by imprisonment of six months and one day to four
years and two months (Art. 360, Revised Penal Code) which

imposable penalty is lodged within the Municipal Trial Courts


jurisdiction under R.A. No. 7691 (Sec. 32 [2]), said law however,
excludes therefrom ** cases falling within the exclusive original
jurisdiction of the Regional Trial Courts **. The Court inBocobo vs.
Estanislao, 72 SCRA 520 and Jalandoni vs. Endaya, 55 SCRA 261,
correctly cited by the Court of Appeals, has laid down the rule that
Regional Trial courts have the exclusive jurisdiction over libel cases,
hence, the expanded jurisdiction conferred by R.A. 7691 to inferior
courts cannot be applied to libel cases.

Conformably with [these] rulings, we now hold that public respondent


committed an error in ordering that the criminal case for libel be tried by the MTC
of Bangued.

For, although RA 7691 was enacted to decongest the clogged dockets of


the Regional Trail Courts by expanding the jurisdiction of first level courts, said law
is of a general character.Even if it is a later enactment, it does not alter the
provision of Article 360 of the RPC, a law of a special nature. Laws vesting
jurisdiction exclusively with a particular court, are special in character, and should
prevail over the Judiciary Act defining the jurisdiction of other courts (such as the
Court of First Instance) which is a general law. A later enactment like RA 7691 does
not automatically override an existing law, because it is a well-settled principle of
construction that, in case of conflict between a general law and a special law, the
latter must prevail regardless of the dates of their enactment. Jurisdiction
conferred by a special law on the RTC must therefore prevail over that granted by
a general law on the MTC.

Moreover, from the provisions of R.A. 7691, there seems to be no manifest


intent to repeal or alter the jurisdiction in libel cases. If there was such intent, then
the amending law should have clearly so indicated because implied repeals are
not favored. As much as possible, effect must be given to all enactments of the
legislature. A special law cannot be repealed, amended or altered by a subsequent
general law by mere implication. Furthermore, for an implied repeal, a precondition must be found, that is, a substantial conflict should exist between the
new and prior laws. Absent an express repeal, a subsequent law cannot be
construed as repealing a prior one unless an irreconcilable inconsistency or
repugnancy exists in the terms of the new and old laws.The two laws, in brief,
must be absolutely incompatible. In the law which broadened the jurisdiction of

the first level courts, there is no absolute prohibition barring Regional Trial Courts
from taking cognizance of certain cases over which they have been priorly granted
special and exclusive jurisdiction. Such grant of the RTC (previously CFI) was
categorically contained in the first sentence of the amended Sec. 32 of B.P. 129.
The inconsistency referred to in Section 6 of RA 7691, therefore, does not apply
to cases of criminal libel.

Lastly, in Administrative Order No. 104-96 issued 21 October 1996, this


Court delineated the proper jurisdiction over libel cases, hence settled the matter
with finality:

RE: DESIGNATION OF SPECIAL COURTS FOR KIDNAPPING,


ROBBERY, CARNAPPING, DANGEROUS DRUGS CASES AND OTHER
HEINOUS CRIMES; INTELLECTUAL PROPERTY RIGHTS VIOLATIONS
AND JURISDICTION IN LIBEL CASES.

xxxx

LIBEL CASES SHALL BE TRIED BY THE REGIONAL TRIAL


COURTS HAVING JURISDICTION OVER THEM TO THE EXCLUSION
OF THE METROPOLITAN TRIAL COURTS, MUNICIPAL TRIAL COURTS
IN CITIES, MUNICIPAL TRIAL COURTS AND MUNICIPAL CIRCUIT
TRIAL COURTS.(Underscoring supplied)[40]

As we have constantly held in Jalandoni, Bocobo, People v. Metropolitan Trial Court of Quezon
City, Br. 32,[41] Manzano, and analogous cases, we must, in the same way, declare herein that the
law, as it still stands at present, dictates that criminal and civil actions for damages in cases of
written defamations shall be filed simultaneously or separately with the RTC to the exclusion of

all other courts. A subsequent enactment of a law defining the jurisdiction of other courts cannot
simply override, in the absence of an express repeal or modification, the specific provision in
the RPC vesting in the RTC, as aforesaid, jurisdiction over defamations in writing or by similar
means.[42] The grant to the Sandiganbayan[43] of jurisdiction over offenses committed in
relation to (public) office, similar to the expansion of the jurisdiction of the MTCs, did not divest
the RTC of its exclusive and original jurisdiction to try written defamation cases regardless of
whether the offense is committed in relation to office. The broad and general phraseology of
Section 4, Presidential Decree No. 1606, as amended by Republic Act No. 8249, [44] cannot be
construed to have impliedly repealed, or even simply modified, such exclusive and original
jurisdiction of the RTC.[45]

Since jurisdiction over written defamations exclusively rests in the RTC without qualification, it is
unnecessary and futile for the parties to argue on whether the crime is committed in relation to
office. Thus, the conclusion reached by the trial court that the respondent committed the alleged
libelous acts in relation to his office as former COMELEC chair, and deprives it of jurisdiction to
try the case, is, following the above disquisition, gross error. This Court, therefore, orders the
reinstatement of Criminal Cases Nos. Q-02-109406 and Q-02-109407 and their remand to the
respective Regional Trial Courts for further proceedings. Having said that, the Court finds
unnecessary any further discussion of the other issues raised in the petitions.

WHEREFORE, premises considered, the consolidated petitions for review


on certiorari are GRANTED. Criminal Cases Nos. Q-02-109406 and Q-02-109407
areREINSTATED and REMANDED to the Regional Trial Court of Quezon City for further
proceedings.

SO ORDERED.

[G.R. Nos. 160054-55. July 21, 2004]

MANOLO P. SAMSON, petitioner, vs. HON. REYNALDO B. DAWAY, in his capacity


as Presiding Judge, Regional Trial Court of Quezon City, Branch 90, PEOPLE
OF THE PHILIPPINES and CATERPILLAR, INC., respondents.
DECISION
YNARES-SANTIAGO, J.:
Assailed in this petition for certiorari is the March 26, 2003 Order[1] of the Regional
Trial Court of Quezon City, Branch 90, which denied petitioners (1) motion to quash the
information; and (2) motion for reconsideration of the August 9, 2002 Order denying his
motion to suspend the arraignment and other proceedings in Criminal Case Nos. Q-02108043-44. Petitioner also questioned its August 5, 2003 Order[2] which denied his motion
for reconsideration.
The undisputed facts show that on March 7, 2002, two informations for unfair
competition under Section 168.3 (a), in relation to Section 170, of the Intellectual Property
Code (Republic Act No. 8293), similarly worded save for the dates and places of
commission, were filed against petitioner Manolo P. Samson, the registered owner of ITTI
Shoes. The accusatory portion of said informations read:
That on or about the first week of November 1999 and sometime prior or subsequent thereto, in
Quezon City, Philippines, and within the jurisdiction of this Honorable Court, above-named
accused, owner/proprietor of ITTI Shoes/Mano Shoes Manufactuirng Corporation located at
Robinsons Galleria, EDSA corner Ortigas Avenue, Quezon City, did then and there willfully,
unlawfully and feloniously distribute, sell and/or offer for sale CATERPILLAR products such as
footwear, garments, clothing, bags, accessories and paraphernalia which are closely identical to
and/or colorable imitations of the authentic Caterpillar products and likewise using trademarks,
symbols and/or designs as would cause confusion, mistake or deception on the part of the buying
public to the damage and prejudice of CATERPILLAR, INC., the prior adopter, user and owner
of the following internationally: CATERPILLAR, CAT, CATERPILLAR & DESIGN, CAT
AND DESIGN, WALKING MACHINES and TRACK-TYPE TRACTOR & DESIGN.
CONTRARY TO LAW.[3]
On April 19, 2002, petitioner filed a motion to suspend arraignment and other
proceedings in view of the existence of an alleged prejudicial question involved in Civil
Case No. Q-00-41446 for unfair competition pending with the same branch; and also in
view of the pendency of a petition for review filed with the Secretary of Justice assailing
the Chief State Prosecutors resolution finding probable cause to charge petitioner with
unfair competition. In an Order dated August 9, 2002, the trial court denied the motion to
suspend arraignment and other proceedings.

On August 20, 2002, petitioner filed a twin motion to quash the informations and
motion for reconsideration of the order denying motion to suspend, this time challenging
the jurisdiction of the trial court over the offense charged. He contended that since under
Section 170 of R.A. No. 8293, the penalty5 of imprisonment for unfair competition does
not exceed six years, the offense is cognizable by the Municipal Trial Courts and not by
the Regional Trial Court, per R.A. No. 7691.
In its assailed March 26, 2003 Order, the trial court denied petitioners twin motions.6 A
motion for reconsideration thereof was likewise denied on August 5, 2003.
Hence, the instant petition alleging that respondent Judge gravely abused its
discretion in issuing the assailed orders.
The issues posed for resolution are (1) Which court has jurisdiction over criminal and
civil cases for violation of intellectual property rights? (2) Did the respondent Judge
gravely abuse his discretion in refusing to suspend the arraignment and other
proceedings in Criminal Case Nos. Q-02-108043-44 on the ground of (a) the existence of
a prejudicial question; and (b) the pendency of a petition for review with the Secretary of
Justice on the finding of probable cause for unfair competition?
Under Section 170 of R.A. No. 8293, which took effect on January 1, 1998, the
criminal penalty for infringement of registered marks, unfair competition, false designation
of origin and false description or representation, is imprisonment from 2 to 5 years and a
fine ranging from Fifty Thousand Pesos to Two Hundred Thousand Pesos, to wit:
SEC. 170. Penalties. Independent of the civil and administrative sanctions imposed by law, a
criminal penalty of imprisonment from two (2) years to five (5) years and a fine ranging from
Fifty thousand pesos (P50,000.00) to Two hundred thousand pesos (P200,000.00), shall be
imposed on any person who is found guilty of committing any of the acts mentioned in Section
155 [Infringement], Section 168 [Unfair Competition] and Section 169.1 [False Designation of
Origin and False Description or Representation].
Corollarily, Section 163 of the same Code states that actions (including criminal and
civil) under Sections 150, 155, 164, 166, 167, 168 and 169 shall be brought before the
proper courts with appropriate jurisdiction under existing laws, thus
SEC. 163. Jurisdiction of Court. All actions under Sections 150, 155, 164 and 166 to 169 shall
be brought before the proper courts with appropriate jurisdiction under existing laws.
(Emphasis supplied)
The existing law referred to in the foregoing provision is Section 27 of R.A. No. 166
(The Trademark Law) which provides that jurisdiction over cases for infringement of
registered marks, unfair competition, false designation of origin and false description or
representation, is lodged with the Court of First Instance (now Regional Trial Court)
SEC. 27. Jurisdiction of Court of First Instance. All actions under this Chapter [V Infringement]
and Chapters VI [Unfair Competition] and VII [False Designation of Origin and False
Description or Representation], hereof shall be brought before the Court of First Instance.

We find no merit in the claim of petitioner that R.A. No. 166 was expressly repealed
by R.A. No. 8293. The repealing clause of R.A. No. 8293, reads
SEC. 239. Repeals. 239.1. All Acts and parts of Acts inconsistent herewith, more particularly
Republic Act No. 165, as amended; Republic Act No. 166, as amended; and Articles 188 and
189 of the Revised Penal Code; Presidential Decree No. 49, including Presidential Decree No.
285, as amended, are hereby repealed. (Emphasis added)
Notably, the aforequoted clause did not expressly repeal R.A. No. 166 in its entirety,
otherwise, it would not have used the phrases parts of Acts and inconsistent herewith;
and it would have simply stated Republic Act No. 165, as amended; Republic Act No.
166, as amended; and Articles 188 and 189 of the Revised Penal Code; Presidential
Decree No. 49, including Presidential Decree No. 285, as amended are hereby repealed.
It would have removed all doubts that said specific laws had been rendered without force
and effect. The use of the phrases parts of Acts and inconsistent herewith only means
that the repeal pertains only to provisions which are repugnant or not susceptible of
harmonization with R.A. No. 8293.7 Section 27 of R.A. No. 166, however, is consistent
and in harmony with Section 163 of R.A. No. 8293. Had R.A. No. 8293 intended to vest
jurisdiction over violations of intellectual property rights with the Metropolitan Trial Courts,
it would have expressly stated so under Section 163 thereof.
Moreover, the settled rule in statutory construction is that in case of conflict between
a general law and a special law, the latter must prevail. Jurisdiction conferred by a special
law to Regional Trial Courts must prevail over that granted by a general law to Municipal
Trial Courts.8
In the case at bar, R.A. No. 8293 and R.A. No. 166 are special laws 9 conferring
jurisdiction over violations of intellectual property rights to the Regional Trial Court. They
should therefore prevail over R.A. No. 7691, which is a general law. 10 Hence, jurisdiction
over the instant criminal case for unfair competition is properly lodged with the Regional
Trial Court even if the penalty therefor is imprisonment of less than 6 years, or from 2 to
5 years and a fine ranging from P50,000.00 to P200,000.00.
In fact, to implement and ensure the speedy disposition of cases involving violations
of intellectual property rights under R.A. No. 8293, the Court issued A.M. No. 02-1-11-SC
dated February 19, 2002 designating certain Regional Trial Courts as Intellectual Property
Courts. On June 17, 2003, the Court further issued a Resolution consolidating jurisdiction
to hear and decide Intellectual Property Code and Securities and Exchange Commission
cases in specific Regional Trial Courts designated as Special Commercial Courts.
The case of Mirpuri v. Court of Appeals,11 invoked by petitioner finds no application
in the present case. Nowhere in Mirpuri did we state that Section 27 of R.A. No. 166 was
repealed by R.A. No. 8293. Neither did we make a categorical ruling therein that
jurisdiction over cases for violation of intellectual property rights is lodged with the
Municipal Trial Courts. The passing remark in Mirpuri on the repeal of R.A. No. 166 by
R.A. No. 8293 was merely a backgrounder to the enactment of the present Intellectual
Property Code and cannot thus be construed as a jurisdictional pronouncement in cases
for violation of intellectual property rights.

Anent the second issue, petitioner failed to substantiate his claim that there was a
prejudicial question. In his petition, he prayed for the reversal of the March 26, 2003 order
which sustained the denial of his motion to suspend arraignment and other proceedings
in Criminal Case Nos. Q-02-108043-44. For unknown reasons, however, he made no
discussion in support of said prayer in his petition and reply to comment. Neither did he
attach a copy of the complaint in Civil Case No. Q-00-41446 nor quote the pertinent
portion thereof to prove the existence of a prejudicial question.
At any rate, there is no prejudicial question if the civil and the criminal action can,
according to law, proceed independently of each other.12 Under Rule 111, Section 3 of
the Revised Rules on Criminal Procedure, in the cases provided in Articles 32, 33, 34 and
2176 of the Civil Code, the independent civil action may be brought by the offended party.
It shall proceed independently of the criminal action and shall require only a
preponderance of evidence.
In the case at bar, the common element in the acts constituting unfair competition
under Section 168 of R.A. No. 8293 is fraud.13 Pursuant to Article 33 of the Civil Code, in
cases of defamation, fraud, and physical injuries, a civil action for damages, entirely
separate and distinct from the criminal action, may be brought by the injured party. Hence,
Civil Case No. Q-00-41446, which as admitted14 by private respondent also relate to
unfair competition, is an independent civil action under Article 33 of the Civil Code. As
such, it will not operate as a prejudicial question that will justify the suspension of the
criminal cases at bar.
Section 11 (c), Rule 116 of the Revised Rules on Criminal Procedure provides
SEC. 11. Suspension of arraignment. Upon motion by the proper party, the arraignment shall be
suspended in the following cases
xxxxxxxxx
(c) A petition for review of the resolution of the prosecutor is pending at either the Department
of Justice, or the Office of the President; Provided, that the period of suspension shall not exceed
sixty (60) days counted from the filing of the petition with the reviewing office.
While the pendency of a petition for review is a ground for suspension of the
arraignment, the aforecited provision limits the deferment of the arraignment to a period
of 60 days reckoned from the filing of the petition with the reviewing office. It follows,
therefore, that after the expiration of said period, the trial court is bound to arraign the
accused or to deny the motion to defer arraignment.
In the instant case, petitioner failed to establish that respondent Judge abused his
discretion in denying his motion to suspend. His pleadings and annexes submitted before
the Court do not show the date of filing of the petition for review with the Secretary of
Justice.15 Moreover, the Order dated August 9, 2002 denying his motion to suspend was
not appended to the petition. He thus failed to discharge the burden of proving that he
was entitled to a suspension of his arraignment and that the questioned orders are

contrary to Section 11 (c), Rule 116 of the Revised Rules on Criminal Procedure. Indeed,
the age-old but familiar rule is that he who alleges must prove his allegations.
In sum, the dismissal of the petition is proper considering that petitioner has not
established that the trial court committed grave abuse of discretion. So also, his failure to
attach documents relevant to his allegations warrants the dismissal of the petition,
pursuant to Section 3, Rule 46 of the Rules of Civil Procedure, which states:
SEC. 3. Contents and filing of petition; effect of non-compliance with requirements. The
petition shall contain the full names and actual addresses of all the petitioners and respondents, a
concise statement of the matters involved, the factual background of the case, and the grounds
relied upon for the relief prayed for.
It shall be filed in seven (7) clearly legible copies together with proof of service thereof on the
respondent with the original copy intended for the court indicated as such by the petitioner, and
shall be accompanied by a clearly legible duplicate original or certified true copy of the
judgment, order, resolution, or ruling subject thereof, such material portions of the record
as are referred to therein, and other documents relevant or pertinent thereto.
xxxxxxxxx
The failure of the petitioner to comply with any of the foregoing requirements shall be
sufficient ground for the dismissal of the petition. (Emphasis added)
WHEREFORE, in view of all the foregoing, the petition is DISMISSED.
SO ORDERED.

G.R. No. 165835

June 22, 2005

MAJOR GENERAL CARLOS F. GARCIA, Petitioner,


vs.
SANDIGANBAYAN and the OFFICE OF THE OMBUDSMAN, Respondents.
DECISION
Tinga, J.:
Petitioner Major General Carlos F. Garcia was the Deputy Chief of Staff for
Comptrollership, J6, of the Armed Forces of the Philippines. Petitioner filed
this Petition for certiorari and prohibition under Rule 65 to annul and set aside public
respondent Sandiganbayans Resolution1 dated 29 October 2004 and Writ of
Preliminary Attachment2 dated 2 November 2004, and to enjoin public respondents
Sandiganbayan and Office of the Ombudsman from further proceeding with any action
relating to the enforcement of the assailed issuances.
On 27 September 2004, Atty. Maria Olivia Elena A. Roxas, Graft Investigation and
Prosecution Officer II of the Field Investigation Office of the Office of the Ombudsman,
after due investigation, filed a complaint against petitioner with public respondent Office
of the Ombudsman, for violation of Sec. 8, in relation to Sec. 11 of Republic Act (R.A.)
No. 6713,3 violation of Art. 183 of the Revised Penal Code, and violation of Section 52
(A)(1), (3) and (20) of the Civil Service Law. Based on this complaint, a case for
Violations of R.A. No. 1379,4 Art. 183 of the Revised Penal Code, and Sec. 8 in relation
to Sec. 11 of R.A. No. 6713, docketed as Case
No. OMB-P-C-04-1132-I, was filed against petitioner.5 Petitioners wife Clarita
Depakakibo Garcia, and their three sons, Ian Carl, Juan Paolo and Timothy Mark, all
surnamed Garcia, were impleaded in the complaint for violation of R.A. No. 1379 insofar
as they acted as conspirators, conduits, dummies and fronts of petitioner in receiving,
accumulating, using and disposing of his ill-gotten wealth.
On the same day, 27 October 2004, the Republic of the Philippines, acting through
public respondent Office of the Ombudsman, filed before the Sandiganbayan, a Petition
with Verified Urgent Ex Parte Application for the Issuance of a Writ of Preliminary
Attachment6 against petitioner, his wife, and three sons, seeking the forfeiture of
unlawfully acquired properties under Sec. 2 of R.A. No. 1379, as amended. The petition
was docketed as Civil Case No. 0193, entitled "Republic of the Philippines vs. Maj. Gen.
Carlos F. Garcia, et al." It was alleged that the Office of the Ombudsman, after
conducting an inquiry similar to a preliminary investigation in criminal cases, has
determined that a prima facie case exists against Maj. Gen. Garcia and the other
respondents therein who hold such properties for, with, or on behalf of, Maj. Gen.
Garcia, since during his incumbency as a soldier and public officer he acquired huge
amounts of money and properties manifestly out of proportion to his salary as such
public officer and his other lawful income, if any.7

Acting on the Republics prayer for issuance of a writ of preliminary attachment, the
Sandiganbayan issued the questioned Resolution granting the relief prayed for. The
corresponding writ of preliminary attachment was subsequently issued on 2 November
2004 upon the filing of a bond by the Republic. On 17 November 2004, petitioner (as
respondent a quo) filed a Motion to Dismiss8 in Civil Case No. 0193 on the ground of
lack of jurisdiction of the Sandiganbayan over forfeiture proceedings under R.A. No.
1379. On even date, petitioner filed the present Petition, raising the same issue of lack
jurisdiction on the part of the Sandiganbayan.
Petitioner argues in this Petition that the Sandiganbayan is without jurisdiction over the
"civil action" for forfeiture of unlawfully acquired properties under R.A. No. 1379,
maintaining that such jurisdiction actually resides in the Regional Trial Courts as
provided under Sec. 29 of the law, and that the jurisdiction of the Sandiganbayan in civil
actions pertains only to separate actions for recovery of unlawfully acquired property
against President Marcos, his family, and cronies as can be gleaned from Sec. 4 of
Presidential Decree (P.D.) No. 1606,10 as amended, and Executive Orders (E.O.) Nos.
1411 and 14-A.12
Theorizing that the Sandiganbayan, under P.D. No. 1606 or the law creating it, was
intended principally as a criminal court, with no jurisdiction over separate civil actions,
petitioner points to President Corazon C. Aquinos issuances after the EDSA
Revolution, namely: (1) E.O. No. 1 creating the Presidential Commission on Good
Government (PCGG) for the recovery of ill-gotten wealth amassed by President
Ferdinand E. Marcos, his family and cronies, (2) E.O. No. 14 which amended P.D. No.
1606 and R.A. No. 1379 by transferring to the Sandiganbayan jurisdiction over civil
actions filed against President Marcos, his family and cronies based on R.A. No. 1379,
the Civil Code and other existing laws, and (3) E.O. No. 14-A whch further amended
E.O. No. 14, P.D. No. 1606 and R.A. No. 1379 by providing that the civil action under
R.A. No. 1379 which may be filed against President Marcos, his family and cronies, may
proceed independently of the criminal action.
Petitioner gathers from the presidential issuances that the Sandiganbayan has been
granted jurisdiction only over the separate civil actions filed against President Marcos,
his family and cronies, regardless of whether these civil actions were for recovery of
unlawfully acquired property under R.A. No. 1379 or for restitution, reparation of
damages or indemnification for consequential damages or other civil actions under the
Civil Code or other existing laws. According to petitioner, nowhere in the amendments
to P.D. No. 1606 and R.A. No. 1379 does it provide that the Sandiganbayan has been
vested jurisdiction over separate civil actions other than those filed against President
Marcos, his family and cronies.13 Hence, the Sandiganbayan has no jurisdiction over
any separate civil action against him, even if such separate civil action is for recovery of
unlawfully acquired property under R.A. No. 1379.
Petitioner further contends that in any event, the petition for forfeiture filed against him is
fatally defective for failing to comply with the jurisdictional requirements under Sec. 2,
R.A. No. 1379, 14 namely: (a) an inquiry similar to a preliminary investigation conducted

by the prosecution arm of the government; (b) a certification to the Solicitor General that
there is reasonable ground to believe that there has been violation of the said law and
that respondent is guilty thereof; and (c) an action filed by the Solicitor General on
behalf of the Republic of the Philippines.15 He argues that only informations for perjury
were filed and there has been no information filed against him for violation of R.A. No.
1379. Consequently, he maintains, it is impossible for the Office of the Ombudsman to
certify that there is reasonable ground to believe that a violation of the said law had
been committed and that he is guilty thereof. The petition is also supposedly bereft of
the required certification which should be made by the investigating City or Provincial
Fiscal (now Prosecutor) to the Solicitor General. Furthermore, he opines that it should
have been the Office of the Solicitor General which filed the petition and not the Office
of the Ombudsman as in this case. The petition being fatally defective, the same should
have been dismissed, petitioner concludes.
In their Comment,16 respondents submit the contrary, noting that the issues raised by
petitioner are not novel as these have been settled in Republic vs.
Sandiganbayan17 which categorically ruled that "there is no issue that jurisdiction over
violations of [R.A.] Nos. 3019 and 1379 now rests with the
Sandiganbayan."18 Respondents argue that under the Constitution19 and prevailing
statutes, the Sandiganbayan is vested with authority and jurisdiction over the petition for
forfeiture under R.A. No. 1379 filed against petitioner. Respondents point to Sec. 4.a (1)
(d) of P.D. 1606, as amended, as the prevailing law on the jurisdiction of
the Sandiganbayan, thus:
Sec. 4. Jurisdiction.The Sandiganbayan shall exercise exclusive original jurisdiction in
all cases involving:
a. Violations of Republic Act No. 3019, as amended, otherwise known as the Anti-Graft
and Corrupt Practices Act, Republic Act No. 1379, and Chapter II, Section 2, Title VII,
Book II of the Revised Penal Code, where one or more of the accused are officials
occupying the following positions in the government, whether in a permanent, acting or
interim capacity, at the time of the commission of the offense:
(1) Officials of the executive branch occupying the positions of regional director and
higher, otherwise classified as Grade 27 and higher of the Compensation and Position
Classification Act of 1989 (Republic Act No. 6758), specifically including:
.
(d) Philippine army and air force colonels, naval captains, and all officers of higher
ranks;
.

As petitioner falls squarely under the category of public positions covered by the
aforestated law, the petition for forfeiture should be within the jurisdiction of the
Sandiganbayan.
Respondents also brush off as inconsequential petitioners argument that the petition for
forfeiture is "civil" in nature and the Sandiganbayan, having allegedly no jurisdiction over
civil actions, therefore has no jurisdiction over the petition, since the same P.D. No.
1606 encompasses all cases involving violations of R.A. No. 3019, irrespective of
whether these cases are civil or criminal in nature. The petition for forfeiture should not
be confused with the cases initiated and prosecuted by the PCGG pursuant to E.O.
Nos. 14 and 14-A, as these are dealt with under a separate subparagraph of P.D. No.
1606, as amended, in particular Sec. 4.c thereof.20 Further, respondents stress that
E.O. Nos. 14 and 14-A exclusively apply to actions for recovery of unlawfully acquired
property against President Marcos, his family, and cronies. It would also not be accurate
to refer to a petition for forfeiture as a "civil case," since it has been held that petitions
for forfeiture are deemed criminal or penal and that it is only the proceeding for its
prosecution which is civil in nature.21
The Office of the Ombudsman filed a separate Comment,22 likewise relying on Republic
v. Sandiganbayan to argue that the Sandiganbayan has jurisdiction over the petition for
forfeiture filed against petitioner. The Ombudsman explains that the grant to the
Sandiganbayan of jurisdiction over violations of R.A. No. 1379 did not change even
under the amendments of
R.A. No. 797523 and R.A. No. 829424, although it came to be limited to cases involving
high-ranking public officials as enumerated therein, including Philippine army and air
force colonels, naval captains, and all other officers of higher rank, to which petitioner
belongs.25
In arguing that it has authority to investigate and initiate forfeiture proceedings against
petitioner, the Office of the Ombudsman refers to both the Constitution 26 and R.A. No.
6770.27 The constitutional power of investigation of the Office of the Ombudsman is
plenary and unqualified; its power to investigate any act of a public official or employee
which appears to be "illegal, unjust, improper or inefficient" covers the unlawful
acquisition of wealth by public officials as defined under R.A. No. 1379. Furthermore,
Sec. 15 (11)28 of R.A. No. 6770 expressly empowers the Ombudsman to investigate
and prosecute such cases of unlawful acquisition of wealth. This authority of the
Ombudsman has been affirmed also in Republic vs. Sandiganbayan.29
The Office of the Ombudsman then refutes petitioners allegation that the petition for
forfeiture filed against him failed to comply with the procedural and formal requirements
under the law. It asserts that all the requirements of R.A. No. 1379 have been strictly
complied with. An inquiry similar to a preliminary investigation was conducted by a
Prosecution Officer of the Office of the Ombudsman. The participation of the Office of
the Solicitor General, claimed by petitioner to be necessary, is actually no longer

required since the Office of the Ombudsman is endowed with the authority to investigate
and prosecute the case as discussed above.30
In addition, the Office of the Ombudsman alleges that the present Petition should be
dismissed for blatant forum-shopping. Even as petitioner had filed a Motion to
Dismiss as regards the petition for forfeiture (docketed as Civil Case No. 0193) before
the Sandiganbayan on the ground of the Sandiganbayans alleged lack of jurisdiction,
he filed the instant Petition raising exactly the same issue, even though the Motion to
Dismiss in Civil Case No. 0193 is still pending resolution.1avvphi1 Worse, it appears
that the Motion to Dismiss and the instant Petition were filed on the same day, 17
November 2004.
Petitioner refutes these arguments in his Reply31 and enunciates that the
Sandiganbayans criminal jurisdiction is separate and distinct from its civil jurisdiction,
and that the Sandiganbayans jurisdiction over forfeiture cases had been removed
without subsequent amendments expressly restoring such civil jurisdiction. His thesis is
that R.A. No. 1379 is a special law which is primarily civil and remedial in nature, the
clear intent of which is to separate theprima facie determination in forfeiture
proceedings from the litigation of the civil action. This intent is further demonstrated by
Sec. 2 of R.A. No. 1379 which grants the authority to make an inquiry similar to a
preliminary investigation being done by the City or Provincial Fiscal, and the authority to
file a petition for forfeiture to the Solicitor General.
Petitioner also points out in his Reply32 to the Comment of the Office of the
Ombudsman, that the use of the phrase "violations of [R.A.] Nos. 3019 and 1379" in
P.D. No. 1606, as amended, implies jurisdiction over cases which are principally
criminal or penal in nature because the concept of "violation" of certain laws necessarily
carries with it the concept of imposition of penalties for such violation. Hence, when
reference was made to "violations of [R.A.] Nos. 3019 and 1379," the only jurisdiction
that can supposedly be implied is criminal jurisdiction, not civil jurisdiction, thereby
highlighting respondent Sandiganbayans lack of jurisdiction over the "civil case" for
forfeiture of ill-gotten wealth. Of course, petitioner does not rule out cases where the
crime carries with it the corresponding civil liability such that when the criminal action is
instituted, the civil action for enforcement of the civil liability is impliedly instituted with it,
and the court having jurisdiction over the criminal action also acquires jurisdiction over
the ancillary civil action. However, petitioner argues that the action for forfeiture subject
of this case is not the ancillary civil action impliedly instituted with the criminal action.
Rather, the petition for forfeiture is an independent civil action over which the
Sandiganbayan has no jurisdiction. Petitioner points to P.D. No. 1606, as amended,
which treats of independent civil actions only in the last paragraph of Sec. 4 thereof:
Any provisions of law or Rules of Court to the contrary notwithstanding, the criminal
action and the corresponding civil action for the recovery of civil liability shall at all times
be simultaneously instituted with, and jointly determined in, the same proceeding by the
Sandiganbayan or the appropriate courts, the filing of the criminal action being deemed
to necessarily carry with it the filing of the civil action, and no right to reserve the filing of

such civil action separately from the criminal action shall be recognized: Provided,
however, That where the civil action had heretofore been filed separately but judgment
therein has not yet been rendered, and the criminal case is hereafter filed with the
Sandiganbayan or the appropriate court, said civil action shall be transferred to the
Sandiganbayan or the appropriate court, as the case may be, for consolidation and joint
determination with the criminal action, otherwise the separate civil action shall be
deemed abandoned.
Petitioner however did not raise any argument to refute the charge of forum-shopping.
The issues for resolution are: (a) whether the Sandiganbayan has jurisdiction over
petitions for forfeiture under R.A. No. 1379; (b) whether the Office of the Ombudsman
has the authority to investigate, initiate and prosecute such petitions for forfeiture; and
(c) whether petitioner is guilty of forum-shopping.
The petition is patently without merit. It should be dismissed.
The seminal decision of Republic v. Sandiganbayan33 squarely rules on the issues
raised by petitioner concerning the jurisdiction of the Sandiganbayan and the authority
of the Office of the Ombudsman. After reviewing the legislative history of the
Sandiganbayan and the Office of the Ombudsman, the Court therein resolved the
question of jurisdiction by the Sandiganbayan over violations of R.A. No. 3019 and R.A.
No. 1379. Originally, it was the Solicitor General who was authorized to initiate forfeiture
proceedings before the then Court of First Instance of the city or province where the
public officer or employee resides or holds office, pursuant to Sec. 2 of R.A. No.
1379. Upon the creation of the Sandiganbayan pursuant to P.D. No. 1486, 34 original and
exclusive jurisdiction over such violations was vested in the said court.35 P.D. No.
160636 was later issued expressly repealing P.D. No. 1486, as well as modifying the
jurisdiction of the Sandiganbayan by removing its jurisdiction over civil actions brought
in connection with crimes within the exclusive jurisdiction of said court.37 Such civil
actions removed from the jurisdiction of the Sandigabayan include those for restitution
or reparation of damages, recovery of instruments and effects of the crime, civil actions
under Articles 32 and 34 of the Civil Code, and forfeiture proceedings provided for
under R.A. No. 1379.38
Subsequently, Batas Pambansa Blg. 12939 abolished the concurrent jurisdiction of the
Sandiganbayan and the regular courts and expanded the exclusive original jurisdiction
of the Sandiganbayan over the offenses enumerated in Sec. 4 of P.D. No. 1606 to
embrace all such offenses irrespective of the imposable penalty. Since this change
resulted in the proliferation of the filing of cases before the Sandiganbayan where the
offense charged is punishable by a penalty not higher than prision correccional or its
equivalent, and such cases not being of a serious nature, P.D. No. 1606 was again
amended by P.D. No. 186040 and eventually by P.D. No. 1861.41
On the foregoing premises alone, the Court in Republic v. Sandiganbayan, deduced
that jurisdiction over violations of R.A. No. 3019 and 1379 is lodged with the

Sandiganbayan.42 It could not have taken into consideration R.A. No. 797543 and R.A.
No. 824944 since both statutes which also amended the jurisdiction of the
Sandiganbayan were not yet enacted at the time. The subsequent enactments only
serve to buttress the conclusion that the Sandiganbayan indeed has jurisdiction over
violations of R.A. No. 1379.
Under R.A. No. 8249, the Sandiganbayan is vested with exclusive original jurisdiction in
all cases involving violations of R.A. No. 3019, R.A. No. 1379, and Chapter II, Sec. 2,
Title VII, Book II of the Revised Penal Code, where one or more of the accused are
officials occupying the following positions whether in a permanent, acting or interim
capacity, at the time of the commission of the offense: (1) Officials of the executive
branch occupying the positions of regional director and higher, otherwise classified as
Grade '27' and higher, of the Compensation and Position Classification Act of 989 (R.A.
No. 6758), specifically including: (a) Provincial governors, vice-governors, members of
the sangguniang panlalawigan, and provincial treasurers, assessors, engineers, and
other city department heads; (b) City mayor, vice-mayors, members of the sangguniang
panlungsod, city treasurers, assessors, engineers, and other city department heads; (c)
Officials of the diplomatic service occupying the position of consul and higher;
(d) Philippine army and air force colonels, naval captains, and all officers of higher rank;
(e) Officers of the Philippine National Police while occupying the position of provincial
director and those holding the rank of senior superintended or higher; (f) City and
provincial prosecutors and their assistants, and officials and prosecutors in the Office of
the Ombudsman and special prosecutor; (g) Presidents, directors or trustees, or
managers of government-owned or controlled corporations, state universities or
educational institutions or foundations; (2) Members of Congress and officials thereof
classified as Grade '27' and up under the Compensation and Position Classification Act
of 1989; (3) Members of the judiciary without prejudice to the provisions of the
Constitution; (4) Chairmen and members of Constitutional Commission, without
prejudice to the provisions of the Constitution; and (5) All other national and local
officials classified as Grade '27' and higher under the Compensation and Position
Classification Act of 1989.45
In the face of the prevailing jurisprudence and the present state of statutory law on the
jurisdiction of the Sandiganbayan, petitioners argumentthat the Sandiganbayan has
no jurisdiction over the petition for forfeiture it being "civil" in nature and the
Sandiganbayan allegedly having no jurisdiction over civil actionscollapses completely.
The civil nature of an action for forfeiture was first recognized in Republic v.
Sandiganbayan, thus: "[T]he rule is settled that forfeiture proceedings are actions in
rem and, therefore, civil in nature."46 Then, Almeda, Sr.
v. Perez,47 followed, holding that the proceedings under R.A. No. 1379 do not terminate
in the imposition of a penalty but merely in the forfeiture of the properties illegally
acquired in favor of the State. It noted that the
procedure outlined in the law leading to forfeiture is that provided for in a civil action. 48

However, the Court has had occasion to rule that forfeiture of illegally acquired property
partakes the nature of a penalty. In Cabal v. Kapunan, Jr.,49 the Court cited voluminous
authorities in support of its declaration of the criminal or penal nature of forfeiture
proceedings, viz:
In a strict signification, a forfeiture is a divestiture of property without compensation, in
consequence of a default or an offense, and the term is used in such a sense in this
article. A forfeiture, as thus defined, is imposed by way of punishment not by the mere
convention of the parties, but by the lawmaking power, to insure a prescribed course of
conduct. It is a method deemed necessary by the legislature to restrain the commission
of an offense and to aid in the prevention of such an offense. The effect of such a
forfeiture is to transfer the title to the specific thing from the owner to the sovereign
power. (23 Am. Jur. 599)
"In Black's Law Dictionary a 'forfeiture' is defined to be 'the incurring of a liability to pay a
definite sum of money as the consequence of violating the provisions of some statute or
refusal to comply with some requirement of law.' It may be said to be a penalty imposed
for misconduct or breach of duty.'" (Com. vs. French, 114 S.W. 255.)
.
"Generally speaking, informations for the forfeiture of goods that seek no judgment of
fine or imprisonment against any person are deemed to be civil proceedings in
rem. Such proceedings are criminal in nature to the extent that where the person using
the res illegally is the owner of rightful possessor of it the forfeiture proceeding is in the
nature of a punishment. They have been held to be so far in the nature of
criminal proceedings that a general verdict on several counts in an information is upheld
if one count is good. According to the authorities such proceedings, where the owner of
the property appears, are so far considered as quasicriminal proceedings as to relieve
the owner from being a witness against himself and to prevent the compulsory
production of his books and papers. . . ." (23 Am. Jur. 612)
.
"Proceedings for forfeitures are generally considered to be civil and in the nature of
proceedings in rem. The statute providing that no judgment or other proceedings in civil
causes shall be arrested or reversed for any defect or want of form is applicable to
them. In some aspects, however, suits for penalties and forfeitures are of quasi-criminal
nature and within the reason of criminal proceedings for all the purposes of . . . that
portion of the Fifth Amendment which declares that no person shall be compelled in any
criminal case to be a witness against himself. The proceeding is one against the owner,
as well as against the goods; for it is his breach of the laws which has to be proved to
establish the forfeiture and his property is sought to be forfeited." (15 Am. Jur., Sec.
104, p. 368)50

Cabal v. Kapunan modified the earlier ruling in Almeda, Sr. v. Perez.51 The Court
in Cabal held that the doctrine laid down in Almeda refers to the purely procedural
aspect of the forfeiture proceedings and has no bearing on the substantial rights of
respondents, particularly their constitutional right against self-incrimination.52 This was
reaffirmed and reiterated in
Republic v. Agoncillo53 and Katigbak v. Solicitor General.54
The Sandiganbayan is vested with jurisdiction over violations of R.A. No. 1379,
entitled "An Act Declaring Forfeiture In Favor of the State Any Property Found to Have
Been Unlawfully Acquired By Any Public Officer or Employee and Providing For the
Proceedings Therefor." What acts would constitute a violation of such a law? A reading
of R.A. No. 1379 establishes that it does not enumerate any prohibited acts the
commission of which would necessitate the imposition of a penalty. Instead, it provides
the procedure for forfeiture to be followed in case a public officer or employee has
acquired during his incumbency an amount of property manifestly out of proportion to
his salary as such public officer or employee and to his lawful income and income from
legitimately acquired property.55 Section 1256 of the law provides a penalty but it is only
imposed upon the public officer or employee who transfers or conveys the unlawfully
acquired property; it does not penalize the officer or employee for making the unlawful
acquisition. In effect, as observed in Almeda, Sr. v. Perez, it imposes the penalty of
forfeiture of the properties unlawfully acquired upon the respondent public officer or
employee.57
It is logically congruent, therefore, that violations of R.A. No. 1379 are placed under the
jurisdiction of the Sandiganbayan, even though the proceeding is civil in nature, since
the forfeiture of the illegally acquired property amounts to a penalty. The soundness of
this reasoning becomes even more obvious when we consider that the respondent in
such forfeiture proceedings is a public officer or employee and the violation of R.A. No.
1379 was committed during the respondent officer or employees incumbency and in
relation to his office. This is in line with the purpose behind the creation of the
Sandiganbayan as an anti-graft courtto address the urgent problem of dishonesty in
public service.58
Following the same analysis, petitioner should therefore abandon his erroneous belief
that the Sandiganbayan has jurisdiction only over petitions for forfeiture filed against
President Marcos, his family and cronies.
We come then to the question of authority of the Office of the Ombudsman to
investigate, file and
prosecute petitions for forfeiture under R.A. No. 1379. This was the main issue resolved
in Republic v. Sandiganbayan.59
Under Sec. 2 of R.A. No. 1379, it was the Solicitor General who was authorized to
initiate forfeiture proceedings before the then Courts of First Instance. P.D. No. Decree

No. 1486 was later issued on 11 June 1978 vesting the Sandiganbayan with jurisdiction
over R.A. No. 1379 forfeiture proceedings. Sec. 12 of P.D. No. 1486 gave the Chief
Special Prosecutor the authority to file and prosecute forfeiture cases. This may be
taken as an implied repeal by P.D. No. 1486 of the jurisdiction of the former Courts of
First Instance and the authority of the Solicitor General to file a petition for forfeiture
under Sec. 2 of R.A. No. 1379 by transferring said jurisdiction and authority to the
Sandiganbayan and the Chief Special Prosecutor, respectively. 60 An implied repeal is
one which takes place when a new law contains some provisions which are contrary to,
but do not expressly repeal those of a former law.61 As a rule, repeals by implication are
not favored and will not be so declared unless it be manifest that the legislature so
intended. Before such repeal is deemed to exist, it must be shown that the statutes or
statutory provisions deal with the same subject matter and that the latter be inconsistent
with the former. The language used in the latter statute must be such as to render it
irreconcilable with what had been formerly enacted. An inconsistency that falls short of
that standard does not suffice. What is needed is a manifest indication of the legislative
purpose to repeal.62
P.D. No. 1486 contains a repealing clause which provides that "[A]ny provision of law,
order, rule or regulation inconsistent with the provisions of this Decree is hereby
repealed or modified accordingly."63 This is not an express repealing clause because it
fails to identify or designate the statutes that are intended to be repealed. Rather, it is a
clause which predicates the intended repeal upon the condition that a substantial
conflict must be found in existing and prior laws.64
The conflict between P.D. No. 1486 and R.A. No. 1379 refers to the jurisdiction over the
forfeiture proceeding and the authority to file the petition for forfeiture. As P.D. No. 1486
grants exclusive jurisdiction and authority to the Sandiganbayan and the Chief Special
Prosecutor, the then Courts of First Instance and Solicitor General cannot exercise
concurrent jurisdiction or authority over such cases. Hence, P.D. No. 1486 and Sec. 2,
R.A. No. 1379 are inconsistent with each other and the former should be deemed to
have repealed the latter.lawphil.net
On 11 June 1978, the same day that P.D. No. 1486 was enacted, P.D. No.
148765 creating the Office of the Ombudsman (then known as the Tanodbayan) was
passed. The Tanodbayan initially had no authority to prosecute cases falling within the
jurisdiction of the Sandiganbayan as provided in Sec. 4 of P.D. No. 1486, such
jurisdiction being vested in the Chief Special Prosecutor as earlier mentioned.
On 10 December 1978, P.D. No. 1606 was enacted expressly repealing P.D. No. 1486.
Issued on the same date was P.D. No. 160766 which amended the powers of the
Tanodbayan to investigate administrative complaints67and created the Office of the
Chief Special Prosecutor.68 P.D. No. 1607 provided said Office of the Chief Special
Prosecutor with exclusive authority to conduct preliminary investigation of all cases
cognizable by the Sandiganbayan, file informations therefor, and direct and control the
prosecution of said cases.69 P.D. No. 1607 also removed from the Chief Special
Prosecutor the authority to file actions for forfeiture under R.A. No. 1379. 70

The rule is that when a law which expressly repeals a prior law is itself repealed, the law
first repealed shall not be thereby revived unless expressly so provided. From this it
may fairly be inferred that the old rule continues in force where a law which repeals a
prior law, not expressly but by implication, is itself repealed; and that in such cases the
repeal of the repealing law revives the prior law, unless the language of the repealing
statute provides otherwise.71 Hence, the repeal of P.D. No. 1486 by P.D. No. 1606
necessarily revived the authority of the Solicitor General to file a petition for forfeiture
under R.A. No. 1379, but not the jurisdiction of the Courts of First Instance over the
case nor the authority of the Provincial or City Fiscals (now Prosecutors) to conduct the
preliminary investigation therefore, since said powers at that time remained in the
Sandiganbayan and the Chief Special Prosecutor.72
The Tanodbayans authority was further expanded by P.D. No. 163073 issued on 18 July
1990. Among other things, the Tanodbayan was given the exclusive authority to
conduct preliminary investigation of all cases cognizable by the Sandiganbayan, to file
informations therefore and to direct and control the prosecution of said cases. 74 The
power to conduct the necessary investigation and to file and prosecute the
corresponding criminal and administrative cases before the Sandiganbayan or the
proper court or administrative agency against any public personnel who has acted in a
manner warranting criminal and disciplinary action or proceedings was also transferred
from the Chief Special Prosecutor to the Tanodbayan.75
Thereafter, P.D. No. 1606 was amended by P.D. Nos. 1860 and 1861 76 which granted
the Tanodbayan the same authority. The present Constitution was subsequently ratified
and then the Tanodbayan became known as the Office of the Special Prosecutor which
continued to exercise its powers except those conferred on the Office of the
Ombudsman created under the Constitution.77 The Office of the Ombudsman was
officially created under R.A. No. 6770.78
At present, the powers of the Ombudsman, as defined by R.A. No. 6770, corollary to
Sec. 13, Art. XI of the Constitution, include the authority, among others, to:
(1) Investigate and prosecute on its own or on complaint by any person, any act or
omission of any public officer or employee, office or agency, when such act or omission
appears to be illegal, unjust, improper or inefficient. It has primary jurisdiction over
cases cognizable by the Sandiganbayan and, in the exercise of this primary jurisdiction,
may take over, at any stage, from any investigatory agency of Government, the
investigation of such cases;79

(11) Investigate and initiate the proper action for the recovery of ill-gotten and/or
unexplained wealth amassed after 25 February 1986 and the prosecution of the parties
involved therein.80

Ostensibly, it is the Ombudsman who should file the petition for forfeiture under R.A.
No. 1379. However, the Ombudsmans exercise of the correlative powers to investigate
and initiate the proper action for recovery of ill-gotten and/or unexplained wealth is
restricted only to cases for the recovery of ill-gotten and/or unexplained wealth
amassed after 25 February 1986.81 As regards such wealth accumulated on or before
said date, the Ombudsman is without authority to commence before the Sandiganbayan
such forfeiture actionsince the authority to file forfeiture proceedings on or before 25
February 1986 belongs to the Solicitor Generalalthough he has the authority to
investigate such cases for forfeiture even before 25 February 1986, pursuant to the
Ombudsmans general investigatory power under Sec. 15 (1) of R.A. No. 6770.82
It is obvious then that respondent Office of the Ombudsman acted well within its
authority in conducting the investigation of petitioners illegally acquired assets and in
filing the petition for forfeiture against him. The contention that the procedural
requirements under Sec. 2 of R.A. No. 1379 were not complied with no longer deserve
consideration in view of the foregoing discussion.
Now to the charge that petitioner is guilty of forum-shopping. Forum-shopping is
manifest whenever a party "repetitively avail[s] of several judicial remedies in different
courts, simultaneously or successively, all substantially founded on the same
transactions and the same essential facts and circumstances, and all raising
substantially the same issues either pending in, or already resolved adversely by, some
other court."83 It has also been defined as "an act of a party against whom an adverse
judgment has been rendered in one forum of seeking and possibly getting a favorable
opinion in another forum, other than by appeal or the special civil action of certiorari, or
the institution of two or more actions or proceedings grounded on the same cause on
the supposition that one or the other court would make a favorable
disposition."84 Considered a pernicious evil, it adversely affects the efficient
administration of justice since it clogs the court dockets, unduly burdens the financial
and human resources of the judiciary, and trifles with and mocks judicial
processes.85 Willful and deliberate forum-shopping is a ground for summary dismissal of
the complaint or initiatory pleading with prejudice and constitutes direct contempt of
court, as well as a cause for administrative sanctions, which may both be resolved and
imposed in the same case where the forum-shopping is found.86
There is ample reason to hold that petitioner is guilty of forum-shopping. The present
petition was filed accompanied by the requisite Verification and Certification Against
Forum Shopping87 in which petitioner made the following representation:
.
3.] As Petitioner, I have not heretofore commenced any other action or proceeding in
the Supreme Court, the Court of Appeals, or any other tribunal or agency, involving the
same issues as that in the above-captioned case.

4.] To the best of my knowledge, no such action or proceeding is pending in the


Supreme Court, the Court of Appeals, or any other tribunal or agency.
5.] If I should hereafter learn that such proceeding has been commenced or is pending
before the Supreme Court, the Court of Appeals, or any other tribunal or agency, I
undertake to report that fact to this Honorable Court within five (5) days from knowledge
thereof.
However, petitioner failed to inform the Court that he had filed a Motion to Dismiss88 in
relation to the petition for forfeiture before the Sandiganbayan. The existence of this
motion was only brought to the attention of this Court by respondent Office of the
Ombudsman in its Comment. A scrutiny of the Motion to Dismiss reveals that petitioner
raised substantially the same issues and prayed for the same reliefs therein as it has in
the instant petition. In fact, the Arguments and Discussion89 in the Petition of petitioners
thesis that the Sandiganbayan has no jurisdiction over separate civil actions for
forfeiture of unlawfully acquired properties appears to be wholly lifted from the Motion to
Dismiss. The only difference between the two is that in the Petition, petitioner raises the
ground of failure of the petition for forfeiture to comply with the procedural requirements
of R.A. No. 1379, and petitioner prays for the annulment of the
Sandiganbayans Resolution dated 29 October 2004 and Writ of Preliminary
Attachment dated 2 November 2004. Nevertheless, these differences are only
superficial. BothPetition and Motion to Dismiss have the same intent of dismissing the
case for forfeiture filed against petitioner, his wife and their sons. It is undeniable that
petitioner had failed to fulfill his undertaking. This is incontestably forum-shopping which
is reason enough to dismiss the petition outright, without prejudice to the taking of
appropriate action against the counsel and party concerned.90 The brazenness of this
attempt at forum-shopping is even demonstrated by the fact that both
the Petition and Motion to Dismiss were filed on the same day, 17 November 2004.
Petitioner should have waited for the resolution of his Motion to Dismiss before resorting
to the petition at hand.
Petitioners counsel of record, Atty. Constantino B. De Jesus, needs to be reminded that
his primary duty is to assist the courts in the administration of justice. As an officer of
the court, his duties to the court are more significant and important than his obligations
to his clients. Any conduct which tends to delay, impede or obstruct the administration
thereof contravenes his oath of office.91 Atty. De Jesus failed to accord due regard, as
he must, the tenets of the legal profession and the mission of our courts of justice. For
this, he should be penalized. Penalties imposed upon lawyers who engaged in forumshopping range from severe censure to suspension from the practice of law. 92 In the
instant case, we deem the imposition of a fine in the amount of P20,000.00 to be
sufficient to make Atty. De Jesus realize the seriousness of his naked abuse of the
judicial process.
WHEREFORE, in view of the foregoing, the Petition is DISMISSED. Atty. Constantino
B. De Jesus is DECLARED in CONTEMPT of this Court and meted a fine of Twenty

Thousand Pesos (P20,000.00) to be paid within ten (10) days from the finality of this D
E C I S I O N. Costs against petitioner.
SO ORDERED.

FERDINAND R. MARCOS, JR.


Petitioner,

G.R. No. 189434

- versus REPUBLIC OF THE PHILIPPINES,


represented by the Presidential Commission
on Good Government,
Respondent.

x--------------------------x
IMELDA ROMUALDEZ-MARCOS,
Petitioner ,

G.R. No. 189505


Present:

- versus REPUBLIC OF THE PHILIPPINES,


Respondent.

BRION, J.,
Acting Chairperson,
ABAD,
PEREZ,
SERENO, and
REYES, JJ.
Promulgated:
April 25, 2012

x - - - - - - - - - - - - - - - - - - - - - - - - - - - - - - - - - - - - - - - - - - - - - - - - - - -x
DECISION
SERENO, J.:
These two consolidated Petitions filed under Rule 45 of the 1997 Rules of Civil Procedure
pray for the reversal of the 2 April 2009 Decision of the Sandiganbayan in Civil Case No. 0141
entitled Republic of the Philippines v. Heirs of Ferdinand E. Marcos and Imelda R.
Marcos.[1] The anti-graft court granted the Motion for Partial Summary Judgment filed by
respondent Republic of the Philippines (Republic) and declared all assets and properties of
Arelma, S.A., an entity created by the late Ferdinand E. Marcos, forfeited in favor of the
government.
On 17 December 1991, the Republic, through the Presidential Commission on Good Government
(PCGG), filed a Petition for Forfeiture[2] before the Sandiganbayan pursuant to the forfeiture law,

Republic Act No. 1379 (R.A. 1379)[3] in relation to Executive Order Nos. 1, 2 and 14.[4] The
petition was docketed as Civil Case No. 0141.
Respondent Republic, through the PCGG and the Office of the Solicitor General (OSG), sought
the declaration of Swiss bank accounts totaling USD 356 million (now USD 658 million), and two
treasury notes worth USD 25 million and USD 5 million, as ill-gotten wealth.[5] The Swiss
accounts, previously held by five groups of foreign foundations,[6]were deposited in escrow with
the Philippine National Bank (PNB), while the treasury notes were frozen by the Bangko Sentral
ng Pilipinas (BSP).
Respondent also sought the forfeiture of the assets of dummy corporations and entities
established by nominees of Marcos and his wife, Petitioner Imelda Romualdez-Marcos, as well as
real and personal properties manifestly out of proportion to the spouses lawful income. This claim
was based on evidence collated by the PCGG with the assistance of the United States Justice
Department and the Swiss Federal Police Department.[7] The Petition for Forfeiture described
among others, a corporate entity by the name Arelma, Inc., which maintained an account and
portfolio in Merrill Lynch, New York, and which was purportedly organized for the same purpose
of hiding ill-gotten wealth.[8]
Before the case was set for pretrial, the Marcos children and PCGG Chairperson
Magtanggol Gunigundo signed several Compromise Agreements (a General Agreement and
Supplemental Agreements) all dated 28 December 1993 for a global settlement of the Marcos
assets. One of the whereas clauses in the General Agreement specified that the Republic obtained
a judgment from the Swiss Federal Tribunal on December 21, 1990, that the Three Hundred Fiftysix Million U.S. dollars (USD 356 million) belongs in principle to the Republic of the Philippines
provided certain conditionalities are met xxx. This Decision was in turn based on the finding of
Zurich District Attorney Peter Cosandey that the deposits in the name of the foundations were of
illegal provenance.[9]
On 18 October 1996, respondent Republic filed a Motion for Summary Judgment and/or
judgment on the pleadings (the 1996 Motion) pertaining to the forfeiture of the USD 356 million.
The Sandiganbayan denied the 1996 Motion on the sole ground that the Marcoses had earlier
moved for approval of the Compromise Agreements, and that this latter Motion took precedence
over that for summary judgment. Petitioner Imelda Marcos filed a manifestation claiming she was
not a party to the Motion for Approval of the Compromise Agreements, and that she owned 90%
of the funds while the remaining 10% belonged to the Marcos estate.[10]

On 10 March 2000, the Republic filed another Motion for Summary Judgment (the 2000
Motion), based on the grounds that: (1) the essential facts that warrant the forfeiture of the funds
subject of the Petition under R.A. 1379 are admitted by respondents in their pleadings and other
submissions; and (2) the respondent Marcoses pretrial admission that they did not have any interest
or ownership over the funds subject of the action for forfeiture tendered no genuine issue or
controversy as to any material fact.
In a 19 September 2000 Decision, the Sandiganbayan initially granted the 2000 Motion,
declaring that the Swiss deposits held in escrow at the PNB were ill-gotten wealth, and, thus,
forfeited in favor of the State.[11] In a Resolution dated 31 January 2002, the Sandiganbayan
reversed its earlier ruling and denied the 2000 Motion. Alleging grave abuse of discretion on the
part of the court in rendering the later Resolution, the Republic filed a Petition for Certiorari with
the Supreme Court. In G.R. No. 152154 entitledRepublic of the Philippines v. Sandiganbayan (for
brevity, the Swiss Deposits Decision),[12] this Court set aside the 31 January 2002 Sandiganbayan
Resolution and reinstated the 19 September 2000 Decision, including the declaration that the Swiss
deposits are ill-gotten wealth. On 18 November 2003, the Court denied with finality petitioner
Marcoses Motion for Reconsideration.
On 16 July 2004, the Republic filed a Motion for Partial Summary Judgment (2004 Motion)
to declare the funds, properties, shares in and interests of ARELMA, wherever they may be
located, as ill-gotten assets and forfeited in favor of the Republic of the Philippines pursuant to
R.A. 1379 in the same manner (that) the Honorable Supreme Court forfeited in favor of the
petitioner the funds and assets of similar Marcos foundations such as AVERTINA, VIBUR,
AGUAMINA, MALER and PALMY.[13] Petitioner contends that: (1) respondents are deemed to
have admitted the allegations of the Petition as regards Arelma; and (2) there is no dispute that the
combined lawful income of the Marcoses is grossly disproportionate to the deposits of their
foundations and dummy corporations, including Arelma. Ferdinand Marcos, Jr., Imelda Marcos,
and Imee Marcos-Manotoc filed their respective Oppositions. Irene Marcos-Araneta filed a Motion
to Expunge on the ground that the proceedings in Civil Case No. 0141 had already terminated.
On 2 April 2009, the Sandiganbayan rendered the assailed Decision granting respondents
Motion for Partial Summary Judgment.[14] It found that the proceedings in Civil Case No. 0141
had not yet terminated, as the Petition for Forfeiture included numerous other properties, which
the Sandiganbayan and Supreme Court had not yet ruled upon. The Republics 1996 Motion was
merely held in abeyance to await the outcome of the global settlement of the Marcos assets.
Further, this development had prompted the Republic to file the 2000 Motion, which was clearly

limited only to the Swiss accounts amounting to USD 356 million. Thus, according to the
Sandiganbayan, its 19 September 2000 Decision as affirmed by the Supreme Court in G.R. No.
152154, was in the nature of a separate judgment over the Swiss accounts and did not preclude a
subsequent judgment over the other properties subject of the same Petition for Forfeiture, such as
those of Arelma.[15] The Sandiganbayan held as follows:
WHEREFORE, considering all the foregoing, the Motion for Partial
Summary
Judgment
dated
July
16,
2004
of
petitioner
is
hereby GRANTED. Accordingly, Partial Summary Judgment is hereby rendered
declaring the assets, investments, securities, properties, shares, interests, and funds
of Arelma, Inc., presently under management and/or in an account at the Meryll
(sic) Lynch Asset Management, New York, U.S.A., in the estimated aggregate
amount of US$3,369,975.00 as of 1983, plus all interests and all other income that
accrued thereon, until the time or specific day that all money or monies are released
and/or transferred to the possession of the Republic of the Philippines, are hereby
forfeited in favor of petitioner Republic of the Philippines.
SO ORDERED.[16]

On 22 October 2009, Ferdinand R. Marcos, Jr. filed the instant Rule 45 Petition,
questioning the said Decision.[17] One week later, Imelda Marcos filed a separate Rule 45
Petition[18] on essentially identical grounds, which was later consolidated with the first Petition.
The grievances of both petitioners boil down to the following issues:
1.
Whether the forfeiture proceeding, Civil Case No. 0141 with the
Sandiganbayan is criminal in nature, such that summary judgment is not allowed;
2.
Whether petitioner Republic complied with Section 3, subparagraphs c, d, and
e of R.A. 1375;
3.
Whether Civil Case No. 0141 has been terminated such that a motion for
partial summary judgment may no longer be allowed; and
4.
Whether in this case there are genuine, triable issues which would preclude
the application of the rule on summary judgment.
I.
Forfeiture proceedings are civil in
nature

Petitioner Ferdinand Marcos, Jr. argues that R.A. 1379 is a penal law; therefore a person charged
under its provisions must be accorded all the rights granted to an accused under the Constitution
and penal laws.[19] He asserts that the Marcoses were entitled to all the substantial rights of an
accused, one of these being the right to present their evidence to a full blown trial as per Section 5
of R.A. 1379.[20] He relies on the 1962 case, Cabal v. Kapunan,[21] where the Court ruled that:
We are not unmindful of the doctrine laid down in Almeda vs. Perez, L18428 (August 30, 1962) in which the theory that, after the filing of respondents'
answer to a petition for forfeiture under Republic Act No. 1379, said petition may
not be amended as to substance pursuant to our rules of criminal procedure, was
rejected by this Court upon the ground that said forfeiture proceeding is civil in
nature. This doctrine refers, however, to the purely procedural aspect of said
proceeding, and has no bearing on the substantial rights of the respondents therein,
particularly their constitutional right against self-incrimination.

This argument fails to convince. Petitioner conveniently neglects to quote from the preceding
paragraphs of Cabal, which clearly classified forfeiture proceedings as quasi-criminal, not
criminal. And even so, Cabal declared that forfeiture cases partake of a quasi-criminal nature only
in the sense that the right against self-incrimination is applicable to the proceedings, i.e., in which
the owner of the property to be forfeited is relieved from the compulsory production of his books
and papers:
Generally speaking, informations for the forfeiture of goods that seek no
judgment of fine or imprisonment against any person are deemed to be civil
proceedings in rem. Such proceedings are criminal in nature to the extent that
where the person using the res illegally is the owner or rightful possessor of it, the
forfeiture proceeding is in the nature of a punishment.
xxx xxx xxx
Proceedings for forfeitures are generally considered to be civil and in the
nature of proceedings in rem. The statute providing that no judgment or other
proceedings in civil cases shall be arrested or reversed for any defect or want of
form is applicable to them. In some aspects, however, suits for penalties and
forfeitures are of quasi-criminal nature and within the reason of criminal
proceedings for all the purposes of * * * that portion of the Fifth Amendment which
declares that no person shall be compelled in any criminal case to be a witness
against himself. The proceeding is one against the owner, as well as against the
goods; for it is his breach of the laws which has to be proved to establish the
forfeiture and his property is sought to be forfeited.
xxx xxx xxx

As already observed, the various constitutions provide that no person shall


be compelled in any criminal case to be a witness against himself. This prohibition
against compelling a person to take the stand as a witness against himself
applies only to criminal, quasi-criminal, and penal proceedings, including a
proceeding civil in form for forfeiture of property by reason of the commission of
an offense, but not a proceeding in which the penalty recoverable is civil or
remedial in nature. (Emphasis supplied.)[22]

The right of the Marcoses against self-incrimination has been amply protected by the
provisions of R.A. 1379, which prohibits the criminal prosecution of individuals for or on account
of any transaction, matter or thing concerning which they are compelled -- after having claimed
the privilege against self-incrimination -- to testify or produce evidence, documentary or
otherwise.[23] Since this cases inception in 1991, petitioners have participated in the hearings,
argued their case, and submitted their pleadings and other documents, never once putting at issue
their right against self-incrimination or the violation thereof.[24]
More importantly, the factual context in the present case is wholly disparate from that
in Cabal, which was originally initiated as an action in personam. Manuel C. Cabal, then Chief of
Staff of the Armed Forces of the Philippines, was charged with graft, corrupt practices,
unexplained wealth, conduct unbecoming of an officer and gentleman, dictatorial tendencies,
giving false statements of his assets and liabilities in 1958 and other equally reprehensible
acts.[25] In contradistinction, the crux of the present case devolves solely upon the recovery of
assets presumptively characterized by the law as ill-gotten, and owned by the State; hence, it is an
action in rem. In Republic v. Sandiganbayan, this Court settled the rule that forfeiture proceedings
are actions in rem and therefore civil in nature.[26] Proceedings under R.A. 1379 do not terminate
in the imposition of a penalty but merely in the forfeiture of the properties illegally acquired in
favor of the State.[27]
As early as Almeda v. Judge Perez,[28] we have already delineated the difference between
criminal and civil forfeiture and classified the proceedings under R.A. 1379 as belonging to the
latter, viz:
Forfeiture proceedings may be either civil or criminal in nature,
and may be in rem or in personam. If they are under a statute such
that if an indictment is presented the forfeiture can be included in
the criminal case, they are criminal in nature, although they may be
civil in form; and where it must be gathered from the statute that the

action is meant to be criminal in its nature it cannot be considered


as civil. If, however, the proceeding does not involve the conviction
of the wrongdoer for the offense charged the proceeding is of a civil
nature; and under statutes which specifically so provide, where the
act or omission for which the forfeiture is imposed is not also a
misdemeanor, such forfeiture may be sued for and recovered in a
civil action.
In the first place a proceeding under the Act (Rep. Act No. 1379) does not
terminate in the imposition of a penalty but merely in the forfeiture of the properties
illegally acquired in favor of the state. (Sec. 6) In the second place the procedure
outlined in the law leading to forfeiture is that provided for in a civil action. Thus
there is a petition (Sec. 3), then an answer (Sec. 4), and lastly, a hearing. The
preliminary investigation which is required prior to the filing of the petition, in
accordance with Sec. 2 of the Act, is provided expressly to be one similar to a
preliminary investigation in a criminal case. If the investigation is only similar to
that in a criminal case, but the other steps in the proceedings are those for civil
proceedings, it stands to reason that the proceeding is not criminal. xxx. (citations
omitted)

Forfeiture cases impose neither a personal criminal liability, nor the civil liability that arises
from the commission of a crime (ex delicto). The liability is based solely on a statute that
safeguards the right of the State to recover unlawfully acquired properties.[29] Executive Order No.
14 (E.O. No. 14), Defining the Jurisdiction Over Cases Involving the Ill-gotten Wealth of Former
President Ferdinand Marcos, authorizes the filing of forfeiture suits that will proceed
independently of any criminal proceedings. Section 3 of E.O. 14 empowered the PCGG to file
independent civil actions separate from the criminal actions.[30]
Thus, petitioners cannot equate the present case with a criminal case and assail the
proceedings before the Sandiganbayan on the bare claim that they were deprived of a full-blown
trial. In affirming the Sandiganbayan and denying petitioners Motion for Reconsideration in the
Swiss Deposits Decision, the Court held:
Section 5 of RA 1379 provides:
The court shall set a date for a hearing which may be open
to the public, and during which the respondent shall be given
ample opportunity to explain, to the satisfaction of the court,
how he has acquired the property in question.

And pursuant to Section 6 of the said law, if the respondent is unable to


show to the satisfaction of the court that he has lawfully acquired the property in
question, then the court shall declare such property forfeited in favor of the State.
xxx xxx xxx
A careful analysis of Section 5 of RA 1379 readily discloses that the word
hearing does not always require the formal introduction of evidence in a trial, only
that the parties are given the occasion to participate and explain how they acquired
the property in question. If they are unable to show to the satisfaction of the court
that they lawfully acquired the property in question, then the court shall declare
such property forfeited in favor of the State. There is no provision in the law that a
full blown trial ought to be conducted before the court declares the forfeiture of the
subject property. Thus, even if the forfeiture proceedings do not reach trial, the
court is not precluded from determining the nature of the acquisition of the property
in question even in a summary proceeding.[31]

As forfeiture suits under R.A. 1379 are civil in nature, it follows that Rule 35 of the Rules
of Court on Summary Judgment may be applied to the present case. This is consistent with our
ruling in the Swiss Deposits Decision upholding the summary judgment rendered by the
Sandiganbayan over the Swiss deposits, which are subject of the same Petition for Forfeiture as
the Arelma assets.
II. Republic complied with Section 3 (c),
(d), and (e) of R.A. 1375

Petitioner Marcos, Jr. argues that there are genuine issues of fact as borne by the Pre-trial
Order, Supplemental Pre-trial Order, and the Pre-trial Briefs of the parties. He laments that the
Republic was unable to meet the necessary averments under the forfeiture law, which requires a
comparison between the approximate amount of property acquired during the incumbency of
Ferdinand Marcos, and the total amount of governmental salaries and other earnings.[32] While the
Petition contained an analysis of Ferdinand Marcoss income from 1965 to 1986 (during his
incumbency), there was purportedly no mention of the latters income from 1940 to 1965 when he
was a practicing lawyer, congressman and senator; other earnings until the year 1985; and real
properties that were auctioned off to satisfy the estate tax assessed by the Bureau of Internal
Revenue.[33]
Petitioner Marcos, Jr. implores us herein to revisit and reverse our earlier ruling in the Swiss
Deposits Decision and argues that the pronouncements in that case are contrary to law and its basic

tenets. The Court in that case allegedly applied a lenient standard for the Republic, but a strict one
for the Marcoses. He finds fault in the ruling therein which was grounded on public policy and the
ultimate goal of the forfeiture law, arguing that public policy is better served if the Court gave
more importance to the substantive rights of the Marcoses.
In accordance with the principle of immutability of judgments, petitioners can no longer use the
present forum to assail the ruling in the Swiss Deposits Decision, which has become final and
executory. Aside from the fact that the method employed by petitioner is improper and redundant,
we also find no cogent reason to revisit the factual findings of the Sandiganbayan in Civil Case
No. 0141, which this Court in the Swiss Deposits Decision found to be thorough and convincing.
In the first place, using a Rule 45 Petition to question a judgment that has already become final is
improper, especially when it seeks reconsideration of factual issues, such as the earnings of the
late President from 1940 to 1965 and the existence of real properties that petitioners claim were
auctioned off to pay the taxes. Secondly, petitioners never raised the existence of these earnings
and real properties at the outset and never mentioned these alleged other incomes by way of
defense in their Answer. In their Answer, and even in their subsequent pleadings, they merely
made general denials of the allegations without stating facts admissible in evidence at the hearing.
As will be discussed later, both the Sandiganbayan and the Supreme Court found that the Marcoses
unsupported denials of matters patently and necessarily within their knowledge were inexcusable,
and that a trial would have served no purpose at all.[34]
R.A. 1379 provides that whenever any public officer or employee has acquired during his
incumbency an amount of property manifestly out of proportion to his salary as such public officer
and to his other lawful income, said property shall be presumed prima facie to have been
unlawfully acquired.[35] The elements that must concur for this prima facie presumption to apply
are the following: (1) the offender is a public officer or employee; (2) he must have acquired a
considerable amount of money or property during his incumbency; and (3) said amount is
manifestly out of proportion to his salary as such public officer or employee and to his other lawful
income and income from legitimately acquired property.
Thus, in determining whether the presumption of ill-gotten wealth should be applied, the relevant
period is incumbency, or the period in which the public officer served in that position. The amount
of the public officers salary and lawful income is compared against any property or amount
acquired for that same period. In the Swiss Deposits Decision, the Court ruled that petitioner
Republic was able to establish the prima facie presumption that the assets and properties acquired

by the Marcoses were manifestly and patently disproportionate to their aggregate salaries as public
officials.[36]
For a petition to flourish under the forfeiture law, it must contain the following:
(a)

The name and address of the respondent.

(b)

The public officer or employment he holds and such other public offices
or employment which he has previously held.

(c)

The approximate amount of property he has acquired during his


incumbency in his past and present offices and employments.

(d)

A description of said property, or such thereof as has been identified


by the Solicitor General.

(e)

The total amount of his government salary and other proper earnings
and incomes from legitimately acquired property, and

(f)

Such other information as may enable the court to determine whether or


not the respondent has unlawfully acquired property during his
incumbency.[37] (Emphasis supplied)

Petitioners claim that the Republic failed to comply with subparagraphs c, d, and e above, because
the latter allegedly never took into account the years when Ferdinand Marcos served as a war
veteran with back pay, a practicing lawyer, a trader and investor, a congressman and senator. We
find this claim to be a haphazard rehash of what has already been conclusively determined by the
Sandiganbayan and the Supreme Court in the Swiss Deposits Decision. The alleged receivables
from prior years were without basis, because Marcos never had a known law office nor any known
clients, and neither did he file any withholding tax certificate that would prove the existence of a
supposedly profitable law practice before he became President. As discussed in the Swiss Deposits
Decision:
The Solicitor General made a very thorough presentation of its case for
forfeiture:
xxx

xxx

xxx

4. Respondent Ferdinand E. Marcos (now deceased and represented by his


Estate/Heirs) was a public officer for several decades continuously and without
interruption as Congressman, Senator, Senate President and President of the
Republic of the Philippines from December 31, 1965 up to his ouster by direct
action of the people of EDSA on February 22-25, 1986.
5. Respondent Imelda Romualdez Marcos (Imelda, for short) the former
First Lady who ruled with FM (Ferdinand Marcos) during the 14-year martial law
regime, occupied the position of Minister of Human Settlements from June 1976
up to the peaceful revolution in February 22-25, 1986. She likewise served once as
a member of the Interim Batasang Pambansa during the early years of martial law
from 1978 to 1984 and as Metro Manila Governor in concurrent capacity as
Minister of Human Settlements.
xxx

xxx

xxx

11. At the outset, however, it must be pointed out that based on the Official
Report of the Minister of Budget, the total salaries of former President Marcos
as President from 1966 to 1976 was 60,000 a year and from 1977 to
1985, 100,000 a year; while that of the former First Lady, Imelda R. Marcos,
as Minister of Human Settlements from June 1976 to February 22-25, 1986
was 75,000 a year.[38]

The Sandiganbayan found that neither the late Ferdinand Marcos nor petitioner Imelda Marcos
filed any Statement of Assets and Liabilities, as required by law, from which their net worth could
be determined. Coupled with the fact that the Answer consisted of general denials and a standard
plea of lack of knowledge or information sufficient to form a belief as to the truth of the allegations
what the Court characterized as foxy replies and mere pretense fairness dictates that what must be
considered as lawful income should only be the accumulated salaries of the spouses and what are
shown in the public documents they submitted, such as their Income Tax Return (ITR) and their
Balance Sheets. The amounts representing the combined salaries of the spouses were admitted by
petitioner Imelda Marcos in paragraph 10 of her Answer, and reflected in the Certification dated
May 27, 1986 issued by then Minister of Budget and Management Alberto Romulo:
Ferdinand E. Marcos, as President
1966-1976

at 60,000/year

660,000

1977-1984

at 100,000/year

800,000

1985

at 110,000/year

110,000

1,570,00
Imelda R. Marcos, as Minister
June 1976-1985

at 75,000/year

718,000

In addition to their accumulated salaries from 1966 to 1985 are the Marcos couple's
combined salaries from January to February 1986 in the amount of 30,833.33.
Hence, their total accumulated salaries amounted to 2,319,583.33. Converted to
U.S. dollars on the basis of the corresponding peso-dollar exchange rates prevailing
during the applicable period when said salaries were received, the total amount had
an equivalent value of $304,372.43.[39]

The date contained in the ITRs and Balance Sheets filed by the Marcoses are summarized in
Schedules A to D submitted as evidence by the Republic. Schedule A showed that from 1965 to
1984, the Marcoses reported Php 16,408,442.00 or USD 2,414,484.91 in total income, comprised
of:
Income
Source

Amount

Percentage

Official
Salaries

- 2,627,581.00

- 16.01%

Legal
Practice

- 11,109,836.00

- 67.71%

Farm Income - 149,700.00

- .91%

Others

- 15.37%

Total

- 2,521,325.00

16,408,442.00 - 100.00%

The amount reported by the Marcos couple as their combined salaries more or less coincided with
the Official Report submitted by the Minister of Budget. Yet what appeared anomalous was the
Php 11,109,836 representing Legal Practice, which accounted for 67% or more than three-fourths

of their reported income. Out of this anomalous amount, Php 10,649,836, or 96%
thereof, represented receivables from prior years during the period 1967 to 1984. The Court cited
the Solicitor Generals findings:
In the guise of reporting income using the cash method under Section 38 of
the National Internal Revenue Code, FM made it appear that he had an extremely
profitable legal practice before he became a President (FM being barred by law
from practicing his law profession during his entire presidency) and that, incredibly,
he was still receiving payments almost 20 years after. The only problem is that in
his Balance Sheet attached to his 1965 ITR immediately preceding his
ascendancy to the presidency he did not show any Receivables from client at
all, much less the 10.65-M that he decided to later recognize as income. There
are no documents showing any withholding tax certificates. Likewise, there is
nothing on record that will show any known Marcos client as he has no known
law office. As previously stated, his net worth was a mere 120,000.00 in
December, 1965. The joint income tax returns of FM and Imelda cannot, therefore,
conceal the skeletons of their kleptocracy.[40]

In addition, the former President also reported a total of Php 2,521,325 which he referred to as
Miscellaneous Items and Various Corporations under Other Income for 1972-1976. Spouses
Marcos did not declare any income from any deposits that may be subject to a 5% withholding
tax, nor did they file any capital gains tax returns from 1960 to 1965. The Bureau of Internal
Revenue attested that there are no records pertaining to the tax transactions of the spouses in
Baguio City, Manila, Quezon City, and Tacloban.
The Balance Sheet attached to the couples ITR for 1965 indicates an ending net worth of Php
120,000, which covered the year immediately preceding their ascendancy to the presidency. As
previously mentioned, the combined salaries of the spouses for the period 1966 to 1986, or in the
two decades that they stayed in power, totaled only USD 304,372.43. In stark contrast, as shown
by Schedule D, computations establish the total net worth of the spouses for the years 1965 until
1984 in the total amount of USD 957,487.75, assuming that the income from legal practice is real
and valid.[41] The combined salaries make up only 31.79% of the spouses total net worth from
1965 to 1984. This means petitioners are unable to account for or explain more than twothirds of the total net worth of the Marcos spouses from 1965 to 1984.
Thus, for the final time, we soundly reiterate that the Republic was able to establish
the prima facie presumption that the assets and properties acquired by the Marcoses were
manifestly and patently disproportionate to their aggregate salaries as public officials. The

Republic presented further evidence that they had bigger deposits beyond their lawful incomes,
foremost of which were the Swiss accounts deposited in the names of five foundations spirited
away by the couple to different countries. Petitioners herein thus failed to overturn this
presumption when they merely presented vague denials and pleaded lack of sufficient knowledge
in their Answer.
In any case, petitioners may no longer question the findings of the Sandiganbayan affirmed
by the Supreme Court in the Swiss Deposits Decision, as these issues have long become the law
of the case in the original Petition for Forfeiture. As held in Philippine Coconut Producers
Federation, Inc. (COCOFED) v. Republic:[42]
Law of the case is a term applied to an established rule that when an
appellate court passes on a question and remands the case to the lower court for
further proceedings, the question there settled becomes the law of the case upon
subsequent appeal. It means that whatever is once irrevocably established as the
controlling legal rule or decision between the same parties in the same case
continues to be the law of the case, so long as the facts on which such decision was
predicated continue to be the facts of the case before the court.
Otherwise put, the principle means that questions of law that have been previously
raised and disposed of in the proceedings shall be controlling in succeeding
instances where the same legal question is raised, provided that the facts on which
the legal issue was predicated continue to be the facts of the case before the court.

In the case at bar, the same legal issues are being raised by petitioners. In fact, petitioner
Marcos Jr. admits outright that what he seeks is a reversal of the issues identical to those already
decided by the Court in the Swiss Deposits Decision.[43] He may not resuscitate, via another
petition for review, the same issues long laid to rest and established as the law of the case.
III. Civil Case No. 0141 has not yet
terminated

Petitioners next argue that the law of the case doctrine should be applied, not to the ruling affirming
the forfeiture, but to the grant of the summary judgment over the Swiss accounts as affirmed by
the Supreme Court in the Swiss Deposits Decision. They contend that since the Courts Decision
mentioned only the deposits under the five Swiss foundations, then the Republic can no longer
seek partial summary judgment for forfeiture over the Arelma account. And since the said Decision
has long become final and has in fact been executed, they insist that the Sandiganbayan has lost
its jurisdiction over the case.

Petitioners are under the mistaken impression that the Swiss Deposits Decision serves as the entire
judgment in Civil Case No. 0141. Just because respondent Republic succeeded in obtaining
summary judgment over the Swiss accounts does not mean it is precluded from seeking partial
summary judgment over a different subject matter covered by the same petition for forfeiture. In
fact, Civil Case No. 0141 pertains to the recovery of all the assets enumerated therein, such as (1)
holding companies, agro-industrial ventures and other investments; (2) landholdings, buildings,
condominium units, mansions; (3) New York properties; (4) bills amounting to Php 27,744,535,
time deposits worth Php 46.4 million, foreign currencies and jewelry seized by the United States
customs authorities in Honolulu, Hawaii; (5) USD 30 million in the custody of the Central Bank
in dollar-denominated Treasury Bills; shares of stock, private vehicles, and real estate in the United
States, among others.[44]
In the enumeration of properties included in the Petition, the Arelma assets were described as
Assets owned by Arelma, Inc., a Panamanian corporation organized in Liechtenstein, for sole
purpose (sic) of maintaining an account in Merrill Lynch, New York.[45] Paragraph 59 of the
Petition for Forfeiture states:
59. FM and Imelda used a number of their close business associations or favorite
cronies in opening bank accounts abroad for the purpose of laundering their filthy
riches. Aside from the foundations and corporations established by their
dummies/nominees to hide their ill-gotten wealth as had already been discussed,
several other corporate entities had been formed for the same purpose, to wit:
(1). ARELMA, INC (T)his was organized for the sole purpose of
maintaining an account and portfolio in Merrill Lynch, New York.
(2). Found among Malacaang documents is a letter dated September 21,
1972 by J.L. Sunier, Senior Vice President of SBC to Mr. Jose V. Campos, a known
Marcos crony (See Annex V-21 hereof). In the said letter, instructions were given
by Sunier to their Panama office to constitute a Panamanian company, the name of
which will be either Larema, Inc. or Arelma, Inc., orRelma, Inc. this company will
have the same set-up as Maler; the appointment of Sunier and Dr. Barbey as
attorneys and appointment of selected people in Panama as directors; the opening
of direct account in the name of the new company with Merrill Lynch, New York,
giving them authority to operate the account, but excluding withdrawals of cash,
securities or pledging of portfolio; and sending of money in favor of the new
company under reference AZUR in order to cut links with the present account
already opened with Merrill Lynch under an individuals name.

(3). Also found was a letter dated November 14, 1972 and signed by Jose
Y. Campos (Annex V-21-a hereof). The letter was addressed to SEC, Geneva, and
Sunier duly authorized by their mutual friend regarding the opening of an account
of Arelma, Inc. with Merrill Lynch, New York to the attention of Mr. Saccardi,
Vice-President.
(4). On May 19, 1983, J. L. Sunier wrote a letter with a reference
SAPPHIRE and a salutation Dear Excellency stating, among others, the current
valuation by Merrill Lynch of the assets of Arelma, Inc. amounting to $3,369,975
(Annex V-21-b hereof).
(5). Included in the documents sent by SBC, Geneva, through the Swiss
Federal Department of Justice and Police were those related to Arelma, Inc. as
follows:
(a) Opening bank documents for Account No. 53.145 A.R. dated September
17, 1972, signed by Dr. Barbey and Mr. Sunier. This was later on cancelled as a
result of the change in attorneys and authorized signatories of the company
(Annexes V-21-c and V-21-d hereof).
(b) Opening bank documents for Account No. 53. 145 A.R. signed by new
attorneys led by Michel Amandruz (Annexes V-21-e and V-21-f hereof).
(c). Bank statements for Account No. 53.145 A.R. with ending balance of
$26.10 as of 12-31-85 (Annex V-21-g and V-21-h hereof).
(d). An informative letter stating that Account 53. 145 A.R. was related to
an account opened with Merrill Lynch Asset Management, Inc., New York for
Arelma, Inc. The opening of this account slowly made Account 53. 145 A.R. an
inactive account (See Annexes V-21-I and V-21-j hereof).[46]

When the Marcos family fled Manila in 1986, they left behind several documents that revealed the
existence of secret bank deposits in Switzerland and other financial centers.[47] These papers,
referred to by respondent as Malacaang documents, detailed how Arelma, Inc.[48] was established.
Attached as Annex V-21 was the Letter of Instruction sent to the Panamanian branch of the Sunier
company to open Arelma. The latter was to have the same set-up as Maler, one of the five Swiss
foundations, subject of the 2000 Motion. Annexes V-21-c to V-21-j pertained to documents to be
used to open an account with Merrill Lynch Asset Management, Inc. in New York.

The Swiss Deposits Decision dealt only with the summary judgment as to the five Swiss accounts,
because the 2000 Motion for Partial Summary Judgment dated 7 March 2000 specifically
identified the five Swiss accounts only. It did not include the Arelma account. There was a prayer
for general reliefs in the 1996 Motion, but as has been discussed, this prayer was dismissed by the
Sandiganbayan. The dismissal was based solely on the existence of the Compromise Agreements
for a global settlement of the Marcos assets, which the Supreme Court later invalidated. The 2000
Motion for Summary Judgment was confined only to the five accounts amounting to USD 356
million held by five Swiss foundations.
As clarified by the Solicitor General during the hearing of 24 March 2000 in the Sandiganbayan:
PJ: The Court is of the impression and the Court is willing to be corrected,
that ones (sic) the plaintiff makes a claim for summary judgment it in fact states it
no longer intends to present evidence and based on this motion to render judgment,
is that correct?
SOL. BALLACILLO: Yes, your Honors.
PJ: In other words, on the basis of pre-trial, you are sayingbecause if we are
talking of a partial claim, then there is summary judgment, unless there is
preliminary issue to the claim which is a matter of stipulation.
SOL. BALLACILLO: We submit, your Honors, that there can be partial
summary judgment on this matter.
PJ: But in this instance, you are making summary judgment on the
entire case?
SOL. BALLACILLO: With respect to the $365 million.
PJ: In the complaint you asked for the relief over several topics. You
have $356 million, $25 million and $5 million. Now with regards to the $365
million, you are asking for summary judgment?
SOL. BALLACILLO: Yes, your Honor.
PJ: And, therefore, you are telling us now, thats it, we need not have to
prove.
SOL. BALLACILLO: Yes, your Honors.[49] (Emphasis supplied.)

The Courts discussion clearly did not include the Arelma account. The dispositive portion of the
Swiss Deposits Decision states:
WHEREFORE, the petition is hereby GRANTED. The assailed
Resolution of the Sandiganbayan dated January 31, 2002 is SET ASIDE. The Swiss
deposits which were transferred to and are now deposited in escrow at the
Philippine National Bank in the estimated aggregate amount of US$658,175,373.60
as of January 31, 2002, plus interest, are hereby forfeited in favor of petitioner
Republic of the Philippines.[50]

Thus, the other properties, which were subjects of the Petition for Forfeiture, but were not included
in the 2000 Motion, can still be subjects of a subsequent motion for summary judgment. To rule
otherwise would run counter to this Courts long established policy on asset recovery which, in
turn, is anchored on considerations of national survival.
E.O. 14, Series of 1986,[51] and Section 1(d) of Proclamation No. 3[52] declared the national policy
after the Marcos regime. The government aimed to implement the reforms mandated by the people:
protecting their basic rights, adopting a provisional constitution, and providing for an orderly
transition to a government under a new constitution. The said Proclamation further states that The
President shall give priority to measures to achieve the mandate of the people to recover ill-gotten
properties amassed by the leaders and supporters of the previous regime and protect the interest of
the people through orders of sequestration or freezing of assets or accounts. One of the whereas
clauses of E.O. 14 entrusts the PCGG with the just and expeditious recovery of such ill-gotten
wealth in order that the funds, assets and other properties may be used to hasten national economic
recovery. These clauses are anchored on the overriding considerations of national interest and
national survival, always with due regard to the requirements of fairness and due process.
With the myriad of properties and interconnected accounts used to hide these assets that are in
danger of dissipation, it would be highly unreasonable to require the government to ascertain their
exact locations and recover them simultaneously, just so there would be one comprehensive
judgment covering the different subject matters.
In any case, the Sandiganbayan rightly characterized their ruling on the 2004 Motion as a separate
judgment, which is allowed by the Rules of Court under Section 5 of Rule 36:
Separate judgments.When more than one claim for relief is presented in an
action, the court, at any stage, upon a determination of the issues material to a

particular claim and all counterclaims arising out of the transaction or occurrence
which is the subject matter of the claim, may render a separate judgment disposing
of such claim. The judgment shall terminate the action with respect to the claim so
disposed of and the action shall proceed as to the remaining claims. In case a
separate judgment is rendered, the court by order may stay its enforcement until the
rendition of a subsequent judgment or judgments and may prescribe such conditions
as may be necessary to secure the benefit thereof to the party in whose favor the
judgment is rendered.[53]

Rule 35 on summary judgments, admits of a situation in which a case is not fully adjudicated on
motion,[54] and judgment is not rendered upon all of the reliefs sought. InPhilippine Business Bank
v. Chua,[55] we had occasion to rule that a careful reading of its Section 4 reveals that a partial
summary judgment was never intended to be considered a final judgment, as it does not [put] an
end to an action at law by declaring that the plaintiff either has or has not entitled himself to recover
the remedy he sues for. In this case, there was never any final or complete adjudication of Civil
Case No. 0141, as the Sandiganbayans partial summary judgment in the Swiss Deposits Decision
made no mention of the Arelma account.
Section 4 of Rule 35 pertains to a situation in which separate judgments were necessary because
some facts existed without controversy, while others were controverted. However, there is nothing
in this provision or in the Rules that prohibits a subsequent separate judgment after a partial
summary judgment on an entirely different subject matterhad earlier been rendered. There is no
legal basis for petitioners contention that a judgment over the Swiss accounts bars a motion for
summary judgment over the Arelma account.
Thus, the Swiss Deposits Decision has finally and thoroughly disposed of the forfeiture case only
as to the five Swiss accounts. Respondents 2004 Motion is in the nature of a separate judgment,
which is authorized under Section 5 of Rule 36. More importantly respondent has brought to our
attention the reasons why a motion for summary judgment over the Arelma account was prompted
only at this stage. In Republic of the Philippines v. Pimentel,[56] a case filed by human rights
victims in the United States decided by the US Supreme Court only in 2008, the antecedents of the
Arelma account were described as follows:
In 1972, Ferdinand Marcos, then President of the Republic, incorporated Arelma,
S.A. (Arelma), under Panamanian law. Around the same time, Arelma opened a
brokerage account with Merrill Lynch, Pierce, Fenner & Smith Inc. (Merrill Lynch)
in New York, in which it deposited $2 million. As of the year 2000, the account
had grown to approximately $35 million.

Alleged crimes and misfeasance by Marcos during his presidency became


the subject of worldwide attention and protest. A class action by and on behalf of
some 9,539 of his human rights victims was filed against Marcos and his estate,
among others. The class action was tried in the United States District Court for the
District of Hawaii and resulted in a nearly $2 billion judgment for the class.
See Hilao v. Estate of Marcos, 103 F.3d 767 (C.A.9 1996). We refer to that
litigation as the Pimentel case and to its class members as the Pimentel class. In a
related action, the Estate of Roger Roxas and Golden Budha [sic] Corporation (the
Roxas claimants) claim a right to execute against the assets to satisfy their own
judgment against Marcos' widow, Imelda Marcos. See Roxas v. Marcos, 89 Hawaii
91, 113-115, 969 P.2d 1209, 1231-1233 (1998).
The Pimentel class claims a right to enforce its judgment by attaching
the Arelma assets held by Merrill Lynch. The Republic and the Commission
claim a right to the assets under a 1955 Philippine law providing that property
derived from the misuse of public office is forfeited to the Republic from the
moment of misappropriation. See An Act Declaring Forfeiture in Favor of the
State Any Property Found To Have Been Unlawfully Acquired by Any Public
Officer or Employee and Providing for the Proceedings Therefor, Rep. Act No.
1379, 51:9 O.G. 4457 (June 18, 1955).
After Marcos fled the Philippines in 1986, the Commission was created to
recover any property he wrongfully took. Almost immediately the Commission
asked the Swiss Government for assistance in recovering assets-including shares in
Arelma-that Marcos had moved to Switzerland. In compliance the Swiss
Government froze certain assets and, in 1990, that freeze was upheld by the Swiss
Federal Supreme Court. In 1991, the Commission asked the Sandiganbayan, a
Philippine court of special jurisdiction over corruption cases, to declare forfeited to
the Republic any property Marcos had obtained through misuse of his office. That
litigation is still pending in the Sandiganbayan. (Citations omitted.)

The pursuit of the Arelma account encountered several hindrances, as it was subject to not one,
but two claims of human rights victims in foreign courts: the Pimentel class and the Roxas
claimants. The government and the PCGG were able to obtain a Stay Order at the appellate level,
but the trial court judge vacated the stay and awarded the Arelma assets to the Pimentel class of
human rights victims.
As early as 1986, the PCGG had already sought assistance from the Swiss government to recover
the Arelma assets; however, it was only in 2000 that the Swiss authorities turned over two Stock
Certificates, which were assets of Arelma. The transfer by Switzerland of the Stock Certificates to

the Republic was made under the same conditions as the bank deposits of the five Swiss
foundations.[57]
Meanwhile, the Pimentel case was tried as a class action before Judge Manuel Real of the United
States District Court for the Central District of California. Judge Real was sitting by designation
in the District of Hawaii after the Judicial Panel on Multidistrict Litigation consolidated the various
human rights Complaints against Marcos in that court.[58] Judge Real directed Merrill Lynch to file
an action for interpleader in the District of Hawaii, where he presided over the matter, and where
the Republic and the PCGG were named as defendants. In Pimentel, the Court further narrates how
Judge Real ruled that the pending litigation in Philippine courts could not determine entitlement
to the Arelma assets:
After being named as defendants in the interpleader action, the Republic and the
Commission asserted sovereign immunity under the Foreign Sovereign Immunities
Act of 1976 (FSIA), 28 U.S.C. 1604. They moved to dismiss pursuant to Rule
19(b), based on the premise that the action could not proceed without them Judge
Real initially rejected the request by the Republic and the Commission to dismiss
the interpleader action. They appealed, and the Court of Appeals reversed. It held
the Republic and the Commission are entitled to sovereign immunity and that
under Rule 19(a) they are required parties (or necessary parties under the old
terminology). See In re Republic of the Philippines, 309 F.3d 1143, 1149-1152
(C.A.9 2002). The Court of Appeals entered a stay pending the outcome of the
litigation in the Sandiganbayan over the Marcos assets.
After concluding that the pending litigation in the Sandiganbayan could not
determine entitlement to the Arelma assets, Judge Real vacated the stay,
allowed the action to proceed, and awarded the assets to the Pimentel class. A
week later, in the case initiated before the Sandiganbayan in 1991, the
Republic asked that court to declare the Arelma assets forfeited, arguing the
matter was ripe for decision. The Sandiganbayan has not yet ruled. In the
interpleader case the Republic, the Commission, Arelma, and PNB appealed
the District Court's judgment in favor of the Pimentel claimants. This time the
Court of Appeals affirmed. Dismissal of the interpleader suit, it held, was not
warranted under Rule 19(b)because, though the Republic and the
Commission were required (necessary) parties under Rule 19(a), their claim
had so little likelihood of success on the merits that the interpleader action
could proceed without them. One of the reasons the court gave was that any
action commenced by the Republic and the Commission to recover the assets
would be barred by New York's 6-year statute of limitations for claims
involving the misappropriation of public property.[59] (Citations omitted)

The American Supreme Court reversed the judgment of the Court of Appeals for the Ninth Circuit
and remanded the case with instructions to order the District Court to dismiss the interpleader
action. The former held that the District Court and the Court of Appeals failed to give full effect
to sovereign immunity when they held that the action could proceed without the Republic and the
Commission:
Comity and dignity interests take concrete form in this case. The claims of
the Republic and the Commission arise from events of historical and political
significance for the Republic and its people. The Republic and the Commission
have a unique interest in resolving the ownership of or claims to the Arelma assets
and in determining if, and how, the assets should be used to compensate those
persons who suffered grievous injury under Marcos. There is a comity interest in
allowing a foreign state to use its own courts for a dispute if it has a right to do so.
The dignity of a foreign state is not enhanced if other nations bypass its courts
without right or good cause. Then, too, there is the more specific affront that could
result to the Republic and the Commission if property they claim is seized by the
decree of a foreign court.[60]

Thus it was only in 2008 that the Republic was finally able to obtain a favorable judgment from
the American Supreme Court with regard to the different claims against the Arelma assets.
Petitioners never intervened or lifted a finger in any of the litigation proceedings involving the
enforcement of judgment against the Arelma assets abroad. We find merit in respondents
observation that petitioner Imelda Marcoss participation in the proceedings in the Philippines,
particularly her invocation of her right against undue deprivation of property, is inconsistent with
her and Ferdinand Marcos, Jr.s insistence that the properties in question do not belong to them,
and that they are mere beneficiaries.[61]
Indeed, it is clear that the Arelma assets are in danger of dissipation. Even as the United States
Supreme Court gave weight to the likely prejudice to be suffered by the Republic when it dismissed
the interpleader in Pimentel, it also considered that the balance of equities may change in due
course. One relevant change may occur if it appears that the Sandiganbayan cannot or will not
issue its ruling within a reasonable period of time. If the Sandiganbayan rules that the Republic
and the Commission have no right to the assets, their claims in some later interpleader suit would
be less substantial than they are now.[62]
IV. Petitioners sham denials justify the
application of summary judgment

As already settled in the Swiss Deposits Decision and reiterated in the discussion above as
the law of the case, the lawful income of the Marcoses is only USD 304,372.43. As discussed in
paragraph 9 of the Petition for Forfeiture, Annex V-21-b states that Arelmas assets as of 19 May
1983 were worth USD 3,369,975.00.[63] The entirety of the lawful income of the Marcoses
represents only 9% of the entire assets of Arelma, which petitioners remain unable to
explain.
In their Answer to the Petition for Forfeiture, petitioners employ the same tactic, consisting
of general denials based on a purported lack of knowledge regarding the whereabouts of the
Arelma assets. Paragraph 32 of the said pleading states:
Respondents specifically DENY paragraph 59 of the Petition insofar as it
alleges that the Marcoses used their cronies and engaged in laundering their filthy
riches for being false and conclusory of the truth being that the Marcoses did not
engage in any such illegal acts and that all the properties they acquired were
lawfully acquired; and specifically DENY the rest for lack of knowledge or
information sufficient to form a belief as to the truth of the allegation since
Respondents are not privy to the alleged transactions.[64]

This particular denial mimics petitioners similar denials of the allegations in the forfeiture
Petition pertaining to the Swiss accounts and is practically identical to paragraphs 7 to 37 of the
Answer. The Swiss Deposits Decision has characterized these as sham denials:
17. Respondents specifically DENY paragraph 18 of the Petition for lack
of knowledge or information sufficient to form a belief as to the truth of the
allegation since Respondents cannot remember with exactitude the contents of the
alleged ITRs.
18. Respondents specifically DENY paragraph 19 of the Petition for lack
of knowledge or information sufficient to form a belief as to the truth of the
allegation since Respondents cannot remember with exactitude the contents of the
alleged ITRs and that they are not privy to the activities of the BIR.
19. Respondents specifically DENY paragraph 20 of the Petition for lack
of knowledge or information sufficient to form a belief as to the truth of the
allegation since Respondents cannot remember with exactitude the contents of the
alleged ITRs.
20. Respondents specifically DENY paragraph 21 of the Petition for lack
of knowledge or information sufficient to form a belief as to the truth of the

allegation since Respondents cannot remember with exactitude the contents of the
alleged ITRs.
21. Respondents specifically DENY paragraph 22 of the Petition for lack
of knowledge or information sufficient to form a belief as to the truth of the
allegation since Respondents cannot remember with exactitude the contents of the
alleged ITRs.
22. Respondents specifically DENY paragraph 23 insofar as it alleges that
Respondents clandestinely stashed the country's wealth in Switzerland and hid the
same under layers and layers of foundation and corporate entities for being false,
the truth being that Respondents aforesaid properties were lawfully acquired.
23. Respondents specifically DENY paragraphs 24, 25, 26, 27, 28, 29 and
30 of the Petition for lack of knowledge or information sufficient to form a
belief as to the truth of the allegation since Respondents were not privy to the
transactions regarding the alleged Azio-Verso-Vibur Foundation accounts, except
that as to Respondent Imelda R. Marcos she specifically remembers that the funds
involved were lawfully acquired.
24. Respondents specifically DENY paragraphs 31, 32, 33, 34, 35, 36,37,
38, 39, 40, and 41 of the Petition for lack of knowledge or information sufficient
to form a belief as to the truth of the allegations since Respondents are not privy
to the transactions and as to such transaction they were privy to they cannot
remember with exactitude the same having occurred a long time ago, except that as
to Respondent Imelda R. Marcos she specifically remembers that the funds
involved were lawfully acquired.
25. Respondents specifically DENY paragraphs 42, 43, 44, 45, and 46, of
the Petition for lack of knowledge or information sufficient to form a belief as
to the truth of the allegations since Respondents were not privy to the transactions
and as to such transaction they were privy to they cannot remember with exactitude
the same having occurred a long time ago, except that as to Respondent Imelda R.
Marcos she specifically remembers that the funds involved were lawfully acquired.
26. Respondents specifically DENY paragraphs 49, 50, 51 and 52, of the
Petition for lack of knowledge or information sufficient to form a belief as to
the truth of the allegationssince Respondents were not privy to the transactions
and as to such transaction they were privy to they cannot remember with exactitude
the same having occurred a long time ago, except that as to Respondent Imelda R.
Marcos she specifically remembers that the funds involved were lawfully acquired.
Upon careful perusal of the foregoing, the Court finds that respondent
Mrs. Marcos and the Marcos children indubitably failed to tender genuine
issues in their answer to the petition for forfeiture. A genuine issue is an issue
of fact which calls for the presentation of evidence as distinguished from an
issue which is fictitious and contrived, set up in bad faith or patently lacking

in substance so as not to constitute a genuine issue for trial. Respondents'


defenses of "lack of knowledge for lack of privity" or "(inability to) recall
because it happened a long time ago" or, on the part of Mrs. Marcos, that "the
funds were lawfully acquired" are fully insufficient to tender genuine issues.
Respondent Marcoses' defenses were a sham and evidently calibrated to
compound and confuse the issues.[65] (Emphasis supplied.)

In the case at bar, petitioners give the same stock answer to the effect that the Marcoses did
not engage in any illegal activities, and that all their properties were lawfully acquired. They fail
to state with particularity the ultimate facts surrounding the alleged lawfulness of the mode of
acquiring the funds in Arelma (which totaled USD 3,369,975.00 back in 1983), considering that
the entirety of their lawful income amounted only to USD 304,372.43, or only 9% of the entire
Arelma fund. Then, as now, they employ what the Court in G.R. No. 152154 characterized as a
negative pregnant, not just in denying the criminal provenance of the Arelma funds, but in the
matter of ownership of the said funds. As discussed by the Court in the first Republic case, cited
by the Sandiganbayan:
Evidently, this particular denial had the earmark of what is called in the law
on pleadings as a negative pregnant, that is, a denial pregnant with the admission
of the substantial facts in the pleading responded to which are not squarely denied.
It was in effect an admission of the averments it was directed at. Stated otherwise,
a negative pregnant is a form of negative expression which carries with it an
affirmation or at least an implication of some kind favorable to the adverse party.
It is a denial pregnant with an admission of the substantial facts alleged in the
pleading.Where a fact is alleged with qualifying or modifying language and the
words of the allegation as so qualified or modified are literally denied, it has
been held that the qualifying circumstances alone are denied while the fact
itself is admitted.[66]

Due to the insufficiency of petitioners denial of paragraph 59 which in effect denies only
the qualifying circumstances, and by virtue of the Courts ruling in the Swiss Deposits Decision,
petitioners are deemed to have admitted the factual antecedents and the establishment of Arelma.
In paragraph 32 of their Answer, they only deny the first few sentences of paragraph 59, while
conveniently neglecting to address subparagraphs 1 to 5 and the opening bank documents
described in 5 (a) to (d) of the Petition for Forfeiture. Paragraphs 1 and 2 of the Petition discusses
the establishment of a Panamanian company to be named either Larema, Inc. or Arelma, Inc., or
Relma, Inc.; the appointment of several people as directors; and the opening of a direct account
with Merrill Lynch. Paragraphs 3 to 5 also of the Petition for Forfeiture detail correspondences
between a J.L. Sunier and a letter addressed to Malacaang with the salutation Dear Excellency.

Regarding the averment of petitioners that they lack knowledge sufficient to form a belief
as to the truth of the above allegations in the Petition for Forfeiture, the Courts discussion in the
Swiss Deposits Decision bears reiterating:
Here, despite the serious and specific allegations against them, the Marcoses
responded by simply saying that they had no knowledge or information sufficient
to form a belief as to the truth of such allegations. Such a general, self-serving claim
of ignorance of the facts alleged in the petition for forfeiture was insufficient to
raise an issue. Respondent Marcoses should have positively stated how it was that
they were supposedly ignorant of the facts alleged.[67]

Petitioners cannot escape the fact that there is manifest disparity between the amount of the
Arelma funds and the lawful income of the Marcoses as shown in the ITRs filed by spouses
Marcos. The Swiss Deposits Decision found that the genuineness of the said ITRs and balance
sheets of the Marcos spouses have already been admitted by petitioners themselves:
Not only that. Respondents answer also technically admitted the
genuineness and due execution of the Income Tax Returns (ITRs) and the balance
sheets of the late Ferdinand E. Marcos and Imelda R. Marcos attached to the
petition for forfeiture, as well as the veracity of the contents thereof.
The answer again premised its denials of said ITRs and balance sheets on
the ground of lack of knowledge or information sufficient to form a belief as to the
truth of the contents thereof. Petitioner correctly points out that respondents' denial
was not really grounded on lack of knowledge or information sufficient to form a
belief but was based on lack of recollection. By reviewing their own records,
respondent Marcoses could have easily determined the genuineness and due
execution of the ITRs and the balance sheets. They also had the means and
opportunity of verifying the same from the records of the BIR and the Office of the
President. They did not.
When matters regarding which respondents claim to have no knowledge or
information sufficient to form a belief are plainly and necessarily within their
knowledge, their alleged ignorance or lack of information will not be considered a
specific denial. An unexplained denial of information within the control of the
pleader, or is readily accessible to him, is evasive and is insufficient to constitute
an effective denial.[68] (Footnotes omitted.)

We find that petitioners have again attempted to delay the goal of asset recovery by their
evasiveness and the expedient profession of ignorance. It is well-established that a profession of
ignorance about a fact that is necessarily within the pleaders knowledge or means of knowing is
as ineffective as no denial at all. On a similar vein, there is a failure by petitioners to properly

tender an issue, which as correctly ruled by the Sandiganbayan, justifies the Republics resort to
summary judgment.
Summary judgment may be allowed where there is no genuine issue as to any material fact
and where the moving party is entitled to a judgment as a matter of law.[69] InYuchengco v.
Sandiganbayan, the Court has previously discussed the importance of summary judgment in
weeding out sham claims or defenses at an early stage of the litigation in order to avoid the expense
and loss of time involved in a trial, viz:
Even if the pleadings appear, on their face, to raise issues, summary
judgment may still ensue as a matter of law if the affidavits, depositions and
admissions show that such issues are not genuine. The presence or absence of a
genuine issue as to any material fact determines, at bottom, the propriety of
summary judgment. A genuine issue, as differentiated from a fictitious or contrived
one, is an issue of fact that requires the presentation of evidence. To the party who
moves for summary judgment rests the onus of demonstrating clearly the absence
of any genuine issue of fact, or that the issue posed in the complaint is patently
unsubstantial so as not to constitute a genuine issue for trial. [70]

Even if in the Answer itself there appears to be a tender of issues requiring trial, yet when
the relevant affidavits, depositions, or admissions demonstrate that those issues are not genuine
but sham or fictitious, the Court is justified in dispensing with the trial and rendering summary
judgment for plaintiff.[71]
Summary judgment, or accelerated judgment as it is sometimes known, may also call for a
hearing so that both the movant and the adverse party may justify their positions. However, the
hearing contemplated (with 10-day notice) is for the purpose of determining whether the issues are
genuine or not, not to receive evidence of the issues set up in the pleadings. In Carcon
Development Corporation v. Court of Appeals, [72] the Court ruled that a hearing is not de riguer.
The matter may be resolved, and usually is, on the basis of affidavits, depositions, and admissions.
This does not mean that the hearing is superfluous; only that the court is empowered to determine
its necessity.
It is the law itself that determines when a summary judgment is proper. Under the rules,
summary judgment is appropriate when there are no genuine issues of fact that call for the
presentation of evidence in a full-blown trial. Even if on their face the pleadings appear to raise
issues, when the affidavits, depositions and admissions show that such issues are not genuine, then

summary judgment as prescribed by the rules must ensue as a matter of law. What is crucial to a
determination, therefore, is the presence or absence of a genuine issue as to any material fact.
When the facts as pleaded appear uncontested or undisputed, then summary judgment is called
for.[73]
Guided by the principles above indicated, we hold that under the circumstances obtaining
in the case at bar, summary judgment is proper. The Sandiganbayan did not commit a reversible
error in granting the corresponding 2004 Motion for Summary Judgment filed by respondent. The
latter is well within its right to avail itself of summary judgment and obtain immediate relief,
considering the insufficient denials and pleas of ignorance made by petitioners on matters that are
supposedly within their knowledge.
These denials and pleas constitute admissions of material allegations under paragraph 59
of the Petition for Forfeiture a tact they have employed repeatedly in Civil Case No. 0141. As
discussed, the purpose of summary judgment is precisely to avoid long drawn litigations and
useless delays.[74] We also affirm the Sandiganbayans findings that the moving party, the Republic,
is now entitled to judgment as a matter of law.
WHEREFORE, the instant Petition is DENIED. The Decision dated 2 April 2009 of the
Sandiganbayan is AFFIRMED. All assets, properties, and funds belonging to Arelma, S.A., with an
estimated aggregate amount of USD 3,369,975 as of 1983, plus all interests and all other income that
accrued thereon, until the time or specific day that all money or monies are released and/or transferred
to the possession of the Republic of the Philippines, are hereby forfeited in favor of Respondent
Republic of the Philippines.

SO ORDERED.

[G.R. Nos. 161784-86. April 26, 2005]

DINAH C. BARRIGA, petitioner, vs. THE HONORABLE SANDIGANBAYAN


(4TH DIVISION) and THE PEOPLE OF THE PHILIPPINES, respondents.
DECISION
CALLEJO, SR., J.:
This is a petition for certiorari under Rule 65 of the Rules of Court for the nullification
of the Resolution[1] of the Sandiganbayan in Criminal Case Nos. 27435 to 27437 denying
the motion to quash the Informations filed by one of the accused, Dinah C. Barriga, and
the Resolution denying her motion for reconsideration thereof.

The Antecedents
On April 3, 2003, the Office of the Ombudsman filed a motion with the Sandiganbayan
for the admission of the three Amended Informations appended thereto. The first
Amended Information docketed as Criminal Case No. 27435, charged petitioner Dinah C.
Barriga and Virginio E. Villamor, the Municipal Accountant and the Municipal Mayor,
respectively, of Carmen, Cebu, with malversation of funds. The accusatory portion reads:
That in or about January 1996 or sometime prior or subsequent thereto, in the Municipality of
Carmen, Province of Cebu, Philippines and within the jurisdiction of this Honorable Court,
above-named accused VIRGINIO E. VILLAMOR and DINAH C. BARRIGA, both public
officers, being then the Municipal Mayor and Municipal Accountant, respectively, of the
Municipality of Carmen, Cebu, and as such, had in their possession and custody public
funds amounting to TWENTY- THREE THOUSAND FORTY-SEVEN AND 20/100 PESOS
(P23,047.20), Philippine Currency, intended for the payment of Five (5) rolls of Polyethylene
pipes to be used in the Corte-Cantumog Water System Project of the Municipality of Carmen,
Cebu, for which they are accountable by reason of the duties of their office, in such capacity and
committing the offense in relation to office, conniving and confederating together and mutually
helping each other, did then and there willfully, unlawfully and feloniously misappropriate, take,
embezzle and convert into their own personal use and benefit said amount of P23,047.20, and
despite demands made upon them to account for said amount, they have failed to do so, to the
damage and prejudice of the government.
CONTRARY TO LAW.[2]
The inculpatory portion of the second Amended Information, docketed as Criminal
Case No. 27436, charging the said accused with illegal use of public funds, reads:
That in or about the month of November 1995, or sometime prior or subsequent thereto, in the
Municipality of Carmen, Province of Cebu, Philippines, and within the jurisdiction of the

Honorable Court, above-named accused VIRGINIO E. VILLAMOR and DINAH C.


BARRIGA, both public officers, being then the Municipal Mayor and Municipal Accountant,
respectively, of the Municipality of Carmen, Cebu, and as such, had in their possession and
control public funds in the amount of ONE THOUSAND THREE HUNDRED FIVE PESOS
(P1,305.00) Philippine Currency, representing a portion of the Central Visayas Water and
Sanitation Project Trust Fund (CVWSP Fund) intended and appropriated for the projects
classified under Level I and III particularly the construction of Deep Well and Spring Box
for Level I projects and construction of water works system for Level III projects of specified
barangay beneficiaries/recipients, and for which fund accused are accountable by reason of the
duties of their office, in such capacity and committing the offense in relation to office, conniving
and confederating together and mutually helping each other, did then and there, willfully
unlawfully and feloniously disburse and use said amount of P1,305.00 for the Spring Box of
Barangay Natimao-an, Carmen, Cebu, a barangay which was not included as a recipient of
CVWSP Trust Fund, thus, accused used said public fund to a public purpose different from
which it was intended or appropriated, to the damage and prejudice of the
government, particularly the barangays which were CVWSP Trust Fund beneficiaries.
CONTRARY TO LAW.[3]
The accusatory portion of the third Amended Information, docketed as Criminal Case
No. 27437, charged the same accused with illegal use of public funds, as follows:
That in or about the month of January 1997, or sometime prior or subsequent thereto, in the
Municipality of Carmen, Province of Cebu, Philippines, and within the jurisdiction of this
Honorable Court, above-named accused Virginio E. Villamor and Dinah C.
Barriga, both public officers, being then the Municipal Mayor and Municipal Accountant,
respectively, of the Municipality of Carmen, Cebu, and as such, had in their possession and
control public funds in the amount of TWO HUNDRED SIXTY-SEVEN THOUSAND FIVE
HUNDRED THIRTY-SEVEN and 96/100 (P267,537.96) PESOS, representing a portion of
the Central Visayas Water and Sanitation Project Trust Fund (CVWSP Fund), intended and
appropriated for the projects classified under Level I and Level III, particularly the construction
of Spring Box and Deep Well for Level I projects and construction of water works system for
Level III projects of specified barangay beneficiaries/ recipients, and for which fund accused are
accountable by reason for the duties of their office, in such capacity and committing the offense
in relation to office, conniving and confederating together and mutually helping each other, did
then and there willfully, unlawfully and feloniously disburse and use said amount of P267,537.96
for the construction and expansion of Barangay Cantucong Water System, a project falling
under Level II of CVWSP, thus, accused used said public funds to a public purpose different
from which it was intended and appropriated, to the damage and prejudice of the
government, particularly the barangay beneficiaries of Levels I and III of CVWSP.
CONTRARY TO LAW.[4]
The Sandiganbayan granted the motion and admitted the Amended Informations.
The petitioner filed a Motion to Quash the said Amended Informations on the ground that
under Section 4 of Republic Act No. 8294, the Sandiganbayan has no jurisdiction over

the crimes charged. She averred that the Amended Informations failed to allege and show
the intimate relation between the crimes charged and her official duties as municipal
accountant, which are conditions sine qua non for the graft court to acquire jurisdiction
over the said offense. She averred that the prosecution and the Commission on Audit
admitted, and no less than this Court held in Tan v. Sandiganbayan,[5] that a municipal
accountant is not an accountable officer. She alleged that the felonies of malversation
and illegal use of public funds, for which she is charged, are not included in Chapter 11,
Section 2, Title VII, Book II, of the Revised Penal Code; hence, the Sandiganbayan has
no jurisdiction over the said crimes. Moreover, her position as municipal accountant is
classified as Salary Grade (SG) 24.
The petitioner also posited that although the Sandiganbayan has jurisdiction over
offenses committed by public officials and employees in relation to their office, the mere
allegation in the Amended Informations that she committed the offenses charged in
relation to her office is not sufficient as the phrase is merely a conclusion of law;
controlling are the specific factual allegations in the Informations that would indicate the
close intimacy between the discharge of her official duties and the commission of the
offenses charged. To bolster her stance, she cited the rulings of this Court in People v.
Montejo,[6] Soller v. Sandiganbayan,[7] and Lacson v. Executive Secretary.[8] She further
contended that although the Amended Informations alleged that she conspired with her
co-accused to commit the crimes charged, they failed to allege and show her exact
participation in the conspiracy and how she committed the crimes charged. She also
pointed out that the funds subject of the said Amended Informations were not under her
control or administration.
On October 9, 2003, the Sandiganbayan issued a Resolution[9] denying the motion of
the petitioner. The motion for reconsideration thereof was, likewise, denied, with the graft
court holding that the applicable ruling of this Court was Montilla v. Hilario,[10] i.e., that an
offense is committed in relation to public office when there is a direct, not merely
accidental, relation between the crime charged and the office of the accused such that,
in a legal sense, the offense would not exist without the office; in other words, the office
must be a constituent element of the crime as defined in the statute. The graft court further
held that the offices of the municipal mayor and the municipal accountant were constituent
elements of the felonies of malversation and illegal use of public funds. The graft court
emphasized that the rulings of this Court in People v. Montejo[11] and Lacson v. Executive
Secretary[12] apply only where the office held by the accused is not a constituent element
of the crimes charged. In such cases, the Information must contain specific factual
allegations showing that the commission of the crimes charged is intimately connected
with or related to the performance of the accused public officers public functions. In fine,
the graft court opined, the basic rule is that enunciated by this Court in Montilla v.
Hilario, and the ruling of this Court in People v. Montejo is the exception.
The petitioner thus filed the instant petition for certiorari under Rule 65 of the Rules
of Court, seeking to nullify the aforementioned Resolutions of the Sandiganbayan. The
petitioner claims that the graft court committed grave abuse of its discretion amounting to
excess or lack of jurisdiction in issuing the same.

In its comment on the petition, the Office of the Special Prosecutor averred that the
remedy of filing a petition for certiorari, from a denial of a motion to quash amended
information, is improper. It posits that any error committed by the Sandiganbayan in
denying the petitioners motion to quash is merely an error of judgment and not of
jurisdiction. It asserts that as ruled by the Sandiganbayan, what applies is the ruling of
this Court in Montilla v. Hilario and not People v. Montejo. Furthermore, the crimes of
malversation and illegal use of public funds are classified as crimes committed by public
officers in relation to their office, which by their nature fall within the jurisdiction of the
Sandiganbayan. It insists that there is no more need for the Amended Informations to
specifically allege intimacy between the crimes charged and the office of the accused
since the said crimes can only be committed by public officers. It further claims that the
petitioner has been charged of malversation and illegal use of public funds in conspiracy
with Municipal Mayor Virginio E. Villamor, who occupies a position classified as SG 27;
and even if the petitioners position as municipal accountant is only classified as SG 24,
under Section 4 of Rep. Act No. 8249, the Sandiganbayan still has jurisdiction over the
said crimes. The Office of the Special Prosecutor further avers that the petitioners claim,
that she is not an accountable officer, is a matter of defense.

The Ruling of the Court


The petition has no merit.
We agree with the ruling of the Sandiganbayan that based on the allegations of the
Amended Informations and Rep. Act No. 8249, it has original jurisdiction over the crimes
of malversation and illegal use of public funds charged in the Amended Informations
subject of this petition.
Rep. Act No. 8249,[13] which amended Section 4 of Presidential Decree No. 1606,
provides, inter alia, that the Sandiganbayan has original jurisdiction over crimes and
felonies committed by public officers and employees, at least one of whom belongs to
any of the five categories thereunder enumerated at the time of the commission of such
crimes.[14] There are two classes of public office-related crimes under subparagraph (b)
of Section 4 of Rep. Act No. 8249: first, those crimes or felonies in which the public office
is a constituent element as defined by statute and the relation between the crime and the
offense is such that, in a legal sense, the offense committed cannot exist without the
office;[15] second, such offenses or felonies which are intimately connected with the public
office and are perpetrated by the public officer or employee while in the performance of
his official functions, through improper or irregular conduct.[16]
The Sandiganbayan has original jurisdiction over criminal cases involving crimes and
felonies under the first classification. Considering that the public office of the accused is
by statute a constituent element of the crime charged, there is no need for the Prosecutor
to state in the Information specific factual allegations of the intimacy between the office
and the crime charged, or that the accused committed the crime in the performance of
his duties. However, the Sandiganbayan likewise has original jurisdiction over criminal
cases involving crimes or felonies committed by the public officers and employees

enumerated in Section (a) (1) to (5) under the second classification if the Information
contains specific factual allegations showing the intimate connection between the offense
charged and the public office of the accused, and the discharge of his official duties or
functions - whether improper or irregular.[17] The requirement is not complied with if the
Information merely alleges that the accused committed the crime charged in relation to
his office because such allegation is merely a conclusion of law.[18]
Two of the felonies that belong to the first classification are malversation defined and
penalized by Article 217 of the Revised Penal Code, and the illegal use of public funds or
property defined and penalized by Article 220 of the same Code. The public office of the
accused is a constituent element in both felonies.
For the accused to be guilty of malversation, the prosecution must prove the following
essential elements:
(a) The offender is a public officer;
(b) He has the custody or control of funds or property by reason of the duties of his office;
(c) The funds or property involved are public funds or property for which he is accountable; and
(d) He has appropriated, taken or misappropriated, or has consented to, or through abandonment
or negligence, permitted the taking by another person of, such funds or property.[19]
For the accused to be guilty of illegal use of public funds or property, the prosecution
is burdened to prove the following elements:
(1) The offenders are accountable officers in both crimes.
(2) The offender in illegal use of public funds or property does not derive any personal gain or
profit; in malversation, the offender in certain cases profits from the proceeds of the crime.
(3) In illegal use, the public fund or property is applied to another public use; in malversation,
the public fund or property is applied to the personal use and benefit of the offender or of another
person.[20]
We agree with the ruling of the Sandiganbayan that the public office of the accused
Municipal Mayor Virginio E. Villamor is a constituent element of malversation and illegal
use of public funds or property. Accused mayors position is classified as SG 27. Since
the Amended Informations alleged that the petitioner conspired with her co-accused, the
municipal mayor, in committing the said felonies, the fact that her position as municipal
accountant is classified as SG 24 and as such is not an accountable officer is of no
moment; the Sandiganbayan still has exclusive original jurisdiction over the cases lodged
against her. It must be stressed that a public officer who is not in charge of public funds
or property by virtue of her official position, or even a private individual, may be liable for
malversation or illegal use of public funds or property if such public officer or private

individual conspires with an accountable public officer to commit malversation or illegal


use of public funds or property.
In United States v. Ponte,[21] the Court, citing Viada, had the occasion to state:
Shall the person who participates or intervenes as co-perpetrator, accomplice or abettor in the
crime of malversation of public funds, committed by a public officer, have the penalties of this
article also imposed upon him? In opposition to the opinion maintained by some jurists and
commentators (among others the learned Pacheco) we can only answer the question
affirmatively, for the same reasons (mutatis mutandis) we have already advanced in Question I
of the commentary on article 314. French jurisprudence has also settled the question in the same
way on the ground that the person guilty of the crimenecessarily aids the other culprit in the acts
which constitute the crime. (Vol. 2, 4th edition, p. 653)
The reasoning by which Groizard and Viada support their views as to the correct interpretation
of the provisions of the Penal Code touching malversation of public funds by a public official, is
equally applicable in our opinion, to the provisions of Act No. 1740 defining and penalizing that
crime, and we have heretofore, in the case of the United States vs. Dowdell (11 Phil. Rep., 4),
imposed the penalty prescribed by this section of the code upon a public official who took part
with another in the malversation of public funds, although it was not alleged, and in fact clearly
appeared, that those funds were not in his hands by virtue of his office, though it did appear that
they were in the hands of his co-principal by virtue of the public office held by him.[22]
The Court has also ruled that one who conspires with the provincial treasurer in
committing six counts of malversation is also a co-principal in committing those offenses,
and that a private person conspiring with an accountable public officer in committing
malversation is also guilty of malversation.[23]
We reiterate that the classification of the petitioners position as SG 24 is of no
moment. The determinative fact is that the position of her co-accused, the municipal
mayor, is classified as SG 27, and under the last paragraph of Section 2 of Rep. Act No.
7975, if the position of one of the principal accused is classified as SG 27, the
Sandiganbayan has original and exclusive jurisdiction over the offense.
We agree with the petitioners contention that under Section 474 of the Local
Government Code, she is not obliged to receive public money or property, nor is she
obligated to account for the same; hence, she is not an accountable officer within the
context of Article 217 of the Revised Penal Code. Indeed, under the said article, an
accountable public officer is one who has actual control of public funds or property by
reason of the duties of his office. Even then, it cannot thereby be necessarily concluded
that a municipal accountant can never be convicted for malversation under the Revised
Penal Code. The name or relative importance of the office or employment is not the
controlling factor.[24] The nature of the duties of the public officer or employee, the fact
that as part of his duties he received public money for which he is bound to account and
failed to account for it, is the factor which determines whether or not malversation is
committed by the accused public officer or employee. Hence, a mere clerk in the
provincial or municipal government may be held guilty of malversation if he or she is
entrusted with public funds and misappropriates the same.

IN LIGHT OF ALL THE FOREGOING, the petition is DENIED for lack of merit. Costs
against the petitioner.
SO ORDERED.

G.R. No. L-14595

May 31, 1960

THE PEOPLE OF THE PHILIPPINES, petitioner,


vs.
HONORABLE GREGORIO MONTEJO, Judge, Court of First Instance, Zamboanga
City and Basilan City, MAYOR LEROY S. BROWN, DETECTIVE JOAQUIN R.
POLLISCO, PATROLMAN GRACIANO LACERNA aliasDODONG, PATROLMAN
MOHAMAD HASBI, SPECIAL POLICEMAN DIONISIO DINGLASA, SPECIAL
POLICEMAN HADJARATIL, SPECIAL POLICEMAN ALO, and JOHN
DOES, respondents.
Acting City Atty. Perfecto B. Querubin for petitioner.
Hon. Gregorio Montejo in his own behalf.
C. A. S. Sipin, Jr. for the other respondents.
CONCEPCION, J.:
This is a special civil action for certiorari , with mandamus and preliminary injunction,
against Hon. Gregorio Montejo, as Judge of the Court of First Instance of the cities of
Zamboanga and Basilan, and the defendants in Criminal Case No. 672 of said court.
In the petition herein, which was filed by the prosecution in said criminal case, it is
prayed that, pending the final determination thereof, a writ of preliminary injunction
issue, enjoining respondent Judge from proceeding with the trial of said case; that, after
due hearing, the rulings of respondent Judge, rejecting some evidence for the
prosecution therein and not permitting the same to propound certain questions, be set
aside; that said respondent Judge be ordered to admit the aforementioned evidence
and permit said questions; and that Senator Roseller Lim be declared, contrary to
another ruling made by respondent Judge, disqualified by the Constitution from
appearing as counsel for the accused in said criminal case. Soon, after the filing of the
petition, we issued the writ of preliminary injunction prayed for, without bond.
In their respective answers, respondents alleged, in substance, that the ruling
complained of are in conformity with law.
Respondents Leroy S. Brown, Mayor of Basilan City, Detective Joaquin R. Pollisco,
Patrolman Graciano Lacerna (alias Dodong) and Mohamad Hasbi, Special Policemen
Dionisio Dinglasa, Moro Yakan, Hadjaratil, Moro Alo and several John Does, are
charged, in said Criminal Case No. 672, with murder. It is alleged in the information
therein that, during May and June, 1958, in the sitio of Tipo-Tipo, district of Lamitan,
City of Basilan, Mayor Brown "organized groups of police patrol and civilian
commandoes", consisting of regular and special policemen, whom he "armed with
pistols and high power guns", and then "established a camp", called sub-police
headquarters hereinafter referred to as sub-station at Tipo-Tipo, Lamitan, which
was placed under his command, orders, direct supervision and control, and in which his
codefendants were stationed; that the criminal complaints were entertained in said sub-

station, in which defendant Pollisco acted as investigating officer and exercised


authority to order the apprehension of persons and their detention in the camp, for days
or weeks, without due process of law and without bringing them to the proper court;
that, on or about June 4, and 5, 1958; one Yokan Awalin Tebag was arrested by order
of Mayor Brown, without any warrant or complaint filed in court, and then brought to,
and detained in, the aforementioned sub-station; that while on the way thereto, said
Awalin Tebag was maltreated, pursuant to instructions of Mayor Brown, concurred in by
Pollisco, to the effect that Tebag be mauled until such time as he shall surrender his
gun; that, once in the sub-station, Tebag, whose hands were securely tied, was
subjected, by defendants Lacerna, Hasbi, Pollisco, Dinglasa, and other special
policemen, to further and more severe torture, in consequence of which Tebag died;
that, in order to simulate that Tebag had been killed by peace officers in the course of
an encounter between the latter and a band of armed bandits of which he formed part,
the body of Tebag was brought, early the next morning, to a nearby isolated field, where
defendant Hasbi fired twice at said dead body from behind, and then an old Japanese
rifle, supplied by Mayor Brown, was placed beside said body; and that, in furtherance of
the aforementioned simulation, a report of said imaginary encounter, mentioning Tebag
as the only member of a band of armed bandits whose identity was known, was
submitted and respondent Hasbi caused one of his companions to shoot him on the left
arm.
During the trial of said criminal case, respondent Judge rejected the following evidence
for the prosecution therein:
1. Exhibit A A report of Capt. F. G. Sarrosa, Commanding Officer of the PC
Detachment in Basilan City, who investigated the case, showing that on June 5, 1958,
he and Lt. Clemente Antonio, PAF, found nine (9) detainees in the Tipo-Tipo substation. This was part of the chain of evidence of the prosecution to prove that persons
used to be detained in the aforementioned sub-station by the main respondents herein,
without either a warrant of arrest or a complaint filed in court.
2. Exhibit C Letter of Atty. Doroteo de Guzman to the officer in charge of the substation, dated June 4, 1958, inquiring as to the whereabouts of Awalin Tebag, who,
according to the letter, was arrested in his house, by policemen, on June 4, 1958. Capt.
Sarrosa took possession of this letter in the course of his aforementioned investigation.
3. Exhibits G, G-1, G-2 and G-3 These are the transcript of the testimony of Tebag's
mother, before the City Fiscal of Basilan City, when she asked an autopsy of the body
of her son.
4. Exhibits J to V Consisting of the following, namely: a sketch of the sub-station;
pictures of several huts therein, indicating their relative positions and distances; a
picture depicting how the body of Tebag was taken from a camarin in the sub-station; a
picture showing how Patrolman Hasbiwas shot by a companion, at this request; and a
picture, Exhibit T, demonstrating how Mayor Brown allegedly gave the Japanese rifle,
Exhibit Y, to Hasbi, to be planted beside Tebag's body.

Although referred to by Yakan Carnain, Arit, Lianson, Kona Amenola, and Asidin, in the
course of their testimony as witnesses for the prosecution, these exhibits were not
admitted in evidence, which were presented to show how they were able to observe the
movements in the sub-station, the same being quite small.
5. Exhibits X (a "barong") and X-1 (a scabbard) Amenola said that these effects were
given to him by Mayor Brown in the latter's office, and that he then saw therein the
Japanese rifle, Exhibit Y, which was later placed beside the dead body of Awalin Tebag.
6. Exhibits DD, DD-1, FF, JJ, KK and LL These show that on April 28, 1958, Yakan
Kallapattoh and Fernandez (Pilnandiz) executed affidavits admitting participation in a
given robbery; that an information therefor (Exh. KK) was filed against them on May 2,
1958, with the municipal court of Basilan City (Criminal Case No. 1774); and that, in
compliance with warrants for their arrest then issued, they were apprehended and
detained in the sub-station, thus corroborating the testimony of prosecution witness
Yakans Amenola, Carnain Asidin and Arip to the effect that Kallapattoh and Fernandez
(Pilnandiz) were together with them, in the aforementioned sub-station, when Tebag
was maltreated and died therein, on June 4, 1958, as well as confirming Pollisco's
statement, Exhibit TT-18, before the City Fiscal of Basilan city, on June 21, 1958,
admitting that Fernandez was in the sub-station on June 5, 1958, on account of the
warrant of arrest adverted to. Through the exhibits in question the prosecution sought,
also, to bolster up its theory that Kallapattoh and Fernandez disappeared from the substation after Tebag's death, because the main respondents herein illegally released
them to prevent them from revealing the circumstances surrounding said event.
7. Exhibits II, II-1, and MM These are sketches of a human body and pictures
purporting to show the points of entrance, as well as of exit, of two (2) bullets wounds
found on the body of Tebag. Respondent Judge rejected these exhibits and did not
allow Dr. Rosalino Reyes, Chief of the Medico-Legal Section of the National Bureau of
Investigation, to answer questions asked by the prosecution, to establish that the
trajectories of said bullets wounds were parallel to each other, which, the prosecution
claims, would have been impossible had Tebag been alive when he sustained said
wounds..
8. Respondent Judge sustained, also, the objections to certain questions propounded to
said Dr. Reyes, to show that the injuries sustained by Tebag in the large intestines must
have been inflicted when Tebag was dead already, and did not allow Dr. Reyes to draw
lines on Exhibits II and MM, indicating the connection between the points of entrance
and those of exit of said wounds.
9. Exhibits Z, Z-1, Z-2 These are records of the office of the City Fiscal of Basilan
City showing that the Japanese rifle, Exhibit Y, two rounds of ammunitions and one
empty shell were received by said Office from the Police Department of Basilan City on
June 17, 1958. These exhibits were presented to show that said rifle tallies with the
description thereof given by prosecution witness Kona Amenola, in his affidavit, dated

June 14, 1958, when said weapon was still in the possession of respondent Pollisco,
and hence, to establish Amenola's veracity.
Likewise, the following rebuttal evidence for the prosecution were rejected by
respondent Judge, viz:
1. Exhibits OO to OO-8 These are daily records of events of the police department,
Lamitan District, Basilan City, including the Tipo-Tipo region. They do not mention the
killing therein, by the police patrol, of any outlaw on June 5, 1958, thereby contradicting
the reports (Exhs. 12 and 12-A) of respondent Pollisco and Hasbi about it. Respondent
Judge did not allow the record clerk of the City Fiscal's office to identify said exhibits,
upon the ground that it was too late to present him although when the exhibits were
marked by the prosecution it reserved the right to identify them as part of official
records.
2. Exhibits PP, QQ to QQ-3 Respondent Pollisco had testified that on June 4, 1958,
Hadji Aisa inquired about one Awalin; that he told Aisa that Awalin was taken by Mayor
Brown to the seat of the city government; and that he (Pollisco) suggested that Datu
Unding be advised not to worry, because there was no evidence against Awalin. To
impeach the veracity of Pollisco, the prosecution presented the exhibits under
consideration, for the same show that one Dong Awalin (who is different from Awalin
Tebag) was apprehended on May 27, 1958, and released on bail on June 23, 1958; that
Pollisco could not have truthfully informed Aisa on June 4, 1958, what Dong Awalin had
been taken by Mayor Brown to the seat of the city government and that there was no
evidence against him; for he was then a detention prisoner; and that Pollisco could not
have had in mind, therefore, said Dong Awalin as the Awalin about whom Aisa had
inquired. Indeed, Exhibits TT-13 to TT-16 show that, testifying before the City Fiscal,
respondent Pollisco said that he twice ordered Patrolman Lacerna on June 4, 1958, to
bring Awalin Tebag to him (Pollisco) for investigation.
3. Exhibits SS to SS-7 These are the testimonies before the City Fiscal, of defense
witness Mohammad Sali who, on cross examination by the prosecution, denied having
given it. Thus the predicate therefor was established by the prosecution which sought
thereby to impeach Sali's veracity.
4. Exhibits TT, TT-1 to TT-25 These are the testimonies, before the City Fiscal of the
main respondents herein, who gave a different story before respondent Judge. The
prosecution thus sought to impeach their veracity as witnesses in their own behalf, after
laying down the predicate in the course of their cross examination.
5. Exhibits UU, UU-1 to UU-3 These are sworn statements made by defendant Hasbi
before the City Fiscal. They were presented in rebuttal, after laying down the predicate,
to impeach his testimony in court.
6. Exhibits RR, RR-1, XX and XX-1 With these exhibits the prosecution tried to rebut
Pollisco's testimony to the effect that prosecution witness Lianson Arip had a grudge

against him, he (Pollisco) having charged him with theft in the City Fiscal's Office. It
appears from said exhibits that Arip's affidavit, implicating Pollisco, was dated June 8,
1958, whereas Pollisco's affidavit charging Arip with theft, was dated June 20, 1958, so
that said statement of Arip could not have been influenced by Pollisco's subsequent act.
In contrast with the severe and rigorous policy used by respondent Judge in dealing
with the aforementioned evidence for the prosecution, petitioner herein cites the
liberality with which the lower court admitted, as evidence for the defense, records of
supposed achievements of the Tipo-Tipo sub-station (Exhibits 9 to 9-G, 10 to 10-I, 17 to
17-C, 19 to 19-A, 20 to 20-I 21 and 22), a congratulatory communication (Exh. 24), and
a letter of commendation to a peace officer assigned thereto (Exh. 7), including an
article in the Philippine Free Press (Exhs. 23 and 23-A).
Upon a review of the record, we are fully satisfied that the lower court had, not only
erred, but, also, committed a grave abuse of discretion in issuing the resolutions
complained of, in rejecting the aforementioned direct and rebuttal evidence for the
prosecution, and in not permitting the same to propound the questions, already
adverted to. It is obvious to us that said direct and rebuttal evidence, as well as the
aforementioned questions, are relevant to the issues involved in Criminal Case No. 627.
Although it is not possible to determine with precision, at this stage of the proceedings,
how far said exhibits may affect the outcome of that case, it is elemental that all parties
therein are entitled to a reasonable opportunity to establish their respective pretense. In
this connection it should be noted that, in the light of the allegations of the amended
information in said case and of the records before us, the issue of the guilt or innocence
of the accused therein is bound to hinge heavily upon the veracity of the opposing
witnesses and the weight attached to their respective testimony. Hence, the parties
should be allowed a certain latitude in the presentation of their evidence lest they may
be so hampered that the ends of justice may eventually be defeated or appear to be
defeated. The danger of leading to such result must be avoided, particularly in cases of
the nature, importance and significance of the one under consideration.
With respect to the question whether or not Senator Roseller Lim may appear as
counsel for the main respondents herein, as defendants in said criminal case, the
Constitution provides that no Senator or Member of the House of Representatives shall
"appear as counsel ... in any criminal case wherein an officer or employee of the
Government is accused of an offense committed in relation of his office ... (Art. VI, Sec.
17, Const. of the Phil.). The issue, therefore, is whether the defendants in Criminal case
No. 672 are "accused of an offense committed in relation" to their office.
A mere perusal of the amended information therein readily elicits an affirmative answer.
It is alleged in said amended information that "Leroy S. Brown, City Mayor of Basilan
City, as such, has organized groups of police patrol and civilian commandoes consisting
of regular policemen and ... special policemen, appointed and provided by him with
pistols and high power guns" and then "established a camp ... at Tipo-Tipo," which is
under his "command, ... supervision and control," where his codefendants were
stationed, entertained criminal complaints and conducted the corresponding

investigations, as well as assumed the authority to arrest and detain persons without
due process of law and without bringing them to the proper court, and that, in line with
this set-up established by said Mayor of Basilan City as such, and acting upon his
orders, his codefendants arrested and maltreated Awalin Tebag, who died in
consequence thereof.
It is apparent from these allegations that, although public office is not an element of the
crime of murder in abstract, as committed by the main respondents herein, according to
the amended information, the offense therein charged is intimately connected with their
respective offices and was perpetrated while they were in the performance, though
improper or irregular, of their official functions. Indeed, they had no personal motive to
commit the crime and they would not have committed it had they not held their aforesaid
offices. The co-defendants of respondent Leroy S. Brown, obeyed his instructions
because he was their superior officer, as Mayor of Basilan City.
The case of Monllito vs. Hilario and Crisologo, 90 Phil., 49, relied upon by respondent
Judge, in overruling the objection of the prosecution to the appearance of Senator
Roseller Lim, is not in point, for, as stated in the decision therein:
From the allegations of the information it does not appear that the official
positions of the accused were connected with the offense charged. In fact, the
attorneys for the prosecution stated that the motives for the crimes were personal
with political character. It does not even appear, nor is there assertion, that the
crimes were committed by the defendants in line of duty or in the performance of
their official functions. (Emphasis supplied.)
Such is not the situation obtaining in the case at bar.
Wherefore, the rulings complained of are set aside and reversed and respondent Judge
is hereby enjoined to admit the aforementioned direct and rebuttal evidence for the
prosecution, as well as to permit the formulation, of the questions already referred to,
with costs against the respondents herein. It is so ordered.

[G.R. No. 144261-62. May 9, 2001]

PRUDENTE D. SOLLER, M.D., PRECIOSA M. SOLLER, M.D., RODOLFO I. SALCEDO,


JOSEFINA B. MORADA, MARIO M. MATINING, and ROMMEL M.
LUARCA, petitioners vs. THE HONORABLE SANDIGANBAYAN and PEOPLE
OF THE PHILIPPINES, respondents.
DECISION
GONZAGA-REYES, J.:
This special civil action for certiorari, prohibition and mandamus raises the issue of the
propriety of the assumption of jurisdiction by the Sandiganbayan[1] in Criminal Cases Nos. 25521
and 25522 both entitled People of the Philippines vs. Prudente D. Soller, Preciosa M. Soller,
Rodolfo Salcedo, Josefina Morada, Mario Matining and Rommel Luarca wherein petitioners are
charged with Obstruction of Apprehension and Prosecution of Criminal Offenders as defined and
penalized under P.D. No. 1829. The grounds for petitioners Motion to Quash the Informations
against them are that only petitioner Prudente D. Soller occupied a position classified as Grade 27
and higher and because the offenses charged were not committed by him in violation of his office
as Municipal Mayor of Bansud, Oriental Mindoro.
It appears that in the evening of March 14, 1997, Jerry Macabael a municipal guard, was shot
and killed along the national highway at Bansud, Oriental Mindoro while driving a motorcycle
together with petitioner Sollers son, Vincent M. Soller. His body was brought to a medical clinic
located in the house of petitioner Dr. Prudente Soller, the Municipal Mayor, and his wife Dr.
Preciosa Soller, who is the Municipal Health Officer. The incident was reported to and investigated
by petitioner SPO4 Mario Matining. An autopsy was conducted on the same night on the cadaver
of Jerry by petitioner Dr. Preciosa Soller with the assistance of petitioner Rodolfo Salcedo,
Sanitary Inspector, and petitioner Josefina Morada, Rural Health Midwife.
On the basis of the foregoing incident, a complaint was later filed against the petitioners by
the widow of Jerry Macabael with the Office of the Ombudsman charging them with conspiracy
to mislead the investigation of the fatal shootout of Jerry Macabael by (a) altering his wound (b)
concealing his brain; (c) falsely stating in police report that he had several gunshot wounds when
in truth he had only one; and d) falsely stating in an autopsy report that there was no blackening
around his wound when in truth there was.
Petitioners spouses Soller denied having tampered with the cadaver of Jerry Macabael, and
claimed, among others that Jerry Macabael was brought to their private medical clinic because it
was there where he was rushed by his companions after the shooting, that petitioner Prudente
Soller, who is also a doctor, was merely requested by his wife Preciosa Soller, who was the
Municipal Health Officer, to assist in the autopsy considering that the procedure involved sawing
which required male strength, and that Mrs. Macabaels consent was obtained before the
autopsy. The two (2) police officers denied having planted three (3) shells at the place where the
shooting took place.

The Office of the Ombudsman recommended the filing of an Information for Obstruction of
Justice (Violation of P.D. 1829), and two (2) Informations[2] were filed with the Sandiganbayan
which were docketed as Criminal Cases Nos. 25521 and 25522. The two (2) informations
respectively read as follows:
Criminal Case No. 25521
The undersigned Graft Investigation Officer I, Office of the Deputy Ombudsman for Luzon,
hereby accuses PRUDENTE SOLLER, PRECIOSA SOLLER, MARIO MATINING, ROMMEL
LUARCA, RODOLFO SALCEDO, and JOSIE MORADA, of committing the offense of
Obstruction of Apprehension and Prosecution of Criminal Offenders as defined and penalized
under Section 1, Paragraph b of P.D. 1829, committed as follows:
That on or about March 14, 1997, prior or subsequent thereto, at the Municipality of Bansud,
Oriental Mindoro and within the jurisdiction of this Honorable Court, the above name accused,
all public officers, then being the Municipal Mayor, Municipal Health Officer, SPO II, PO 1,
Sanitary Inspector and Midwife, respectively, all of said municipality, conspiring and
confederating with one another, did then and there wilfully, unlawfully, and criminally alter and
suppress the gunshot wound and conceal the brain of JERRY MACABAEL with intent to impair
its veracity, authenticity, and availability as evidence in the investigation of criminal case for
murder against the accused Vincent Soller, the son of herein respondents.
CONTRARY TO LAW.
Criminal Case No. 25522
The undersigned Graft Investigation Officer, I, Office of the Deputy Ombudsman for Luzon,
hereby accuses PRUDENTE SOLLER, PRECIOSA SOLLER, MARIO MATINING, ROMMEL
LUARCA, RODOLFO SALCEDO, and JOSIE MORADA, of committing the offense of
Obstruction of Apprehension and Prosecution of Criminal Offenders as defined and penalized
under Section 1, Paragraph b of P.D. 1829, committed as follows:
That on or about March 14, 1997, prior or subsequent thereto, at the Municipality of Bansud,
Oriental Mindoro and within the jurisdiction of this Honorable Court, the above name accused,
all public officers, then being the Municipal Mayor, Municipal health Officer, SPO II, PO 1,
Sanitary Inspector and Midwife, respectively, all of said municipality, conspiring and
confederating with one another, did then and there wilfully, unlawfully, and criminal give false
and fabricated information in the autopsy report and police report to mislead or prevent the law
enforcement agency, from apprehending the offender by reporting that there are several gunshot
wounds in the body of the victim, JERRY MACABAEL and that there is no tattooing
(blackening) around the wound of the said victim when in truth and in fact, there is only one
gunshot wound and there is tattooing (blackening) around the wound which would indicate that
the victim was shot by Vincent Soller, the son of the herein respondents spouses Prudente and
Preciosa Soller.
CONTRARY TO LAW.

Petitioners filed a Motion to Quash on the principal ground that the Sandiganbayan had no
jurisdiction over the offenses charged; this motion was opposed by respondent People. In its
assailed Order dated April 14, 2000, the Sandiganbayan denied petitioners Motion to Quash on the
ground that the accusation involves the performance of the duties of at least one (1) of the accused
public officials, and if the Mayor is indeed properly charged together with that official, then the
Sandiganbayan has jurisdiction over the entire case and over all the co-accused. The Order stated
that the accused is the Mayor of the municipality where the alleged incident took place and,
therefore, any attempt to deviate or to present false evidence in connection with a criminal offense
committed in his municipality for which he is charged would be an offense also in which the
accused Mayor would be probably held accountable before this Court.
Motion for Reconsideration of the above order was filed on the premise that it is not among
the functions of the mayor to conduct autopsies so that any misdeed, if indeed there was any, could
not be an offense which would put him under the jurisdiction of the court. Motion for
Reconsideration was denied, the Sandiganbyan ruling that:
The enumeration of the functions of the mayor indicate very clearly that he is the primary
executive and, therefore, necessarily the primary peace officer of the municipality, for which
reason, any action on his part which deviates from that function is an office-related offense. In
this particular instance, the accused is charged for having cooperated or co-participated with
another public official of lower rank in the same municipality in the supposed falsification of the
results of an autopsy. Additionally, even if the functions of an autopsy were totally unrelated to
any of the administrative or executive functions over which the mayor may have supervision
and, more specially, control, the fact of the matter is that the jurisdiction of the Court covers not
only the offenses committed by the officials of Grade Level 27 or higher as the principal accused
but even where such officials are also accused together with some other public officials who may
be at a level below Grade Level 27 in connection with the performance of their duties.
In this instance, accused Mayor Prudente D. Soller, Sr. who occupies a position at Grade Level
27, is co-accused with his wife, the Municipal Health Officer who occupies a position at Grade
Level 24, so that, necessarily, the offense attributed to the lower ranking officer elevates the
entire case to this Court primarily because somebody over whom this Court has jurisdiction, the
Mayor, is accused together with the lower ranking officer.[3]
Hence, this petition alleging thatRESPONDENT SANDIGANBAYAN ACTED WITHOUT OR IN EXCESS OF
JURISDICTION OR WITH GRAVE ABUSE OF DISCRETION AMOUNTING TO LACK OF
JURISDICTION IN HOLDING THAT IT HAS JURISDICTION OVER THE OFFENSE
CHARGED IN SUBJECT CRIMINAL CASES NOS. 25521 and 25522.[4]
Citing Section 4 of P.D. 1606 as amended, which defines the jurisdiction of the
Sandiganbayan, petitioners claim that for an offense to fall within the jurisdiction of the
Sandiganbayan, the offense must have been committed by the officials enunciated in paragraph
(a) in relation to their office, i.e. it should be intimately connected with the office of the offender,
and should have been perpetrated while the offender was in the performance of his official
functions. Moreover, these requisites must all be alleged in the information. Petitioners assert that

in the subject criminal cases, the Informations do not contain factual averments showing that they
committed the acts charged in relation to their office, i.e., the acts charged are intimately connected
with their respective offices and were perpetrated by them while they were in the performance of
their duties and functions.
On the other hand, respondent People of the Philippines, represented by the Office of the
Ombudsman, through the Office of the Special Prosecutor, posits that even if the offense charged
was not committed by the accused while in the performance of his official functions, the same
could still be considered done in relation to his office if the acts were committed in line of
duty. Respondents position is that an offense may be considered committed in relation to office if
it arose from misuse or abuse of public office or from non-performance of an official duty or
function; thus the offense of falsifying autopsy and police reports is office-related considering that
among the duties and functions of the municipal mayor in the exercise of general supervision and
control over all programs, projects, services and activities of the municipal government, is that he
shall ensure that all executive officials and employees of the municipality faithfully discharge their
duties and functions. The fact that the informations do not allege that the acts charged were
committed by petitioner Prudente Soller while he was in the performance of his official functions
or duties is not a fatal defect, as the conclusion of law that his acts are in violation of his duties as
municipal mayor could necessarily be deduced from the informations.
Petitioners, in their Reply, reiterate that the factual averments in the Information were fatally
defective in view of the absence of any specific allegation that would indicate that the crimes
charged were committed by the defendants in line of duty or in the performance of their official
functions.
The petition is meritorious.
The rule is that in order to ascertain whether a court has jurisdiction or not, the provisions of
the law should be inquired into.[5] Furthermore, the jurisdiction of the court must appear clearly
from the statute law or it will not be held to exist. It cannot be presumed or implied. For this
purpose in criminal cases, the jurisdiction of the court is determined by the law at the time of the
commencement of the action.[6]
The action here was instituted with the filing of the Informations on May 25, 1999 charging
the petitioners with the offense of Obstruction of Apprehension and Prosecution of Criminal
Offenders as defined and penalized under Section 1, Paragraph b of P.D. 1829. The applicable
statutory provisions are those of P.D. No. 1606 as last amended by the Republic Act No.
8249. Section 4 of P.D. No. 1606 as amended provides insofar as pertinent:
SEC. 4. Jurisdiction - The Sandiganbayan shall exercise exclusive original jurisdiction in all
cases involving:
a. Violations of Republic Act No. 3019, as amended, otherwise known as the Anti-Graft and
Corruption Practices Act, Republic Act No. 1379, and Chapter II, Section 2, Title VII, Book II of
the Revised Penal Code, where one or more of the accused are officials occupying the following
positions in the government, whether in a permanent, acting or interim capacity, at the time of
the commission of the offense:
xxx xxx xxx

(5) All other national and local officials classified as Grade 27 and higher under the
Compensation and Position Classification Act of 1989.
xxx xxx xxx
b. Other offenses or felonies whether simple or complexed with other crime committed by the
public officials and employees mentioned in subsection a of this section in relation to their
office.
xxx xxx xxx
In cases where none of the accused are occupying positions corresponding to salary Grade 27 or
higher, as prescribed in the said Republic Act 6758, or military and PNP officers mentioned
above, exclusive original jurisdiction thereof shall be vested in the proper regional trial court,
metropolitan trial court, municipal trial court, and municipal circuit trial court, as the case may
be, pursuant to their jurisdictions as provided by Batas Pambansa Blg. 129, amended.
xxx xxx xxx
In Binay vs. Sandiganbayan,[7] this Court held that the Municipal Mayor, who occupies Salary
Grade 27 in the hierarchy of positions in the government under Republic Act No. 6758 and the
Index of Occupational Services. Position Titles and Salary Grades, falls within the exclusive
original jurisdiction of the Sandiganbayan.
The bone of contention here is whether the offenses charged may be considered as committed
in relation to their office as this phrase is employed in the above-quoted Section 4.
As early as Montilla vs. Hilario,[8] this Court has interpreted the requirement that an offense
be committed in relation to the office to mean that the offense cannot exist without the office or
that the office must be a constituent element of the crime as defined and punished in Chapter Two
to Six, Title Seven of the Revised Penal Code (referring to the crimes committed by the public
officers). People vs. Montejo[9] enunciated the principle that the offense must be intimately
connected with the office of the offender and perpetrated while he was in the performance, though
improper or irregular of his official functions. The Court, speaking through Chief Justice
Concepcion said that although public office is not an element of the crime of murder in (the)
abstract, the facts in a particular case may show that xxx the offense therein charged is intimately connected with (the accuseds) respective offices
and was perpetrated while they were in the performance though improper or irregular, of their
official functions.Indeed (the accused) had no personal motive to commit the crime and they
would not have committed it had they not held their aforesaid offices. The co-defendants of
respondent Leroy S. Brown obeyed his instructions because he was their superior officer, as
Mayor of Basilan City.[10]
The cited rulings in Montilla vs. Hilario and in People vs. Montejo were reiterated in Sanchez vs.
Demetriou,[11] Republic vs. Asuncion,[12] and Cunanan vs. Arceo.[13] The case of Republic vs.
Asuncioncategorically pronounced that the fact that offense was committed in relation to the office
must be alleged in the information:

That the public officers or employees committed the crime in relation to their office, must,
however, be alleged in the information for the Sandiganbayan to have jurisdiction over a case
under Section 4 (a) (2). This allegation is necessary because of the unbending rule that
jurisdiction is determined by the allegations of the information.[14]
For this purpose what is controlling is not whether the phrase committed in violation to public
office appears in the information; what determines the jurisdiction of the Sandiganbayan is the
specific factual allegation in the information that would indicate close intimacy between the
discharge of the accuseds official duties and the commission of the offense charged in order to
qualify the crime as having been committed in relation to public office.[15]
In this case, the Informations subject of Criminal Cases Nos. 25521 and 25522 quoted earlier,
fail to allege that petitioners had committed the offenses charged in relation to their offices. Neither
are there specific allegations of facts to show the intimate relation/connection between the
commission of the offense charged and the discharge of official functions of the offenders, i.e. that
the obstruction of and apprehension and prosecution of criminal offenders was committed in
relation to the office of petitioner Prudente Soller, whose office as Mayor is included in the
enumeration in Section 4 (a) of P.D. 1606 as amended. Although the petitioners were described as
being all public officers, then being the Municipal Mayor, Municipal Health Officer, SPO II, PO
I, Sanitary Inspector and Midwife, there was no allegation that the offense of altering and
suppressing the gunshot wound of the victim with intent to impair the veracity, authenticity and
availability as evidence in the investigation of the criminal case for murder (Criminal Case No.
25521) or of giving false and fabricated information in the autopsy report and police report to
mislead the law enforcement agency and prevent the apprehension of the offender (Criminal Case
No. 25522) was done in the performance of official function. Indeed the offenses defined in P.D.
1829 may be committed by any person whether a public officer or a private citizen, and
accordingly public office is not an element of the offense. Moreover, the Information in Criminal
Case No. 25522 states that the fabrication of information in the police and autopsy report would
indicate that the victim was shot by Vincent Soller, the son of herein petitioners spouses Prudente
and Preciosa Soller. Thus there is a categorical indication that the petitioners spouses Soller had a
personal motive to commit the offenses and they would have committed the offenses charged even
if they did not respectively hold the position of Municipal Mayor or Municipal Health Officer.
A cursory reading of the duties and functions of the Municipal Mayor as enumerated in
Section 444 of the Local Government Code will readily show that the preparation of police and
autopsy reports and the presentation and gathering of evidence in the investigation of criminal
cases are not among such duties and functions, and the broad responsibility to maintain peace and
order cannot be a basis for construing that the criminal acts imputed to petitioner Mayor fall under
his functions as Municipal Mayor.[16] What is obvious is that petitioners spouses probably acted as
the parents of the alleged assailant and if at all, were motivated by personal reasons rather than
official duty.
Consequently, for failure to show in the informations that the charges were intimately
connected with the discharge of the official functions of accused Mayor Soller, the offenses
charged in the subject criminal cases fall within the exclusive original function of the Regional
Trial Court, not the Sandiganbayan.

WHEREFORE, the petition is GRANTED and the challenged orders are SET ASIDE and
declared NULL and VOID for lack of jurisdiction. No costs.
SO ORDERED.

HANNAH EUNICE D. SERANA, G.R. No. 162059


Petitioner,
Present:
YNARES-SANTIAGO, J.,
Chairperson,
- versus - AUSTRIA-MARTINEZ,
CORONA,*
NACHURA, and
REYES, JJ.
SANDIGANBAYAN and Promulgated:
PEOPLE OF THE PHILIPPINES,
Respondents. January 22, 2008
x--------------------------------------------------x
DECISION

REYES, R.T., J.:


CAN the Sandiganbayan try a government scholar** accused, along with her brother, of
swindling government funds?
MAAARI bang litisin ng Sandiganbayan ang isang iskolar ng bayan, at ang kanyang
kapatid, na kapwa pinararatangan ng estafa ng pera ng bayan?
The jurisdictional question is posed in this petition for certiorari assailing the
Resolutions[1] of the Sandiganbayan, Fifth Division, denying petitioners motion to quash the
information and her motion for reconsideration.
The Antecedents
Petitioner Hannah Eunice D. Serana was a senior student of the University of the PhilippinesCebu. A student of a state university is known as a government scholar. She was appointed by then
President Joseph Estrada on December 21, 1999 as a student regent of UP, to serve a one-year
term starting January 1, 2000 and ending on December 31, 2000.
In the early part of 2000, petitioner discussed with President Estrada the renovation of
Vinzons Hall Annex in UP Diliman.[2] On September 4, 2000, petitioner, with her siblings and

relatives, registered with the Securities and Exchange Commission the Office of the Student
Regent Foundation, Inc. (OSRFI).[3]
One of the projects of the OSRFI was the renovation of the Vinzons Hall Annex.[4] President
Estrada gave Fifteen Million Pesos (P15,000,000.00) to the OSRFI as financial assistance for the
proposed renovation. The source of the funds, according to the information, was the Office of the
President.
The renovation of Vinzons Hall Annex failed to materialize.[5] The succeeding student regent,
Kristine Clare Bugayong, and Christine Jill De Guzman, Secretary General of the KASAMA sa
U.P., a system-wide alliance of student councils within the state university, consequently filed a
complaint for Malversation of Public Funds and Property with the Office of the Ombudsman.[6]
On July 3, 2003, the Ombudsman, after due investigation, found probable cause to indict
petitioner and her brother Jade Ian D. Serana for estafa, docketed as Criminal Case No. 27819 of
the Sandiganbayan.[7] The Information reads:
The undersigned Special Prosecution Officer III, Office of the Special
Prosecutor, hereby accuses HANNAH EUNICE D. SERANA and JADE IAN D.
SERANA of the crime of Estafa, defined and penalized under Paragraph 2(a),
Article 315 of the Revised Penal Code, as amended committed as follows:
That on October, 24, 2000, or sometime prior or subsequent thereto, in
Quezon City, Metro Manila, Philippines, and within the jurisdiction of this
Honorable Court, above-named accused, HANNAH EUNICE D. SERANA, a
high-ranking public officer, being then the Student Regent of the University of the
Philippines, Diliman, Quezon City, while in the performance of her official
functions, committing the offense in relation to her office and taking advantage of
her position, with intent to gain, conspiring with her brother, JADE IAN D.
SERANA, a private individual, did then and there wilfully, unlawfully and
feloniously defraud the government by falsely and fraudulently representing to
former President Joseph Ejercito Estrada that the renovation of the Vinzons Hall of
the University of the Philippines will be renovated and renamed as President Joseph
Ejercito Estrada Student Hall, and for which purpose accused HANNAH EUNICE
D. SERANA requested the amount of FIFTEEN MILLION PESOS
(P15,000,000.00), Philippine Currency, from the Office of the President, and the
latter relying and believing on said false pretenses and misrepresentation gave and
delivered to said accused Land Bank Check No. 91353 dated October 24, 2000 in
the amount of FIFTEEN MILLION PESOS (P15,000,000.00), which check was
subsequently encashed by accused Jade Ian D. Serana on October 25, 2000 and
misappropriated for their personal use and benefit, and despite repeated demands

made upon the accused for them to return aforesaid amount, the said accused failed
and refused to do so to the damage and prejudice of the government in the aforesaid
amount.
CONTRARY TO LAW. (Underscoring supplied)
Petitioner moved to quash the information. She claimed that the Sandiganbayan does not
have any jurisdiction over the offense charged or over her person, in her capacity as UP student
regent.
Petitioner claimed that Republic Act (R.A.) No. 3019, as amended by R.A. No. 8249,
enumerates the crimes or offenses over which the Sandiganbayan has jurisdiction.[8]It has no
jurisdiction over the crime of estafa.[9] It only has jurisdiction over crimes covered by Title VII,
Chapter II, Section 2 (Crimes Committed by Public Officers), Book II of the Revised Penal Code
(RPC). Estafa falling under Title X, Chapter VI (Crimes Against Property), Book II of the RPC is
not within the Sandiganbayans jurisdiction.
She also argued that it was President Estrada, not the government, that was duped. Even
assuming that she received the P15,000,000.00, that amount came from Estrada, not from the
coffers of the government.[10]
Petitioner likewise posited that the Sandiganbayan had no jurisdiction over her person. As a
student regent, she was not a public officer since she merely represented her peers, in contrast to
the other regents who held their positions in an ex officio capacity. She added that she was a simple
student and did not receive any salary as a student regent.
She further contended that she had no power or authority to receive monies or funds. Such power
was vested with the Board of Regents (BOR) as a whole. Since it was not alleged in the information
that it was among her functions or duties to receive funds, or that the crime was committed in
connection with her official functions, the same is beyond the jurisdiction of the Sandiganbayan
citing the case of Soller v. Sandiganbayan.[11]
The Ombudsman opposed the motion.[12] It disputed petitioners interpretation of the law. Section
4(b) of Presidential Decree (P.D.) No. 1606 clearly contains the catch-all phrase in relation to
office, thus, the Sandiganbayan has jurisdiction over the charges against petitioner. In the same
breath, the prosecution countered that the source of the money is a matter of defense. It should be
threshed out during a full-blown trial.[13]

According to the Ombudsman, petitioner, despite her protestations, was a public officer. As a
member of the BOR, she had the general powers of administration and exercised the corporate
powers of UP. Based on Mechems definition of a public office, petitioners stance that she was not
compensated, hence, not a public officer, is erroneous. Compensation is not an essential part of
public office. Parenthetically, compensation has been interpreted to include allowances. By this
definition, petitioner was compensated.[14]
Sandiganbayan Disposition
In a Resolution dated November 14, 2003, the Sandiganbayan denied petitioners motion for lack
of merit.[15] It ratiocinated:
The focal point in controversy is the jurisdiction of the Sandiganbayan over this
case.
It is extremely erroneous to hold that only criminal offenses covered by Chapter II,
Section 2, Title VII, Book II of the Revised Penal Code are within the jurisdiction
of this Court. As correctly pointed out by the prosecution, Section 4(b) of R.A. 8249
provides that the Sandiganbayan also has jurisdiction over other offenses
committed by public officials and employees in relation to their office. From this
provision, there is no single doubt that this Court has jurisdiction over the offense
of estafa committed by a public official in relation to his office.
Accused-movants claim that being merely a member in representation of the
student body, she was never a public officer since she never received any
compensation nor does she fall under Salary Grade 27, is of no moment, in view of
the express provision of Section 4 of Republic Act No. 8249 which provides:
Sec. 4. Jurisdiction The Sandiganbayan shall exercise exclusive original
jurisdiction in all cases involving:
(A) x x x
(1) Officials of the executive branch occupying the positions of regional director
and higher, otherwise classified as Grade 27 and higher, of the Compensation and
Position Classification Act of 1989 (Republic Act No. 6758), specifically
including:
xxxx
(g) Presidents, directors or trustees, or managers of government-owned or
controlled corporations, state universities or educational institutions or
foundations. (Italics supplied)

It is very clear from the aforequoted provision that the Sandiganbayan has original
exclusive jurisdiction over all offenses involving the officials enumerated in
subsection (g), irrespective of their salary grades, because the primordial
consideration in the inclusion of these officials is the nature of their responsibilities
and functions.
Is accused-movant included in the contemplated provision of law?
A meticulous review of the existing Charter of the University of the Philippines
reveals that the Board of Regents, to which accused-movant belongs, exclusively
exercises the general powers of administration and corporate powers in the
university, such as: 1) To receive and appropriate to the ends specified by law such
sums as may be provided by law for the support of the university; 2) To prescribe
rules for its own government and to enact for the government of the university such
general ordinances and regulations, not contrary to law, as are consistent with the
purposes of the university; and 3) To appoint, on recommendation of the President
of the University, professors, instructors, lecturers and other employees of the
University; to fix their compensation, hours of service, and such other duties and
conditions as it may deem proper; to grant to them in its discretion leave of absence
under such regulations as it may promulgate, any other provisions of law to the
contrary notwithstanding, and to remove them for cause after an investigation and
hearing shall have been had.
It is well-established in corporation law that the corporation can act only through
its board of directors, or board of trustees in the case of non-stock corporations. The
board of directors or trustees, therefore, is the governing body of the corporation.
It is unmistakably evident that the Board of Regents of the University of
the Philippines is performing functions similar to those of the Board of Trustees of
a non-stock corporation. This draws to fore the conclusion that being a member of
such board, accused-movant undoubtedly falls within the category of public
officials upon whom this Court is vested with original exclusive jurisdiction,
regardless of the fact that she does not occupy a position classified as Salary Grade
27 or higher under the Compensation and Position Classification Act of 1989.
Finally, this court finds that accused-movants contention that the same of P15
Million was received from former President Estrada and not from the coffers of the
government, is a matter a defense that should be properly ventilated during the trial
on the merits of this case.[16]
On November 19, 2003, petitioner filed a motion for reconsideration.[17] The motion was
denied with finality in a Resolution dated February 4, 2004.[18]
Issue

Petitioner is now before this Court, contending that THE RESPONDENT COURT
COMMITTED GRAVE ABUSE OF DISCRETION AMOUNTING TO LACKAND/OR
EXCESS OF JURISDICTION IN NOT QUASHING THE INFORMATION AND DISMISING
THE CASE NOTWITHSTANDING THAT IS HAS NO JURISDICTION OVER THE
OFFENSE CHARGED IN THE INFORMATION.[19]
In her discussion, she reiterates her four-fold argument below, namely: (a) the Sandiganbayan has
no jurisdiction over estafa; (b) petitioner is not a public officer with Salary Grade 27 and she paid
her tuition fees; (c) the offense charged was not committed in relation to her office; (d) the funds
in question personally came from President Estrada, not from the government.
Our Ruling
The petition cannot be granted.
Preliminarily, the denial of a motion to
quash is not correctible by certiorari.
We would ordinarily dismiss this petition for certiorari outright on procedural
grounds. Well-established is the rule that when a motion to quash in a criminal case is denied, the
remedy is not a petition for certiorari, but for petitioners to go to trial, without prejudice to
reiterating the special defenses invoked in their motion to quash.[20]Remedial measures as regards
interlocutory orders, such as a motion to quash, are frowned upon and often dismissed.[21] The
evident reason for this rule is to avoid multiplicity of appeals in a single action.[22]
In Newsweek, Inc. v. Intermediate Appellate Court,[23] the Court clearly explained and
illustrated the rule and the exceptions, thus:
As a general rule, an order denying a motion to dismiss is merely
interlocutory and cannot be subject of appeal until final judgment or order is
rendered. (Sec. 2 of Rule 41). The ordinary procedure to be followed in such a case
is to file an answer, go to trial and if the decision is adverse, reiterate the issue on
appeal from the final judgment. The same rule applies to an order denying a motion
to quash, except that instead of filing an answer a plea is entered and no appeal lies
from a judgment of acquittal.
This general rule is subject to certain exceptions. If the court, in denying the
motion to dismiss or motion to quash, acts without or in excess of jurisdiction or
with grave abuse of discretion, then certiorari or prohibition lies. The reason is that

it would be unfair to require the defendant or accused to undergo the ordeal and
expense of a trial if the court has no jurisdiction over the subject matter or offense,
or is not the court of proper venue, or if the denial of the motion to dismiss or
motion to quash is made with grave abuse of discretion or a whimsical and
capricious exercise of judgment. In such cases, the ordinary remedy of appeal
cannot be plain and adequate. The following are a few examples of the exceptions
to the general rule.
In De Jesus v. Garcia (19 SCRA 554), upon the denial of a motion to
dismiss based on lack of jurisdiction over the subject matter, this Court granted the
petition for certiorari and prohibition against the City Court of Manila and directed
the respondent court to dismiss the case.
In Lopez v. City Judge (18 SCRA 616), upon the denial of a motion to quash
based on lack of jurisdiction over the offense, this Court granted the petition for
prohibition and enjoined the respondent court from further proceeding in the case.

In Enriquez v. Macadaeg (84 Phil. 674), upon the denial of a motion to


dismiss based on improper venue, this Court granted the petition for prohibition
and enjoined the respondent judge from taking cognizance of the case except to
dismiss the same.
In Manalo v. Mariano (69 SCRA 80), upon the denial of a motion to dismiss
based on bar by prior judgment, this Court granted the petition for certiorari and
directed the respondent judge to dismiss the case.
In Yuviengco v. Dacuycuy (105 SCRA 668), upon the denial of a motion to
dismiss based on the Statute of Frauds, this Court granted the petition
for certiorari and dismissed the amended complaint.
In Tacas v. Cariaso (72 SCRA 527), this Court granted the petition
for certiorari after the motion to quash based on double jeopardy was denied by
respondent judge and ordered him to desist from further action in the criminal case
except to dismiss the same.
In People v. Ramos (83 SCRA 11), the order denying the motion to quash
based on prescription was set aside on certiorari and the criminal case was
dismissed by this Court.[24]
We do not find the Sandiganbayan to have committed a grave abuse of discretion.
The jurisdiction of the Sandiganbayan is
set by P.D. No. 1606, as amended, not by
R.A. No. 3019, as amended.

We first address petitioners contention that the jurisdiction of the Sandiganbayan is


determined by Section 4 of R.A. No. 3019 (The Anti-Graft and Corrupt Practices Act, as
amended). We note that petitioner refers to Section 4 of the said law yet quotes Section 4 of P.D.
No. 1606, as amended, in her motion to quash before the Sandiganbayan.[25]She repeats the
reference in the instant petition for certiorari[26] and in her memorandum of authorities.[27]

We cannot bring ourselves to write this off as a mere clerical or typographical error. It
bears stressing that petitioner repeated this claim twice despite corrections made by the
Sandiganbayan.[28]
Her claim has no basis in law. It is P.D. No. 1606, as amended, rather than R.A. No. 3019,
as amended, that determines the jurisdiction of the Sandiganbayan. A brief legislative history of
the statute creating the Sandiganbayan is in order. The Sandiganbayan was created by P.D. No.
1486, promulgated by then President Ferdinand E. Marcos onJune 11, 1978. It was promulgated to
attain the highest norms of official conduct required of public officers and employees, based on
the concept that public officers and employees shall serve with the highest degree of responsibility,
integrity, loyalty and efficiency and shall remain at all times accountable to the people.[29]
P.D. No. 1486 was, in turn, amended by P.D. No. 1606 which was promulgated on
December 10, 1978. P.D. No. 1606 expanded the jurisdiction of the Sandiganbayan.[30]
P.D. No. 1606 was later amended by P.D. No. 1861 on March 23, 1983, further altering
the Sandiganbayan jurisdiction. R.A. No. 7975 approved on March 30, 1995 made succeeding
amendments to P.D. No. 1606, which was again amended on February 5, 1997 by R.A. No.
8249. Section 4 of R.A. No. 8249 further modified the jurisdiction of the Sandiganbayan. As it
now stands, the Sandiganbayan has jurisdiction over the following:
Sec. 4. Jurisdiction. - The Sandiganbayan shall exercise exclusive original
jurisdiction in all cases involving:
A. Violations of Republic Act No. 3019, as amended, other known as the AntiGraft and Corrupt Practices Act, Republic Act No. 1379, and Chapter II, Section 2,
Title VII, Book II of the Revised Penal Code, where one or more of the accused are

officials occupying the following positions in the government, whether in a


permanent, acting or interim capacity, at the time of the commission of the offense:
(1) Officials of the executive branch occupying the positions of regional director
and higher, otherwise classified as Grade 27 and higher, of the Compensation and
Position Classification Act of 989 (Republic Act No. 6758), specifically including:
(a) Provincial governors, vice-governors, members of the sangguniang
panlalawigan, and provincial treasurers, assessors, engineers, and other city
department heads;
(b) City mayor, vice-mayors, members of the sangguniang panlungsod, city
treasurers, assessors, engineers, and other city department heads;
(c) Officials of the diplomatic service occupying the position of consul and
higher;
(d) Philippine army and air force colonels, naval captains, and all officers of higher
rank;
(e) Officers of the Philippine National Police while occupying the position of
provincial director and those holding the rank of senior superintended or higher;

(f) City and provincial prosecutors and their assistants, and officials and
prosecutors in the Office of the Ombudsman and special prosecutor;
(g) Presidents, directors or trustees, or managers of government-owned or
controlled corporations, state universities or educational institutions or foundations.
(2) Members of Congress and officials thereof classified as Grade 27 and up under
the Compensation and Position Classification Act of 1989;
(3) Members of the judiciary without prejudice to the provisions of the
Constitution;
(4) Chairmen and members of Constitutional Commission, without prejudice to the
provisions of the Constitution; and
(5) All other national and local officials classified as Grade 27 and higher under the
Compensation and Position Classification Act of 1989.
B. Other offenses of felonies whether simple or complexed with other
crimes committed by the public officials and employees mentioned in subsection a
of this section in relation to their office.

C. Civil and criminal cases filed pursuant to and in connection with


Executive Order Nos. 1, 2, 14 and 14-A, issued in 1986.
In cases where none of the accused are occupying positions corresponding to Salary
Grade 27 or higher, as prescribed in the said Republic Act No. 6758, or military
and PNP officer mentioned above, exclusive original jurisdiction thereof shall be
vested in the proper regional court, metropolitan trial court, municipal trial court,
and municipal circuit trial court, as the case may be, pursuant to their respective
jurisdictions as provided in Batas Pambansa Blg. 129, as amended.
The Sandiganbayan shall exercise exclusive appellate jurisdiction over final
judgments, resolutions or order of regional trial courts whether in the exercise of
their own original jurisdiction or of their appellate jurisdiction as herein provided.
The Sandiganbayan shall have exclusive original jurisdiction over petitions for the
issuance of the writs of mandamus, prohibition, certiorari, habeas corpus,
injunctions, and other ancillary writs and processes in aid of its appellate
jurisdiction and over petitions of similar nature, including quo warranto, arising or
that may arise in cases filed or which may be filed under Executive Order Nos. 1,
2, 14 and 14-A, issued in 1986: Provided, That the jurisdiction over these petitions
shall not be exclusive of the Supreme Court.
The procedure prescribed in Batas Pambansa Blg. 129, as well as the implementing
rules that the Supreme Court has promulgated and may thereafter promulgate,
relative to appeals/petitions for review to the Court of Appeals, shall apply to
appeals and petitions for review filed with the Sandiganbayan. In all cases elevated
to the Sandiganbayan and from the Sandiganbayan to the Supreme Court, the Office
of the Ombudsman, through its special prosecutor, shall represent the People of the
Philippines, except in cases filed pursuant to Executive Order Nos. 1, 2, 14 and 14A, issued in 1986.
In case private individuals are charged as co-principals, accomplices or accessories
with the public officers or employees, including those employed in governmentowned or controlled corporations, they shall be tried jointly with said public officers
and employees in the proper courts which shall exercise exclusive jurisdiction over
them.
Any provisions of law or Rules of Court to the contrary notwithstanding, the
criminal action and the corresponding civil action for the recovery of civil liability
shall, at all times, be simultaneously instituted with, and jointly determined in, the
same proceeding by the Sandiganbayan or the appropriate courts, the filing of the
criminal action being deemed to necessarily carry with it the filing of the civil
action, and no right to reserve the filing such civil action separately from the
criminal action shall be recognized: Provided, however, That where the civil action
had heretofore been filed separately but judgment therein has not yet been
rendered, and the criminal case is hereafter filed with the Sandiganbayan or the

appropriate court, said civil action shall be transferred to the Sandiganbayan or the
appropriate court, as the case may be, for consolidation and joint determination with
the criminal action, otherwise the separate civil action shall be deemed abandoned.
Upon the other hand, R.A. No. 3019 is a penal statute approved on August 17, 1960. The
said law represses certain acts of public officers and private persons alike which constitute graft
or corrupt practices or which may lead thereto.[31] Pursuant to Section 10 of R.A. No. 3019, all
prosecutions for violation of the said law should be filed with the Sandiganbayan.[32]
R.A. No. 3019 does not contain an enumeration of the cases over which the Sandiganbayan
has jurisdiction. In fact, Section 4 of R.A. No. 3019 erroneously cited by petitioner, deals not with
the jurisdiction of the Sandiganbayan but with prohibition on private individuals. We quote:
Section 4. Prohibition on private individuals. (a) It shall be unlawful for
any person having family or close personal relation with any public official to
capitalize or exploit or take advantage of such family or close personal relation by
directly or indirectly requesting or receiving any present, gift or material or
pecuniary advantage from any other person having some business, transaction,
application, request or contract with the government, in which such public official
has to intervene. Family relation shall include the spouse or relatives by
consanguinity or affinity in the third civil degree. The word close personal relation
shall include close personal friendship, social and fraternal connections, and
professional employment all giving rise to intimacy which assures free access to
such public officer.
(b) It shall be unlawful for any person knowingly to induce or cause any
public official to commit any of the offenses defined in Section 3 hereof.
In fine, the two statutes differ in that P.D. No. 1606, as amended, defines the jurisdiction
of the Sandiganbayan while R.A. No. 3019, as amended, defines graft and corrupt practices and
provides for their penalties.
Sandiganbayan has jurisdiction over
the offense of estafa.
Relying on Section 4 of P.D. No. 1606, petitioner contends that estafa is not among those
crimes cognizable by the Sandiganbayan. We note that in hoisting this argument, petitioner
isolated the first paragraph of Section 4 of P.D. No. 1606, without regard to the succeeding
paragraphs of the said provision.

The rule is well-established in this jurisdiction that statutes should receive a sensible
construction so as to avoid an unjust or an absurd conclusion.[33] Interpretatio talis in ambiguis
semper fienda est, ut evitetur inconveniens et absurdum. Where there is ambiguity, such
interpretation as will avoid inconvenience and absurdity is to be adopted. Kung saan mayroong
kalabuan, ang pagpapaliwanag ay hindi dapat maging mahirap at katawa-tawa.
Every section, provision or clause of the statute must be expounded by reference to each
other in order to arrive at the effect contemplated by the legislature.[34] The intention of the
legislator must be ascertained from the whole text of the law and every part of the act is to be taken
into view.[35] In other words, petitioners interpretation lies in direct opposition to the rule that a
statute must be interpreted as a whole under the principle that the best interpreter of a statute is the
statute itself.[36] Optima statuti interpretatrix est ipsum statutum. Ang isang batas ay marapat na
bigyan ng kahulugan sa kanyang kabuuan sa ilalim ng prinsipyo na ang pinakamainam na
interpretasyon ay ang mismong batas.
Section 4(B) of P.D. No. 1606 reads:
B. Other offenses or felonies whether simple or complexed with other
crimes committed by the public officials and employees mentioned in subsection a
of this section in relation to their office.
Evidently, the Sandiganbayan has jurisdiction over other felonies committed by public officials in
relation to their office. We see no plausible or sensible reason to exclude estafaas one of the
offenses included in Section 4(B) of P.D. No. 1606. Plainly, estafa is one of those other
felonies. The jurisdiction is simply subject to the twin requirements that (a) the offense is
committed by public officials and employees mentioned in Section 4(A) of P.D. No. 1606, as
amended, and that (b) the offense is committed in relation to their office.
In Perlas, Jr. v. People,[37] the Court had occasion to explain that the Sandiganbayan has
jurisdiction over an indictment for estafa versus a director of the National Parks Development
Committee, a government instrumentality. The Court held then:

The National Parks Development Committee was created originally as an


Executive Committee on January 14, 1963, for the development of the Quezon
Memorial, Luneta and other national parks (Executive Order No. 30). It was later

designated as the National Parks Development Committee (NPDC) on February 7,


1974 (E.O. No. 69). On January 9, 1966, Mrs. Imelda R. Marcos and Teodoro F.
Valencia were designated Chairman and Vice-Chairman respectively (E.O. No. 3).
Despite an attempt to transfer it to the Bureau of Forest Development, Department
of Natural Resources, on December 1, 1975 (Letter of Implementation No. 39,
issued pursuant to PD No. 830, dated November 27, 1975), the NPDC has remained
under the Office of the President (E.O. No. 709, dated July 27, 1981).
Since 1977 to 1981, the annual appropriations decrees listed NPDC as a
regular government agency under the Office of the President and allotments for its
maintenance and operating expenses were issued direct to NPDC (Exh. 10-A,
Perlas, Item Nos. 2, 3).
The Sandiganbayans jurisdiction over estafa was reiterated with greater firmness
in Bondoc v. Sandiganbayan.[38] Pertinent parts of the Courts ruling in Bondoc read:
Furthermore, it is not legally possible to transfer Bondocs cases to the
Regional Trial Court, for the simple reason that the latter would not have
jurisdiction over the offenses. As already above intimated, the inability of the
Sandiganbayan to hold a joint trial of Bondocs cases and those of the government
employees separately charged for the same crimes, has not altered the nature of the
offenses charged, as estafa thru falsification punishable by penalties higher
than prision correccional or imprisonment of six years, or a fine of P6,000.00,
committed by government employees in conspiracy with private persons, including
Bondoc. These crimes are within the exclusive, original jurisdiction of the
Sandiganbayan. They simply cannot be taken cognizance of by the regular courts,
apart from the fact that even if the cases could be so transferred, a joint trial would
nonetheless not be possible.
Petitioner UP student regent
is a public officer.
Petitioner also contends that she is not a public officer. She does not receive any salary or
remuneration as a UP student regent. This is not the first or likely the last time that We will be
called upon to define a public officer. In Khan, Jr. v. Office of the Ombudsman, We ruled that it is
difficult to pin down the definition of a public officer.[39] The 1987 Constitution does not define
who are public officers. Rather, the varied definitions and concepts are found in different
statutes and jurisprudence.
In Aparri v. Court of Appeals,[40] the Court held that:

A public office is the right, authority, and duty created and conferred by
law, by which for a given period, either fixed by law or enduring at the pleasure of
the creating power, an individual is invested with some portion of the sovereign
functions of the government, to be exercise by him for the benefit of the public
([Mechem Public Offices and Officers,] Sec. 1). The right to hold a public office
under our political system is therefore not a natural right. It exists, when it exists at
all only because and by virtue of some law expressly or impliedly creating and
conferring it (Mechem Ibid., Sec. 64). There is no such thing as a vested interest or
an estate in an office, or even an absolute right to hold office. Excepting
constitutional offices which provide for special immunity as regards salary and
tenure, no one can be said to have any vested right in an office or its salary (42 Am.
Jur. 881).
In Laurel v. Desierto,[41] the Court adopted the definition of Mechem of a public office:
A public office is the right, authority and duty, created and conferred by
law, by which, for a given period, either fixed by law or enduring at the pleasure of
the creating power, an individual is invested with some portion of the sovereign
functions of the government, to be exercised by him for the benefit of the
public. The individual so invested is a public officer.[42]
Petitioner claims that she is not a public officer with Salary Grade 27; she is, in fact, a
regular tuition fee-paying student. This is likewise bereft of merit. It is not only the salary grade
that determines the jurisdiction of the Sandiganbayan. The Sandiganbayan also has jurisdiction
over other officers enumerated in P.D. No. 1606. In Geduspan v. People,[43] We held that while
the
first part
of Section
4(A)
covers
only
officials
with
Salary Grade 27 and higher, its second part specifically includes other
executive officials whose positions may not be of Salary Grade 27 and higher but who are by
express provision of law placed under the jurisdiction of the said court. Petitioner falls under the
jurisdiction of the Sandiganbayan as she is placed there by express provision of law.[44]
Section 4(A)(1)(g) of P.D. No. 1606 explictly vested the Sandiganbayan with jurisdiction
over Presidents, directors or trustees, or managers of government-owned or controlled
corporations, state universities or educational institutions or foundations. Petitioner falls under this
category. As the Sandiganbayan pointed out, the BOR performs functions similar to those of a
board of trustees of a non-stock corporation.[45] By express mandate of law, petitioner is, indeed,
a public officer as contemplated by P.D. No. 1606.

Moreover, it is well established that compensation is not an essential element of public


office.[46] At most, it is merely incidental to the public office.[47]
Delegation of sovereign functions is essential in the public office. An investment in an
individual of some portion of the sovereign functions of the government, to be exercised by him
for the benefit of the public makes one a public officer.[48]
The administration of the UP is a sovereign function in line with Article XIV of the
Constitution. UP performs a legitimate governmental function by providing advanced instruction
in literature, philosophy, the sciences, and arts, and giving professional and technical
training.[49] Moreover, UP is maintained by the Government and it declares no dividends and is
not a corporation created for profit.[50]
The offense charged was committed
in relation to public office, according
to the Information.
Petitioner likewise argues that even assuming that she is a public officer, the Sandiganbayan would
still not have jurisdiction over the offense because it was not committed in relation to her office.
According to petitioner, she had no power or authority to act without the approval of the
BOR. She adds there was no Board Resolution issued by the BOR authorizing her to contract with
then President Estrada; and that her acts were not ratified by the governing body of the state
university. Resultantly, her act was done in a private capacity and not in relation to public office.
It is axiomatic that jurisdiction is determined by the averments in the information.[51] More than
that, jurisdiction is not affected by the pleas or the theories set up by defendant or respondent in
an answer, a motion to dismiss, or a motion to quash.[52] Otherwise, jurisdiction would become
dependent almost entirely upon the whims of defendant or respondent.[53]
In the case at bench, the information alleged, in no uncertain terms that petitioner, being
then a student regent of U.P., while in the performance of her official functions,committing the
offense in relation to her office and taking advantage of her position, with intent to gain, conspiring
with her brother, JADE IAN D. SERANA, a private individual, did then and there wilfully,
unlawfully and feloniously defraud the government x x x. (Underscoring supplied)

Clearly, there was no grave abuse of discretion on the part of the Sandiganbayan when it did not
quash the information based on this ground.
Source of funds is a defense that should
be raised during trial on the merits.
It is contended anew that the amount came from President Estradas private funds and not from the
government coffers. Petitioner insists the charge has no leg to stand on.
We cannot agree. The information alleges that the funds came from the Office of the President and
not its then occupant, President Joseph Ejercito Estrada. Under the information, it is averred that
petitioner requested the amount of Fifteen Million Pesos (P15,000,000.00), Philippine
Currency, from the Office of the President, and the latter relying and believing on said false
pretenses and misrepresentation gave and delivered to said accused Land Bank Check No. 91353
dated October 24, 2000 in the amount of Fifteen Million Pesos (P15,000,000.00).
Again, the Court sustains the Sandiganbayan observation that the source of the P15,000,000 is a
matter of defense that should be ventilated during the trial on the merits of the instant case.[54]
A lawyer owes candor, fairness
and honesty to the Court.
As a parting note, petitioners counsel, Renato G. dela Cruz, misrepresented his reference
to Section 4 of P.D. No. 1606 as a quotation from Section 4 of R.A. No. 3019. A review of his
motion to quash, the instant petition for certiorari and his memorandum, unveils the
misquotation. We urge petitioners counsel to observe Canon 10 of the Code of Professional
Responsibility, specifically Rule 10.02 of the Rules stating that a lawyer shall not misquote or
misrepresent.
The Court stressed the importance of this rule in Pangan v. Ramos,[55] where Atty Dionisio
D. Ramos used the name Pedro D.D. Ramos in connection with a criminal case. The Court ruled
that Atty. Ramos resorted to deception by using a name different from that with which he was
authorized. We severely reprimanded Atty. Ramos and warned that a repetition may warrant
suspension or disbarment.[56]

We admonish petitioners counsel to be more careful and accurate in his citation. A lawyers
conduct before the court should be characterized by candor and fairness.[57]The administration of
justice would gravely suffer if lawyers do not act with complete candor and honesty before the
courts.[58]
WHEREFORE, the petition is DENIED for lack of merit.
SO ORDERED.

G.R. No. 191894

July 15, 2015

DANILO A. DUNCANO, Petitioner,


vs.
HON. SANDIGANBAYAN (2nd DIVISION), and HON. OFFICE OF THE SPECIAL
PROSECUTOR, Respondents.
DECISION
PERALTA, J.:
This petition for certiorari under Rule 65 of the Rules of Court (Rules) with prayer for
issuance of preliminary injunction and/or temporary restraining order seeks to reverse
and set aside the August 18, 2009 Resolution1 and February 8, 2010 Order2 of
respondent Sandiganbayan Second Division in Criminal Case No. SB-09-CRM-0080,
which denied petitioner's Motion to Dismiss on the ground of la9k of jurisdiction.
The facts are plain and undisputed.
Petitioner Danilo A. Duncano is, at the time material to the case, the Regional Director
of the Bureau of Internal Revenue (BIR) with Salary Grade 26 as classified under
Republic Act (R.A.) No. 6758.3 On March 24, 2009,4 the Office of the Special Prosecutor
(OSP), Office of the Ombudsman, filed a criminal case against him for violation of
Section 8, in relation to Section 11 of R.A. No. 6713,5 allegedly committed as follows:
That on or about April 15, 2003, or sometime prior or subsequent thereto, in Quezon
City, Philippines, and within the jurisdiction of this Honorable Court, accused
DANILODUNCANO y ACIDO, a high ranking public officer, being the Regional Director
of Revenue Region No. 7, of the Bureau of Internal Revenue, Quezon City, and as such
is under an obligation to accomplish and submit declarations under oath of his assets,
liabilities and net worth and financial and business interests, did then and there, wilfully,
unlawfully and criminally fail to disclose in his Sworn Statement of Assets and Liabilities
and Networth (SALN) for the year 2002, his financial and business interests/connection
in Documail Provides Corporation and Don Plus Trading of which he and his family are
the registered owners thereof, and the 1993 Nissan Patrol motor vehicle registered in
the name of his son VINCENT LOUIS P. DUNCANO which are part of his assets, to the
damage and prejudice of public interest.
CONTRARY TO LAW.6
Prior to his arraignment, petitioner filed a Motion to Dismiss With Prayer to Defer the
Issuance of Warrant of Arrest7 before respondent Sandiganbayan Second Division. As
the OSP alleged, he admitted that he is a Regional Director with Salary Grade 26. Citing
Inding v. Sandiganbayan8 and Serana v. Sandiganbayan, et al.,9 he asserted that under
Presidential Decree (P.D.) No. 1606, as amended by Section 4 (A) (1) of R.A No.
8249,10 the Sandiganbayan has no jurisdiction to try and hear the case because he is

an official of the executive branch occupying the position of a Regional Director but with
a compensation that is classified as below Salary Grade 27.
In its Opposition,11 the OSP argued that a reading of Section 4 (A) (1) (a) to (g) of the
subject law would clearly show that the qualification as to Salary Grade 27 and higher
applies only to officials of the executive branch other than the Regional Director and
those specifically enumerated. This is so since the term "Regional Director" and "higher"
are separated by the conjunction "and," which signifies that these two positions are
different, apart and distinct, words but are conjoined together "relating one to the other"
to give effect to the purpose of the law. The fact that the position of Regional Director
was specifically mentioned without indication as to its salary grade signifies the
lawmakers intention that officials occupying such position, regardless of salary grade,
fall within the original and exclusive jurisdiction of the Sandiganbayan. This issue, it is
claimed, was already resolved in Inding. Finally, the OSP contended that the filing of the
motion to dismiss is premature considering that the Sandiganbayan has yet to acquire
jurisdiction over the person of the accused.
Still not to be outdone, petitioner invoked the applicability of Cuyco v.
Sandiganbayan12 and Organo v. Sandiganbayan13 in his rejoinder.
On August 18, 2009, the Sandiganbayan Second Division promulgated its Resolution,
disposing: WHEREFORE, in the light of the foregoing, the Court hereby DENIES the
instant Motion to Dismiss for being devoid of merit. Let a Warrant of Arrest be therefore
issued against the accused.
SO ORDERED.14
The respondent court ruled that the position of Regional Director is one of those
exceptions where the Sandiganbayan has jurisdiction even if such position is not Salary
Grade 27. It was opined that Section 4 (A) (1) of R.A No. 8249 unequivocally provides
that respondent court has jurisdiction over officials of the executive branch of the
government occupying the position of regional director and higher, otherwise classified
as Salary Grade 27 and higher, of R.A. No. 6758, including those officials who are
expressly enumerated in subparagraphs (a) to (g). In support of the ruling, this Courts
pronouncements in Indingand Binay v. Sandiganbayan15 were cited.
Petitioner filed a Motion for Reconsideration, but it was denied;16 Hence, this petition.
Instead of issuing a temporary restraining order or writ of preliminary injunction, the
Court required respondents to file a comment on the petition without necessarily giving
due course thereto.17 Upon compliance of the OSP, a Rejoinder (supposedly a Reply)
was filed by petitioner.
At the heart of the controversy is the determination of whether, according to P.D. No.
1606, as amended by Section 4 (A) (1) of R.A No. 8249, only Regional Directors with
Salary Grade of 27 and higher, as classified under R.A. No. 6758, fall within the

exclusive jurisdiction of the Sandiganbayan. Arguing that he is not included among the
public officials specifically enumerated in Section 4 (A) (1) (a) to (g) of the law and
heavily relying as well on Cuyco, petitioner insists that respondent court lacks
jurisdiction over him, who is merely a Regional Director with Salary Grade 26. On the
contrary, the OSP maintains that a Regional Director, irrespective of salary grade, falls
within the exclusive original jurisdiction of the Sandiganbayan. We find merit in the
petition.
The creation of the Sandiganbayan was mandated by Section 5, Article XIII of the 1973
Constitution.18 By virtue of the powers vested in him by the Constitution and pursuant to
Proclamation No. 1081, dated September 21, 1972, former President Ferdinand E.
Marcos issued P.D. No. 1486.19 The decree was later amended by P.D. No.
1606,20Section 20 of Batas Pambansa Blg. 129,21 P.D. No. 1860,22 and P.D. No.
1861.23
With the advent of the 1987 Constitution, the special court was retained as provided for
in Section 4, Article XI thereof.24 Aside from Executive Order Nos. 1425 and 14-a,26 and
R.A. 7080,27 which expanded the jurisdiction of the Sandiganbayan, P.D. No. 1606 was
further modified by R.A. No. 7975,28 R.A. No. 8249,29 and just this year, R.A. No.
10660.30
For the purpose of this case, the relevant provision is Section 4 of R.A. No. 8249, which
states: SEC. 4. Section 4 of the same decree is hereby further amended to read as
follows:
"SEC. 4. Jurisdiction. The Sandiganbayan shall exercise exclusive original jurisdiction
in all cases involving:
"A. Violations of Republic Act No. 3019, as amended, otherwise known as the Anti-Graft
and Corrupt Practices Act, Republic Act No. 1379, and Chapter II, Section 2, Title VII,
Book II of the Revised Penal Code, where one or more of the accused are officials
occupying the following positions in the government, whether in a permanent, acting or
interim capacity, at the time of the commission of the offense:
"(1) Officials of the executive branch occupying the positions of regional director and
higher, otherwise classified as Grade 27 and higher, of the Compensation and Position
Classification Act of 1989 (Republic Act No. 6758), specifically including:
"(a) Provincial governors, vice-governors, members of the sangguniang
panlalawigan, and provincial treasurers, assessors, engineers, and other
provincial department heads;
"(b) City mayor, vice-mayors, members of the sangguniang panlungsod, city
treasurers, assessors, engineers, and other city department heads;

"(c) Officials of the diplomatic service occupying the position of consul and
higher;
"(d) Philippine army and air force colonels, naval captains, and all officers of
higher rank;
"(e) Officers of the Philippine National Police while occupying the position of
provincial director and those holding the rank of senior superintendent or higher;
"(f) City and provincial prosecutors and their assistants, and officials and
prosecutors in the Office of the Ombudsman and special prosecutor;
"(g) Presidents, directors or trustees, or managers of government-owned or
controlled corporations, state universities or educational institutions or
foundations.
"(2) Members of Congress and officials thereof classified as Grade 27 and up
under the Compensation and Position Classification Act of 1989;
"(3) Members of the judiciary without prejudice to the provisions of the
Constitution;
"(4) Chairmen and members of Constitutional Commission, without prejudice to
the provisions of the Constitution; and
"(5) All other national and local officials classified as Grade 27 and higher under
the Compensation and Position Classification Act of 1989.
"B. Other offenses or felonies whether simple or complexed with other crimes
committed by the public officials and employees mentioned in subsection a of this
section in relation to their office.
"C. Civil and criminal cases filed pursuant to and in connection with Executive Order
Nos. 1, 2, 14 and 14-A, issued in 1986.
x x x"
Based on the afore-quoted, those that fall within the original jurisdiction of the
Sandiganbayan are: (1) officials of the executive branch with Salary Grade 27 or higher,
and (2) officials specifically enumerated in Section 4 (A) (1) (a) to (g), regardless of their
salary grades.31 While the first part of Section 4 (A) covers only officials of the executive
branch with Salary Grade 27 and higher, its second part specifically includes other
executive officials whose positions may not be of Salary Grade 27 and higher but who
are by express provision of law placed under the jurisdiction of the Sandiganbayan. 32

That the phrase "otherwise classified as Grade 27 and higher" qualifies "regional
director and higher" is apparent from the Sponsorship Speech of Senator Raul S. Roco
on Senate Bill Nos. 1353and 844, which eventually became R.A. Nos. 7975 and 8249,
respectively:
As proposed by the Committee, the Sandiganbayan shall exercise original jurisdiction
over the cases assigned to it only in instances where one or more of the principal
accused are officials occupying the positions of regional director and higher or are
otherwise classified as Grade 27 and higher by the Compensation and Position
Classification Act of 1989, whether in a permanent, acting or interim capacity at the time
of the commission of the offense. The jurisdiction, therefore, refers to a certain grade
upwards, which shall remain with the Sandiganbayan.33 (Emphasis supplied)
To speed up trial in the Sandiganbayan, Republic Act No. 7975 was enacted for that
Court to concentrate on the "larger fish" and leave the "small fry" to the lower courts.
This law became effective on May 6, 1995 and it provided a two-pronged solution to the
clogging of the dockets of that court, to wit:
It divested the Sandiganbayan of jurisdiction over public officials whose salary grades
were at Grade "26" or lower, devolving thereby these cases to the lower courts, and
retaining the jurisdiction of the Sandiganbayan only over public officials whose salary
grades were at Grade "27" or higher and over other specific public officials holding
important positions in government regardless of salary grade; x x x34 (Emphasis
supplied)
The legislative intent is to allow the Sandiganbayan to devote its time and expertise to
big-time cases involving the so-called "big fishes" in the government rather than those
accused who are of limited means who stand trial for "petty crimes," the so-called "small
fry," which, in turn, helps the court decongest its dockets.35
Yet, those that are classified as Salary Grade 26 and below may still fall within the
jurisdiction of the Sandiganbayan, provided that they hold the positions enumerated by
the law.36 In this category, it is the position held, not the salary grade, which determines
the jurisdiction of the Sandiganbayan.37 The specific inclusion constitutes an exception
to the general qualification relating to "officials of the executive branch occupying the
positions of regional director and higher, otherwise classified as Grade 27 and higher,
of the Compensation and Position Classification Act of 1989."38 As ruled in Inding:
Following this disquisition, the paragraph of Section 4 which provides that if the accused
is occupying a position lower than SG 27, the proper trial court has jurisdiction, can only
be properly interpreted as applying to those cases where the principal accused is
occupying a position lower than SG 27 and not among those specifically included in the
enumeration in Section 4 a. (1) (a) to (g). Stated otherwise, except for those officials
specifically included in Section 4 a. (1) (a) to (g), regardless of their salary grades, over
whom the Sandiganbayan has jurisdiction, all other public officials below SG 27 shall be
under the jurisdiction of the proper trial courts "where none of the principal accused are

occupying positions corresponding to SG 27 or higher." By this construction, the entire


Section 4 is given effect. The cardinal rule, after all, in statutory construction is that the
particular words, clauses and phrases should not be studied as detached and isolated
expressions, but the whole and every part of the statute must be considered in fixing the
meaning of any of its parts and in order to produce a harmonious whole. And courts
should adopt a construction that will give effect to every part of a statute, if at all
possible. Ut magis valeat quam pereat or that construction is to be sought which gives
effect to the whole of the statute its every word.39
Thus, to cite a few, We have held that a member of the Sangguniang Panlungsod,40 a
department manager of the Philippine Health Insurance Corporation (Philhealth), 41 a
student regent of the University of the Philippines,42 and a Head of the Legal
Department and Chief of the Documentation with corresponding ranks of VicePresidents and Assistant Vice-President of the Armed Forces of the Philippines
Retirement and Separation Benefits System (AFP-RSBS)43 fall within the jurisdiction of
the Sandiganbayan.
Petitioner is not an executive official with Salary Grade 27 or higher. Neither does he
hold any position particularly enumerated in Section 4 (A) (1) (a) to (g). As he correctly
argues, his case is, in fact, on all fours with Cuyco.1avvphi1Therein, the accused was
the Regional Director of the Land Transportation Office, Region IX, Zamboanga City,
but at the time of the commission of the crime in 1992, his position was classified as
Director II with Salary Grade 26.44 It was opined: Petitioner contends that at the time of
the commission of the offense in 1992, he was occupying the position of Director II,
Salary Grade 26, hence, jurisdiction over the cases falls with the Regional Trial Court.
We sustain petitioner's contention.
The Sandiganbayan has no jurisdiction over violations of Section 3(a) and (e), Republic
Act No. 3019, as amended, unless committed by public officials and employees
occupying positions of regional director and higher with Salary Grade "27" or higher,
under the Compensation and Position Classification Act of 1989 (Republic Act No.
6758) in relation to their office.
In ruling in favor of its jurisdiction, even though petitioner admittedly occupied the
position of Director II with Salary Grade "26" under the Compensation and Position
Classification Act of 1989 (Republic Act No. 6758), the Sandiganbayan incurred in
serious error of jurisdiction, and acted with grave abuse of discretion amounting to lack
of jurisdiction in suspending petitioner from office, entitling petitioner to the reliefs
prayed for.45
In the same way, a certification issued by the OIC Assistant Chief, Personnel Division
of the BIR shows that, although petitioner is a Regional Director of the BIR, his position
is classified as Director II with Salary Grade 26.46

There is no merit in the OSPs allegation that the petition was prematurely filed on the
ground that respondent court has not yet acquired jurisdiction over the person of
petitioner. Records disclose that when a warrant of arrest was issued by respondent
court, petitioner voluntarily surrendered and posted a cash bond on September 17,
2009.Also, he was arraigned on April 14, 2010,prior to the filing of the petition on April
30, 2010.
WHEREFORE, the foregoing considered, the instant petition for certiorari is GRANTED.
The August 18, 2009 Resolution and February 8, 2010 Order of the Sandiganbayan
Second Division, which denied petitioner's Motion to Dismiss on the ground of lack of
jurisdiction, are REVERSED AND SET ASIDE.
SO ORDERED.

G.R. No. 159747

April 13, 2004

GREGORIO B. HONASAN II, petitioner,


vs.
THE PANEL OF INVESTIGATING PROSECUTORS OF THE DEPARTMENT OF
JUSTICE (LEO DACERA, SUSAN F. DACANAY, EDNA A. VALENZUELA AND
SEBASTIAN F. CAPONONG, JR.), CIDG-PNP- P/DIRECTOR EDUARDO
MATILLANO, and HON. OMBUDSMAN SIMEON V. MARCELO, respondents.

DECISION

AUSTRIA-MARTINEZ, J.:
On August 4, 2003, an affidavit-complaint was filed with the Department of Justice
(DOJ) by respondent CIDG-PNP/P Director Eduardo Matillano. It reads in part:

2. After a thorough investigation, I found that a crime of coup d'etat was indeed
committed by military personnel who occupied Oakwood on the 27 th day of July
2003 and Senator Gregorio "Gringo"Honasan, II
3.
4. The said crime was committed as follows:
4.1 On June 4, 2003, at on or about 11 p.m., in a house located in San Juan,
Metro Manila, a meeting was held and presided by Senator Honasan. Attached
as Annex "B" is the affidavit of Perfecto Ragil and made an integral part of
this complaint.

4.8 In the early morning of July 27, 2003, Capt. Gerardo Gambala, for and
in behalf of the military rebels occupying Oakwood, made a public
statement aired on nation television, stating their withdrawal of support to
the chain of command of the AFP and the Government of President Gloria
Macapagal Arroyo and they are willing to risk their lives in order to achieve
the National Recovery Agenda of Sen. Honasan, which they believe is the
only program that would solve the ills of society. . . . (Emphasis supplied).

The Sworn Statement of AFP Major Perfecto Ragil referred to by PNP/P Director
Matillano is quoted verbatim, to wit:
1. That I am a member of the Communication Electronics and Information
Systems Services, Armed Forces of the Philippines with the rank of Major;
2. That I met a certain Captain Gary Alejano of the Presidential Security Guard
(PSG) during our Very Important Person (VIP) Protection Course sometime in
last week of March 2003;
3. That sometime in May 2003, Captain Alejano gave me a copy of the pamphlet
of the National Recovery Program (NRP) and told me that: "Kailangan ng Bansa
ng taong kagaya mo na walang bahid ng corruption kaya basahin mo ito
(referring to NRP) pamphlet. I took the pamphlet but never had the time to read
it;
4. That sometime in the afternoon of June 4, 2003, Captain Alejano invited me to
join him in a meeting where the NRP would be discussed and that there would be
a special guest;
5. That Capt. Alejano and I arrived at the meeting at past 9 o'clock in the evening
of June 4, 2003 in a house located somewhere in San Juan, Metro Manila;
6. That upon arrival we were given a document consisting of about 3-4 pages
containing discussion of issues and concerns within the framework of NRP and
we were likewise served with dinner;
7. That while we were still having dinner at about past 11 o'clock in the evening,
Sen. Gregorio "Gringo" Honasan arrived together with another fellow who was
later introduced as Capt. Turingan;
8. That after Sen. Honasan had taken his dinner, the meeting proper started
presided by Sen. Honasan;
9. That Sen. Honasan discussed the NRP, the graft and corruption in the
government including the military institution, the judiciary, the executive branch
and the like;
10. That the discussion concluded that we must use force, violence and armed
struggle to achieve the vision of NRP. At this point, I raised the argument that it is
my belief that reforms will be achieved through the democratic processes and not
thru force and violence and/or armed struggle. Sen. Honasan countered that "we
will never achieve reforms through the democratic processes because the people
who are in power will not give up their positions as they have their vested
interests to protect." After a few more exchanges of views, Sen. Honasan
appeared irritated and asked me directly three (3) times: "In ka ba o out?" I then

asked whether all those present numbering 30 people, more or less, are really
committed, Sen. Honasan replied: "Kung kaya nating pumatay sa ating mga
kalaban, kaya din nating pumatay sa mga kasamahang magtataksil." I decided
not to pursue further questions;
11. That in the course of the meeting, he presented the plan of action to achieve
the goals of NRP, i.e., overthrow of the government under the present leadership
thru armed revolution and after which, a junta will be constituted and that junta
will run the new government. He further said that some of us will resign from the
military service and occupy civilian positions in the new government. He also said
that there is urgency that we implement this plan and that we would be notified of
the next activities.
12. That after the discussion and his presentation, he explained the rites that we
were to undergo-some sort of "blood compact". He read a prayer that sounded
more like a pledge and we all recited it with raised arms and clenched fists. He
then took a knife and demonstrated how to make a cut on the left upper inner
arm until it bleeds. The cut was in form of the letter "I" in the old alphabet but was
done in a way that it actually looked like letter "H". Then, he pressed his right
thumb against the blood and pressed the thumb on the lower middle portion of
the copy of the Prayer. He then covered his thumb mark in blood with tape. He
then pressed the cut on his left arm against the NRP flag and left mark of letter
"I" on it. Everybody else followed;
13. That when my turn came, I slightly made a cut on my upper inner arm and
pricked a portion of it to let it bleed and I followed what Senator HONASAN did;
14. That I did not like to participate in the rites but I had the fear for my life with
what Senator HONASAN said that "kaya nating pumatay ng kasamahan";
15. That after the rites, the meeting was adjourned and we left the place;
16. That I avoided Captain Alejano after that meeting but I was extra cautious
that he would not notice it for fear of my life due to the threat made by Senator
HONASAN during the meeting on June 4, 2003 and the information relayed to
me by Captain Alejano that their group had already deeply established their
network inside the intelligence community;
17. That sometime in the first week of July 2003, Captain Alejano came to see
me to return the rifle that he borrowed and told me that when the group arrives at
the Malacaang Compound for "D-DAY", my task is to switch off the telephone
PABX that serves the Malacaang complex. I told him that I could not do it. No
further conversation ensued and he left;
18. That on Sunday, July 27, 2003, while watching the television, I saw flashed
on the screen Lieutenant Antonio Trillanes, Captain Gerardo Gambala, Captain

Alejano and some others who were present during the June 4 th meeting that I
attended, having a press conference about their occupation of the Oakwood
Hotel. I also saw that the letter "I" on the arm bands and the banner is the same
letter "I" in the banner which was displayed and on which we pressed our wound
to leave the imprint of the letter "I";
19. That this Affidavit is being executed in order to attest the veracity of the
foregoing and in order to charge SENATOR GREGORIO "GRINGO" HONASAN,
Capt. FELIX TURINGAN, Capt. GARY ALEJANO, Lt. ANTONIO TRILLANES,
Capt. GERARDO GAMBALA and others for violation of Article 134-A of the
Revised Penal Code for the offense of "coup d'etat". (Emphasis supplied)
The affidavit-complaint is docketed as I.S. No. 2003-1120 and the Panel of Investigating
Prosecutors of the Department of Justice (DOJ Panel for brevity) sent a subpoena to
petitioner for preliminary investigation.
On August 27, 2003, petitioner, together with his counsel, appeared at the DOJ. He filed
a Motion for Clarification questioning DOJ's jurisdiction over the case, asserting that
since the imputed acts were committed in relation to his public office, it is the Office of
the Ombudsman, not the DOJ, that has the jurisdiction to conduct the corresponding
preliminary investigation; that should the charge be filed in court, it is the
Sandiganbayan, not the regular courts, that can legally take cognizance of the case
considering that he belongs to the group of public officials with Salary Grade 31; and
praying that the proceedings be suspended until final resolution of his motion.
Respondent Matillano submitted his comment/opposition thereto and petitioner filed a
reply.
On September 10, 2003, the DOJ Panel issued an Order, to wit:
On August 27, 2003, Senator Gregorio B. Honasan II filed through counsel a
"Motion to Clarify Jurisdiction". On September 1, 2003, complainant filed a
Comment/Opposition to the said motion.
The motion and comment/opposition are hereby duly noted and shall be passed
upon in the resolution of this case.
In the meantime, in view of the submission by complainant of additional
affidavits/evidence and to afford respondents ample opportunity to controvert the
same, respondents, thru counsel are hereby directed to file their respective
counter-affidavits and controverting evidence on or before September 23, 2003. 1
Hence, Senator Gregorio B. Honasan II filed the herein petition for certiorari under Rule
65 of the Rules of Court against the DOJ Panel and its members, CIDG-PNP-P/Director
Eduardo Matillano and Ombudsman Simeon V. Marcelo, attributing grave abuse of
discretion on the part of the DOJ Panel in issuing the aforequoted Order of September

10, 2003 on the ground that the DOJ has no jurisdiction to conduct the preliminary
investigation.
Respondent Ombudsman, the Office of Solicitor General in representation of
respondents DOJ Panel, and Director Matillano submitted their respective comments.
The Court heard the parties in oral arguments on the following issues:
1) Whether respondent Department of Justice Panel of Investigators has
jurisdiction to conduct preliminary investigation over the charge of coup
d'etat against petitioner;
2) Whether Ombudsman-DOJ Circular No. 95-001 violates the Constitution and
Republic Act No. 6770 or Ombudsman Act of 1989; and
3) Whether respondent DOJ Panel of Investigators committed grave abuse of
discretion in deferring the resolution of the petitioner's motion to clarify jurisdiction
considering the claim of the petitioner that the DOJ Panel has no jurisdiction to
conduct preliminary investigation.
After the oral arguments, the parties submitted their respective memoranda. The
arguments of petitioner are:
1. The Office of the Ombudsman has jurisdiction to conduct the preliminary
investigation over all public officials, including petitioner.
2. Respondent DOJ Panel is neither authorized nor deputized under OMB-DOJ
Joint Circular No. 95-001 to conduct the preliminary investigation involving
Honasan.
3. Even if deputized, the respondent DOJ Panel is still without authority since
OMB-DOJ Joint Circular No. 95-001 is ultra vires for being violative of the
Constitution, beyond the powers granted to the Ombudsman by R.A. 6770 and
inoperative due to lack of publication, hence null and void.
4. Since petitioner is charged with coup de 'etat in relation to his office, it is the
Office of the Ombudsman which has the jurisdiction to conduct the preliminary
investigation.
5. The respondent DOJ Panel gravely erred in deferring the resolution of
petitioner's Motion to Clarify Jurisdiction since the issue involved therein is
determinative of the validity of the preliminary investigation.
6. Respondent DOJ Panel gravely erred when it resolved petitioner's Motion in
the guise of directing him to submit Counter-Affidavit and yet refused and/or

failed to perform its duties to resolve petitioner's Motion stating its legal and
factual bases.
The arguments of respondent DOJ Panel are:
1. The DOJ has jurisdiction to conduct the preliminary investigation on petitioner
pursuant to Section 3, Chapter I, Title III, Book IV of the Revised Administrative
Code of 1987 in relation to P.D. No. 1275, as amended by P.D. No. 1513.
2. Petitioner is charged with a crime that is not directly nor intimately related to
his public office as a Senator. The factual allegations in the complaint and the
supporting affidavits are bereft of the requisite nexus between petitioner's office
and the acts complained of.
3. The challenge against the constitutionality of the OMB-DOJ Joint Circular, as a
ground to question the jurisdiction of the DOJ over the complaint below, is
misplaced. The jurisdiction of the DOJ is a statutory grant under the Revised
Administrative Code. It is not derived from any provision of the joint circular which
embodies the guidelines governing the authority of both the DOJ and the Office
of the Ombudsman to conduct preliminary investigation on offenses charged in
relation to public office.
4. Instead of filing his counter-affidavit, petitioner opted to file a motion to clarify
jurisdiction which, for all intents and purposes, is actually a motion to dismiss that
is a prohibited pleading under Section 3, Rule 112 of the Revised Rules of
Criminal Procedure. The DOJ Panel is not required to act or even recognize it
since a preliminary investigation is required solely for the purpose of determining
whether there is a sufficient ground to engender a well founded belief that a
crime has been committed and the respondent is probably guilty thereof and
should be held for trial. The DOJ panel did not outrightly reject the motion of
petitioner but ruled to pass upon the same in the determination of the probable
cause; thus, it has not violated any law or rule or any norm of discretion.
The arguments of respondent Ombudsman are:
1. The DOJ Panel has full authority and jurisdiction to conduct preliminary
investigation over the petitioner for the reason that the crime of coup d'etat under
Article No. 134-A of the Revised Penal Code (RPC) may fall under the
jurisdiction of the Sandiganbayan only if the same is committed "in relation to
office" of petitioner, pursuant to Section 4, P.D. No. 1606, as amended by R.A.
No. 7975 and R.A. No. 8249.
2. Petitioner's premise that the DOJ Panel derives its authority to conduct
preliminary investigation over cases involving public officers solely from the
OMB-DOJ Joint Circular No. 95-001 is misplaced because the DOJ's concurrent
authority with the OMB to conduct preliminary investigation of cases involving

public officials has been recognized in Sanchez vs. Demetriou (227 SCRA 627
[1993]) and incorporated in Section 4, Rule 112 of the Revised Rules of Criminal
Procedure.
3. Petitioner's assertion that the Joint Circular is ultra vires and the DOJ cannot
be deputized by the Ombudsman en masse but must be given in reference to
specific cases has no factual or legal basis. There is no rule or law which
requires the Ombudsman to write out individualized authorities to deputize
prosecutors on a per case basis. The power of the Ombudsman to deputize DOJ
prosecutors proceeds from the Constitutional grant of power to request
assistance from any government agency necessary to discharge its functions, as
well as from the statutory authority to so deputize said DOJ prosecutors under
Sec. 31 of RA 6770.
4. The Joint Circular which is an internal arrangement between the DOJ and the
Office of the Ombudsman need not be published since it neither contains a penal
provision nor does it prescribe a mandatory act or prohibit any under pain or
penalty. It does not regulate the conduct of persons or the public, in general.
The Court finds the petition without merit.
The authority of respondent DOJ Panel is based not on the assailed OMB-DOJ Circular
No. 95-001 but on the provisions of the 1987 Administrative Code under Chapter I, Title
III, Book IV, governing the DOJ, which provides:
Sec. 1. Declaration of policy - It is the declared policy of the State to provide the
government with a principal law agency which shall be both its legal counsel
and prosecution arm; administer the criminal justice system in accordance with
the accepted processes thereof consisting in the investigation of the crimes,
prosecution of offenders and administration of the correctional system;
Sec. 3. Powers and Functions - To accomplish its mandate, the Department shall
have the following powers and functions:

(2) Investigate the commission of crimes, prosecute offenders and


administer the probation and correction system; (Emphasis supplied)
and Section 1 of P.D. 1275, effective April 11, 1978, to wit:
SECTION 1. Creation of the National Prosecution Service; Supervision and
Control of the Secretary of Justice. There is hereby created and established a
National Prosecution Service under the supervision and control of the Secretary
of Justice, to be composed of the Prosecution Staff in the Office of the Secretary
of Justice and such number of Regional State Prosecution Offices, and Provincial

and City Fiscal's Offices as are hereinafter provided, which shall be primarily
responsible for the investigation and prosecution of all cases involving
violations of penal laws. (Emphasis supplied)
Petitioner claims that it is the Ombudsman, not the DOJ, that has the jurisdiction to
conduct the preliminary investigation under paragraph (1), Section 13, Article XI of the
1987 Constitution, which confers upon the Office of the Ombudsman the power
to investigate on its own, or on complaint by any person, any act or omission of any
public official, employee, office or agency, when such act or omission appears to be
illegal, unjust, improper, or inefficient. Petitioner rationalizes that the 1987
Administrative Code and the Ombudsman Act of 1989 cannot prevail over the
Constitution, pursuant to Article 7 of the Civil Code, which provides:
Article 7. Laws are repealed only by subsequent ones, and their violation or nonobservance shall not be excused by disuse, or custom or practice to the contrary.
When the courts declare a law to be inconsistent with the Constitution, the former
shall be void and the latter shall govern.
Administrative or executive acts, orders and regulations shall be valid only when they
are not contrary to the laws or the Constitution.
and Mabanag vs. Lopez Vito.2
The Court is not convinced. Paragraph (1) of Section 13, Article XI of the
Constitution, viz:
SEC. 13. The Office of the Ombudsman shall have the following powers,
functions, and duties:
1. Investigate on its own, or on complaint by any person, any act or omission of
any public official, employee, office or agency, when such act or omission
appears to be illegal, unjust, improper, or inefficient.
does not exclude other government agencies tasked by law to investigate and
prosecute cases involving public officials. If it were the intention of the framers of the
1987 Constitution, they would have expressly declared the exclusive conferment of the
power to the Ombudsman. Instead, paragraph (8) of the same Section 13 of the
Constitution provides:
(8) Promulgate its rules of procedure and exercise such other powers or perform
such functions or duties as may be provided by law.
Accordingly, Congress enacted R.A. 6770, otherwise known as "The Ombudsman Act
of 1989." Section 15 thereof provides:

Sec. 15. Powers, Functions and Duties. - The Office of the Ombudsman shall
have the following powers, functions and duties:
(1) Investigate and prosecute on its own or on complaint by any person, any act
or omission of any public officer or employee, office or agency, when such act or
omission appears to be illegal, unjust, improper or inefficient. It has primary
jurisdiction over cases cognizable by the Sandiganbayan and, in the
exercise of this primary jurisdiction, it may take over, at any stage, from
any investigatory agency of the government, the investigation of such
cases.
. (Emphasis supplied)
Pursuant to the authority given to the Ombudsman by the Constitution and the
Ombudsman Act of 1989 to lay down its own rules and procedure, the Office of the
Ombudsman promulgated Administrative Order No. 8, dated November 8, 1990,
entitled, Clarifying and Modifying Certain Rules of Procedure of the Ombudsman, to wit:
A complaint filed in or taken cognizance of by the Office of the Ombudsman
charging any public officer or employee including those in government-owned or
controlled corporations, with an act or omission alleged to be illegal, unjust,
improper or inefficient is an Ombudsman case. Such a complaint may be the
subject of criminal or administrative proceedings, or both.
For purposes of investigation and prosecution, Ombudsman cases
involving criminal offenses may be subdivided into two classes, to wit: (1)
those cognizable by the Sandiganbayan, and (2) those falling under the
jurisdiction of the regular courts. The difference between the two, aside
from the category of the courts wherein they are filed, is on the authority to
investigate as distinguished from the authority to prosecute, such cases.
The power to investigate or conduct a preliminary investigation on any
Ombudsman case may be exercised by an investigator or prosecutor of the
Office of the Ombudsman, or by any Provincial or City Prosecutor or their
assistance, either in their regular capacities or as deputized Ombudsman
prosecutors.
The prosecution of cases cognizable by the Sandiganbayan shall be under
the direct exclusive control and supervision of the Office of the
Ombudsman. In cases cognizable by the regular Courts, the control and
supervision by the Office of the Ombudsman is only in Ombudsman cases
in the sense defined above. The law recognizes a concurrence of
jurisdiction between the Office of the Ombudsman and other investigative
agencies of the government in the prosecution of cases cognizable by
regular courts. (Emphasis supplied)

It is noteworthy that as early as 1990, the Ombudsman had properly differentiated the
authority to investigate cases from the authority to prosecute cases. It is on this note
that the Court will first dwell on the nature or extent of the authority of the Ombudsman
to investigate cases. Whence, focus is directed to the second sentence of paragraph
(1), Section 15 of the Ombudsman Act which specifically provides that the Ombudsman
has primary jurisdiction over cases cognizable by the Sandiganbayan, and, in the
exercise of this primary jurisdiction, it may take over, at any stage, from any
investigating agency of the government, the investigation of such cases.
That the power of the Ombudsman to investigate offenses involving public officers or
employees is not exclusive but is concurrent with other similarly authorized agencies of
the government such as the provincial, city and state prosecutors has long been settled
in several decisions of the Court.
In Cojuangco, Jr. vs. Presidential Commission on Good Government, decided in 1990,
the Court expressly declared:
A reading of the foregoing provision of the Constitution does not show that the
power of investigation including preliminary investigation vested on the
Ombudsman is exclusive.3
Interpreting the primary jurisdiction of the Ombudsman under Section 15 (1) of the
Ombudsman Act, the Court held in said case:
Under Section 15 (1) of Republic Act No. 6770 aforecited, the Ombudsman has
primary jurisdiction over cases cognizable by the Sandiganbayan so that it may
take over at any stage from any investigatory agency of the government, the
investigation of such cases. The authority of the Ombudsman to investigate
offenses involving public officers or employees is not exclusive but is
concurrent with other similarly authorized agencies of the
government. Such investigatory agencies referred to include the PCGG and
the provincial and city prosecutors and their assistants, the state
prosecutors and the judges of the municipal trial courts and municipal
circuit trial court.
In other words the provision of the law has opened up the authority to
conduct preliminary investigation of offenses cognizable by the
Sandiganbayan to all investigatory agencies of the government duly
authorized to conduct a preliminary investigation under Section 2, Rule 112
of the 1985 Rules of Criminal Procedure with the only qualification that the
Ombudsman may take over at any stage of such investigation in the
exercise of his primary jurisdiction.4 (Emphasis supplied)
A little over a month later, the Court, in Deloso vs. Domingo,5 pronounced that the
Ombudsman, under the authority of Section 13 (1) of the 1987 Constitution, has
jurisdiction to investigate any crime committed by a public official, elucidating thus:

As protector of the people, the office of the Ombudsman has the power, function
and duty to "act promptly on complaints filed in any form or manner against
public officials" (Sec. 12) and to "investigate x x x any act or omission of any
public official x x x when such act or omission appears to be illegal, unjust,
improper or inefficient." (Sec. 13[1].) The Ombudsman is also empowered to
"direct the officer concerned," in this case the Special Prosecutor, "to take
appropriate action against a public official x x x and to recommend his
prosecution" (Sec. 13[3]).
The clause "any [illegal] act or omission of any public official" is broad enough to
embrace any crime committed by a public official. The law does not qualify the
nature of the illegal act or omission of the public official or employee that the
Ombudsman may investigate. It does not require that the act or omission be
related to or be connected with or arise from, the performance of official duty.
Since the law does not distinguish, neither should we.
The reason for the creation of the Ombudsman in the 1987 Constitution and for
the grant to it of broad investigative authority, is to insulate said office from the
long tentacles of officialdom that are able to penetrate judges' and fiscals' offices,
and others involved in the prosecution of erring public officials, and through the
exertion of official pressure and influence, quash, delay, or dismiss investigations
into malfeasances and misfeasances committed by public officers. It was
deemed necessary, therefore, to create a special office to investigate all criminal
complaints against public officers regardless of whether or not the acts or
omissions complained of are related to or arise from the performance of the
duties of their office. The Ombudsman Act makes perfectly clear that the
jurisdiction of the Ombudsman encompasses "all kinds of malfeasance,
misfeasance, and non-feasance that have been committed by any officer or
employee as mentioned in Section 13 hereof, during his tenure of office" (Sec.
16, R.A. 6770).
.........
Indeed, the labors of the constitutional commission that created the Ombudsman
as a special body to investigate erring public officials would be wasted if its
jurisdiction were confined to the investigation of minor and less grave offenses
arising from, or related to, the duties of public office, but would exclude those
grave and terrible crimes that spring from abuses of official powers and
prerogatives, for it is the investigation of the latter where the need for an
independent, fearless, and honest investigative body, like the Ombudsman, is
greatest.6
At first blush, there appears to be conflicting views in the rulings of the Court in
the Cojuangco, Jr. case and theDeloso case. However, the contrariety is more apparent
than real. In subsequent cases, the Court elucidated on the nature of the powers of the
Ombudsman to investigate.

In 1993, the Court held in Sanchez vs. Demetriou,7 that while it may be true that the
Ombudsman has jurisdiction to investigate and prosecute any illegal act or omission of
any public official, the authority of the Ombudsman to investigate is merely a primary
and not an exclusive authority, thus:
The Ombudsman is indeed empowered under Section 15, paragraph (1) of RA
6770 to investigate and prosecute any illegal act or omission of any public
official. However as we held only two years ago in the case of Aguinaldo vs.
Domagas,8 this authority "is not an exclusive authority but rather a shared or
concurrent authority in respect of the offense charged."
Petitioners finally assert that the information and amended information filed in
this case needed the approval of the Ombudsman. It is not disputed that the
information and amended information here did not have the approval of the
Ombudsman. However, we do not believe that such approval was necessary at
all. In Deloso v. Domingo, 191 SCRA 545 (1990), the Court held that the
Ombudsman has authority to investigate charges of illegal acts or omissions on
the part of any public official, i.e., any crime imputed to a public official. It must,
however, be pointed out that the authority of the Ombudsman to
investigate "any [illegal] act or omission of any public official" (191 SCRA
550) is not an exclusive authority but rather a shared or concurrent
authority in respect of the offense charged, i.e., the crime of sedition. Thus,
the non-involvement of the office of the Ombudsman in the present case does
not have any adverse legal consequence upon the authority of the panel of
prosecutors to file and prosecute the information or amended information.
In fact, other investigatory agencies of the government such as the
Department of Justice in connection with the charge of sedition, and the
Presidential Commission on Good Government, in ill gotten wealth cases,
may conduct the investigation.9 (Emphasis supplied)
In Natividad vs. Felix,10 a 1994 case, where the petitioner municipal mayor contended
that it is the Ombudsman and not the provincial fiscal who has the authority to conduct a
preliminary investigation over his case for alleged Murder, the Court held:
The Deloso case has already been re-examined in two cases, namely Aguinaldo
vs. Domagas andSanchez vs. Demetriou. However, by way of amplification, we
feel the need for tracing the history of the legislation relative to the jurisdiction of
Sandiganbayan since the Ombudsman's primary jurisdiction is dependent on the
cases cognizable by the former.
In the process, we shall observe how the policy of the law, with reference to the
subject matter, has been in a state of flux.
These laws, in chronological order, are the following: (a) Pres. Decree No. 1486,
-- the first law on the Sandiganbayan; (b) Pres. Decree No. 1606 which expressly

repealed Pres. Decree No. 1486; (c) Section 20 of Batas Pambansa Blg. 129; (d)
Pres. Decree No. 1860; and (e) Pres. Decree No. 1861.
The latest law on the Sandiganbayan, Sec. 1 of Pres. Decree No. 1861 reads as
follows:
"SECTION 1. Section 4 of Presidential Decree No. 1606 is hereby
amended to read as follows:
'SEC. 4. Jurisdiction. The Sandiganbayan shall exercise:
'(a) Exclusive original jurisdiction in all cases involving:
...
(2) Other offenses or felonies committed by public officers and
employees in relation to their office, including those employed in
government-owned or controlled corporation, whether simple or
complexed with other crimes, where the penalty prescribed by law
is higher thatprision correccional or imprisonment for six (6) years,
or a fine of P6,000: PROVIDED, HOWEVER, that offenses or
felonies mentioned in this paragraph where the penalty prescribed
by law does not exceed prision correccional or imprisonment for six
(6) years or a fine of P6,000 shall be tried by the proper Regional
Trial Court, Metropolitan Trial Court, Municipal Trial Court and
Municipal Circuit Trial Court."
A perusal of the aforecited law shows that two requirements must concur under
Sec. 4 (a) (2) for an offense to fall under the Sandiganbayan's jurisdiction,
namely: the offense committed by the public officer must be in relation to his
office and the penalty prescribed be higher then prision correccional or
imprisonment for six (6) years, or a fine of P6,000.00.11
Applying the law to the case at bench, we find that although the second
requirement has been met, the first requirement is wanting. A review of these
Presidential Decrees, except Batas Pambansa Blg. 129, would reveal that the
crime committed by public officers or employees must be "in relation to their
office" if it is to fall within the jurisdiction of the Sandiganbayan. This phrase
which is traceable to Pres. Decree No. 1468, has been retained by Pres. Decree
No. 1861 as a requirement before the Ombudsman can acquire primary
jurisdiction on its power to investigate.
It cannot be denied that Pres. Decree No. 1861 is in pari materia to Article
XI, Sections 12 and 13 of the 1987 Constitution and the Ombudsman Act of
1989 because, as earlier mentioned, the Ombudsman's power to investigate
is dependent on the cases cognizable by the Sandiganbayan. Statutes are

in pari materia when they relate to the same person or thing or to the same
class of persons or things, or object, or cover the same specific or
particular subject matter.
It is axiomatic in statutory construction that a statute must be interpreted,
not only to be consistent with itself, but also to harmonize with other laws
on the same subject matter, as to form a complete, coherent and intelligible
system. The rule is expressed in the maxim, "interpretare et concordare
legibus est optimus interpretandi," or every statute must be so construed
and harmonized with other statutes as to form a uniform system of
jurisprudence. Thus, in the application and interpretation of Article XI,
Sections 12 and 13 of the 1987 Constitution and the Ombudsman Act of
1989, Pres. Decree No. 1861 must be taken into consideration. It must be
assumed that when the 1987 Constitution was written, its framers had in
mind previous statutes relating to the same subject matter. In the absence
of any express repeal or amendment, the 1987 Constitution and the
Ombudsman Act of 1989 are deemed in accord with existing statute,
specifically, Pres. Decree No. 1861.12 (Emphasis supplied)
R.A. No. 8249 which amended Section 4, paragraph (b) of the Sandiganbayan Law
(P.D. 1861) likewise provides that for other offenses, aside from those enumerated
under paragraphs (a) and (c), to fall under the exclusive jurisdiction of the
Sandiganbayan, they must have been committed by public officers or employees in
relation to their office.
In summation, the Constitution, Section 15 of the Ombudsman Act of 1989 and Section
4 of the Sandiganbayan Law, as amended, do not give to the Ombudsman exclusive
jurisdiction to investigate offenses committed by public officers or employees. The
authority of the Ombudsman to investigate offenses involving public officers or
employees is concurrent with other government investigating agencies such as
provincial, city and state prosecutors. However, the Ombudsman, in the exercise of its
primary jurisdiction over cases cognizable by the Sandiganbayan, may take over, at any
stage, from any investigating agency of the government, the investigation of such
cases.
In other words, respondent DOJ Panel is not precluded from conducting any
investigation of cases against public officers involving violations of penal laws but if the
cases fall under the exclusive jurisdiction of the Sandiganbayan, then respondent
Ombudsman may, in the exercise of its primary jurisdiction take over at any stage.
Thus, with the jurisprudential declarations that the Ombudsman and the DOJ have
concurrent jurisdiction to conduct preliminary investigation, the respective heads of said
offices came up with OMB-DOJ Joint Circular No. 95-001 for the proper guidelines of
their respective prosecutors in the conduct of their investigations, to wit:
OMB-DOJ JOINT CIRCULAR NO. 95-001

Series of 1995
TO: ALL GRAFT INVESTIGATION/SPECIAL PROSECUTION OFFICERS OF
THE OFFICE OF THE OMBUDSMAN
ALL REGIONAL STATE PROSECUTORS AND THEIR ASSISTANTS,
PROVINCIAL/CITY PROSECUTORS AND THEIR ASSISTANTS, STATE
PROSECUTORS AND PROSECUTING ATTORNEYS OF THE DEPARTMENT
OF JUSTICE.
SUBJECT: HANDLING COMPLAINTS FILED AGAINST PUBLIC OFFICERS
AND EMPLOYEES, THE CONDUCT OF PRELIMINARY INVESTIGATION,
PREPARATION OF RESOLUTIONS AND INFORMATIONS AND
PROSECUTION OF CASES BY PROVINCIAL AND CITY PROSECUTORS AND
THEIR ASSISTANTS.
x-------------------------------------------------------------------------------------------------------x
In a recent dialogue between the OFFICE OF THE OMBUDSMAN and the
DEPARTMENT OF JUSTICE, discussion centered around the latest
pronouncement of the supreme court on the extent to which the ombudsman
may call upon the government prosecutors for assistance in the investigation and
prosecution of criminal cases cognizable by his office and the conditions under
which he may do so. Also discussed was Republic Act No. 7975 otherwise
known as "an act to strengthen the functional and structural organization of the
sandiganbayan, amending for the purpose presidential decree no. 1606, as
amended" and its implications on the jurisdiction of the office of the Ombudsman
on criminal offenses committed by public officers and employees.
Concerns were expressed on unnecessary delays that could be caused by
discussions on jurisdiction between the OFFICE OF THE OMBUDSMAN and the
department of justice, and by procedural conflicts in the filing of complaints
against public officers and employees, the conduct of preliminary investigations,
the preparation of resolutions and informations, and the prosecution of cases by
provincial and city prosecutors and their assistants as deputized prosecutors of
the ombudsman.
Recognizing the concerns, the office of the ombudsman and the department of
justice, in a series of consultations, have agreed on the following guidelines to be
observed in the investigation and prosecution of cases against public officers and
employees:
1. Preliminary investigation and prosecution of offenses committed by public
officers and employees in relation to office whether cognizable by the
sandiganbayan or the regular courts, and whether filed with the office of the

ombudsman or with the office of the provincial/city prosecutor shall be under the
control and supervision of the office of the ombudsman.
2. Unless the Ombudsman under its Constitutional mandate finds reason to
believe otherwise, offenses not in relation to office and cognizable by the regular
courts shall be investigated and prosecuted by the office of the provincial/city
prosecutor, which shall rule thereon with finality.
3. Preparation of criminal information shall be the responsibility of the
investigating officer who conducted the preliminary investigation. Resolutions
recommending prosecution together with the duly accomplished criminal
informations shall be forwarded to the appropriate approving authority.
4. Considering that the office of the ombudsman has jurisdiction over public
officers and employees and for effective monitoring of all investigations and
prosecutions of cases involving public officers and employees, the office of the
provincial/city prosecutor shall submit to the office of the ombudsman a monthly
list of complaints filed with their respective offices against public officers and
employees.
Manila, Philippines, October 5, 1995.
(signed)

(signed)

TEOFISTO T. GUINGONA,
JR.
Secretary
Department of Justice

ANIANO A. DESIERTO
Ombudsman
Office of the Ombudsman

A close examination of the circular supports the view of the respondent Ombudsman
that it is just an internal agreement between the Ombudsman and the DOJ.
Sections 2 and 4, Rule 112 of the Revised Rules on Criminal Procedure on Preliminary
Investigation, effective December 1, 2000, to wit:
SEC. 2. Officers authorized to conduct preliminary investigationsThe following may conduct preliminary investigations:
(a) Provincial or City Prosecutors and their assistants;
(b) Judges of the Municipal Trial Courts and Municipal Circuit Trial Courts;
(c) National and Regional State Prosecutors; and

(d) Other officers as may be authorized by law.


Their authority to conduct preliminary investigation shall include all crimes
cognizable by the proper court in their respective territorial jurisdictions.
SEC. 4. Resolution of investigating prosecutor and its review. - If the
investigating prosecutor finds cause to hold the respondent for trial, he shall
prepare the resolution and information, He shall certify under oath in the
information that he, or as shown by the record, an authorized officer, has
personally examined the complainant and his witnesses; that there is reasonable
ground to believe that a crime has been committed and that the accused is
probably guilty thereof; that the accused was informed of the complaint and of
the evidence submitted against him; and that he was given an opportunity to
submit controverting evidence. Otherwise, he shall recommend the dismissal of
the complaint.
Within five (5) days from his resolution, he shall forward the record of the case to
the provincial or city prosecutor or chief state prosecutor, or to the Ombudsman
or his deputy in cases of offenses cognizable by the Sandiganbayan in the
exercise of its original jurisdiction. They shall act on the resolution within ten
(10) days from their receipt thereof and shall immediately inform the parties of
such action.
No complaint or information may be filed or dismissed by an investigating
prosecutor without the prior written authority or approval of the provincial
or city prosecutor or chief state prosecutor or the Ombudsman or his
deputy.
Where the investigating prosecutor recommends the dismissal of the complaint
but his recommendation is disapproved by the provincial or city prosecutor or
chief state prosecutor or the Ombudsman or his deputy on the ground that a
probable cause exists, the latter may, by himself file the information against the
respondent, or direct another assistant prosecutor or state prosecutor to do so
without conducting another preliminary investigation.
If upon petition by a proper party under such rules as the Department of Justice
may prescribe or motu proprio, the Secretary of Justice reverses or modifies the
resolution of the provincial or city prosecutor or chief state prosecutor, he shall
direct the prosecutor concerned either to file the corresponding information
without conducting another preliminary investigation, or to dismiss or move for
dismissal of the complaint or information with notice to the parties. The same
Rule shall apply in preliminary investigations conducted by the officers of the
Office of the Ombudsman. (Emphasis supplied)
confirm the authority of the DOJ prosecutors to conduct preliminary investigation of
criminal complaints filed with them for offenses cognizable by the proper court within

their respective territorial jurisdictions, including those offenses which come within the
original jurisdiction of the Sandiganbayan; but with the qualification that in offenses
falling within the original jurisdiction of the Sandiganbayan, the prosecutor shall, after
their investigation, transmit the records and their resolutions to the Ombudsman
or his deputy for appropriate action. Also, the prosecutor cannot dismiss the
complaint without the prior written authority of the Ombudsman or his deputy, nor can
the prosecutor file an Information with the Sandiganbayan without being deputized by,
and without prior written authority of the Ombudsman or his deputy.
Next, petitioner contends that under OMB-Joint Circular No. 95-001, there is no showing
that the Office of the Ombudsman has deputized the prosecutors of the DOJ to conduct
the preliminary investigation of the charge filed against him.
We find no merit in this argument. As we have lengthily discussed, the Constitution, the
Ombudsman Act of 1989, Administrative Order No. 8 of the Office of the Ombudsman,
the prevailing jurisprudence and under the Revised Rules on Criminal Procedure, all
recognize and uphold the concurrent jurisdiction of the Ombudsman and the DOJ to
conduct preliminary investigation on charges filed against public officers and
employees.
To reiterate for emphasis, the power to investigate or conduct preliminary investigation
on charges against any public officers or employees may be exercised by an
investigator or by any provincial or city prosecutor or their assistants, either in their
regular capacities or as deputized Ombudsman prosecutors. The fact that all
prosecutors are in effect deputized Ombudsman prosecutors under the OMB-DOJ
Circular is a mere superfluity. The DOJ Panel need not be authorized nor deputized by
the Ombudsman to conduct the preliminary investigation for complaints filed with it
because the DOJ's authority to act as the principal law agency of the government and
investigate the commission of crimes under the Revised Penal Code is derived from the
Revised Administrative Code which had been held in the Natividad case13 as not being
contrary to the Constitution. Thus, there is not even a need to delegate the conduct of
the preliminary investigation to an agency which has the jurisdiction to do so in the first
place. However, the Ombudsman may assert its primary jurisdiction at any stage of the
investigation.
Petitioner's contention that OMB-DOJ Joint Circular No. 95-001 is ineffective on the
ground that it was not published is not plausible. We agree with and adopt the
Ombudsman's dissertation on the matter, to wit:
Petitioner appears to be of the belief, although NOT founded on a proper reading
and application of jurisprudence, that OMB-DOJ Joint Circular No. 95-001, an
internal arrangement between the DOJ and the Office of the Ombudsman, has to
be published.
As early as 1954, the Honorable Court has already laid down the rule in the case
of People vs. Que Po Lay, 94 Phil. 640 (1954) that only circulars and

regulations which prescribe a penalty for its violation should be published before
becoming effective, this, on the general principle and theory that before the
public is bound by its contents, especially its penal provision, a law, regulation or
circular must first be published and the people officially and specifically informed
of said contents and its penalties: said precedent, to date, has not yet been
modified or reversed. OMB-DOJ Joint Circular No. 95-001 DOES NOT contain
any penal provision or prescribe a mandatory act or prohibit any, under pain or
penalty.
What is more, in the case of Tanada v. Tuvera, 146 SCRA 453 (1986), the
Honorable Court ruled that:
Interpretative regulations and those merely internal in nature, that is, regulating
only the personnel of the administrative agency and not the public, need not be
published. Neither is publication required of the so-called letters of instructions
issued by administrative superiors concerning the rules or guidelines to be
followed by their subordinates in the performance of their duties. (at page 454.
emphasis supplied)
OMB-DOJ Joint Circular No. 95-001 is merely an internal circular between the
DOJ and the Office of the Ombudsman, outlining authority and responsibilities
among prosecutors of the DOJ and of the Office of the Ombudsman in the
conduct of preliminary investigation. OMB-DOJ Joint Circular No. 95-001 DOES
NOT regulate the conduct of persons or the public, in general.
Accordingly, there is no merit to petitioner's submission that OMB-DOJ Joint
Circular No. 95-001 has to be published.14
Petitioner insists that the Ombudsman has jurisdiction to conduct the preliminary
investigation because petitioner is a public officer with salary Grade 31 so that the case
against him falls exclusively within the jurisdiction of the Sandiganbayan. Considering
the Court's finding that the DOJ has concurrent jurisdiction to investigate charges
against public officers, the fact that petitioner holds a Salary Grade 31 position does not
by itself remove from the DOJ Panel the authority to investigate the charge of coup
d'etat against him.
The question whether or not the offense allegedly committed by petitioner is one of
those enumerated in the Sandiganbayan Law that fall within the exclusive jurisdiction of
the Sandiganbayan will not be resolved in the present petition so as not to pre-empt the
result of the investigation being conducted by the DOJ Panel as to the questions
whether or not probable cause exists to warrant the filing of the information against the
petitioner; and to which court should the information be filed considering the presence of
other respondents in the subject complaint.
WHEREFORE, the petition for certiorari is DISMISSED for lack of merit.

SO ORDERED.
G.R. No. 175750-51

April 2, 2014

SILVERINA E. CONSIGNA, Petitioner,


vs.
PEOPLE OF THE PHILIPPINES, THE HON. SANDIGANBAYAN (THIRD DIVISION),
and EMERLINA MOLETA,Respondents.
DECISION
PEREZ, J.:
For review on certiorari is the Decision1 of the Honorable Sandiganbayan dated 12
December 2006, finding Silverina E. Consigna (petitioner) guilty for violation of Section
3(e) of Republic Act (R.A.) No. 3019, otherwise known as Anti-Graft and Corrupt
Practices Act, and Estafa, as defined and penalized under Article 315 (2)(a) of the
Revised Penal Code (RPC).
The facts as culled from the records are as follows:
On or about 14 June 1994, petitioner, the Municipal Treasurer of General Luna, Surigao
del Norte, together with Jose Herasmio, obtained as loan from private respondent
Hermelina Moleta (Moleta), the sum of P320,000.00, to pay for the salaries of the
employees of the municipality and to construct the municipal gymnasium as the
municipalitys Internal Revenue Allotment (IRA) had not yet arrived. As payment,
petitioner issued three (3) Land Bank of the Philippines (LBP) checks signed by Jaime
Rusillon (Rusillon), the incumbent mayor of the Municipality of General Luna: (1) Check
No. 11281104 for P130,000.00 dated 14 June 1994; (2) Check No. 9660500
forP130,000.00 dated 14 June 1994; and (3) Check No. 9660439 for P60,000.00 dated
11 July 1994.
Between 15 June 1994 and 18 August 1994, in several attempts on different occasions,
Moleta demanded payment from petitioner and Rusillon, but to no avail.
Thus, on 18 August 1994, Moleta deposited the three (3) LBP checks to her account in
Metrobank-Surigao Branch. Upon presentation for payment, Metrobank returned the
checks to Moleta as the checks had no funds. The following day, Moleta again
deposited the checks. This time, however, she deposited the checks to her LBP
account. Upon presentation for payment, the checks were again returned for the
reason, "Signature Not on File." Upon verification, LBP informed Moleta that the
municipalitys account was already closed and transferred to Development Bank of the
Philippines, and that petitioner, the municipal treasurer, has been relieved from her
position.

Hence, Moleta filed with the Sandiganbayan two (2) sets of Information against
petitioner, in the latters capacity as Municipal Treasurer and Rusillon, in his capacity as
Municipal Mayor of General Luna, Surigao del Norte, to wit:
(1) Criminal Case No. 24182 - Sec. 3(e) of R.A. 3019, otherwise known as AntiGraft and Corrupt Practices Act:
That on or about 15 June 1994, or sometime after said date, at the General
Luna, Surigao del Norte, and within the jurisdiction of this Honorable Court
accused Municipal Treasurer Silverina Consigna (with Salary Grade below 27),
and Municipal Mayor Jaime Rusillon (with Salary Grace 27) did then and there,
willfully and unlawfully, with evident bad faith, in cooperation with each other, and
taking advantage of their official positions and in the discharge for the functions
as such, borrow the amount of P320,000.00 from one Emerlina Moleta to whom
they misrepresented to be for the municipality of General Luna, when in fact the
same is not; and fail to pay back said amount thereby causing undue injury to
said Emerlina Moleta in the amount of P320,000.00.2
(2) Criminal Case No. 24183 Art. 315 of the RPC, otherwise known as Estafa:
That on or about 15 June 1994, or sometime after said date, at the General
Luna, Surigao del Norte, and within the jurisdiction of this Honorable Court,
accused Municipal Treasurer Silverina Consigna (with Salary Grade below 27),
and Municipal Mayor Jaime Rusillon (with Salary Grade 27), did then and there,
willfully and unlawfully, with evident bad faith, in cooperation with each other,
representing themselves to be transacting in behalf of the [M]unicipality of Gen.
Luna, in truth and in fact they are not, contract a loan from one Emerlina Moleta
in the amount of P320,000.00 for which they issued three (3) checks: LBP Check
No. 11281104 dated 14 June 1994 in the amount of P130,000.00, LBP Check
No. 9660500 dated 14 June 1994 in the amount of P130,000.00, and LBP Check
no. 9660439 dated 11 July 1994 in the amount ofP60,000.00, all in favor of said
Emerlina Moleta, knowing fully well that the account belongs to the Municipality
of the (sic) Gen. Luna, and that they have no personal funds [of] the same
account such that upon presentation of the said checks to the bank, the same
were dishonored and refused payment, to the damage and prejudice of said
Emerlina Moleta in the amount of P320,000.00.3
As defense, petitioner argued that the court a quo has no jurisdiction because (1) the
crime as charged did not specify the provision of law allegedly violated, i.e., the specific
type of Estafa; and (2) Sec. 3(e) of RA 3019 does not fall within the jurisdiction of the
court a quo because the offense as charged can stand independently of public office
and public office is not an element of the crime.4
The court a quo admitted that the Information for violation of Estafa did not specify the
provision of law allegedly violated.5 However, based on the allegations of deceit and

misrepresentation, the court a quo allowed the prosecution to indict petitioner and
Rusillon under Art. 315 (2)(a) of the RPC.
On the charge of graft and corruption, petitioner argued that, "[w]hen allegations in the
information do not show that the official position of the [petitioner] was connected with
the offense charged, the accused is not charged with an offense in relation to her official
functions".6 Petitioner, citing Lacson v. The Executive Secretary,7 further argued:
x x x [M]ere allegation in the information "that the offense was committed by the
accused public officer in relation to his office is not sufficient. That phrase is a mere
conclusion of law not a factual averment that would show the close intimacy between
the offense charged and the discharge of accuseds official duties." 8
Petitioner also contends that there was no fraud or misrepresentation. By demanding
payment from Rusillon, Moleta attested that there exists no fraud or misrepresentation.
In petitioners words, " why will she [Moleta] insist payment from [Rusillon] if she has
no knowledge that the money loaned have reached him?" 9
On the other hand, Rusillon maintained that he had no participation in the acts
committed by petitioner. Based on his testimony, he signed the three (3) checks to pay
the following: (1) payroll of the following day; (2) daily expenses of the municipal
building; (3) construction of the municipal gymnasium; and (4) health offices medical
supplies.10 As found by the court a quo, "the only link of Rusillon to [petitioner] with
respect to the loan transaction is his signature on the three (3) checks which [petitioner]
used as security to Moleta."11
After trial, the Sandiganbayan, on 12 December 2006, found petitioner guilty, but
exonerated Rusillon. The dispositive portion of the Decision reads:12
WHEREFORE, in view of the foregoing, judgment is hereby rendered as follows:
(1) In Criminal Case No. 24182, accused SILVERINA E. CONSIGNA is found
GUILTY beyond reasonable doubt of violation of Section 3(e) of the Republic Act
No. 3019, and is hereby SENTENCED to suffer the penalty of imprisonment of
six (6) years and one (1) month to eight (8) years.
Accused JAIME RUSILLON is ACQUITTED for failure of the prosecution to prove
his guilt with moral certainty.
(2) In Criminal Case No. 24183, accused SILVERINA E. CONSIGNA is found
GUILTY beyond reasonable doubt of Estafa under Article 315 (2)(a) of the
Revised Penal Code, and is hereby SENTENCED to the indeterminate prison
term of six (6) years and one (1) day of prision mayor as MINIMUM, to twenty
(20) years of reclusion temporal as MAXIMUM.

Accused JAIME RUSILLON is ACQUITTED as his guilt was not proven with
moral certainty.
(3) Accused SILVERIA E. CONSIGNA is ordered to pay private complainant
Emerlina F. Moleta the amount of PhP368,739.20 by way of actual damages;
PhP30,000.00 as moral damages, and the costs of suit; and
(4) The hold departure order against accused JAIME RUSILLON in connection
with these cases is hereby LIFTED.
Hence, this Petition.
Noticeably, the petitioner formulated its arguments, thus:
a. The court a quo committed grave abuse of discretion in making its finding of facts
which amounts to lack of jurisdiction.
xxxx
b. The court a quo committed grave abuse of discretion when it convicted the accused
on "false pretense, fraudulent act or means" made or executed prior to or
simultaneously with the commission of fraud.
xxxx
c. The court a quo committed grave abuse of discretion when it made a conclusion that
the petitioner acted with manifest partiality, evident bad faith or inexcusable negligence
to justify its conclusion that all the elements of violations of Section 3(e) of RA 3019 are
present."13
Preliminarily, We here note a common disorder in petitions that mingle the concepts
involved in a Petition for Review under Rule 45 and in the special civil action of certiorari
under Rule 65, as a prevalent practice of litigants to cure a lapsed appeal.
We shall discuss the distinction.
With regard to the period to file a petition, in Rule 45, the period within which to file is
fifteen (15) days from notice of the judgment or final order or resolution appealed
from.14 In contrast to Rule 65, the petition should be filed not later than sixty (60) days
from notice of the judgment, order or resolution.15
Regarding the subject matter, a review on certiorari under Rule 45 is generally limited to
the review of legal issues; the Court only resolves questions of law which have been
properly raised by the parties during the appeal and in the petition. 16 A Rule 65 review,
on the other hand, is strictly confined to the determination of the propriety of the trial
courts jurisdiction whether it has jurisdiction over the case and if so, whether the

exercise of its jurisdiction has or has not been attended by grave abuse of discretion
amounting to lack or excess of jurisdiction.17 Otherwise stated, errors of judgment are
the proper subjects of a Rule 45 petition; errors of jurisdiction are addressed in a Rule
65 petition.
The special civil action of certiorari under Rule 65 is resorted to only in the absence of
appeal or any plain, speedy and adequate remedy in the ordinary course of law. 18 So
when appeal, or a petition for review is available, certiorari cannot be resorted to;
certiorari is not a substitute for a lapsed or lost appeal.19 A Rule 65 certiorari petition
cannot be a substitute for a Rule 45 petition so as to excuse the belatedness in filing the
correct petition. Where an appeal is available, certiorari will not prosper, even if the
ground therefor is grave abuse of discretion.20
Grave abuse of discretion means "such capricious and whimsical exercise of judgment
as is equivalent to lack of jurisdiction, or, in other words where the power is exercised in
an arbitrary or despotic manner by reason of passion or personal hostility, and it must
be so patent and gross as to amount to an evasion of positive duty or to a virtual refusal
to perform the duty enjoined or to act at all in contemplation of law.21
Petitioner was correct when she filed a Petition for Review under Rule 45. However,
instead of raising errors of judgment as a proper subject of a petition for review under
Rule 45, the petition formulated jurisdictional errors purportedly committed by the court
a quo, i.e., whether or not the court a quo committed grave abuse of discretion, 22 which
is the proper subject of a Petition for Certiorari under Rule 65. Noticeably, the petition
does not allege any bias, partiality or bad faith by the court a quo in its
proceedings;23 and the petition does not raise a denial of due process in the
proceedings before the Sandiganbayan.24
Importantly, however, the petition followed the period specified in Rule 45. It was timely
filed. For that reason, we excuse the repeated referral to the supposed grave abuse of
discretion of the Sandiganbayan and treat the petition as, nonetheless, one for review of
the questioned decision. We thus recast the arguments as:
I. Whether or not the court a quo committed a reversible error for finding
petitioner guilty of estafa, based on information which does not specifically
designate the provision allegedly violated.
II. Whether or not petitioner is guilty of estafa as penalized under Art. 315 (2)(a)
of the RPC.
III. Whether or not petitioner is guilty of Sec. 3 (e) of RA 3019.
The Petition must fail.
1. On the first issue, petitioner insists that even if the court a quo already admitted that
the Information failed to specifically identify the mode or manner by which estafa was

committed by petitioner, it nonetheless went on to convict her by relying on the


allegation in the Information of deceit and misrepresentation and applying par. (2)(a),
Art. 315 of the RPC.
Entrenched in jurisprudence is the dictum that the real nature of the criminal charge is
determined not from the caption or preamble of the information, or from the specification
of the provision of law alleged to have been violated, which are mere conclusions of
law, but by the actual recital of the facts in the complaint or information. 25As held in
People v. Dimaano:26
For complaint or information to be sufficient, it must state the name of the accused; the
designation of the offense given by the statute; the acts or omissions complained of as
constituting the offense; the name of the offended party; the approximate time of the
commission of the offense, and the place wherein the offense was committed. What is
controlling is not the title of the complaint, nor the designation of the offense charge or
the particular law or part thereof allegedly violated, these being mere conclusions of law
made by the prosecutor, but the description of the crime charged and the particular
facts therein recited. The acts or omissions complained of must be alleged in such form
as is sufficient to enable a person of common understanding to know what offense is
intended to be charged, and enable the court to pronounce proper judgment. No
information for a crime will be sufficient if it does not accurately and clearly allege the
elements of the crime charged. Every element of the offense must be stated in the
information. What facts and circumstances are necessary to be included therein must
be determined by reference to the definitions and essentials of the specified crimes. The
requirement of alleging the elements of a crime in the information is to inform the
accused of the nature of the accusation against him so as to enable him to suitably
prepare his defense. The presumption is that the accused has no independent
knowledge of the facts that constitute the offense. (Emphasis supplied)
As early in United States v. Lim San,27 this Court has determined that:
From a legal point of view, and in a very real sense, it is of no concern to the accused
what is the technical name of the crime of which he stands charged. It in no way aids
him in a defense on the merits. x x x. That to which his attention should be directed, and
in which he, above all things else, should be most interested, are the facts alleged. The
real question is not did he commit a crime given in the law some technical and specific
name, but did he perform the acts alleged in the body of the information in the manner
therein set forth. If he did, it is of no consequence to him, either as a matter of
procedure or of substantive right, how the law denominates the crime which those acts
constitute. The designation of the crime by name in the caption of the information from
the facts alleged in the body of that pleading is a conclusion of law made by the fiscal.
In the designation of the crime the accused never has a real interest until the trial has
ended. For his full and complete defense he need not know the name of the crime at all.
It is of no consequence whatever for the protection of his substantial rights. The real
and important question to him is, "Did you perform the acts alleged in the manner
alleged?" not "Did you commit a crime named murder." If he performed the acts alleged,

in the manner stated, the law determines what the name of the crime is and fixes the
penalty therefor. It is the province of the court alone to say what the name of the crime
is or what it is named. x x x. (Emphasis and underscoring supplied)
Petitioners argument is as outdated as it is erroneous. The averments in the two (2)
sets of Information against petitioner and Rusillon clearly stated facts and
circumstances constituting the elements of the crime of estafa as to duly inform them of
the nature and cause of the accusation, sufficient to prepare their respective defenses.
2. Contrary to the submission of petitioner, false pretense and fraudulent acts attended
her transaction with Moleta. The law explicitly provides that in the prosecution for Estafa
under par. (2)(a), Art. 315 of the RPC, it is indispensable that the element of deceit,
consisting in the false statement or fraudulent representation of the accused, be made
prior to, or at least simultaneously with the commission of the fraud, it being essential
that such false statement or representation constitutes the very cause or the only motive
which induced the offended party to part with his money. Paragraph 2(a), Art. 315 of the
RPC provides:
Art. 315. Swindling (estafa). Any person who shall defraud another by any of the
means mentioned hereinbelow x x x:
xxxx
2. By means of any of the following false pretenses or fraudulent acts executed prior to
or simultaneously with the commission of the fraud:
xxxx
(a) By using fictitious name, or falsely pretending to possess power, influence,
qualifications, property, credit, agency, business or imaginary transactions, or by means
of other similar deceits.
xxxx
The elements of estafa by means of deceit, whether committed by false pretenses or
concealment, are the following: (a) there must be a false pretense, fraudulent act or
fraudulent means; (b) such false pretense, fraudulent act or fraudulent means must be
made or executed prior to or simultaneously with the commission of the fraud; (c) the
offended party must have relied on the false pretense, fraudulent act or fraudulent
means, that is, he was induced to part with his money or property because of the false
pretense, fraudulent act or fraudulent means; and (d) as a result thereof, the offended
party suffered damage.28
As borne by the records, petitioners representations were outright distortions of the
truth perpetrated for the sole purpose of inducing Moleta to hand to her the amount
of P320,000.00 purportedly for the Municipality of General Luna. Being the Municipal

Treasurer, there was reason for Moleta to rely on petitioners representations that
money is needed for the payment of the employees salary as well as for the
construction of the gymnasium. There was also a ring of truth to the deception that the
share of the municipality from the IRA is forthcoming. Added to this, petitioners
representations were even supported by the issuance of three (3) LBP checks to
guarantee payment taken from the account of the municipality and signed by no less
than the municipal mayor, giving the impression that the loaned amount would indeed
be utilized for public purposes.
As the court a quo correctly observed:
It is undisputed that Consigna obtained a loan from Moleta for the reason that the
municipality lacked funds for the June 15, 1994 payroll of the employees and materials
of the gymnasium. However, several circumstances point to the fact that Consignas
representation has no basis.
She contradicted her own testimony that at the time she borrowed from Moleta on June
14, 1994, the municipality suffered a shortage of funds, with her admission that when
she was relieved as a municipal treasurer, the Municipality had more than 1 million in
Land Bank from the IRA of P600,000.00 a month for the past three months x x x. This
means that when she left her post before the second week of July x x x, the municipality
had money from the April to June 1994 IRA, enough to meet the need of P320,000.00. x
x x29
The circumstances and the reason behind the issuance of the three (3) checks given to
Moleta by petitioner was testified to by Rusillon:
He was the incumbent mayor of the Municipality of General Luna, Surigao del Norte, in
1994. In the morning of June 14, 1994, he received the amount of P268,800.00 from
accused Consigna, as evidenced by a voucher (Exh. 1) signed by him on the same day.
The money was to be used for the purchase of materials for the gymnasium of the
municipality which construction started in 1992. After signing the voucher, he ordered
Consigna to prepare a check for P130,000.00 (Exh. 2) for the June 15, 1994 payroll of
the municipalitys employees. After the check was prepared, he again ordered Consigna
to make another two checks, one for P130,000.00 (Exh. 3) dated June 14, 1994
intended for the expenses of the municipal building and for the daily transactions of the
municipality in the following days, and the other check was for P60,000.00 (Exh. 4)
dated July 11, 1994 for the purchase of medicines for the municipalitys health office.
The latter check was postdated to July because it would be charged against the IRA in
the 3rd quarter of 1994 since they bought medicines at that time on a quarterly basis as
the budget allowed only P240,000.00 per year for such expenditure."30
3. Anent the issue on the alleged grave abuse of discretion amounting to lack of
jurisdiction committed by the court a quo when it took cognizance of Criminal Case No.
24182, charging petitioner for "taking advantage of her official position and the
discharge of the functions as such," petitioner averred that the charge was erroneous

because borrowing of money is not a function of a Municipal Treasurer under the Local
Government Code. Petitioner asserts that the last sentence of Sec. 3(e) of RA 3019
cannot cover her.
We find such reasoning misplaced.
The following are the essential elements of violation of Sec. 3(e) of RA 3019:
1. The accused must be a public officer discharging administrative, judicial or
official functions;
2. He must have acted with manifest partiality, evident bad faith or inexcusable
negligence; and
3. That his action caused any undue injury to any party, including the
government, or giving any private party unwarranted benefits, advantage or
preference in the discharge of his functions.31
There is no doubt that petitioner, being a municipal treasurer, was a public officer
discharging official functions when she misused such position to be able to take out a
loan from Moleta, who was misled into the belief that petitioner, as municipal treasurer,
was acting on behalf of the municipality.
In Montilla v. Hilario,32 this Court described the "offense committed in relation to the
office" as:
[T]he relation between the crime and the office contemplated by the Constitution is, in
our opinion, direct and not accidental. To fall into the intent of the Constitution, the
relation has to be such that, in the legal sense, the offense cannot exist without the
office. In other words, the office must be a constituent element of the crime as defined in
the statute, such as, for instance, the crimes defined and punished in Chapter Two to
Six, Title Seven, of the Revised Penal Code.
Public office is not of the essence of murder. The taking of human life is either murder
or homicide whether done by a private citizen or public servant, and the penalty is the
same except when the perpetrator, being a public functionary took advantage of his
office, as alleged in this case, in which event the penalty is increased.
But the use or abuse of office does not adhere to the crime as an element; and even as
an aggravating circumstance, its materiality arises not from the allegations but on the
proof, not from the fact that the criminals are public officials but from the manner of the
commission of the crime. (Emphasis supplied)
In this case, it was not only alleged in the Information, but was proved with certainty
during trial that the manner by which petitioner perpetrated the crime necessarily relates
to her official function as a municipal treasurer. Petitioners official function created in

her favor an impression of authority to transact business with Moleta involving


government financial concerns. There is, therefore, a direct relation between the
commission of the crime and petitioners office the latter being the very reason or
consideration that led to the unwarranted benefit she gained from Moleta, for which the
latter suffered damages in the amount of P320,000.00. It was just fortunate that Rusillon
instructed the bank to stop payment of the checks issued by petitioner, lest, the victim
could have been the Municipality of General Luna.
As regards the two other elements, the Court explained in Cabrera v.
Sandiganbayan33 that there are two (2) ways by which a public official violates Sec. 3(e)
of R.A. No. 3019 in the performance of his functions, namely: (a) by causing undue
injury to any party, including the Government; or (b) by giving any private party any
unwarranted benefits, advantage or preference. The accused may be charged under
either mode or under both.34 This was reiterated in Quibal v. Sandiganbayan,35 where
the Court held that the use of the disjunctive term "or" connotes that either act qualifies
as a violation of Sec. 3(e) of R.A. No. 3019.
In this case, petitioner was charged of violating Sec. 3(e) of R.A. No. 3019 under the
alternative mode of "causing undue injury" to Moleta committed with evident bad faith,
for which she was correctly found guilty. "Evident bad faith" connotes not only bad
judgment but also palpably and patently fraudulent and dishonest purpose to do moral
obliquity or conscious wrongdoing for some perverse motive or ill will. "Evident bad
faith" contemplates a state of mind affirmatively operating with furtive design or with
some motive of self-interest or ill will or for ulterior purposes,36 which manifested in
petitioners actuations and representation.
The inevitable conclusion is that petitioner capitalized on her official function to commit
the crimes charged. Without her position, petitioner would not have induced Moleta to
part with her money. In the same vein, petitioner could not have orchestrated a scheme
of issuing postdated checks meddling with the municipalitys coffers and defiling the
mayors signature. As correctly found by the court a quo:
x x x Likewise worthy of stress is [petitioners] failure to establish that the amount she
disbursed to Rusillon came from the money she loaned from Moleta. If indeed
the P268,800.00 advanced to Rusillon was charged against the loan, then, this should
have been reflected in the municipalitys books of accounts. The same is true with
theP320,000.00 and the P32,000.00 given to Moleta if the proceeds of the loan really
went to the municipalitys treasury. It is a standard accounting procedure that every
transaction must be properly entered in the books of accounts of the municipality. A
cash that comes in is a debit to the asset account and every loan incurred is a credit to
the liability account.37
Given the above disquisition, it becomes superfluous to dwell further on the issue raised
by petitioner that Sec. 3(e) applies only to officers and employees of offices or
government corporations charged with the grant of licenses or other concessions.
Nonetheless, to finally settle the issue, the last sentence of the said provision is not a

restrictive requirement which limits the application or extent of its coverage. This has
long been settled in our ruling in Mejorada v. Sandiganbayan,38 where we categorically
declared that a prosecution for violation of Sec. 3(e) of the Anti-Graft Law will lie
regardless of whether or not the accused public officer is "charged with the grant of
licenses or permits or other concessions." Quoted hereunder is an excerpt from
Mejorada:39
Section 3 cited above enumerates in eleven subsections the corrupt practices of any
public officers (sic) declared unlawful. Its reference to "any public officer" is without
distinction or qualification and it specifies the acts declared unlawful. We agree with the
view adopted by the Solicitor General that the last sentence of paragraph [Section 3] (e)
is intended to make clear the inclusion of officers and employees of officers (sic) or
government corporations which, under the ordinary concept of "public officers" may not
come within the term. It is a strained construction of the provision to read it as applying
exclusively to public officers charged with the duty of granting licenses or permits or
other concessions. (Emphasis and underscoring supplied)
The above pronouncement was reiterated in Cruz v. Sandiganbayan, 40 where the Court
affirmed the Mejorada ruling that finally puts to rest any erroneous interpretation of the
last sentence of Sec. 3(e) of the Anti-Graft Law.
All the elements of the crimes as charged are present in the case at bar.1wphi1 All
told, this Court finds no justification to depart from the findings of the lower court.
Petitioner failed to present any cogent reason that would warrant a reversal of the
Decision assailed in this petition.
WHEREFORE, the petition is DENIED. The Decision of the Sandiganbayan in Criminal
Case No. 24182-83 is AFFIRMED in toto.
SO ORDERED.

G.R. No. 128096 January 20, 1999


PANFILO M. LACSON, petitioner,
vs.
THE EXECUTIVE SECRETARY, THE SANDIGANBAYAN, OFFICE OF THE SPECIAL
PROSECUTOR, THE DEPARTMENT OF JUSTICE, MYRNA ABALORA, NENITA
ALAP-AP, IMELDA PANCHO MONTERO, and THE PEOPLE OF THE
PHILIPPINES, respondent.
ROMEO M. ACOP AND FRANCISCO G. ZUBIA, JR., petitioner-intervenors.

MARTINEZ, J.:
The constitutionality of Sections 4 and 7 of Republic Act No. 8249 an act which
further defines the jurisdiction of the Sandiganbayan is being challenged in this
petition for prohibition and mandamus. Petitioner Panfilo Lacson, joined by petitionersintervenors Romeo Acop and Francisco Zubia, Jr., also seeks to prevent the
Sandiganbayan from proceedings with the trial of Criminal Cases Nos. 23047-23057
(for multiple murder) against them on the ground of lack of jurisdiction.
The antecedents of this case, as gathered from the parties' pleadings and documentary
proofs, are as follows:
In the early morning of May 18, 1995, eleven (11) persons believed to be members of
the Kuratong Baleleng gang, reportedly an organized crime syndicate which had been
involved in a spate of bank robberies in Metro Manila, where slain along Commonwealth
Avenue in Quezon City by elements of the Anti-Bank Robbery and Intelligence Task
Group (ABRITG) headed by Chieff Superintendent Jewel Canson of the Philippine
National Police (PNP). The ABRITG was composed of police officers from the Traffic
Management Command (TMC) led by petitioner-intervenor Senior Superintendent
Francisco Zubia, Jr.; Presidential Anti-Crime Commission Task Force Habagat
(PACC-TFH) headed by petitioner Chief Superintendent Panfilo M. Lacson; Central
Police District Command (CPDC) led by Chief Superintendent Ricardo de Leon; and the
Criminal Investigation Command (CIC) headed by petitioner-intervenor Chief
Superintendent Romeo Acop.
Acting on a media expose of SPO2 Eduardo delos Reyes, a member of the CIC, that
what actually transpired at dawn of May 18, 1995 was a summary execution (or a rub
out) and not a shoot-out between the Kuratong Baleleng gang members and the
ABRITG, Ombudsman Aniano Desierto formed a panel of investigators headed by the
Deputy Ombudsman for Military Affairs, Bienvenido Blancaflor, to investigate the
incident. This panel later absolved from any criminal liability all the PNP officers and

personal allegedly involved in May 18, 1995 incident, with a finding that the said incident
was a legitimate police operation. 1
However, a review board led by Overall Deputy Ombudsman Francisco Villa modified
modified the Blancaflor panel's finding and recommended the indictment for multiple
murder against twenty-six (26) respondents, including herein petitioner and intervenors.
The recommendation was approved by the Ombudsman except for the withdrawal of
the charges against Chief Supt. Ricardo de Leon.
Thus, on November 2, 1995, petitioner Panfilo Lacson was among those charged as
principal in eleven (11) information for murder 2 before the Sandiganbayan's Second
Division, while intervenors Romeo Acop and Francisco Zubia, Jr. were among those
charged in the same informations as accessories after-in-the-fact.
Upon motion by all the accused in the 11 information, 3 the Sandiganbayan allowed
them to file a motion for reconsideration of the Ombudsman's action. 4
After conducting a reinvestigation, the Ombudsman filed on March 1, 1996 eleven (11)
amended informations 5before the Sandiganbayan, wherein petitioner was charged only
as an accessory, together with Romeo Acop and Francisco Zubia, Jr. and other. One of
the accused 6 was dropped from the case.
On March 5-6, 1996, all the accused filed separate motions questioning the jurisdiction
of the Sandiganbayan, asserting that under the amended informations, the cases fall
within the jurisdiction of the Regional Trial Court pursuant to Section 2 (paragraphs a
and c) of Republic Act No. 7975. 7 They contend that the said law limited the jurisdiction
of the Sandiganbayan to cases where one or more of the "principal accused" are
government officials with Salary Grade (SG) 27 or higher, or PNP officials with the rank
of Chief Superintendent (Brigadier General) or higher. The highest ranking principal
accused in the amended informations has the rank of only a Chief Inspector, and none
has the equivalent of at least SG 27.
Thereafter, in a Resolution 8 dated May 8, 1996 (promulgated on May 9, 1996), penned
by Justice Demetriou, with Justices Lagman and de Leon concurring, and Justices
Balajadia and Garchitorena dissenting, 9 the Sandiganbayan admitted the amended
information and ordered the cases transferred to the Quezon City Regional Trial Court
which has original and exclusive jurisdiction under R.A. 7975, as none of the principal
accused has the rank of Chief Superintendent or higher.
On May 17, 1996, the Office of the Special Prosecutor moved for a reconsideration,
insisting that the cases should remain with the Sandiganbayan. This was opposed by
petitioner and some of the accused.
While these motions for reconsideration were pending resolution, and even before the
issue of jurisdiction cropped up with the filing of the amended informations on March 1,
1996, House Bill No. 2299 10 and No. 1094 11(sponsored by Representatives Edcel C.

Lagman and Lagman and Neptali M. Gonzales II, respectively), as well as Senate Bill
No. 844 12 (sponsored by Senator Neptali Gonzales), were introduced in Congress,
defining expanding the jurisdiction of the Sandiganbayan. Specifically, the said bills
sought, among others, to amend the jurisdiction of the Sandiganbayan by deleting the
word "principal" from the phrase "principal accused" in Section 2 (paragraphs a and c)
of R.A. No. 7975.
These bills were consolidated and later approved into law as R.A. No. 8249 13 by the
President of the Philippines on February 5, 1997.
Subsequently, on March 5, 1997, the Sandiganbayan promulgated a
Resolution 14 denying the motion for reconsideration of the Special Prosecutor, ruling
that it "stands pat in its resolution dated May 8, 1996."
On the same day 15 the Sandiganbayan issued and ADDENDUM to its March 5, 1997
Resolution, the pertinent portion of which reads:
After Justice Lagman wrote the Resolution and Justice Demetriou
concurred in it, but before Justice de Leon. Jr. rendered his concurring
and dissenting opinion, the legislature enacted Republic Act 8249 and the
President of the Philippines approved it on February 5, 1997. Considering
the pertinent provisions of the new law, Justices Lagman and Demetriou
are now in favor of granting, as they are now granting, the Special
Prosecutor's motion for reconsideration. Justice de Leon has already done
so in his concurring and dissenting opinion.
xxx xxx xxx
Considering that three of the accused in each of these cases are PNP
Chief Superintendents: namely, Jewel T. Canson, Romeo M. Acop and
Panfilo M. Lacson, and that trial has not yet begun in all these cases in
fact, no order of arrest has been issued this court has competence to
take cognizance of these cases.
To recapitulate, the net result of all the foregoing is that by the vote of 3 of
2, the court admitted the Amended Informations in these cases by the
unanimous vote of 4 with 1 neither concurring not dissenting, retained
jurisdiction to try and decide the cases 16 (Empahasis supplied)
Petitioner now questions the constitutionality of Section 4 of R.A. No. 8249, including
Section 7 thereof which provides that the said law "shall apply to all cases pending in
any court over which trial has not begun as to the approval hereof." Petitioner argues
that:
a) The questioned provisions of the statute were introduced by the authors
thereof in bad faith as it was made to precisely suit the situation in which

petitioner's cases were in at the Sandiganbayan by restoring jurisdiction


thereof to it, thereby violating his right to procedural due process and the
equal protection clause of the Constitution. Further, from the way the
Sandiganbayan has foot-dragged for nine (9) months the resolution of a
pending incident involving the transfer of the cases to the Regional Trial
Court, the passage of the law may have been timed to overtake such
resolution to render the issue therein moot, and frustrate the exercise of
petitioner's vested rights under the old Sandiganbayan law (RA 7975)
b) Retroactive application of the law is plan from the fact that it was again
made to suit the peculiar circumstances in which petitioner's cases were
under, namely, that the trial had not yet commenced, as provided in
Section 7, to make certain that those cases will no longer be remanded to
the Quezon City Regional Trial Court, as the Sandiganbayan alone should
try them, thus making it an ex post facto legislation and a denial of the
right of petitioner as an accused in Criminal Case Nos. 23047-23057 to
procedural due process.
c) The title of the law is misleading in that it contains the aforesaid
"innocuous" provisions in Sections 4 and 7 which actually expands rather
than defines the old Sandiganbayan law (RA 7975), thereby violating the
one-title one-subject requirement for the passage of statutes under
Section 26 (1), Article VI of the Constitution. 17
For their part, the intervenors, in their petition-in-intervention, add that "while Republic
Act No. 8249 innocuously appears to have merely expanded the jurisdiction of the
Sandiganbayan, the introduction of Section 4 and 7 in said statute impressed upon it the
character of a class legislation and an ex-post facto statute intended to apply
specifically to the accused in the Kuratong Baleleng case pending before the
Sandiganbayan. 18 They further argued that if their case is tried before the
Sandiganbayan their right to procedural due process would be violated as they could no
longer avail of the two-tiered appeal to the Sandiganbayan, which they acquired under
R.A. 7975, before recourse to the Supreme Court.
Both the Office of the Ombudsman and the Solicitor-General filed separate pleadings in
support of the constitutionality of the challenged provisions of the law in question and
praying that both the petition and the petition-in-intervention be dismissed.
This Court then issued a Resolution 19 requiring the parties to file simultaneously within
a nonextendible period of ten (10) days from notice thereof additional memoranda on
the question of whether the subject amended informations filed a Criminal Case Nos.
23047-23057 sufficiently allege the commission by the accused therein of the crime
charged within the meaning Section 4 b of Republic Act No. 8249, so as to bring the
said cases within the exclusive original jurisdiction of the Sandiganbayan.

The parties, except for the Solicitor General who is representing the People of the
Philippines, filed the required supplemental memorandum within the nonextendible
reglementary period.
The established rule is that every law has in its favor the presumption of
constitutionality, and to justify its nullification there must be a clear and unequivocal
breach of the Constitution, not a doubtful and argumentative one. 20 The burden of
proving the invalidity of the law lies with those who challenge it. That burden, we regret
to say, was not convincingly discharged in the present case.
The creation of the Sandiganbayn was mandated in Section 5, Article XIII of the 1973
Constitution, which provides:
Sec. 5. The Batasang Pambansa shall create a special court, to be known
as Sandiganbayan, which shall have jurisdiction over criminal and civil
cases involving graft and corrupt practices and such other offenses
committed by public officers and employees including those in
government-owned or controlled corporations, in relation to their office as
may be determined by law.
The said special court is retained in the new (1987) Constitution under the following
provisions in Article XI, Section 4:
Sec. 4. The present anti-graft court known as the Sandiganbayan shall
continue to function and exercise its jurisdiction as now or hereafter may
be provided by law.
Pursuant to the constitutional mandate, Presidential Decree No. 1486 21 created the
Sandiganbayan. Thereafter, the following laws on the Sandiganbayan, in chronological
order, were enacted: P.D. No. 1606, 22 Section 20 of Batas Pambansa Blg. 123, 23 P.D.
No. 1860, 24 P.D. No. 1861, 25 R.A. No. 7975, 26 and R.A. No. 8249. 27 Under the latest
amendments introduced by Section 4 of R.A. No. 8249, the Sandiganbayan has
jurisdiction over the following cases:
Sec 4. Section 4 of the same decree [P.D. No. 1606, as amended] is
hereby further amended to read as follows:
Sec. 4. Jurisdiction The Sandiganbayan shall exercise exclusive
original jurisdiction in all cases involving:
a. Violations of Republic Act No. 3019, as amended, otherwise known as
the Anti-Graft and Corrupt Practices Act, Republic Act No. 1379, and
Chapter II, Section 2, Titile VII, Book II of the Revised Penal Code,
where one or more of the accused are officials occupying the following
positions in the government, whether in a permanent, acting or interim
capacity, at the time of the commission of the offense:

(1) Officials of the executive branch occupying the positions of regional


director and higher, otherwise classified as Grade "27" and higher, of the
Compensation and Position Classification Act of 1989 (Republic Act No.
6758), specifically including:
(a) Provincial governors, vice-governors, members of the
sangguniang panlalawigan, and provincial treasurers,
assessors, engineers, and other provincial department
heads;
(b) City mayors, vice-mayors, members of the sangguniang
panlungsod, city treasurers, assessors, engineers, and other
city department heads;
(c) Officials of the diplomatic service occupying the position
of consul and higher;
(d) Philippine Army and air force colonels, naval captains,
and all officers of higher rank;
(e) Officers of the Philippines National Police while
occupying the position of provincial director and those
holding the rank of senior superintendent or higher.
(f) City of provincial prosecutors and their assistants, and
officials and prosecutors in the Office of the Ombudsman
and special prosecutor;
(g) Presidents, directors or trustees or managers of
government-owned or controlled corporations, state
universities or educational institutions or foundations;
(2) Members of Congress or officials thereof classified as-Grade "27" and
up under the Compensation and Position Classification Act of 1989;
(3) Members of the judiciary without prejudice to the provisions of the
Constitution;
(4) Chairman and members of the Constitutional Commissions, without
prejudice to the provisions of the Constitution;
(5) All other national and local officials classified as Grade "27" or higher
under the Compensation and Position Classification Act of 1989.

b. Other offenses or felonies whether simple or complexed with other


crimes committed by the public officials and employees mentioned in
Subsection a of this section in relation to their office.
c. Civil and criminal cases filed pursuant to and connection with Executive
Orders Nos. 1,2, 14 and 14-A, issued in 1986.
In cases where none of the accused are occupying positions
corresponding to salary Grade "27" or higher, as prescribed in the said
Republic Act 6758, or military and PNP officers mentioned above,
exclusive original jurisdiction thereof shall be vested in the proper regional
trial court, metropolitan trial court, municipal trial court, and municipal
circuit trial court, as the case may be, pursuant to their jurisdictions as
privided in Batas Pambansa Blg. 129, as amended.
The Sandiganbayan shall exercise exclusive appellate jurisdiction over
final judgments, resolutions or orders of regional trial courts whether in the
exercise of their own original jurisdiction or of their appellate jurisdiction as
herein provided.
The Sandiganbayan shall have exclusive original jurisdiction over petitions
of the issuance of the writs of mandamus, prohibition, certiorari, habeas
corpus, injunctions, and other ancillary writs and processes in aid of its
appellate jurisdiction and over petitions of similar nature, including quo
warranto, arising or that may arise in cases filed or which may be filed
under Executive Order Nos. 1, 2, 14 and 14-A, issued in 1986: Provided,
That the jurisdiction over these petitions shall not be exclusive of the
Supreme Court.
The procedure prescribed in Batas Pambansa Blg. 129, as well as the
implementing rules that the Supreme Court has promulgated and may
hereafter promulgate, relative to appeals/petitions for review to the Court
of Appeals, shall apply to appeals and petitions for review filed with the
Sandiganbayan. In all cases elevated to the Sandiganbayan and from the
Sandiganbayan to the Supreme Court, the Office of the Ombudsman,
through its special prosecutor, shall represent the People of the
Philippines, except in cases filed pursuant to Executive Order Nos. 1, 2,
14, and 4-A, issued in 1986.
In case private individuals are charged as co-principals, accomplices or
accessories with the public officers or employee, including those
employed in government-owned or controlled corporations, they shall be
tried jointly with said public officers and employees in the proper courts
which shall exercise exclusive jurisdiction over them.
xxx xxx xxx (Emphasis supplied)

Sec. 7 of R.A. No. 8249 states:


Sec. 7. Transitory provision This act shall apply to all cases pending in
any court over which trial has not begun as of the approval hereof.
(Emphasis supplied)
The Sandiganbayan law prior to R.A. 8249 was R.A. 7975. Section 2 of R.A. 7975
provides:
Sec. 2. Section 4 of the same decree [Presidential Decree No. 1606, as
amended) is hereby further amended to read as follows:
Sec 4. Jurisdiction The Sandiganbayan shall exercise exclusive original
jurisdiction in all cases involving:
a. Violations of Republic Act No. 3019, as amended, otherwise known as
the Anti-Graft and Corrupt Practices Act, Republic Act No. 1379, and
Chapter II, Section 2, Title VII, Book II of the Revised Penal Code,
where one or more of the pricipal accused are afficials occupying the
following positions in the government, whether in a permanent, acting or
interim capacity, at the time of the commission of the offense:
(1) Officials of the executive branch occupying the positions of regional
director and higher, otherwise classified as Grade "27" and higher, of the
Compensation and Position Classification Act of 1989 (Republic Act No.
6758), specifically including:
(a) Provincial governors, vice-governors, members of the
sangguniang panlalawigan, and provincial treasurers,
assessors, engineer, and other provincial department heads;
(b) City mayors, vice-mayors, members of the sangguniang
panlungsod, city treasurers, assessors, engineers, and other
city department heads;
(c) Officials of the diplomatic service occupying the position
of consul and higher;
(d) Philippine Army and air force colonels, naval captains,
and all officers of higher rank;
(e) PNP chief superintendent and PNP officers of higher
rank;

(f) City and provincial prosecutors and their assistants, and


officials and prosecutors in the Office of the Ombudsman
and special prosecutor;
(g) Presidents, directors or trustees, or managers of
government-owned or controlled corporations, state
universities or educational institutions or foundations;
(2) Members of Congress or officials thereof classified as Grade "27" and
up under the Compensation and Position Classification Act of 1989;
(3) Members of the judiciary without prejudice to the provisions of the
Constitution;
(4) Chairman and members of the Constitutional Commissions, without
prejudice to the provisions of the Constitution;
(5) All other national and local officials classified as Grade "27" or higher
under the Compensation and Position Classification Act of 1989.
b. Other offenses or felonies committed by the public officials and
employees mentioned in Subsection a of this section in relation to their
office.
c. Civil and criminal cases files pursuant to and in connection with
Executive Order Nos. 1, 2, 14, and 4-A.
In cases where none of the principal accused are occupying positions
corresponding to salary Grade "27" or higher, as presribed in the said
Republic Act 6758, or PNP officers occupying the rank of superintendent
or higher, or their equivalent, exclusive jurisdiction thereof shall be vested
in the proper regional trial court, metropolitan trial court, municipal trial
court, and municipal circuit trial court, as the case may be, pursuant to
their respective jurisdictions as provided in Batas Pambansa Blg. 129.
The Sandiganbayan shall exercise exclusive appellate jurisdiction on
appelas from the final judgment, resolutions or orders of regular court
where all the accused are occupying positions lower than grade "27," or
not otherwise covered by the preceding enumeration.
xxx xxx xxx
In case private individuals are charged as co-principals, accomplices or
accessories with the public officers or employees, including those
employed in government-owned or controlled corporations, they shall be

tried jointly with said public officers and employees in the proper courts
which shall have exclusive jurisdiction over them.
xxx xxx xxx (Emphasis supplied)
Sec. 7 of R.A. No. 7975 reads:
Sec. 7. Upon the effectivity of this Act, all criminal cases in which trial has
not begun in the Sandiganbayan shall be referred to the proper courts.
Under paragraphs a and c, Section 4 of R.A. 8249, the word "principal" before the word
"accused" appearing in the above-quoted Section 2 (paragraphs a and c) of R.A. 7975,
was deleted. It is due to this deletion of the word "principal" that the parties herein are at
loggerheads over the jurisdiction of the Sandiganbayan. Petitioner and intervenors,
relying on R.A. 7975, argue that the Regional Trial Court, not the Sandiganbayan, has
jurisdiction over the subject criminal cases since none of the principal accused under
the amended information has the rank of Superintendent 28 or higher. On the other
hand, the Office of the Ombudsman, through the Special Prosecutor who is tasked to
represent the People before the Supreme Court except in certain cases, 29 contends
that the Sandiganbayan has jurisdiction pursuant to R.A. 8249.
A perusal of the aforequoted Section 4 of R.A. 8249 reveals that to fall under the
exclusive original jurisdiction of the Sandiganbayan, the following requisites must
concur: (1) the offense committed is a violation of (a) R.A. 3019, as amended (the AntiGraft and Corrupt Practices Act), (b) R.A. 1379 (the law on ill-gotten wealth), (c)
Chapter II, Section 2, Title VII, Book II of the Revised Penal Code (the law on
bribery), 30 (d) Executive Order Nos. 1, 2, 14, and 14-A, issued in 1986 (sequestration
cases), 31 or (e) other offenses or felonies whether simple or complexed with other
crimes; (2) the offender comitting the offenses in items (a), (b), (c) and (e) is a public
official or employee 32 holding any of the positions enumerated in paragraph a of
Section 4; and (3) the offense committed is in relation to the office.
Considering that herein petitioner and intervenors are being charged with murder which
is a felony punishable under Title VIII of the Revised Penal Code, the governing on the
jurisdictional offense is not paragraph a but paragraph b, Section 4 of R.A. 8249. This
paragraph b pertains to "other offenses or felonies whether simple or complexed with
other crimes committed by the public officials and employees mentioned in subsection a
of (Section 4, R.A. 8249) in relation to their office. "The phrase" other offenses or
felonies" is too broad as to include the crime of murder, provided it was committed in
relation to the accused's officials functions. Thus, under said paragraph b, what
determines the Sandiganbayan's jurisdiction is the official position or rank of the
offender that is, whether he is one of those public officers or employees enumerated
in paragraph a of Section 4. The offenses mentioned in pargraphs a, b and c of the
same Section 4 do not make any reference to the criminal participation of the accused
public officer as to whether he is charged as a principal, accomplice or accessory. In
enacting R.A. 8249, the Congress simply restored the original provisions of P.D. 1606

which does not mention the criminal participation of the public officer as a requisite to
determine the jurisdiction of the Sandiganbayan.
Petitioner and entervenors' posture that Section 4 and 7 of R.A. 8249 violate their right
to equal protection of the law 33 because its enactment was particularly directed only to
the Kuratong Baleleng cases in the Sandiganbayan, is a contention too shallow to
deserve merit. No concrete evidence and convincing argument were presented to
warrant a declaration of an act of the entire Congress and signed into law by the highest
officer of the co-equal executive department as unconstitutional. Every classification
made by law is presumed reasonable. Thus, the party who challenges the law must
present proof of arbitrariness. 34
It is an established precept in constitutional law that the guaranty of the equal protection
of the laws is not violated by a legislation based on reasonable classification. The
classification is reasonable and not arbitrary when there is concurrence of four
elements, namely:
(1) it must rest on substantial distinction;
(2) it must be germane to the purpose of the law;
(3) must not be limited to existing conditions only, and
(4) must apply equaly to all members of the same class, 35
all of which are present in this case.
The challengers of Sections 4 and 7 of R.A. 8249 failed to rebut the presumption of
constitutionality and reasonables of the questioned provisions. The classification
between those pending cases involving the concerned public officials whose trial has
not yet commence and whose cases could have been affected by the amendments of
the Sandiganbayan jurisdiction under R.A. 8249, as against those cases where trial had
already started as of the approval of the law, rests on substantial distinction that makes
real differences. 36 In the first instance, evidence against them were not yet presented,
whereas in the latter the parties had already submitted their respective proofs,
examined witnesses and presented documents. Since it is within the power of Congress
to define the jurisdiction of courts subject to the constitutional limitations, 37 it can be
reasonably anticipated that an alteration of that jurisdiction would necessarily affect
pending cases, which is why it has to privide for a remedy in the form of a transitory
provision. Thus, petitioner and intervenors cannot now claim that Sections 4 and 7
placed them under a different category from those similarly situated as them. Precisely,
paragraph a of Section 4 provides that it shall apply to "all case involving" certain public
officials and, under the transitory provision in Section 7, to "all cases pending in any
court." Contrary to petitioner and intervenors' argument, the law is not particularly
directed only to the Kuratong Baleleng cases. The transitory provision does not only
cover cases which are in the Sandiganbayan but also in "any court." It just happened

that Kuratong Baleleng cases are one of those affected by the law. Moreover, those
cases where trial had already begun are not affected by the transitory provision under
Section 7 of the new law (R.A. 8249).
In their futile attempt to have said sections nullified, heavy reliance is premised on what
is perceived as bad faith on the part of a Senator and two Justices of the
Sandiganbaya 38 for their participation in the passage of the said provisions. In
particular, it is stressed that the Senator had expressed strong sentiments against those
officials involved in the Kuratong Baleleng cases during the hearings conducted on the
matter by the committee headed by the Senator. Petitioner further contends that the
legislature is biased against him as he claims to have been selected from among the 67
million other Filipinos as the object of the deletion of the word "principal" in paragraph a,
Section 4 of P.D. 1606, as amended, and of the transitory provision of R.A. 8249. 39 R.A
8249, while still a bill, was acted, deliberated, considered by 23 other Senators and by
about 250 Representatives, and was separately approved by the Senate and House of
Representatives and, finally, by the President of the Philippines.
On the perceived bias that the Sandiganbayan Justices allegedly had against petitioner
during the committe hearings, the same would not constitute sufficient justification to
nullify an otherwise valid law. Their presence and participation in the legislative hearings
was deemed necessary by Congress since the matter before the committee involves
the graft court of which one is the head of the Sandiganbayan and the other a member
thereof. The Congress, in its plenary legislative powers, is particularly empowered by
the Constitution to invite persons to appear before it whenever it decides to conduct
inquiries in aid of legislation. 40
Petitioner and entervenors further further argued that the retroactive application of R.A.
8249 to the Kuratong Baleleng cases constitutes an ex post facto law 41 for they are
deprived of their right to procedural due process as they can no longer avail of the twotiered appeal which they had allegedly acquired under R.A. 7975.
Again, this contention is erroneous. There is nothing ex post facto in R.A. 8249.
In Calder v. Bull, 42 an ex post facto law is one
(a) which makes an act done criminal before the passing of
the law and which was innocent when committed, and
punishes such action; or
(b) which aggravates a crime or makes it greater than when
it was committed; or
(c) which changes the punishment and inflicts a greater
punishment than the law annexed to the crime when it was
committed.

(d) which alters the legal rules of evidence and recieves less
or different testimony that the law required at the time of the
commission of the offense on order to convict the
defendant. 43
(e) Every law which, in relation to the offense or its
consequences, alters the situation of a person to his
disadvantage. 44
This Court added two more to the list, namely:
(f) that which assumes to regulate civil rights and remedies
only but in effect imposes a penalty or deprivation of a right
which when done was lawful;
(g) deprives a person accussed of crime of some lawful
protection to which he has become entitled, such as the
protection of a former conviction or acquittal, or a
proclamation of a amnesty. 45
Ex post facto law, generally, prohibits retrospectivity of penal laws. 46 R.A. 8249 is not
penal law. It is a substantive law on jurisdiction which is not penal in character. Penal
laws are those acts of the Legislature which prohibit certain acts and establish penalties
for their violations; 47 or those that define crimes, treat of their nature, and provide dor
their punishment. 48 R.A 7975, which amended P.D. 1606 as regards the
Sandiganbayan's jurisdiction, its mode of appeal and other procedural matters, has
been declared by the Court as not a penal law, but clearly a procedural statute, i.e. one
which prescribes rules of procedure by which courts applying laws of all kinds can
properly administer justice. 49 Not being a penal law, the retroactive application of R.A.
8249 cannot be challenged as unconstitutional.
Petitioner's and entervenors' contention that their right to a two-tiered appeal which they
acquired under R.A. 7975 has been diluted by the enactment of R.A. 8249, is incorrect.
The same contention has already been rejected by the court several
times 50 considering that the right to appeal is not a natural right but statutory in nature
that can be regulated by law. The mode of procedure provided for in the statutory right
of appeal is not included in the prohibition against ex post facto laws. 51 R.A. 8249
pertains only to matters of procedure, and being merely an amendatory statute it does
not partake the nature of an ex post facto law. It does not mete out a penalty and,
therefore, does not come within the prohibition. 52 Moreover, the law did not alter the
rules of evidence or the mode of trial. 53 It has been ruled that adjective statutes may be
made applicable to actions pending and unresolved at the time of their passage. 54
In any case; R.A. 8249 has preserved the accused's right to appeal to the Supreme
Court to review questions of law. 55 On the removal of the intermediate review of facts,

the Supreme Court still has the power of review to determine if he presumption of
innocence has been convincing overcome. 56
Another point. The challenged law does not violate the one-title-one-subject provision of
the Constitution. Much emphasis is placed on the wording in the title of the law that it
"defines" the Sandiganbayan jurisdiction when what it allegedly does is to "expand" its
jurisdiction. The expantion in the jurisdiction of the Sandiganbayan, if it can be
considered as such, does not have to be expressly stated in the title of the law because
such is the necessary consequence of the amendments. The requirement that every bill
must only have one subject expressed in the title 57 is satisfied if the title is
comprehensive enough, as in this case, to include subjects related to the general
purpose which the statute seeks to achieve. 58 Such rule is liberally interpreted and
should be given a practical rather than a technical construction. There is here sufficient
compliance with such requirement, since the title of R.A. 8249 expresses the general
subject (involving the jurisdiction of the Sandiganbayan and the amendment of P.D.
1606, as amended) and all the provisions of the law are germane to that general
subject. 59 The Congress, in employing the word "define" in the title of the law, acted
within its power since Section 2, Article VIII of the Constitution itself empowers the
legislative body to "define, prescribe, and apportion the jurisdiction of various courts. 60
There being no unconstitutional infirmity in both the subject amendatory provision of
Section 4 and the retroactive procedural application of the law as provided in Section 7
of R.A. No. 8249, we shall now determine whether under the allegations in the
Informations, it is the Sandiganbayan or Regional Trial Court which has jurisdictions
over the multiple murder case against herein petitioner and entervenors.
The jurisdiction of a court is defined by the Constitution or statute. The elements of that
definition must appear in the complaint or information so as to ascertain which court has
jurisdiction over a case. Hence the elementary rule that the jurisdiction of a court is
determined by the allegations in the complaint or informations, 61 and not by the
evidence presented by the parties at the trial. 62
As stated earlier, the multiple murder charge against petitioner and intervenors falls
under Section 4 [paragraph b] of R.A. 8249. Section 4 requires that the offense charged
must be committed by the offender in relation to his office in order for the
Sandiganbayan to have jurisdiction over it. 63 This jurisdictional requirement is in
accordance with Section 5, Article XIII of the 1973 Constitution which mandated that the
Sandiganbayan shall have jurisdiction over criminal cases committed by the public
officers and employees, including those in goverment-owned or controlled corporations,
"in relation to their office as may be determined by law." This constitutional mandate
was reiterated in the new (1987) Constitution when it declared in Section 4 thereof that
the Sandiganbayan shall continue to function and exercise its jurisdiction as now or
hereafter may be provided by law.
The remaining question to be resolved then is whether the offense of multiple murder
was committed in relation to the office of the accussed PNP officers.

In People vs. Montejo, 64 we held that an offense is said to have been committed in
relation to the office if it (the offense) is "intimately connected" with the office of the
offender and perpetrated while he was in the performance of his official
functions. 65 This intimate relation between the offense charged and the discharge of
official duties "must be alleged in the informations." 66
As to how the offense charged be stated in the informations, Section 9, Rule 110 of the
Revised Rules of Court mandates:
Sec. 9 Couse of accusation The acts or omissions complied of as
constituting the offense must be stated in ordinary and concise
language without repetition not necessarily in the terms of the statute
defining the offense, but in such from as is sufficient to enable a person of
common understanding to know what offense is intended to be charged,
and enable the court to pronounce proper judgment. (Emphasis supplied)
As early as 1954 we pronounced that "the factor that characterizes the charge is the
actual recital of the facts." 67The real nature of the criminal charge is determined not
from the caption or preamble of the informations nor from the specification of the
provision of law alleged to have been violated, they being conclusions of law, but by the
actual recital of facts in the complaint or information. 68
The noble object or written accusations cannot be overemphasized. This was explained
in U.S. v. Karelsen: 69
The object of this written accusations was First; To furnish the accused
with such a descretion of the charge against him as will enable him to
make his defense and second to avail himself of his conviction or acquittal
for protection against a further prosecution for the same cause and third,
to inform the court of the facts alleged so that it may decide whether they
are sufficient in law to support a conviction if one should be had. In order
that the requirement may be satisfied, facts must be stated, not
conclusions of law. Every crime is made up of certain acts and
intent these must be set forth in the complaint with reasonable
particularly of time, place, names (plaintiff and defendant) and
circumstances. In short, the complaint must contain a specific
allegation of every fact andcircumstance necessary to constitute the crime
charged. (Emphasis supplied)
It is essential, therefore, that the accused be informed of the facts that are imputed to
him as "he is presumed to have no indefendent knowledge of the facts that constitute
the offense." 70
Applying these legal principles and doctrines to the present case, we find the amended
informations for murder against herein petitioner and intervenors wanting of specific

factual averments to show the intimate relation/connection between the offense charged
and the discharge of official function of the offenders.
In the present case, one of the eleven (11) amended informations 71 for murder reads:
AMENDED INFORMATIONS
The undersigned Special Prosecution Officer III. Office of the Ombudsman
hereby accuses CHIEF INSP. MICHAEL RAY AQUINO, CHIEF INSP.
ERWIN T. VILLACORTE, SENIOR INSP. JOSELITO T. ESQUIVEL,
INSP. RICARDO G. DANDAN, SPO4 VICENTE P. ARNADO, SPO4
ROBERTO F. LANGCAUON, SPO2 VIRGILIO V. PARAGAS, SPO2
ROLANDO R. JIMENEZ, SPO1 WILFREDO C. CUARTERO, SPO1
ROBERTO O. AGBALOG, SPO1 OSMUNDO B. CARINO, CHIEF SUPT.
JEWEL F. CANSON, CHIEF SUPT. ROMEO M. ACOP, CHIEF SUPT.
PANFILO M. LACSON, SENIOR SUPT. FRANCISCO G. ZUBIA JR.,
SUPT. ALMARIO A. HILARIO, CHIEF INSP. CESAR O. MANCAO III,
CHIEF INSP. GIL L. MENESES, SENIOR INSP. GLENN DUMLAO,
SENIOR INSP. ROLANDO ANDUYAN, INSP. CEASAR TANNAGAN,
SPO3 WILLY NUAS, SPO3 CICERO S. BACOLOD, SPO2 NORBERTO
LASAGA, PO2 LEONARDO GLORIA, and PO2 ALEJANDRO G.
LIWANAG of the crime of Murder as defined and penalize under Article
248 of the Revised Penal Code committed as follows
That on or about May 18, 1995 in Mariano Marcos Avenue, Quezon City
Philippines and within the jurisdiction of his Honorable Court, the accused
CHIEF INSP. MICHAEL RAY AQUINO, CHIEF INSP. ERWIN T.
VILLACORTE, SENIOR INSP. JOSELITO T. ESQUIVEL, INSP.
RICARDO G. DANDAN, SPO4 VICENTE ARNADO, SPO4 ROBERTO F.
LANGCAUON, SPO2 VIRGILIO V. PARAGAS, SPO2 ROLANDO R.
JIMENEZ, SPO1 WILFREDO C. CUARTERO, SPO1 ROBERTO O.
AGBALOG, and SPO1 OSMUNDO B. CARINO, all taking advantage of
their public and official positions as officers and members of the Philippine
National Police and committing the acts herein alleged in relation to their
public office, conspiring with intent to kill and using firearms with treachery
evident premeditation and taking advantage of their superior strenghts did
then and there willfully unlawfully and feloniously shoot JOEL AMORA,
thereby inflicting upon the latter mortal wounds which caused his
instantaneous death to the damage and prejudice of the heirs of the said
victim.
That accused CHIEF SUPT. JEWEL F. CANSON, CHIEF SUPT. ROMOE
M. ACOP, CHIEF SUPT. PANFILO M. LACSON, SENIOR SUPT.
FRANCISCO G. ZUBIAM JR., SUPT. ALMARIO A. HILARIO, CHIEF
INSP. CESAR O. MANCAO II, CHIEF INSP. GIL L. MENESES, SENIOR
INSP. GLENN DUMLAO, SENIOR INSP. ROLANDO ANDUYAN, INSP.

CEASAR TANNAGAN, SPO3 WILLY NUAS, SPO3 CICERO S.


BACOLOD, PO2 ALEJANDRO G. LIWANAG committing the acts in
relation to office as officers and members of the Philippine National Police
are charged herein as accessories after-the-fact for concealing the crime
herein above alleged by among others falsely representing that there
where no arrest made during the read conducted by the accused herein at
Superville Subdivision, Paranaque, Metro Manila on or about the early
dawn of May 18, 1995.
CONTRARY LAW.
While the above-quoted information states that the above-named principal accused
committed the crime of murder "in relation to thier public office, there is, however, no
specific allegation of facts that the shooting of the victim by the said principal accused
was intimately related to the discharge of their official duties as police officers. Likewise,
the amended information does not indicate that the said accused arrested and
investigated the victim and then killed the latter while in their custody.
Even the allegations concerning the criminal participation of herein petitioner and
intevenors as among the accessories after-the-facts, the amended information is vague
on this. It is alleged therein that the said accessories concelead "the crime herein-above
alleged by, among others, falsely representing that there were no arrests made during
the raid conducted by the accused herein at Superville Subdivision, Paranaque Metro
Manila, on or about the early dawn of May 18, 1995." The sudden mention of the
"arrests made during the raid conducted by the accused" surprises the reader. There is
no indication in the amended information that the victim was one of those arrested by
the accused during the "raid." Worse, the raid and arrests were allegedly conducted "at
Superville Subdivision, Paranaque, Metro Manila" but, as alleged in the immediately
preceding paragraph of the amended information, the shooting of the victim by the
principal accused occurred in Mariano Marcos Avenue, Quezon City." How the raid,
arrests and shooting happened in the two places far away from each other is puzzling.
Again, while there is the allegation in the amended information that the said accessories
committed the offense "in relation to office as officers and members of the (PNP)," we,
however, do not see the intimate connection between the offense charged and the
accused's official functions, which, as earlier discussed, is an essential element in
determining the jurisdiction of the Sandiganbayan.
The stringent requirement that the charge be set forth with such particularly as will
reasonably indicate the exact offense which the accused is alleged to have committed
in relation to his office was, sad to say, not satisfied. We believe that the mere allegation
in the amended information that the offense was committed by the accused public
officer in relation to his office is not sufficient. That phrase is merely a conclusion
between of law, not a factual avernment that would show the close intimacy between
the offense charged and the discharge of the accused's official duties.

In People vs. Magallanes, 72 where the jurisdiction between the Regional Trial Court and
the Sandiganbayan was at issue, we ruled:
It is an elementary rule that jurisdiction is determined by the allegations in
the complaint or information and not by the result of evidence after trial.
In (People vs) Montejo (108 Phil 613 (1960), where the amended
information alleged
Leroy S. Brown City Mayor of Basilan City, as such, has
organized groups of police patrol and civilian commandoes
consisting of regular policeman and . . . special policemen
appointed and provided by him with pistols and higher power
guns and then established a camp . . . at Tipo-tipo which is
under his command . . . supervision and control where his
co-defendants were stationed entertained criminal
complaints and conducted the corresponding investigations
as well as assumed the authority to arrest and detain person
without due process of law and without bringing them to the
proper court, and that in line with this set-up established by
said Mayor of Basilan City as such, and acting upon his
orders his co-defendants arrested and maltreated Awalin
Tebag who denied in consequence thereof.
we held that the offense charged was committed in relation to the office of
the accused because it was perpetreated while they were in the
performance, though improper or irregular of their official functions and
would not have been committed had they not held their office, besides, the
accused had no personal motive in committing the crime thus, there was
an intimate connection between the offense and the office of the accused.
Unlike in Montejo the informations in Criminal Cases Nos. 15562 and
15563 in the court below do not indicate that the accused arrested and
investigated the victims and then killed the latter in the course of the
investigation. The informations merely allege that the accused for the
purpose of extracting or extortin the sum of P353,000.00 abducted,
kidnapped and detained the two victims, and failing in their common
purpose they shot; and killed the said victims. For the purpose of
determining jurisdiction, it is these allegations that shall control, and not
the evidence presented by the prosecution at the trial.
In the aforecited case of People vs. Montejo, it is noteworthy that the phrase committed
in relation to public office "does not appear in the information, which only signifies that
the said phrase is not what determines the jurisdiction of the Sandiganbayan. What is
controlling is the specific factual allegations in the information that would indicate the
close intimacy between the discharge of the accused's official duties and the

commission of the offense charged, in order to qualify the crime as having been
committed in relation to public office.
Consequently, for failure to show in the amended informations that the charge of murder
was intimately connected with the discharge of official functions of the accused PNP
officers, the offense charged in the subject criminal cases is plain murder and,
therefore, within the exclusive original jurisdiction of the Regional Trial Court,73 not the
Sandiganbayan.
WHEREFORE, the constitutionality of Sections 4 and 7 of R.A. 8249 is hereby
sustained. The Addendum to the March 5, 1997 Resolution of the Sandiganbayan is
REVERSED. The Sandiganbayan is hereby directed to transfer Criminal Cases Nos.
23047 to 23057 (for multiple murder) to the Regional Trial Court of Quezon City which
has exclusive original jurisdiction over the said cases.1wphi1.nt
SO ORDERED.

G.R. No. 141710

March 3, 2004

EVELYN V. RODRIGUEZ, AND ANDRES ABONITA, JR., petitioners,


vs.
SANDIGANBAYAN, AND PEOPLE OF THE PHILIPPINES, respondents.

DECISION

CARPIO-MORALES, J.:
The January 17, 2000 three separate Orders of the Sandiganbayan denying petitioners
motion to quash the second amended information,1 denying the motion to defer
arraignment,2 and entering a plea of "not guilty" for petitioners in light of their refusal to
plead to the information,3 are assailed in the present petition for certiorari.
The antecedents of the case are as follows:
On September 24, 1996, acting upon an information that rampant illegal logging
activities have been going on in different areas of Taytay, Palawan, a joint team
composed of the Economic Intelligence and Investigation Bureau (EIIB), the Provincial
Environment and Natural Resources Office (PENRO), the Philippine National Police
(PNP) Tiniguiban Command, the Bantay Palawan, and the Philippine Marines
confiscated freshly cut/processed ipil lumber at Sitio Maypa, Barangay Pancol, Taytay.
The cutting and sawing of the lumber, which were alleged to have been done under the
supervision of Pancol Barangay Captain Pedro Samaniego upon orders of herein
petitioner Mayor Evelyn Rodriguez and Association of Barangay Captains President
Roberto Rodriguez, were without proper permit or license.
Due to the unavailability of trucks to haul all the lumber to Puerto Princesa for
safekeeping, some were hauled inside the Rural Agriculture Center (RAC) Compound
of Taytay and left under the custody of 2nd Lt. Ernan Libao.
On September 25, 1997, Barangay Captain Rodriguez appeared at the RAC Compound
demanding the release of the lumber by presenting a letter-request addressed to the
CENRO to salvage old cut timber, duly indorsed by Mayor Rodriguez. As the request
did not bear the approval of the CENRO, it was denied.
On October 5, 1997, Pancol Barangay Captain Pedro Samaniego and the other herein
petitioner, Igang Barangay Captain Andres Abonita, Jr., went to the RAC Compound
upon orders of Mayor Rodriguez to haul the lumber to the Municipal Hall, but the officer-

in-charge refused to release the same without the advice of EIIB authorities. On even
date, acting upon the orders of Mayor Rodriguez, Barangay Captain Abonita returned to
the RAC Compound accompanied by two fully armed policemen who then and there
forcibly took possession, hauled, and transferred the lumber to the Municipal Hall of
Taytay.
On November 7, 1996, Enrique A. Cuyos, Sr. of the EIIB, Region IV-A, Palawan filed
complaints for robbery4 and violation of Section 1(b), P.D. No. 18295 (Decree Penalizing
Obstruction of Apprehension and Prosecution of Criminal Offenders) against petitioners
Mayor Rodriguez and Barangay Captain Abonita before the Provincial Prosecution
Office of Palawan.
By Resolution6 of February 18, 1997, the Deputized Ombudsman Investigator
recommended the filing of an information against petitioners for violation of Section 1(b),
P.D. No. 1829,7 and the forwarding of the records of the case to the Office of the
Ombudsman-Luzon for review and further proceedings, petitioner Mayor Rodriguez
being a public officer and the charge against her being work-connected.
Following its review of the case, the Office of the Deputy Ombudsman-Luzon, by a Joint
Review Action8 of October 19, 1998, resolved to, as it did file an information 9 for
violation of Section 1(b) P.D. 1829 on December 8, 1998 against petitioners before the
Sandiganbayan, docketed as Criminal Case No. 25065.
A warrant of arrest10 was accordingly issued against petitioners on December 14, 1998.
Before the 1st Division of the Sandiganbayan, petitioner Mayor Rodriguez voluntarily
surrendered and posted a cash bond on January 4, 1999, 11 as did Barangay Captain
Abonita on January 29, 1999.12
On January 27, 1999, petitioners filed a Motion to Defer Arraignment, 13 they having filed
on even date a Motion to Quash14. By Order15 of January 29, 1999, the Sandiganbayan
reset the arraignment to February 26, 1999.
During the scheduled arraignment on February 26, 1999, the special prosecutor moved
to defer the arraignment as recommended changes in the information were not yet
acted upon by the Ombudsman. Without objection from petitioners counsel, the
arraignment was reset16 to April 8, 1999.
In the meantime, the special prosecutor filed on April 6, 1999 an opposition 17 to
petitioners Motion to Quash.
Subsequently, the Sandiganbayan, acting upon a Motion to Admit Information 18 which
was filed by the special prosecutor, admitted the amended information by Order19 of
April 8, 1999.
Petitioners filed on April 26, 1999 a Motion to Quash20 the amended information, to
which motion the special prosecutor filed a comment/opposition21 on June 9, 1999,

explaining that the belated filing thereof was due to the transfer of the records of the
Office of the Special Prosecutor to its new office at the Sandiganbayan Centennial
Building in Quezon City.
Thereafter or on June 28, 1999, the special prosecutor filed another Ex-parte Motion to
Admit Amended Information22 which was set for hearing on November 25, 1999. The
scheduled hearing on November 25, 1999 was, however, cancelled and reset 23 to
December 3, 1999 upon urgent motion by petitioners counsel upon the ground that on
said date, he needed to appear before the Metropolitan Trial Court of Mandaluyong.
By Order24 of December 3, 1999, the Sandiganbayan granted the motion to admit
amended information, denied the motion to quash the amended information, and
ordered the arraignment of petitioners on January 17, 2000.
On January 14, 2000, petitioners filed a Motion to Quash/Dismiss25 the second
amended information.
During the scheduled arraignment of petitioners on January 17, 2000, the
Sandiganbayan issued in open court the assailed separate orders denying petitioners
motion to quash the second amended information,26 denying the motion to defer
arraignment,27 and entering a plea of "not guilty" for both accused28 herein petitioners,
which orders petitioners allege have been rendered with grave abuse of discretion.
Petitioners argue that the pendency of the preliminary investigation of the case which
dragged for almost three years is unreasonable or unjustifiable and violates their
constitutional rights as accused to due process,29 they citing Tatad v.
Sandiganbayan.30 They add that the repeated and ex-parte amendment of the
information by the Ombudsman resulted to inordinate delay in bringing the case to trial,
which is a ground for dismissal of the information under Section 13, in relation to
Section 7 of R.A. 8493 (The Speedy Trial Act of 1998).31
Petitioners likewise argue that the simultaneous filing by the Ombudsman of two
informations against them, one before the Sandiganbayan (Criminal Case No. 25065),
and the other before the Regional Trial Court in Puerto Princesa City (Criminal Case
No. 14959), involving the same subject matter constitutes forum shopping which is
expressly prohibited under the Supreme Court Revised Circular No. 28-91 directing the
summary dismissal of multiple complaints or charges, and necessarily places both of
them in "double danger of conviction and punishment for the same offense." 32
Petitioners additionally question the jurisdiction of the Sandiganbayan, they arguing that
they are not tasked with the enforcement and implementation of P.D. No. 705
(REVISED FORESTRY CODE OF THE PHILIPPINES) as neither of them are law
enforcement officers or prosecutors but are mere executive officials of their respective
local government units with entirely different official functions and, as such, the
accusation against them is not in relation to their office.33 Petitioners thus conclude that
the Sandiganbayan has no jurisdiction over the subject matter of the case, as Section 4

of R.A. 8249 limits the jurisdiction of the Sandiganbayan to those offenses defined and
penalized in Chapter II, Section 2, Title VII, Book II of the Revised Penal Code. 34
The petition fails.
Tatad v. Sandiganbayan35 cited by petitioners has a different factual setting from the
present case. The cases against Tatad remained dormant for almost three years,
drawing this Court to dismiss them in light of the following observations: political
motivation played a vital role in activating and propelling the prosecutorial process;
there was a blatant departure from established procedures prescribed by law for the
conduct of a preliminary investigation; and the long delay in resolving the preliminary
investigation could not be justified on the basis of the record. 36
From the records of the case at bar, it is gathered that the Provincial Prosecutor of
Palawan took only three months, from November 7, 1996 to February 18, 1997, to come
up with its resolution finding probable cause against petitioners. The Deputy
Ombudsman for Luzon took eight months to review the case and come up with the joint
review action on October 19, 1998. On the other hand, the Office of the Ombudsman
acted on the case for around two months. Considering that the records were passed
upon by three offices, the period of preliminary investigation, which did not exceed two
years, cannot be deemed to have violated petitioners constitutionally guaranteed rights
to procedural due process and to a speedy disposition of cases.
As Ty-Dazo v. Sandiganbayan37 instructs:
The right to a speedy disposition of cases, like the right to a speedy trial, is
deemed violated only when the proceedings [are] attended by vexatious,
capricious, and oppressive delays; or when the unjustified postponements of the
trial are asked for and secured, or when without cause or unjustifiable motive, a
long period of time is allowed to elapse without the party having his case tried. In
the determination of whether or not the right has been violated, the factors that
maybe considered and balanced are: the length of the delay, the reasons for
such delay, the assertion or failure to assert such right by the accused, and the
prejudice caused by the delay.
A mere mathematical reckoning of the time involved, therefore, would not be
sufficient. In the application of the constitutional guarantee of the right to speedy
disposition of cases, particular regard must also be taken of the facts and
circumstances peculiar to each case.38
Parenthetically, as reflected in the following observation of the Sandiganbayan,
petitioners themselves contributed to the delay, thus:
With respect to the alleged delay of the filing of the Information and for the delay
in finally getting the case ready for arraignment, Prosecutor Evelyn T. Lucero has
stated that, to a certain extent, the claim is valid although the delay is caused not

unreasonably but because of the exercise of the right of the accused to


determine whether or not they could be charged under the Information for which
they have filed Motions to Quash; thus, the delay cannot be considered
unreasonable nor the grounds for setting aside the amended Information as it
now stands.39 (Underscoring supplied)
The rule is well settled that the right to a speedy disposition of cases, like the right to a
speedy trial, is deemed violated only when the proceeding is attended by vexatious,
capricious, and oppressive delay.40
In further pressing for the dismissal of the case, petitioners cite Sections 741 and 1342 of
R.A. 8493, averring that the unreasonable delay in bringing them to arraignment is a
ground for the dismissal of the case, they having been arraigned only on January 17,
2000, after several and repeated amendments of the information.
The records show, however, that it was on account of petitioners continuous filing of
motions that the arraignment was deferred.
Under Section 2 of Supreme Court Circular No. 38-98, Implementing Rules for
R.A.8493, the pendency of petitioners motion to quash takes the case out from the time
limit for arraignment (and pre-trial) provided under Section 2 of said law.
Sec. 2. Time Limit for Arraignment and Pre-trial. - The arraignment, and the pretrial if the accused pleads not guilty to the crime charged, shall be held within
thirty (30) days from the date the court acquires jurisdiction over the person of
the accused. The period of the pendency of a motion to quash, or for a bill of
particulars, or other cause justifying suspension of arraignment shall be
excluded. (Underscoring supplied)
On the claim of petitioners that the Sandiganbayan should be faulted for granting the
repeated amendments of the information by the Ombudsman, suffice it to state that an
information may be amended in form or in substance without leave of court at any time
before an accused enters his plea.43
In another attempt at having the case dismissed, petitioners aver that the Ombudsman
committed forum shopping by filing the same information before the Sandiganbayan
and the Regional Trial Court of Puerto Princesa, Palawan in violation of Supreme Court
Circular No. 28-91 (Additional Requisites for Petitions filed with the Supreme Court and
the Court of Appeals to Prevent Forum Shopping or Multiple Filing of Petitions and
Complaints).
Assuming arguendo that indeed the same information for violation of Section 1(b) of
P.D. 1829 was also filed before the Regional Trial Court of Puerto Princesa, Palawan,
then as the People by the Office of the Ombudsman through the Special Prosecutor
contends in its Memorandum, "since the Information in Criminal Case No. 25065 was
filed with the Sandiganbayan on December 8, 1988, while the information before the

regional Trial Court was allegedly filed on February 24, 1999, then, if there is any case
to be dismissed for forum shopping, that case should be the one before the Regional
trial Court, as it was the second action filed."44
Petitioners further assail the jurisdiction of Sandiganbayan over the offense for which
they were indicted.
Lamentably, petitioners may well have been confused regarding the charge against
them for instead of showing that the offense with which they were charged - violation of
Section 1(b) of P.D. 1829 (obstruction of justice) - is not in relation to their office, they
argued that they are not tasked with the enforcement and implementation of P.D. No.
705 the offense subject of the investigation which petitioners allegedly obstructed or
interfered with.
Petitioners are charged not for violation of P.D. 705 but of P.D. 1829, hence, petitioners
argument that the act complained of was not done in relation to their office to take the
case out of the jurisdiction of the Sandiganbayan does not lie.
At all events, Republic Act 8249, which amended Presidential Decree No. 1606,
provides that as long as one (or more) of the accused is an official of the executive
branch occupying position otherwise classified as Grade 27 and higher of the
Compensation and Position Classification Act of 1989,45 the Sandiganbayan
exercisesexclusive original jurisdiction over offenses or felonies committed by public
officials whether simple or complexed with other crimes committed by the public
officials and employees in relation to their office.46(Emphasis and underscoring
supplied)
For purposes of vesting jurisdiction with the Sandiganbayan, the crux of the issue is
whether petitioner Mayor Rodriguez, who holds a position of "Grade 27" under the Local
Government Code of 199147, committed the offense charged in relation to her office.
In Montilla v. Hilario48, this Court laid down the principle that for an offense to be
committed in relation to the office, the relation between the crime and the office must be
direct and not accidental, in that in the legal sense, the offense can not exist without the
office.49
As an exception to Montilla, this Court, in People v. Montejo,50 held that although public
office is not an element of an offense charged, as long as the offense charged in the
information is intimately connected with the office and is alleged to have been
perpetrated while the accused was in the performance, though improper or irregular, of
his official functions, there being no personal motive to commit the crime and had the
accused would not have committed it had he not held the aforesaid office, 51 the accused
is held to have been indicted for "an offense committed in relation" to his office.
Applying the exception laid down in Montejo, this Court in Cunanan v. Arceo,52 held that
although public office is not an element of the crime of murder as it may be committed

by any person, whether a public officer or a private citizen, the circumstances under
which the therein petitioner, who was a member of the Philippine National Police, shot
and killed the victim in the course of trying to restore local public order, bring the therein
petitioners case squarely within the meaning of an "offense committed in relation to the
[accuseds] public office."53
In the present case, public office is not an essential element of the offense of
obstruction of justice under Section 1(b) of P.D. 1829. The circumstances surrounding
the commission of the offense alleged to have been committed by petitioner Rodriguez
are such, however, that the offense may not have been committed had said petitioner
not held the office of the mayor. As found during the preliminary investigation, petitioner
Rodriguez, in the course of her duty as Mayor, who is tasked to exercise general and
operational control and supervision over the local police forces54, used her influence,
authority and office to call and command members of the municipal police of Taytay to
haul and transfer the lumber which was still subject of an investigation for violation of
P.D. 705.
The joint-counter affidavits55 signed by petitioners during the preliminary investigation
quoted the letter of petitioner Mayor Rodriguez to the municipal police officers, viz:
To SPO1 Juanito G. Gan and
PO2 Emmanuel Nangit;
PNP Members of Taytay
Municipal Police Office,
Taytay Palawan
Upon receipt of this order you are hereby directed to proceed to Sitio Igang,
Poblacion Taytay, Palawan, at the compound of the Rural Agricultural Center[, in
order to] haul the flitches ipil lumber intended for the projects of the Municipal
Government of Taytay and to turn over to the DENR office of Taytay, Palawan.
For immediate strict compliance. 56
Reference to this above-quoted letter of petitioner Rodriguez is found in both the
Resolution57 of the Deputized Ombudsman Investigator of the Provincial Prosecution
Office of Palawan and the Joint Review Action58 of the Graft Investigation Officer-Luzon.
What determines the jurisdiction of a court is the nature of the action pleaded as
appearing from the allegations in the information59. The averment in the information that
petitioner Rodriguez, as municipal mayor, took advantage of her office and caused the
hauling of the lumber to the municipal hall to obstruct the investigation of the case for
violation of P.D. 705 effectively vested jurisdiction over the offense on the
Sandiganbayan. Thus, the amended information reads:
AMENDED INFORMATION

The undersigned Special Prosecution Officer II, Office of the Special Prosecutor,
hereby accuses EVELYN VILLABERT RODRIGUEZ and ANDREWS BONITA
JR. of Violation of Section 1(b), Presidential Decree No. 1829 committed as
follows:
"That on or about October 5, 1996, at Sitio Igang, Barangay Poblacion,
Municipality of Taytay, Province of Palawan, and within the jurisdiction of this
Honorable Court, accused EVELYN VILLABERT RODRIGUEZ and ANDREWS
ABONITA JR., both public officers, being the Municipal Mayor and Barangay
Captain of Barangay Igang of the same municipality, respectively, committing the
offense in relation to their office and taking advantage of the same, confederating
and conspiring with each other enter the compound of the Rural Agricultural
Center (RAC) at Sitio Igang, Poblacion, Taytay, Palawan and while inside with
force, intimidation and against the will of the one officially detailed thereat, 2LT.
ERNAN O. LIBAO, did then and there willfully, feloniously, unlawfully, knowingly
and forcibly haul 93 pieces or 2.577.32 board feet of assorted dimensions of ipil
lumber, that were officially confiscated by a joint team of EIIB, PENRO, BANTAY
PALAWAN, PNP-TINIGUIBAN COMMAND and PHILIPPINE MARINES,
stockpiled inside the RAC for safekeeping while waiting for available
transportation to haul the same to Puerto Princesa City, and brought the same
ipil lumber within the compound of the Municipal Hall of Taytay, with the
primordial purpose of suppressing or concealing the said ipil lumber as evidence
in the investigation of the case for violation of P.D. 705, as amended.60 (Italics
supplied)
There being no flaw or infirmity then in the amended information, respondent
Sandiganbayan did not commit grave abuse of discretion amounting to lack or excess of
jurisdiction in issuing the order of January 17, 2000, denying petitioners motion to
quash.
The orders of the Sandiganbayan denying the motion to defer arraignment and entering
a plea of not guilty for petitioners in light of their refusal to plead were accordingly
rendered without any grave abuse of discretion.
WHEREFORE, the instant petition is hereby DISMISSED for lack of merit.
SO ORDERED.

[G.R. No. 152398. April 14, 2005]

EDGAR CRISOSTOMO, petitioner, vs. SANDIGANBAYAN, respondent.


DECISION
CARPIO, J.:
The Case
This is an appeal by certiorari under Rule 65 of the Revised Rules on Civil Procedure
of the Sandiganbayan Resolutions promulgated on 17 September 2001 and 14 January
2002, denying the Motion for Reconsideration filed by petitioner SPO1 Edgar Crisostomo
(Crisostomo) assailing the courts Decision[1] promulgated on 28 November 2000. The
Decision found Crisostomo guilty of the crime of murder and sentenced him to suffer the
indeterminate penalty of twelve (12) years, five (5) months and eleven (11) days of prision
mayor as minimum, to eighteen (18) years, eight (8) months and one (1) day of reclusion
temporal as maximum.
The Charge
On 19 October 1993, Crisostomo, a member of the Philippine National Police and a
jail guard at the Solano Municipal Jail was charged with the murder of Renato Suba
(Renato), a detention prisoner at the Solano Municipal Jail. The Information alleged that
Crisostomo conspired with his co-accused, Dominador C. dela Cruz (dela Cruz), Efren
M. Perez (Perez), Raki T. Anggo (Anggo), Randy A. Lumabo (Lumabo), Rolando M.
Norberte (Norberte) and Mario Calingayan (Calingayan), all inmates at the Solano
Municipal Jail, in murdering Renato. The Information reads in full:
That on or about the 14th day of February 1989, in Solano, Nueva Vizcaya, Philippines, and
within the jurisdiction of this Honorable Court, the above-named accused Pat. Edgar T.
Crisostomo, a public officer, being then a member of the Philippine National Police (PNP)
stationed at Solano Police Station and a jailer thereat, taking advantage of his public position and
thus committing the offense in relation to his office, conspiring, confederating and conniving
with his co-accused who are inmates of the Solano Municipal Jail, namely: Dominador C. dela
Cruz, Efren M. Perez, Raki T. Anggo, Randy A. Lumabo, Rolando M. Norberte and Mario B.
Calingayan, with intent to kill and with treachery, taking advantage of superior strength and with
the aid of armed men or employing means to weaken the defense or of means or persons to
insure or afford impunity, did then and there wil[l]fully, unlawfully and feloniously attack and
assault one Renato Suba, a detention prisoner, with the use of rough-surfaced instruments,
including fist blows, inflicting upon him serious injuries causing his internal organs to be badly
damaged such as his liver, messentery and stomach resulting to the death of said Renato Suba to
the damage and prejudice of the heirs of the latter.
CONTRARY TO LAW.[2]

Arraignment and Plea


On 15 December 1993, Crisostomo assisted by counsel, pleaded not guilty to the
crime charged.[3] Thereafter, trial ensued.
Version of the Prosecution
On 13 February 1989, Renato was detained at the municipal jail in Solano, Nueva
Vizcaya for allegedly hitting the head of one Diosdado Lacangan. The following day, 14
February 1989, at 5:00 p.m., Renatos brother Rizalino Suba (Rizalino) visited him at the
municipal jail. Renato asked Rizalino to bring him blanket, toothbrush, clothes and food.
Rizalino left the municipal jail that day at 5:20 p.m. At that time, Renato was in good
physical condition and did not complain of any bodily pain. Renato was 26 years old,
single, and was employed in a logging concession.
At 9:00 p.m., a barangay councilman informed Rizalino that policemen assigned at
the Solano municipal jail wanted Rizalino to go to the municipal building. Rizalino arrived
at the municipal jail at 9:10 p.m. and saw his brother Renato already dead on the floor
outside his cell.
Renato was detained alone in the third cell, one of the four cells at the municipal jail.
Although each of the four cells had an iron grill door equipped with a padlock, the doors
were usually left open. The keys to the padlocks were with the jail guard. There was a
common front door, which no one could enter but the jail guard. Only one jail guard at a
time was assigned at the municipal jail. Crisostomo was the one on duty at the time of the
death of Renato. At no time was Renato brought out of the cell during his detention on 13
February 1989 until his death in the evening of the following day. Crisostomos position in
relation to the cell where the victim was killed was such that Crisostomo as jail guard
could have heard if not seen what was going on inside the cell at the time that Renato
was killed.
There are unexplained discrepancies in the list of detainees/prisoners and police
blotter. The list of detainees/prisoners dated 20 February 1989 shows that there were
eight prisoners on 14 February 1989, including Renato, but after Renatos death, only six
were turned over by Crisostomo to the incoming jail guard. On 15 February 1989, nine
prisoners/detainees were on the list, including Renato who was already dead. However,
the police blotter shows that only six prisoners were under custody. The persons who
were detained with Renato at the time of his death were released without being
investigated by the Solano police.
Renato did not commit suicide. His body bore extensive injuries that could have been
inflicted by several persons. The exhumation and autopsy reports ruled out suicide as the
cause of Renatos death. The deafening silence of the inmates and the jail guard,
Crisostomo, point to a conspiracy. Crisostomos guilt is made apparent when he jumped
bail during trial.
Version of the Defense

The presentation of evidence for Crisostomos defense was deemed waived for his
failure to appear at the scheduled hearings despite notice.
Calingayan, Crisostomos co-accused, was the sole witness for the defense.
Calingayan was only 16 years old at the time that he was charged with the murder of
Renato. Calingayan denied killing Renato.
Calingayan was detained at the Solano Municipal Jail on 12 February 1989 because
his brother-in-law, Patrolman Feliciano Leal (Leal), also a jail guard, had him arrested for
pawning some of the belongings of Leal. Leal told Calingayan that he had him detained
for safekeeping to teach him a lesson.
Renato was detained on 13 February 1989. Calingayan learned that Renato was
detained for hitting somebodys head.
There were four cells at the municipal jail. Calingayan was detained with five other
inmates in the second cell. Renato was detained alone in the third cell. The four cells had
their own separate doors with padlocks but each door was always open. It was up to the
inmates to close the doors. A common door leading to the four cells was always
padlocked and no one could enter the door without the jail guards permission. The jail
guard had the keys to the cells and the common door. Only one jail guard was assigned
to guard the cells. Crisostomo was the jail guard on duty at the time that Renato died.
Calingayan was in jail for three days or until 15 February 1989. Calingayan last saw
Renato alive between 5 to 6 p.m. of 14 February 1989. Just as Calingayan was about to
take a bath after 6 p.m., he saw Renato lying down. One of the inmates asked for Renatos
food because he did not like to eat his food. After taking a bath, Calingayan went back to
his cell and played cards with his three cellmates whose names he could not recall.
Calingayan did not leave his cell during the four hours that he played cards but one of his
cellmates went out.
Calingayan discovered Renatos body on 14 February 1989 between 9:00 p.m. to
10:00 p.m. Calingayan went to the fourth cell, where the comfort room was located, to
urinate. While urinating, Calingayan saw at the corner of the cell a shadow beside him. A
bulb at the alley lighted the cell. Calingayan ran away and called the other inmates, telling
them that the person in cell number four was in the dark place. The other inmates ran
towards the place and shouted si kuwan, si kuwan. Crisostomo was in the room at the
left side from where Calingayan was detained, about fifteen meters away. Upon hearing
the shouts, Crisostomo opened the main door. Once inside the cell, Crisostomo instructed
the inmates to bring down Renatos body that was hanging from the iron bars of the
window of the cell. At that time, Calingayan did not notice what was used in hanging
Renato but when the body was brought outside, Calingayan saw that Renato had hanged
himself with a thin blanket.
The four cells are not similar in area and size. The cell where Renato stayed is the
smallest. The cells are separated by a partition made of hollow blocks as high as the
ceiling. The four cells are in one line so that if you are in one cell you cannot see what is
happening in the other cells. The inmates could go to any of the four cells in the prison
but they could not get out of the main door without the permission of the jail guard. The

comfort room is in the fourth cell, which is also open so that the inmates would not
anymore ask for the key from the office of the jail guard.
The blanket that Renato used to hang himself was tied to the iron grills of the window
of the cell. The window is small, only about two feet by one and one-half feet with eight
iron bars. The window is nine feet from the floor.
No other person was admitted on 14 February 1989. Calingayan does not have a
grudge against Renato. He could not recall if there was any untoward incident between
Renato and the other inmates. The Solano police investigated Calingayan the next
morning.
The Ruling of the Sandiganbayan
Only Crisostomo and Calingayan stood trial. The other accused, dela Cruz, Perez,
Anggo, Lumabo and Norberte were at large. The Sandiganbayan found sufficient
circumstantial evidence to convict Crisostomo and Calingayan of murder.
The Sandiganbayan relied on the autopsy and exhumation reports in disregarding the
defense theory that Renato committed suicide by hanging himself with a blanket.
The Sandiganbayan thus held:
Premises considered, accused Edgar Crisostomo and Mario Calingayan are hereby found guilty
of the crime of murder.
xxx
There being no attending mitigating or aggravating circumstance in the case of accused Edgar
Crisostomo, and taking into consideration the Indeterminate Sentence Law, he is hereby
sentenced to suffer the penalty of imprisonment for the period from twelve (12) years, five (5)
months and eleven (11) days of prision mayor, minimum, to eighteen (18) years, eight (8)
months and one (1) day of reclusion temporal, maximum.
xxx
As to the other accused, Dominador C. Dela Cruz, Efren M. Perez, Raki T. Anggo, Randy A.
Lumabo and Rolando M. Norberte, considering they are still at-large up to the present time, let
an alias warrant of arrest be issued against them. In the meantime, the cases against them are
hereby ordered archived.
SO ORDERED.[4]
The Issues
Crisostomo continues to assail the Sandiganbayans jurisdiction. He raises the
following issues:

WHETHER THE SANDIGANBAYAN HAS JURISDICTION OVER THE CRIME OF


MURDER CHARGED AGAINST CRISOSTOMO WHO IS A SENIOR POLICE OFFICER 1
(SPO1) AT THE TIME OF THE FILING OF THE INFORMATION AGAINST HIM.
EVEN ASSUMING ARGUENDO THAT THE RESPONDENT COURT HAS JURISDICTION,
WHETHER THE SANDIGANBAYAN COMMITTED GRAVE ABUSE OF DISCRETION
AMOUNTING TO LACK OR EXCESS OF JURISDICTION WHEN IT RULED THAT
CRISOSTOMO IS GUILTY OF HAVING CONSPIRED IN THE MURDER OF RENATO
DESPITE THE SANDIGANBAYANS ADMISSION IN ITS DECISION THAT THERE IS NO
DIRECT EVIDENCE THAT WILL SHOW THE PARTICIPATION OF CRISOSTOMO IN
THE DEATH OF THE VICTIM.[5]
The Courts Ruling
The Sandiganbayan had jurisdiction to try the case. However, the prosecution failed
to prove Crisostomo and Calingayans guilt beyond reasonable doubt. Thus, we acquit
Crisostomo and Calingayan.
The Sandiganbayan had Jurisdiction to Try the Case
Crisostomo argues that the Sandiganbayan was without jurisdiction to try the case.
Crisostomo points out that the crime of murder is not listed in Section 4 of Presidential
Decree No. 1606 (PD 1606) as one of the crimes that the Sandiganbayan can try.
Crisostomo faults the Sandiganbayan for not applying the ruling in Sanchez v.
Demetriou[6] to this case. In Sanchez v. Demetriou, the Court ruled that public office
must be a constituent element of the crime as defined in the statute before the
Sandiganbayan could acquire jurisdiction over a case. Crisostomo insists that there is no
direct relation between the commission of murder and Crisostomos public office.
Crisostomo further contends that the mere allegation in the Information that the offense
was committed in relation to Crisostomos office is not sufficient to confer jurisdiction on
the Sandiganbayan. Such allegation without the specific factual averments is merely a
conclusion of law, not a factual averment that would show the close intimacy between the
offense charged and the discharge of Crisostomos official duties.
We are not convinced.
Since the crime was committed on 14 February 1989, the applicable provision of law
is Section 4 of PD 1606, as amended by Presidential Decree No. 1861 (PD 1861), which
took effect on 23 March 1983. The amended provision reads:
Sec. 4. Jurisdiction. The Sandiganbayan shall exercise:
(a) Exclusive original jurisdiction in all cases involving:
xxx
(2) Other offenses or felonies committed by public officers and employees in
relation to their office, including those employed in government-owned or controlled

corporations, whether simple or complexed with other crimes, where the penalty
prescribed by law is higher than prision correccional or imprisonment for six (6) years,
or a fine of P6,000.00: PROVIDED, HOWEVER, that offenses or felonies mentioned in
this paragraph where the penalty prescribed by law does not
exceed prision correccional or imprisonment for six (6) years or a fine of P6,000.00 shall
be tried by the proper Regional Trial Court, Metropolitan Trial Court, Municipal Trial
Court and Municipal Circuit Trial Court.
Crisostomo was charged with murder, the penalty for which is reclusion temporal in its
maximum period to death, a penalty within the jurisdiction of the Sandiganbayan.
Crisostomo would have the Court believe that being a jail guard is a mere incidental
circumstance that bears no close intimacy with the commission of murder. Crisostomos
theory would have been tenable if the murdered victim was not a prisoner under his
custody as a jail guard. The function of a jail guard is to insure the safe custody and
proper confinement of persons detained in the jail. In this case, the Information alleges
that the victim was a detention prisoner when Crisostomo, the jail guard, conspired with
the inmates to kill him.
Indeed, murder and homicide will never be the main function of any public office. No
public office will ever be a constituent element of murder. When then would murder or
homicide, committed by a public officer, fall within the exclusive and original jurisdiction
of the Sandiganbayan? People v. Montejo[7] provides the answer. The Court explained
that a public officer commits an offense in relation to his office if he perpetrates the offense
while performing, though in an improper or irregular manner, his official functions and he
cannot commit the offense without holding his public office. In such a case, there is an
intimate connection between the offense and the office of the accused. If the information
alleges the close connection between the offense charged and the office of the accused,
the case falls within the jurisdiction of the Sandiganbayan. People v. Montejo is an
exception that Sanchez v. Demetriou recognized.
Thus, the jurisdiction of the Sandiganbayan over this case will stand or fall on this
test: Does the Information allege a close or intimate connection between the offense
charged and Crisostomos public office?
The Information passes the test.
The Information alleged that Crisostomo a public officer, being then a member of the
Philippine National Police (PNP) stationed at Solano Police Station and a jailer thereat,
taking advantage of his public position and thus committing the offense in relation to his
office conspired, confederated and connived with his co-accused who are inmates of the
Solano Municipal Jail to kill Renato, a detention prisoner.
If the victim were not a prisoner, the Information would have to state particularly the
intimate relationship between the offense charged and the accused public officers office
to vest jurisdiction on the Sandiganbayan. This is not the case here. The law restrains the
liberty of a prisoner and puts him under the custody and watchful eyes of his jail guard.
Again, the two-fold duties of a jail guard are to insure the safe custody and
proper confinement of persons detained in the jail. The law restricts access to a

prisoner. However, because of the very nature of the work of a jail guard, he has access
to the prisoner. Crisostomo, as the jail guard, could not have conspired with the
inmates to murder the detention prisoner in his cell if Crisostomo were not a jailer.
The Information accused Crisostomo of murdering a detention prisoner, a crime that
collides directly with Crisostomos office as a jail guard who has the duty to insure the safe
custody of the prisoner. Crisostomos purported act of killing a detention prisoner, while
irregular and contrary to Crisostomos duties, was committed while he was performing his
official functions. The Information sufficiently apprised Crisostomo that he stood accused
of committing the crime in relation to his office, a case that is cognizable by the
Sandiganbayan, not the Regional Trial Court. There was no prejudice to Crisostomos
substantive rights.
Assuming that the Information failed to allege that Crisostomo committed the crime
in relation to his office, the Sandiganbayan still had jurisdiction to try the case. The
Information was filed with the Sandiganbayan on 19 October 1993. Deloso v.
Domingo,[8] promulgated on 21 November 1990, did not require that the information
should allege that the accused public officer committed the offense in relation to his office
before the Sandiganbayan could assume jurisdiction over the case. The ruling in Deloso
v. Domingo relied solely on PD 1606.
Aguinaldo v. Domagas,[9] promulgated on 26 September 1991, modified Deloso v.
Domingo. Aguinaldo v. Domagas clarified that offenses specified in Section 4(a)(2) of
PD 1606, as amended by PD 1861, must be committed by public officers and employees
in relation to their office and the information must allege this fact. The succeeding cases
of Sanchez v. Demetriou[10] and Natividad v. Felix,[11] reiterated the Aguinaldo v.
Domagas ruling.
However, despite the subsequent cases clarifying Deloso v. Domingo, the Court
in Republic v. Asuncion,[12] promulgated on 11 March 1994, applied the ruling in Deloso
v. Domingo. Since the effects of the misapprehension of Deloso v. Domingo doctrine
were still persistent, the Court set out the following directives in Republic v. Asuncion:
The dismissal then of Criminal Case No. Q-91-23224 solely on the basis of Deloso vs.
Domingo was erroneous. In the light of Aguinaldo and Sanchez, and considering the absence of
any allegation in the information that the offense was committed by private respondent in
relation to his office, it would even appear that the RTC has exclusive jurisdiction over the case.
However, it may yet be true that the crime of homicide charged therein was committed by the
private respondent in relation to his office, which fact, however, was not alleged in the
information probably because Deloso vs. Domingo did not require such an allegation. In view of
this eventuality and the special circumstances of this case, and to avoid further delay, if not
confusion, we shall direct the court a quo to conduct a preliminary hearing in this case to
determine whether the crime charged in Criminal Case No. Q-91-23224 was committed by the
private respondent in relation to his office. If it be determined in the affirmative, then it shall
order the transfer of the case to the Sandiganbayan which shall forthwith docket and proceed
with the case as if the same were originally filed with it. Otherwise, the court a quo shall set
aside the challenged orders, proceed with the trial of the case, and render judgment thereon.

Republic v. Asuncion ordered the trial court to conduct a preliminary hearing to


determine whether the accused public officer committed the crime charged while
performing his office. If so, the trial court must order the transfer of the case to the
Sandiganbayan as if the same were originally filed with the Sandiganbayan.
In the present case, the Information was filed with the Sandiganbayan upon the
recommendation of the Office of the Deputy Ombudsman in a Resolution dated 30 June
1993. That Crisostomo committed the crime in relation to his office can be gleaned from
the Deputy Ombudsmans resolution as it stated that: (1) Crisostomo was the jail guard
on duty at the time that Renato was killed; (2) from the time that Crisostomo assumed his
duty up to the discovery of Renatos body, no one had entered the jail and no one could
enter the jail, as it was always locked, without the permission of the jail guard; (3) the key
is always with the jail guard; (4) Renato sustained severe and multiple injuries inflicted by
two or more persons indicating conspiracy; and (5) the relative position of the jail guard
to the cell is in such a way that any activity inside the cell could be heard if not seen by
the jail guard.
Based on the foregoing findings, as well as on the Deloso v. Domingo ruling and the
Courts instructions in Republic v. Asuncion, the Sandiganbayan had every reason to
assume jurisdiction over this case. Crisostomo waited until the very last stage of this case,
the rendition of the verdict, before he questioned the Sandiganbayans jurisdiction.
Crisostomo is already estopped from questioning the Sandiganbayans jurisdiction.[13]
Crisostomos Guilt was not Proven Beyond Reasonable Doubt
In the exercise of the Courts judicial discretion, this petition for certiorari will be treated
as an appeal from the decision of the Sandiganbayan to prevent the manifest miscarriage
of justice[14] in a criminal case involving a capital offense. An appeal in a criminal case
opens the entire case for review.[15] The reviewing tribunal can correct errors though
unassigned in the appeal, or even reverse the lower courts decision on grounds other
than those the parties raised as errors.[16]
In this case, the prosecution had the burden to prove first, the conspiracy to murder
Renato, and second, Crisostomos complicity in the conspiracy. The prosecution must
prove that Renatos death was not the result of suicide but was produced by a deliberate
intent to kill him with the attendant circumstances that would qualify the killing to murder.
Since Crisostomo had no direct hand in the killing of Renato, the conviction could only be
sustained if the murder was carried out through a conspiracy between Crisostomo and
his co-accused, the inmates. It must be proven beyond reasonable doubt that
Crisostomos action and inaction were all part of a scheme to murder Renato.
Renato was Killed with Deliberate Intent
To prove that Renatos death is a case of homicide or murder, there must be
incontrovertible evidence, direct or circumstantial, that he was deliberately killed. [17] Intent
to kill can be deduced from the weapons used by the malefactors, the nature, location
and number of wounds sustained by the victim and the words uttered by the malefactors
before, at the time or immediately after the killing of the victim. [18] If the victim dies
because of a deliberate act of the malefactor, intent to kill is conclusively presumed. [19]

The prosecution established that Renato did not commit suicide. Witnesses for the
prosecution vouched that Renato was in good health prior to his death. Calingayan, the
sole witness for the defense, did not point out that there was any thing wrong with Renato
prior to his death. The autopsy and exhumation reports debunked the defenses theory
that Renato hanged himself to death. Renatos injuries were so massive and grave that it
would have been impossible for these injuries to have been self-inflicted by Renato.
The extent of Renatos injuries indicates the perpetrators deliberate intent to kill him.
Dr. Ruben M. Agobung (Dr. Agobung), the NBI Medico Legal Officer [20] who exhumed
and re-autopsied Renatos body, stated in his affidavit[21] that Renato sustained several
external and internal injuries, the most significant of which are the ruptured liver, torn
messentery and torn stomach. The injuries caused massive intra-abdominal hemorrhage
that ultimately caused Renatos death. Dr. Agobung further declared that Renatos injuries
could bring about death in a matter of minutes to a few hours from the time of infliction, if
not promptly and properly attended to by a competent surgeon.
Renatos internal injuries were so severe that the injuries could not have been
sustained prior to his detention at the Solano Municipal Jail. If this were so, Renato would
have experienced continuous and severe body pains and he would have fallen into shock,
which could have been obvious even to those who are not doctors. Dr. Agobung also
concluded that Renatos injuries could have been inflicted by the application of
considerable force with the use of a hard and rough surface as well as hard smooth
surface instruments, fist blows included.
While the blanket that was tied around Renatos neck caused abrasion and contusion
on the neck area, these injuries, however, did not cause Renatos death because the
blood vessels on his neck were still intact.[22] The Exhumation Report[23] and Exhumation
Findings[24] stated that Renato died due to hemorrhagic shock, secondary to multiple
internal organ injuries. These findings lead to the inevitable conclusion that Renato was
killed with deliberate intent and his body was hanged just to simulate suicide.
Prosecution Failed to Prove Crisostomos Involvement in the Killing
No direct evidence linked Crisostomo to the killing of Renato. The prosecution relied
on circumstantial evidence to prove that there was a conspiracy to kill Renato and
Crisostomo participated in carrying out the conspiracy. Circumstantial evidence consists
of proof of collateral facts and circumstances from which the existence of the main fact
may be inferred according to reason and common experience.[25] Section 4, Rule 133 of
the Revised Rules of Evidence states that circumstantial evidence is sufficient if: (a) there
is more than one circumstance; (b) the facts from which the inferences are derived are
proven; (c) the combination of all the circumstances is such as to produce a conviction
beyond reasonable doubt.
In convicting Crisostomo, the Sandiganbayan cited the following circumstantial
evidence:
1. The deceased, Renato Suba, was brought to the police station on the night of February 13,
1989 for investigation for allegedly hitting the head of a certain Diosdado Lacangan; and that

after investigation, the deceased was brought to the detention cell (tsn, hearing of April 21, 1994,
pp. 5-11).
2. On the following day at 5:00 oclock in the afternoon, the deceased was visited by his brother,
Rizalino Suba; that the deceased asked his brother to bring him a blanket, toothbrush, clothes and
foods (ibid, pp. 13-14).
3. Rizalino Suba left the municipal jail on February 14, 1989, at almost 5:20 p.m., while his other
brother, Rolando, brought the things to the deceased in jail; and that Rolando left their house at
about 5:30 p.m. and came back at 6:00 oclock in which Rizalino asked him (Rolando) if he
(Renato Suba) was able to finish the food that he sent and he answered in the affirmative (ibid,
pp. 16, 18-19).
4. At that time, the deceased was in good health and in good condition and that he was not
complaining anything about his body; and that the deceased was then 26 years old, single and
had finished advance ROTC and worked in a logging concession (ibid, pp. 16-18).
5. Accused Mario Calingayan saw the deceased still alive lying down after 6:00 p.m. when he
was about to take a bath; and that after taking a bath, he (witness) went to his cell and played
cards with his three (3) cellmates (whose names he could not recall) for about four (4) hours (tsn,
hearing of April 4, 1995, pp. 16-17).
6. At around 9:00 oclock of the same day, Mr. Baldovino, a barangay councilman, informed
them that they should go to the municipal building as per request of the policemen; that Rizalino
Suba, first asked his uncle David Suba and Manuel Rollo, a barangay councilman, to accompany
him; that they arrived at the municipal building at 9:10 p.m. and they saw that the deceased was
already lying dead on the cement floor outside the cell 1 in the municipal building (tsn, hearing
of April 21, 1994, pp. 20-22).
7. Accused Mario Calingayan was detained with five (5) others at the second cell among four (4)
cells in the jail; that the deceased, Renato Suba, was detained alone at the third cell (tsn, hearing
of April 4, 1995, pp. 6-7).
8. The four (4) cells, although having their own separate doors, made of iron grills and equipped
each with a padlock, were always open; that it was up to them whether to close the doors; that
the keys of the padlocks are held by the guard; and that any detention prisoner could go to any
cell inside the prison (ibid, pp. 7-8, 21, 23).
9. There was a common door located in front, leading inside to the cells which no one could
enter because it is padlocked, except with the jail guards permission; and that the comfort room
is located in the 4thcell which is not equipped with a padlock so that if you want to go to the
comfort room, you do not anymore need the key in the office of the jail guard (ibid, p. 22).
10. There is only one guard assigned in the cells and accused Edgar Crisostomo was the one who
was rendering duty at the time of the death of the victim (ibid, pp. 9, 13).

11. There was no other person who was admitted on February 12, 13 and 14, 1989, and there was
no instance when Suba was brought out of the prison cell from the time he was detained on
February 14, 1989 (ibid, p. 29).
12. The persons who were detained together with the deceased at the time of his death were
released without any investigation having been conducted by the local police (tsn, hearing of
April 21, 1994, pp. 28-29).
13. The apparent inconsistency in the list of detainees/prisoners dated February 20, 1989 (Exhibit
I) and the police blotter (Exhibits J and J-1) whereby in the former there were eight prisoners on
February 14, 1989 including the victim but only six were turned over by accused Crisostomo to
the incoming jail guard after the death of the victim; the list contains nine (9) detainees/prisoners
on February 15, 1989 which includes the victim, who was then dead, while the police blotter
shows that only six prisoners were under their custody. Why the apparent inconsistency?
14. Accused Mario Calingayans claim that he was detained on February 12, 1989, which is
contrary to the master list of detainees showing that he was detained only on February 14, 1989
(tsn, hearing of April 4, 1995, p. 19).
15. Accused Mario Calingayans allegation that when Renato Suba was brought outside, he saw
that he hanged himself with a thin blanket (tsn, hearing of April 4, 1995, pp. 12-13) which was
what the policemen also told the brother of the victim (tsn, hearing of April 21, 1994, pp. 23-24).
16. After the prosecution rested its case and after co-accused Mario Calingayan was finished
with his testimony in court, accused Edgar Crisostomo jumped bail and up to this day had
remained at large (Rollo, pp. 297-298, 305).
17. The fact that accused Dominador C. Dela Cruz, Efren M. Perez, Raki T. Anggo, Randy A.
Lumabo and Rolando M. Norberte are also still at-large.[26]
The Sandiganbayan also relied on the Memorandum Report[27] dated 22 October
1991 of Oscar Oida, then National Bureau of Investigation (NBI) Regional Director for
Region II, who evaluated the NBIs investigation of the case. The Sandiganbayan quoted
the following portions of the report:
xxx
5. That when he [victim] was brought to the Solano Municipal Jail at around 12:00
midnight on 14 February 1989 (the same was corrected by witness Oscar
Oida to be February 13, 1989 when he testified in open court), he was
accompanied by his brother, Rizalino Suba, his cousin, Rodolfo Suba and
Brgy. Councilman Manuel Rulloda in good physical condition with no
injuries[;]
6. That when Luis Suba, father of the victim, Renato Suba, visited him in jail at
around 8:00 a.m., on February 14, 1989 and brought food for his breakfast,

he was in good physical condition, and did not complain of any physical injury
or pain. In fact, he was able to eat all the food[;]
7. That when Rizalino Suba, brother of the victim, visited the latter at around 5:00
oclock p.m. on 14 February 1989, victim was in good spirit and never
complained of any injury or bodily pain. He was in good physical condition.
He even requested that he be brought his clothes, beddings and food[;]
8. That when Rolando Suba, another brother of victim, brought the clothes,
bedding and food as requested by the latter at around 6:00 oclock p.m. on 14
February 1989, he was in good physical condition and did not complain of any
injury or body pain[;]
9. That the good physical condition of victim, Renato Suba was even
corroborated by his four co-inmates, namely, Arki Anggo, Randy A. Lumabo,
Rolando M. Norberte and Mario B. Calingayan and by the jailer, Pat. Edgar
T. Crisostomo, when he was placed under detention in the Solano Municipal
Jail;
10. That the jailer Pat. Edgar Crisostomo from the time he assumed his tour of
duty from 4:00 oclock p.m. on 14 February 1989, up to the time the victim was
discovered allegedly dead and hanging inside the jail at 9:00 oclock p.m. on
that same day, nobody entered the jail and no one would enter said jail, as it
was always locked, without the permission of the jailer. The key is always with
the jailer;
11. That the only companions of the victim at the time of the discovery of his
death on 14 February 1989 at around 9:00 p.m. were his six (6) co-inmates
namely: Dominador C. dela Cruz, Edren M. Perez, Raki T. Anggo, Randy A.
Luma[b]o, Rolando M. Norbert[e] and Mario Calingayan;
12. That definitely the cause of death was not suicide by hanging but due to
several injuries sustained by the victim. The most significant and remarkable
of which are the ruptured liver, torn messentery and a torn stomach which
injuries resulted into massive intra-abdominal hemorrhage that ultimately
caused the death of said victim per autopsy examination;
13. That said injuries can bring about death in a matter of minutes to a few hours
if not promptly and properly attended by a competent surgeon;
14. That said injuries could not have been sustained by victim before he was
detained at the Solano Municipal Jail as he could have been experiencing
continuous severe pain which can easily be observed by the policemen who
arrested him on 14 February 1989 at around 12:00 midnight and therefore
should have been brought to the hospital and not confined in the detention
cell;
15. That the several injuries sustained by victim were caused by hard roughsurfaced as well as hard smooth surfaced instruments, fist blows included;

16. That the multiple injuries and the gravity of the injuries sustained by victim
indicate that they were inflicted by more than two persons;
17. That the nature of the injuries sustained by victim were almost in one
particular part of the body, shown by the fact that the internal organs badly
damaged were the liver, messentery and stomach indicating that the victim
was defenseless and helpless thus affording the assailants to pounce on
continuously with impunity almost on one spot of the body of the victim. The
victim could have been held by two or more assailants while the others were
alternately or giving victim blows on his body with hard rough surfaced as well
as hard smooth surfaced instruments, fist blows included;
18. That with the location and gravity of the injuries sustained by victim, the
persons who inflicted the injuries know fully well that victim will die and knew
the consequences of their acts;
19. That the motive was revenge, as victim before he was killed, hit in the head
a certain Diosdado Lacangan with a wood causing serious injury. Lacangan
was in serious condition at the time victim was killed[;]
20. That the claims of the Solano police and the six (6) co-inmates of victim that
the latter committed suicide by hanging is only a cover up to hide a heinous
offense[;]
21. That the extreme silence of the suspects regarding the death of victim is so
deafening that it established only one thing, conspiracy. It is unusual for a
person not to volunteer information as to who could be the author of the
offense if he is not a participant to a heinous offense particularly in this case
where the circumstances show that there can be no other person responsible
for the death of the victim except the suspects in this instant case[;]
22. That the victim was killed between 6:00 PM to 9:00 PM on 14 February 1989
inside the Solano Municipal Jail[;]
xxx
The relative position of the jailer to the cell where victim was killed was such that the jailer and
the policemen present, could hear if not see what was going inside the cell at the time the victim
was killed. The injuries sustained by victim could not be inflicted without victim shouting and
crying for help. Even the assailants when they inflicted these injuries on victim could not avoid
making loud noises that could attract the attention of the police officers present. Conspiracy to
kill the victim among the inmates and the police officers was clearly established from the
circumstances preceding and after the killing of victim.[28]
In sum, the Sandiganbayan believed that Crisostomo took part in the conspiracy to
kill Renato because of these three circumstances: 1) Crisostomo as the jail guard on duty
at the time of Renatos killing had in his possession the keys to the main door and the
cells; (2) Crisostomo was in such a position that he could have seen or heard the killing
of Renato; and (3) there are discrepancies between the list of detainees/prisoners and

the police blotter. According to the Sandiganbayan, there is a prima facie case against
Crisostomo.
Except for the extensive injuries that Renatos body bore, there is no other evidence
that proves that there was a prior agreement between Crisostomo and the six inmates to
kill Renato. In People v. Corpuz,[29] one of the inmates killed by the other inmates
sustained stab wounds that were possibly inflicted by ten persons. The Court ruled that
conspiracy could not be inferred from the manner that the accused inmates attacked their
fellow inmate because there was no sufficient showing that all the accused inmates acted
pursuant to a previous common accord. Each of the accused inmates was held liable for
his individual act.
Although no formal agreement is necessary to establish conspiracy because
conspiracy may be inferred from the circumstances attending the commission of the
crime, yet conspiracy must be established by clear and convincing evidence. [30] Even if
all the malefactors joined in the killing, such circumstance alone does not satisfy the
requirement of conspiracy because the rule is that neither joint nor simultaneous action
is per se sufficient proof of conspiracy.[31] Conspiracy must be shown to exist as clearly
and convincingly as the commission of the offense itself.[32]
Thus, even assuming that Renato was simultaneously attacked, this does not prove
conspiracy. The malefactors who inflicted the fatal injuries may have intended by their
own separate acts to bring about the death of the victim. [33] No evidence was presented
to show that Crisostomo and the inmates planned to kill Renato or that Crisostomos overt
acts or inaction facilitated the alleged plan to kill Renato. The prosecution had the burden
to show Crisostomos intentional participation to the furtherance of the common design
and purpose.
The pieces of circumstantial evidence are not sufficient to create a prima facie case
against Crisostomo. When the three circumstances are examined with the other evidence
on record, it becomes all the more clear that these circumstances do not lead to a logical
conclusion that Crisostomo lent support to an alleged conspiracy to murder Renato.
First, while Crisostomo as jail guard had in his possession the keys to the main door
and individual cells, there is no proof that Crisostomo allowed an outsider inside the
prison. Calingayan, the sole witness for the defense, testified that no new detainee was
admitted from 13 to 14 of February 1989.[34] The NBI Report[35] relied upon by the
Sandiganbayan confirms Calingayans testimony that nobody entered the jail and that
Renatos only companions inside the jail were the six inmates.[36]
There is also no proof that Crisostomo purposely left the individual cells open to allow
the inmates to attack Renato who was alone in the third cell. Calingayan, who was
detained ahead of Renato,[37] testified that while each of the four cells had a padlock, the
cells had always been kept open.[38] The inmates had always been allowed to enter the
cells and it was up to the inmates to close the doors of the cells.[39] The inmates could
freely go to the fourth cell, which was the inmates comfort room so that they would no
longer ask for the key from the jail guard every time the inmates would use the comfort
room.[40]

Second, the Sandiganbayan should not have absolutely relied on the NBI
Report[41] stating that Crisostomo as jail guard was in such a position that he could have
seen or heard the killing. The prosecution failed to establish that Crisostomo actually saw
and heard the killing of Renato.
Based on Calingayans testimony, it was not impossible for Crisostomo not to have
actually seen and heard the killing of Renato. On cross-examination, Calingayan testified
that all of the cells were in one line.[42] Crisostomos office was at the left side of the cells
about 15 meters away from cell number two, the cell where Calingayan was
detained.[43] Hollow blocks from the floor to the ceiling separated each of the four
cells.[44] With the partition, an inmate in one cell could not see what was happening in the
other cells.[45] Calingayan further testified that Renatos body was in a dark place,[46] as it
was lighted from outside only by a bulb at the alley, at the corridor.[47]
Since Renatos body was found in cell number four, this would make the distance
between Crisostomos office and the crime scene more than 15 meters. Crisostomo could
not have had a full view of cell number four because of the distance between Crisostomos
office and cell number four, the partitions of the four cells and poor lighting in the jail.
Calingayans description of the jail, the cells, the location of Renatos body and
Crisostomos actual position was not contradicted by the prosecution. There is no other
evidence on record that describes the layout and conditions of the jail at the time of
Renatos death.
The prosecution had the burden to present evidence that Crisostomo indeed saw and
heard Renatos killing and Crisostomo consented to the killing as part of the plan to kill
Renato. The absence of such evidence does not preclude the possibility that Renato was
covertly killed and the sounds were muffled to conceal the crime from Crisostomo, the jail
guard. Or Crisostomo as jail guard was simply negligent in securing the safety of the
inmates under his custody. If Crisostomo were negligent, this would be incompatible with
conspiracy because negligence denotes the absence of intent while conspiracy involves
a meeting of the minds to commit a crime.[48] It was the prosecutions burden to limit the
possibilities to only one: that Crisostomo conspired with the inmates to kill Renato. The
prosecution failed to do so.
Third, the prosecution was not clear as to the implication of the discrepancies
between the list of detainees/prisoners and police blotter to the conspiracy to murder
Renato. The prosecution did not even pinpoint which of the two documents is the accurate
document. The prosecution merely asked: why the apparent inconsistency? [49]
Courts must judge the guilt or innocence of the accused based on facts and not on
mere conjectures, presumptions or suspicions.[50] The inconsistency between the two
documents without anything more remains as merely that an inconsistency. The
inconsistency does not even have any bearing on the prosecutions conspiracy theory.
The NBI Report and Calingayans testimony stated that six inmates were with Renato
inside the jail. This was also the same number of inmates turned over by Crisostomo to
the incoming jail guard after Renatos death.[51]
The alleged motive for Renatos killing was to avenge the attack on Lacangan who
was then in a serious condition because Renato hit him on the head with a piece of wood.

No evidence was presented to link Crisostomo to Lacangan or to show what compelling


motive made Crisostomo, a jail guard, abandon his duty and instead facilitate the killing
of an inmate under his custody. Motive is generally held to be immaterial because it is not
an element of the crime.[52] However, motive becomes important when the evidence on
the commission of the crime is purely circumstantial or inconclusive.[53] Motive is thus vital
in this case.
Clearly, the Sandiganbayan had no basis to convict Crisostomo because the
prosecution failed to produce the evidence necessary to overturn the presumption of
innocence. The insufficiency of evidence was the same reason why the National Police
Commission dismissed the administrative case for grave misconduct (murder) against
Crisostomo on 24 October 1990.[54] The circumstances in this case did not constitute an
unbroken chain that would lead to a reasonable conclusion that Crisostomo played a role
in the inmates supposed preconceived effort to kill Renato. Thus, Crisostomo must be
acquitted.
The deafening silence of all of the accused does not necessarily point to a conspiracy.
In the first place, not all of the accused remained silent. Calingayan put himself on the
witness stand. Calingayan further claimed that the Solano police investigated him and his
handwritten statements were taken the morning following Renatos death. [55] Secondly, an
accused has the constitutional right to remain silent and to be exempt from being
compelled to be a witness against himself.[56]
A judgment of conviction must be predicated on the strength of the evidence for the
prosecution and not on the weakness of the evidence for the defense. [57] The
circumstantial evidence in this case is not sufficient to create a prima facie case to shift
the burden of evidence to Crisostomo. Moreover, Calingayans testimony inured to
Crisostomos favor. The supposed waiver of presentation of evidence did not work against
Crisostomo because the prosecution failed to prove Crisostomos guilt beyond reasonable
doubt.
In Salvatierra v. CA,[58] upon ruling for the defendants acquittal, the Court
disregarded the issue of whether the defendants jumped bail for failing to attend trial and
whether their absence should be considered as flight and as evidence of guilt. Even with
this ruling in Salvatierra v. CA, which is applicable to this case, and Crisostomos failure
to question the violation of his right to procedural due process before the Court, we cannot
simply ignore the Sandiganbayans grave abuse of discretion.
The records show that the Sandiganbayan set the hearing of the defenses
presentation of evidence on 21, 22 and 23 June 1995. The 21 June 1995 hearing was
cancelled because of lack of quorum in the regular membership of the Sandiganbayans
Second Division and upon the agreement of the parties. [59] The hearing was reset the
next day, 22 June 1995. Crisostomo and his counsel failed to attend the 22 June 1995
hearing. The Sandiganbayan, on the very same day, issued an order [60] directing the
issuance of a warrant for the arrest of Crisostomo and ordering the confiscation of his
surety bond. The order further declared that Crisostomo had waived his right to present
evidence because of his non-appearance at yesterdays and todays scheduled
hearings.[61] The Sandiganbayan terminated the trial and gave the parties thirty days

within which to file their memoranda, after which, with or without the memoranda, the
case would still be deemed submitted for decision.
The Sandiganbayans error is obvious. Strictly speaking, Crisostomo failed to appear
only on the 22 June 1995 hearing. Crisostomos appearance on the 21 June 1995 hearing
would not have mattered because the hearing on this date was cancelled for lack of
quorum of justices in the Sandiganbayans Second Division.
Under Section 2(c), Rule 114 and Section 1(c), Rule 115 of the Rules of Court,
Crisostomos non-appearance during the 22 June 1995 trial was merely a waiver of his
right to be present for trial on such date only and not for the succeeding trial
dates.[62] Section 1(c) of Rule 115 clearly states that:
xxx The absence of the accused without any justifiable cause at the trial on a particular date of
which he had notice shall be considered a waiver of his right to be present during that trial. When
an accused under custody had been notified of the date of the trial and escapes, he shall be
deemed to have waived his right to be present on said date and on all subsequent trial dates until
custody is regained.
Moreover, Crisostomos absence on the 22 June 1995 hearing should not have been
deemed as a waiver of his right to present evidence. While constitutional rights may be
waived, such waiver must be clear and must be coupled with an actual intention to
relinquish the right.[63] Crisostomo did not voluntarily waive in person or even through his
counsel the right to present evidence. The Sandiganbayan imposed the waiver due to the
agreement of the prosecution, Calingayan, and Calingayans counsel.
In criminal cases where the imposable penalty may be death, as in the present case,
the court is called upon to see to it that the accused is personally made aware of the
consequences of a waiver of the right to present evidence.[64] In fact, it is not enough that
the accused is simply warned of the consequences of another failure to attend the
succeeding hearings.[65] The court must first explain to the accused personally in clear
terms the exact nature and consequences of a waiver.[66] Crisostomo was not even
forewarned. The Sandiganbayan simply went ahead to deprive Crisostomo of his right to
present evidence without even allowing Crisostomo to explain his absence on the 22 June
1995 hearing.
Clearly, the waiver of the right to present evidence in a criminal case involving a grave
penalty is not assumed and taken lightly. The presence of the accused and his counsel
is indispensable so that the court could personally conduct a searching inquiry into the
waiver.[67] Moreover, the searching inquiry must conform to the procedure recently
reiterated in People v. Beriber,[68] to wit:
1. The trial court shall hear both the prosecution and the accused with their respective counsel on
the desire or manifestation of the accused to waive the right to present evidence and be heard.
2. The trial court shall ensure the attendance of the prosecution and especially the accused with
their respective counsel in the hearing which must be recorded. Their presence must be duly
entered in the minutes of the proceedings.

3. During the hearing, it shall be the task of the trial court to


a. ask the defense counsel a series of question[s] to determine whether he had
conferred with and completely explained to the accused that he had the right to
present evidence and be heard as well as its meaning and consequences, together
with the significance and outcome of the waiver of such right. If the lawyer for
the accused has not done so, the trial court shall give the latter enough time to
fulfill this professional obligation.
b. inquire from the defense counsel with conformity of the accused whether he
wants to present evidence or submit a memorandum elucidating on the
contradictions and insufficiency of the prosecution evidence, if any or in default
thereof, file a demurrer to evidence with prior leave of court, if he so believes
that the prosecution evidence is so weak that it need not even be rebutted. If there
is a desire to do so, the trial court shall give the defense enough time for this
purpose.
c. elicit information about the personality profile of the accused, such as his age,
socio-economic status, and educational background, which may serve as a
trustworthy index of his capacity to give a free and informed waiver.
d. all questions posed to the accused should be in a language known and understood
by the latter, hence, the record must state the language used for this purpose as
well as reflect the corresponding translation thereof in English.
If no waiver of the right to present evidence could be presumed from Crisostomos
failure to attend the 22 June 1995 hearing, with more reason that flight could not be
logically inferred from Crisostomos absence at that hearing. Crisostomos absence did not
even justify the forfeiture of his bail bond. A bail bond may be forfeited only in instances
where the presence of the accused is specifically required by the court or the Rules of
Court and, despite due notice to the bondsmen to produce him before the court on a given
date, the accused fails to appear in person as so required. [69] Crisostomo was not
specifically required by the Sandiganbayan or the Rules of Court to appear on the 22
June 1995 hearing. Thus, there was no basis for the Sandiganbayan to order the
confiscation of Crisostomos surety bond and assume that Crisostomo had jumped bail.
Prior to his absence on the 22 June 1995 hearing, Crisostomo had regularly attended
the hearings of the case. When it was Crisostomos turn to present his evidence, Atty.
Anecio R. Guades (Atty. Guades), Crisostomos former counsel, instructed Crisostomo to
wait for the notice of hearing from him and the Sandiganbayan. Crisostomo did not receive
any notice from the Sandiganbayan or from Atty. Guades who disappeared without
informing Crisostomo of his new office address. Upon notification of the promulgation of
the case scheduled on 28 November 2000, Crisostomo voluntarily appeared before the
Sandiganbayan. Crisostomo then terminated the services of Atty. Guades and engaged
the services of another counsel. In the omnibus motion for new trial filed by Crisostomos
new counsel, Crisostomo denied that he went into hiding. If given the chance, Crisostomo

would have presented his pay slips and certificates of attendance to prove that he had
been reporting for work at the Police Station in Solano, Nueva Vizcaya. [70]
We could not absolutely fault the Sandiganbayan for not correcting its 22 June 1995
Order. The Sandiganbayan lost the opportunity to review the order when Crisostomos
new counsel changed his legal strategy by withdrawing the omnibus motion for new trial
and instead sought the nullification of the Sandiganbayans decision for lack of jurisdiction
over the case.
However, the withdrawal of the omnibus motion could not erase the Sandiganbayans
violation of Crisostomos right to procedural due process and Atty. Guades gross
negligence. Atty. Guades failed to protect his clients interest when he did not notify
Crisostomo of the scheduled hearings and just vanished without informing Crisostomo
and the Sandiganbayan of his new office address. The 22 June 1995 Order was served
on Atty. Guades but he did not even comply with the directive in the Order to explain in
writing his absence at the 21 and 22 June 1995 hearings. Atty. Guades did not file the
memorandum in Crisostomos behalf required by the same Order. Atty. Guades did not
also question the violation of Crisostomos right to procedural due process. The
subsequent notices of hearing and promulgation were not served on Atty. Guades as he
could not be located in the building where his office was located.[71]
Clearly, Atty. Guadess negligence was so gross that it should not prejudice
Crisostomos constitutional right to be heard,[72] especially in this case when the imposable
penalty may be death. At any rate, the remand of the case is no longer necessary. [73] The
prosecutions evidence failed to overturn the constitutional presumption of innocence
warranting Crisostomos acquittal.
The Sandiganbayan imposed an indeterminate sentence on Crisostomo. The
Indeterminate Sentence Law (ISL) is not applicable to persons convicted of offenses
punished with the death penalty or reclusion perpetua.[74] Since Crisostomo was accused
of murder, the penalty for which is reclusion temporal in its maximum period to death, the
Sandiganbayan should have imposed the penalty in its medium period since it found no
aggravating circumstance.[75] The medium period of the penalty is reclusion perpertua.
Calingayan must be also Acquitted
The Sandiganbayan cited only two circumstances as evidence of Calingayans guilt.
The Sandiganbayan held that Calingayans claim that he was detained on 12 February
1989 is contrary to the master list of detainees showing that Calingayan was detained on
14 February 1989.[76] Second is Calingayans allegation that when Renato Suba was
brought outside, he saw that he hanged himself with a thin blanket, which was what the
policemen also told the brother of the victim.[77] The Sandiganbayan did not elaborate on
this circumstance. The Sandiganbayan was apparently suspicious of Renatos knowledge
of the material that was used to hang Renato.
Renato could have been killed by two or more inmates or possibly even by all of the
inmates. However, since no conspiracy was proven to exist in this case, the perpetrators
of the crime needed to be identified and their independent acts had to be proven. [78] The
two circumstances that were held against Calingayan are not sufficient proof that

Calingayan was one of the inmates who killed Renato. Thus, Calingayan must be also
acquitted.
Section 11(a) of Rule 122 of the Rules of Court provides that [a]n appeal taken by
one or more [of] several accused shall not affect those who did not appeal, except insofar
as the judgment of the appellant court is favorable and applicable to the latter. In this
case, only Crisostomo questioned the jurisdiction and decision of the Sandiganbayan.
However, the evidence against Crisostomo and Calingayan are inextricably linked as their
conviction hinged on the prosecutions unproven theory of conspiracy. Thus, Crisostomos
acquittal, which is favorable and applicable to Calingayan, should benefit Calingayan. [79]
WHEREFORE, the Decision of the Sandiganbayan in Criminal Case No. 19780
convicting appellant EDGAR CRISOSTOMO and co-accused MARIO B. CALINGAYAN
is hereby REVERSED. EDGAR CRISOSTOMO and co-accused MARIO B.
CALINGAYAN are ACQUITTED of the crime of murder and ordered immediately released
from prison, unless held for another lawful cause. The Director of Prisons is directed to
report to this Court compliance within five (5) days from receipt of this Decision. No costs.
SO ORDERED.

G.R. Nos. 111771-77 November 9, 1993


ANTONIO L. SANCHEZ, petitioner,
vs.
The Honorable HARRIET O. DEMETRIOU (in her capacity as Presiding Judge of
Regional Trial Court, NCR, Branch 70, Pasig), The Honorable FRANKLIN DRILON
(in his capacity as Secretary of Justice), JOVENCITO R. ZUO, LEONARDO C.
GUIYAB, CARLOS L. DE LEON, RAMONCITO C. MISON, REYNALDO J. LUGTU,
and RODRIGO P. LORENZO, the last six respondents in their official capacities as
members of the State Prosecutor's Office), respondents.
Mario E. Ongkiko and Marciano P. Brion, Jr. for petitioner.
The Solicitor General for respondents.

CRUZ, J.:
There is probably no more notorious person in the country today than Mayor Antonio L.
Sanchez of Calauan, Laguna, who stands accused of an unspeakable crime. On him,
the verdict has already been rendered by many outraged persons who would
immediately impose on him an angry sentence. Yet, for all the prejudgments against
him, he is under our Constitution presumed innocent as long as the contrary has not
been proved. Like any other person accused of an offense, he is entitled to the full and
vigilant protection of the Bill of Rights.
Sanchez has brought this petition to challenge the order of the respondent judge
denying his motion to quash the informations for rape with homicide filed against him
and six other persons. We shall treat it as we would any other suit filed by any litigant
hoping to obtain a just and impartial judgment from this Court.
The pertinent facts are as follows:
On July 28, 1993, the Presidential Anti-Crime Commission requested the filing of
appropriate charges against several persons, including the petitioner, in connection with
the rape-slay of Mary Eileen Sarmenta and the killing of Allan Gomez.
Acting on this request, the Panel of State Prosecutors of the Department of Justice
conducted a preliminary investigation on August 9, 1993. Petitioner Sanchez was not
present but was represented by his counsel, Atty. Marciano Brion, Jr.
On August 12, 1993, PNP Commander Rex Piad issued an "invitation" to the petitioner
requesting him to appear for investigation at Camp Vicente Lim in Canlubang, Laguna.
It was served on Sanchez in the morning of August 13,1993, and he was immediately
taken to the said camp.

At a confrontation that same day, Sanchez was positively identified by Aurelio Centeno,
and SPO III Vivencio Malabanan, who both executed confessions implicating him as a
principal in the rape-slay of Sarmenta and the killing of Gomez. The petitioner was then
placed on "arrest status" and taken to the Department of Justice in Manila.
The respondent prosecutors immediately conducted an inquest upon his arrival, with
Atty. Salvador Panelo as his counsel.
After the hearing, a warrant of arrest was served on Sanchez. This warrant was issued
on August 13, 1993, by Judge Enrico A. Lanzanas of the Regional Trial Court of Manila,
Branch 7, in connection with Criminal Cases Nos. 93-124634 to 93-124637 for violation
of Section 8, in relation to Section 1, of R.A. No. 6713. Sanchez was forthwith taken to
the CIS Detention Center, Camp Crame, where he remains confined.
On August 16, 1993, the respondent prosecutors filed with the Regional Trial Court of
Calamba, Laguna, seven informations charging Antonio L. Sanchez, Luis Corcolon,
Rogelio Corcolon, Pepito Kawit, Baldwin Brion, Jr., George Medialdea and Zoilo Ama
with the rape and killing of Mary Eileen Sarmenta.
On August 26, 1993, Judge Eustaquio P. Sto. Domingo of that court issued a warrant
for the arrest of all the accused, including the petitioner, in connection with the said
crime.
The respondent Secretary of Justice subsequently expressed his apprehension that the
trial of the said cases might result in a miscarriage of justice because of the tense and
partisan atmosphere in Laguna in favor of the petitioner and the relationship of an
employee, in the trial court with one of the accused. This Court thereupon ordered the
transfer of the venue of the seven cases to Pasig, Metro Manila, where they were raffled
to respondent Judge Harriet Demetriou.
On September 10, 1993, the seven informations were amended to include the killing of
Allan Gomez as an aggravating circumstance.
On that same date, the petitioner filed a motion to quash the informations substantially
on the grounds now raised in this petition. On September 13, 1993, after oral
arguments, the respondent judge denied the motion. Sanchez then filed with this Court
the instant petition for certiorari and prohibition with prayer for a temporary restraining
order/writ of injunction.
The petitioner argues that the seven informations filed against him should be quashed
because: 1) he was denied the right to present evidence at the preliminary investigation;
2) only the Ombudsman had the competence to conduct the investigation; 3) his
warrantless arrest is illegal and the court has therefore not acquired jurisdiction over
him, 4) he is being charged with seven homicides arising from the death of only two
persons; 5) the informations are discriminatory because they do not include Teofilo

Alqueza and Edgardo Lavadia; and 6) as a public officer, he can be tried for the offense
only by the Sandiganbayan.
The respondents submitted a Comment on the petition, to which we required a Reply
from the petitioner within a non-extendible period of five days. 1 The Reply was filed five
days late. 2 The Court may consider his non-compliance an implied admission of the
respondents' arguments or a loss of interest in prosecuting his petition, which is a
ground for its dismissal. Nevertheless, we shall disregard this procedural lapse and
proceed to discuss his petition on the basis of the arguments before us.
The Preliminary Investigation.
The records of the hearings held on August 9 and 13, 1993, belie the petitioner's
contention that he was not accorded the right to present counter-affidavits.
During the preliminary investigation on August 9, 1993, the petitioner's counsel, Atty.
Marciano Brion, manifested that his client was waiving the presentation of a counteraffidavit, thus:
Atty. Brion, Jr.:
[W]e manifest that after reviewing them there is nothing to rebut or
countermand all these statements as far as Mayor Sanchez is concerned,
We are not going to submit any counter-affidavit.
ACSP Zuo to Atty. Brion:
xxx xxx xxx
Q. So far, there are no other statements.
A. If there is none then, we will not submit any counteraffidavit because we believe there is nothing to rebut or
countermand with all these statements.
Q. So, you are waiving your submission of counter-affidavit?
A. Yes, your honor, unless there are other witnesses who
will come up soon. 3
Nonetheless, the head of the Panel of Prosecutors, respondent Jovencito Zuo, told
Atty. Brion that he could still file a counter-affidavit up to August 27, 1993. No such
counter-affidavit was filed.
During the hearing on August 1'3, 1993, respondent Zuo furnished the petitioner's
counsel, this time Atty. Salvador Panelo, with copies of the sworn statements of

Centeno and Malabanan, and told him he could submit counter-affidavits on or before
August 27, 1993. The following exchange ensued:
ACSP Zuo:
For the record, we are furnishing to you the sworn statement
of witness Aurelio Centeno y Roxas and the sworn
statement of SPO3 Vivencio Malabanan y Angeles.
Do I understand from you that you are again waiving the
submission of counter-affidavit?
Atty. Panelo:
Yes.
ACSP Zuo:
So, insofar as the respondent, Mayor Antonio Sanchez is
concerned, this case is submitted for resolution. 4
On the other hand, there is no support for the petitioner's subsequent manifestation that
his counsel, Atty. Brion, was not notified of the inquest held on August 13, 1993, and
that he was not furnished with the affidavits sworn to on that date by Vivencio
Malabanan and Aurelio Centeno, or with their supplemental affidavits dated August 15,
1993. Moreover, the above-quoted excerpt shows that the petitioner's counsel at the
hearing held on August 13, 1993, was not Atty. Brion but Atty. Panelo.
The petitioner was present at that hearing and he never disowned Atty. Panelo as his
counsel. During the entire proceedings, he remained quiet and let this counsel speak
and argue on his behalf. It was only in his tardy Reply that he has suddenly bestirred
himself and would now question his representation by this lawyer as unauthorized and
inofficious.
Section 3, Paragraph (d), Rule 112 of the Rules of Court, provides that if the respondent
cannot be subpoenaed or, if subpoenaed, does not submit counter-affidavits, the
investigating officer shall base his resolution on the evidence presented by the
complainant.
Just as the accused may renounce the right to be present at the preliminary
investigation 5, so may he waive the right to present counter-affidavits or any other
evidence in his defense.
At any rate, it is settled that the absence of a preliminary investigation does not impair
the validity of the information or otherwise render the same defective and neither does it

affect the jurisdiction of the court over the case or constitute a ground for quashing the
information. 6
If no preliminary investigation has been held, or if it is flawed, the trial court may, on
motion of the accused, order an investigation or reinvestigation and hold the
proceedings in the criminal case in abeyance. 7 In the case at bar, however, the
respondent judge saw no reason or need for such a step. Finding no arbitrariness in her
factual conclusions, we shall defer to her judgment.
Jurisdiction of the Ombudsman
Invoking the case of Deloso v. Domingo, 8 the petitioner submits that the proceedings
conducted by the Department of Justice are null and void because it had no jurisdiction
over the case. His claim is that it is the Office of the Ombudsman that is vested with the
power to conduct the investigation of all cases involving public officers like him, as the
municipal mayor of Calauan, Laguna.
The Ombudsman is indeed empowered under Section 15, paragraph (1) of R.A. 6770 to
investigate and prosecute, any illegal act or omission of any public official. However, as
we held only two years ago in the case ofAguinaldo v. Domagas, 9 this authority "is not
an exclusive authority but rather a shared or concurrent authority in. respect of the
offense charged."
Petitioners finally assert that the information and amended information
filed in this case needed the approval of the Ombudsman. It is not
disputed that the information and amended information here did not have
the approval of the Ombudsman. However, we do not believe that such
approval was necessary at all. In Deloso v. Domingo, 191 SCRA. 545
(1990), the Court held that the Ombudsman has authority to investigate
charges of illegal or omissions on the part of any public official, i.e., any
crime imputed to a public official. It must, however, be pointed out that the
authority of the Ombudsman to investigate "any [illegal] act or omission of
any public official" (191 SCRA at 550) isnot an exclusive authority but
rather a shared or concurrent authority in respect of the offense here
charged, i.e., the crime of sedition. Thus, the non-involvement of the office
of the Ombudsman in the present case does not have any adverse legal
consequence upon the authority the panel of prosecutors to file and
prosecute the information or amended information.
In fact, other investigatory agencies, of the government such as the Department of
Justice, in connection with the charge of sedition, 10 and the Presidential Commission
on Good Government, in ill-gotten wealth cases, 11 may conduct the investigation,
The Arrest
Was petitioner Sanchez arrested on August 13, 1993?

"Arrest" is defined under Section 1, Rule 113 of the Rules of Court as the taking of a
person into custody in order that he may be bound to answer for the commission of an
offense. Under Section 2 of the same Rule, an arrest is effected by an actual restraint of
the person to be arrested or by his voluntary submission to the custody of the person
making the arrest.
Application of actual force, manual touching of the body, physical restraint or a formal
declaration of arrest is not, required. It is enough that there be an intent on the part of
one of the parties to arrest the other and an intent onthe part of the other to submit,
under the belief and impression that submission is necessary. 12
The petitioner was taken to Camp Vicente Lim, Canlubang, Laguna, by virtue of a letterinvitation issued by PNP Commander Rex Piad requesting him to appear at the said
camp for investigation.
In Babst v. National Intelligence Board 13 this Court declared:
Be that as it may, it is not idle to note that ordinarily, an invitation to attend
a hearing and answer some questions, which the person invited may heed
or refuse at his pleasure, is not illegal or constitutionally objectionable.
Under certain circumstances, however, such an invitation can easily
assume a different appearance. Thus, where the invitation comes from a
powerful group composed predominantly of ranking military officers issued
at a time when the country has just emerged from martial rule and when
the suspension of the privilege of the writ of habeas corpus has not
entirely been lifted, and the designated interrogation site is a military
camp, the same can be easily taken,not as a strictly voluntary
invitation which it purports to be, but as an authoritative command which
one can only defy at his peril. . . . (Emphasis supplied)
In the case at bar, the invitation came from a high-ranking military official and the
investigation of Sanchez was to be made at a military camp. Although in the guise of a
request, it was obviously a command or an order of arrest that the petitioner could
hardly he expected to defy. In fact, apparently cowed by the "invitation," he went without
protest (and in informal clothes and slippers only) with the officers who had come to
fetch him.
It may not be amiss to observe that under R.A. No. 7438, the requisites of a "custodial
investigation" are applicable even to a person not formally arrested but merely "invited"
for questioning.
It should likewise be noted that at Camp Vicente Lim, the petitioner was placed on
"arrest status" after he was pointed to by Centeno and Malabanan as the person who
first raped Mary Eileen Sarmenta. Respondent Zuo himself acknowledged during the
August 13, 1993 hearing that, on the basis of the sworn statements of the two state
witnesses, petitioner had been "arrested."

We agree with the petitioner that his arrest did not come under Section 5, Rule 113 of
the Rules of Court, providing as follows:
Sec. 5. Arrest without warrant; when lawful. A peace officer or a private
person may, without a warrant, arrest a person:
(a) When, in his presence, the person to be arrested has committed, is
actually committing, or is attempting to commit an offense;
(b) When an offense has in fact just been committed and he has personal
knowledge of facts indicating that the person to be arrested has committed
it; and
(c) When the person to be arrested is a prisoner who has escapes from a
penal establishment or place where he is serving final judgment or
temporarily confined while his case is pending, or has escaped while
being transferred from one confinement to another.
It is not denied that the arresting officers were not present when the petitioner allegedly
participated in the killing of Allan Gomez and the rape-slay of Mary Eileen Sarmenta.
Neither did they have any personal knowledge that the petitioner was responsible
therefor because the basis of the arrest was the sworn statements of Centeno and
Malabanan. Moreover, as the rape and killing of Sarmenta allegedly took place on June
28-June 29, 1993, or forty-six days before the date of the arrest, it cannot be said that
the offense had "in fact just been committed" when the petitioner was arrested.
The original warrantless arrest of the petitioner was doubtless illegal. Nevertheless, the
Regional Trial Court lawfully acquired jurisdiction over the person of the petitioner by
virtue of the warrant of arrest it issued on August 26, 1993 against him and the other
accused in connection with the rape-slay cases. It was belated, to be sure, but it was
nonetheless legal.
Even on the assumption that no warrant was issued at all, we find that the trial court still
lawfully acquired jurisdiction over the person of the petitioner. The rule is that if the
accused objects to the jurisdiction of the court over his person, he may move to quash
the information, but only on that ground. If, as in this case, the accused raises other
grounds in the motion to quash, he is deemed to have waived that objection and to have
submitted his person to the jurisdiction of that court. 14
The Court notes that on August 13, 1993, after the petitioner was unlawfully arrested,
Judge Lanzanas issued a warrant of arrest against Antonio L. Sanchez in connection
with Criminal Cases Nos. 93-124634 to 93-124637 for violation of R.A No.
6713. 15 Pending the issuance of the warrant of arrest for the rape-slay cases, this first
warrant served as the initial justification for his detention.

The Court also adverts to its uniform ruling that the filing of charges, and the issuance of
the corresponding warrant of arrest, against a person invalidly detained will cure the
defect of that detention or at least deny him the right to be released because of such
defect. * Applicable by analogy to the case at bar is Rule 102 Section 4 of the Rules of
Court that:
Sec, 4. When writ is not allowed or discharge authorized. If it appears
that the person alleged to be restrained of his liberty is in the custody of an
officer under process issued by a court or judge or by virtue of a judgment
or order of a court of record, and that the court or judge had jurisdiction to
issue the process, render the judgment, or make the order, the writ shall
not be allowed; or if the jurisdiction appears after the writ is allowed, the
person shall not be discharged by reason of any informality or defect in
the process, judgment, or order. Nor shall, anything in this rule be held to
authorize the discharge of a person charged with or convicted of an
offense in the Philippines or of a person suffering imprisonment under
lawful judgment.
In one case, 16 the petitioner, sued on habeas corpus on the ground that she had been
arrested by virtue of a John Doe warrant. In their return, the respondents declared that a
new warrant specifically naming her had been issued, thus validating her detention.
While frowning at the tactics of the respondents, the Court said:
The, case has, indeed, become moot and academic inasmuch as the new
warrant of arrest complies with the requirements of the Constitution and
the Rules of Court regarding the particular description of the person to be
arrested. While the first warrant was unquestionably void, being a general
warrant, release of the petitioner for that reason will be a futile act as it will
be followed by her immediate re-arrest pursuant to the new and valid
warrant, returning her to the same prison she will just have left. This Court
will not participate in such a meaningless charade.
The same doctrine has been consistently followed by the Court,
the Umil case. 18

17

more recently in

The Informations
The petitioner submits that the seven informations charging seven separate homicides
are absurd because the two victims in these cases could not have died seven times.
This argument was correctly refuted by the Solicitor General in this wise:
Thus, where there are two or more offenders who commit rape, the
homicide committed on the occasion or by reason of each rape, must be
deemed as a constituent of the special complex crime of rape with

homicide. Therefore, there will be as many crimes of rape with homicide


as there are rapes committed.
In effect, the presence of homicide qualifies the crime of rape, thereby
raising its penalty to the highest degree. Thus, homicide committed on the
occasion or by reason of rape, loses its character as an independent
offense, but assumes a new character, and functions like a qualifying
circumstance. However,by fiction of law, it merged with rape to constitute
an constituent element of a special complex crime of rape with homicide
with a specific penalty which is in the highest degree, i.e. death (reduced
to reclusion perpetua with the suspension of the application of the death
penalty by the Constitution).
It is clearly provided in Rule 110 of the Rules of Court that:
Sec. 13. Duplicity of offense. A complaint or information must charge but
one offense, except only in those cases in which existing laws prescribe a
simple punishment for various offenses.
Rape with homicide comes within the exception under R.A. 2632 and R.A. 4111,
amending the Revised Penal Code.
The petitioner and his six co-accused are not charged with only one rape committed by
him in conspiracy with the other six. Each one of the seven accused is charged with
having himself raped Sarmenta instead of simply helping Sanchez in committing only
one rape. In other words, the allegation of the prosecution is that the girl was raped
seven times, with each of the seven accused taking turns in abusing her with the
assistance of the other six. Afterwards, their lust satisfied, all seven of them decided to
kill and thus silence Sarmenta.
Every one of the seven accused is being charged separately for actually raping
Sarmenta and later killing her instead of merely assisting the petitioner in raping and
then slaying her. The separate informations filed against each of them allege that each
of the seven successive rapes is complexed by the subsequent slaying of Sarmenta
and aggravated by the killing of Allan Gomez by her seven attackers. The separate
rapes were committed in succession by the seven accused, culminating in the slaying of
Sarmenta.
It is of course absurd to suggest that Mary Eileen Sarmenta and Allan Gomez were
killed seven times, but the informations do not make such a suggestion. It is the
petitioner who does so and is thus hoist by his own petard.
The Alleged Discrimination
The charge of discrimination against the petitioner because of the non-inclusion of
Teofilo Alqueza and Edgardo Lavadia in the informations must also be dismissed.

While the prosecuting officer is required by law to charge all those who in his opinion,
appear to be guilty, he nevertheless cannot be compelled to include in the information a
person against whom he believes no sufficient evidence of guilt exists. 19 The
appreciation of the evidence involves the use of discretion on the part of the prosecutor,
and we do not find in the case at bar a clear showing by the petitioner of a grave abuse
of such discretion. 20
The decision of the prosecutor may be reversed or modified by the Secretary of Justice
or in special cases by the President of the Philippines. 21 But even this Court cannot
order the prosecution of a person against whom the prosecutor does not find sufficient
evidence to support at least a prima facie case. The courts try and absolve or convict
the accused but as a rule have no part in the initial decision to prosecute him.
The possible exception is where there is an unmistakable showing of a grave abuse of
discretion that will justify judicial intrusion into the precincts of the executive. But in such
a case the proper remedy to call for such exception is a petition
for mandamus, not certiorari or prohibition. 22 Moreover, before resorting to this relief,
the party seeking the inclusion of another person as a co-accused in the same case
must first avail itself of other adequate remedies such as the filing of a motion for such
inclusion. 23
At any rate, it is a preposterous contention that because no charges have been filed
against Alqueza and Lavadia, the charges against the petitioner and his co-accused
should also be dropped.
Jurisdiction of the Sandiganbayan
The petitioner argued earlier that since most of the accused were incumbent public
officials or employees at the time of the alleged commission of the crimes, the cases
against them should come under the jurisdiction of the Sandiganbayan and not of the
regular courts. This contention was withdrawn in his Reply but we shall discuss it just
the same for the guidance of all those concerned.
Section 4, paragraph (a) of P.D. No, 1606, as amended by P.D. No.1861, provides:
Sec. 4. Jurisdiction. The Sandiganbayan shall exercise:
a) Exclusive original jurisdiction in all cases involving:
(1) Violations of Republic Act No. 3019, as amended,
otherwise known as the Anti-Graft and Corrupt Practices Act,
Republic Act No. 1379, and Chapter II, Section 2, Title VII of
the Revised Penal Code:
(2) Other offenses or felonies committed by public officers
and employees in relation to their office, including those

employed in government-owned or controlled corporations,


whether simple or complexed with other crimes, where the
penalty prescribed by law is higher than prision
correccional or imprisonment for six (6) years, or a fine of
P6,000.00. . . . (Emphasis supplied)
The crime of rape with homicide with which the petitioner stands charged obviously
does not fall under paragraph (1), which deals with graft and corruption cases. Neither
is it covered by paragraph (2) because it is not an offense committed in relation to the
office of the petitioner.
In Montilla v, Hilario, 24 this Court described the "offense committed in relation to the
office" as follows:
[T]he relation between the crime and the office contemplated by the
Constitution is, in our opinion, direct and not accidental. To fall into the
intent of the Constitution, the relation has to be such that, in the legal
sense, the offense cannot exist without the office. In other words, the
office must be a constituent element of the crime as defined in the statute,
such as, for instance, the crimes defined and punished in Chapter Two to
Six, Title Seven, of the Revised Penal Code.
Public office is not of the essence of murder. The taking of human life is
either murder or homicide whether done by a private citizen or public
servant, and the penalty is the same except when the perpetrator. being a
public functionary took advantage of his office, as alleged in this case, in
which event the penalty is increased.
But the use or abuse of office does not adhere to the crime as an element;
and even as an aggravating circumstance, its materiality arises not from
the allegations but on the proof, not from the fact that the criminals are
public officials but from the manner of the commission of the crime
There is no direct relation between the commission of the crime of rape with homicide
and the petitioner's office as municipal mayor because public office is not an essential
element of the crime charged. The offense can stand independently of the office.
Moreover, it is not even alleged in the information that the commission of the crime
charged was intimately connected with the performance of the petitioner's official
functions to make it fall under the exception laid down in People v. Montejo. 25
In that case, a city mayor and several detectives were charged with murder for the
death of a suspect as a result of a "third degree" investigation held at a police
substation. The appearance of a senator as their counsel was questioned by the
prosecution on the ground that he was inhibited by the Constitution from representing
them because they were accused of an offense committed in relation to their office. The
Court agreed. It held that even if their position was not an essential ingredient of the

offense, there was nevertheless an intimate connection between the office and the
offense, as alleged in the information, that brought it within the definition of an offense
"committed in relation to the public office."
As Chief Justice Concepcion said:
It is apparent from these allegations that, although public office is not an
element of the crime of murder in abstract, as committed by the main
respondents herein, according to the amended information, the offense
therein charged is intimately connected with their respective offices and
was perpetrated while they were in the performance, though improper or
irregular, of their official functions. Indeed they had no personal motive to
commit the crime and they would not have committed it had they not held
their aforesaid offices. The co-defendants of respondent Leroy S. Brown,
obeyed his instructions because he was their superior officer, as Mayor of
Basilan City. (Emphasis supplied).
We have read the informations in the case at bar and find no allegation therein that the
crime of rape with homicide imputed to the petitioner was connected with the discharge
of his functions as municipal mayor or that there is an "intimate connection" between the
offense and his office. It follows that the said crime, being an ordinary offense, is triable
by the regular courts and not the Sandiganbayan.
Conclusion
As above demonstrated, all of the grounds invoked by the petitioner are not supported
by the facts and the applicable law and jurisprudence. They must, therefore, all be
rejected. In consequence, the respondent judge, who has started the trial of the criminal
cases against the petitioner and his co-accused, may proceed therewith without further
hindrance.
It remains to stress that the decision we make today is not a decision on the merits of
the criminal cases being tried below. These will have to be decided by the respondent
judge in accordance with the evidence that is still being received. At this time, there is
yet no basis for judgment, only uninformed conjecture. The Court will caution against
such irrelevant public speculations as they can be based only on imperfect knowledge if
not officious ignorance.
WHEREFORE, the petition is DISMISSED. The respondent judge is DIRECTED to
continue with the trial of Criminal Cases Nos. 101141, 101142, 101143, 101144,
101145, 101146 and 101147 and to decide them with deliberate dispatch.
SO ORDERED.

[G.R. No. 154886. July 28, 2005]

LUDWIG H. ADAZA, petitioner, vs. SANDIGANBAYAN (the First DIVISION


composed of Justices GREGORIO S. ONG, CATALINO R. CASTANEDA, JR.
and FRANCISCO H. VILLARUZ, JR. and THE PEOPLE OF THE PHILIPPINES
represented by SPECIAL PROSECUTION OFFICE, respondents.
DECISION
CARPIO-MORALES, J.:
Before this Court is a petition for certiorari under Rule 65 of the Rules of Court
assailing the June 19, 2002 Decision[1] and July 3, 2002 Resolution[2] of the
Sandiganbayan finding petitioner Ludwig H. Adaza (petitioner) guilty beyond reasonable
doubt of Falsification of Public Document penalized under Article 172, in relation to Article
171, paragraph 1 of the Revised Penal Code and denying his motion for reconsideration,
respectively.
Culled from the records of the case are the following facts:
Sometime in 1996, the Department of Public Works and Highways (DPWH) of the
District of Zamboanga del Norte awarded to the Parents and Teachers Association
(PTA) of Manawan National High School (MNHS) in Manawan, Jose Dalman,
Zamboanga del Norte a contract for the construction of a school building consisting of two
classrooms at an agreed consideration of P111,319.50.[3] Petitioner at that time was
municipal mayor of Jose Dalman.
1st

The project was completed on June 24, 1997 per Certificate of Completion and
Turnover for Custody[4] issued by the DPWH, but the PTA failed to receive the last
installment payment therefor in the amount of P20,847.17.[5]
Upon verification with the DPWH, PTA President Felix Mejorada (Mejorada) was
informed by Hazel Pearanda (Pearanda), Cashier II of the 1st Engineering District of
Zamboanga del Norte, that the check for P20,847.17 had been released to petitioner.[6]
Mejorada thereupon went to the Office of the Auditor of the DPWH and requested
that he be furnished with certified true copies of the relevant documents pertaining to the
contract, including the disbursement voucher and the corresponding check representing
the last payment made by the DPWH for the project.[7]
Confronted with Disbursement Voucher No. B-1019707309[8] issued by the DPWH,
Engineering District, Sta. Isabel, Dipolog City, in the amount of P20,847.17 for
payment to him as PTA President, approved by District Engineer Jesus T. Estimo,
Mejorada detected that the signature above his printed name thereon acknowledging
receipt of the check from Releasing Officer-Cashier Pearanda was not his. And he noticed
that petitioners signature was affixed on the voucher.[9]
1st

Upon perusal of DBP Check No. 0000718668[10] dated July 18, 1997 issued to payee
PTA Pres. By: Felix Mejorada and drawn by OIC Assistant District Engineer Jesus G. Sy

and District Engineer Estimo, Mejorada noticed that there were two signatures at the
dorsal portion thereof, his forged signature and another which he found to be that of
Aristela Adaza (Aristela), wife of petitioner.[11]
Asked by Mejorada to explain the circumstances behind the release of the check,
Pearanda related that one afternoon in July 1997, petitioner approached her and inquired
whether the check for the final installment payment on the contract was already prepared,
to which she replied that the check was ready but that it could not be released without
claimant Mejorada affixing his signature on the disbursement voucher. Pearanda further
related that petitioner offered to take the disbursement voucher and have it signed by
Mejorada, hence, she handed it to petitioner but kept the check in her custody; and when
petitioner returned the voucher to Pearanda later that day, the check already bore a
signature purporting to be that of Mejorada.[12]
Continuing, Pearanda related that petitioner thereupon requested that the
corresponding check be given to him in behalf of Mejorada. [13] In order to exculpate
herself from any liability, Pearanda asked petitioner to sign the voucher before releasing
the check. Petitioner obliged by affixing his signature on the space below the purported
signature of Mejorada. Pearanda then released the check to petitioner.
The check was allegedly encashed by Aristela on July 22, 1997.[14]
Mejorada was later to claim that on November 2, 1997, petitioner went to his house
informing him, in the presence of his brother Rotchel Mejorada and his nephew Anecito
Mejorada, that he would be paid within the week. No payment was, however, made. [15]
On December 16, 1997, Mejorada repaired to the National Bureau of Investigation
(NBI), Dipolog City where he filed a complaint against petitioner and his wife Aristela, and
executed a Sworn Statement.[16]
On January 6, 1998, Pearanda likewise executed a Sworn Statement [17] before the
NBI.
The complaint, for falsification of public document, was forwarded to the Office of the
Ombudsman where it was docketed as Case No. OMB-MIN-98-0096. During the
pendency of the preliminary investigation, Mejorada executed an Affidavit of
Desistance[18] dated May 8, 1998 alleging that his and the PTAs claims had been paid in
full by the spouses Adaza and requesting that the cases against them be dismissed or
considered withdrawn.
Petitioner and Aristela subsequently filed their Joint Counter-Affidavit[19] dated May
28, 1998, stating that Mejoradas claim had already been paid in full and that they had not
in any way benefited from the proceeds of the subject disbursement voucher and check
as the proceeds thereof were actually paid to the laborers who constructed the school
building pursuant to the contract. They likewise stated that there was only a
communication gap between them and Mejorada and that after the records have been
reconciled and verified, Mejorada was convinced that the money in question had been
paid to the laborers.

On July 31, 1998, the Office of the Ombudsman issued a Resolution [20] finding
probable cause against petitioner and Aristela. The dispositive portion of the Resolution
reads, quotedverbatim:
WHEREFORE, premises considered, this Office finds probable cause to conclude that the crimes
(sic) of Falsification of Public Document are (sic) probably committed [by] Mayor Ludwig
Adaza and another crime of Falsification of Public Document was probably committed by
respondents (sic) Mayor and his co-respondent wife. Accordingly, let the appropriate
Informations be filed in court.
SO RESOLVED.[21]
On even date, petitioner was charged in two Informations filed before the Sandiganbayan.
The inculpatory portion of the first, docketed as Criminal Case No. 24854, reads as
follows:
That sometime on or about 18 July 1997, or shortly subsequent thereto, in Dipolog City,
Philippines and within the jurisdiction of this Honorable Court, the accused Ludwig Adaza, a
public officer being then the Mayor with salary grade 27 of Jose Dalman, Zamboanga del
Norte, while in the performance of his official duties, committing the offense in relation to his
official function and taking advantage of his public position, did there and then, wilfully,
unlawfully and feloniously, falsify a public document, namely Disbursement Voucher No. B1019707309 of the DPWH 1st Engineering District, Dipolog City, bycounterfeiting therein the
signature of Felix Mejorada when in truth and in fact, as the accused well knew, Felix Mejorada
did not affix his signature on the document and did not authorize the accused to affix Mejoradas
signature therein.
CONTRARY TO LAW.[22] (Underscoring supplied)
Petitioner was charged together with Aristela in the second Information, docketed as
Criminal Case No. 24853, the inculpatory portion of which reads:
That sometime on or about 18 July 1997, or shortly subsequent thereto, in Dipolog City,
Philippines and within the jurisdiction of this Honorable Court, the accused Ludwig Adaza, a
public officer being then the Mayor with salary grade 27 of Jose Dalman, Zamboanga del Norte,
while in the performance of his official duties, committing the offense in relation to his official
function and taking advantage of his public position, conspiring, cooperating and confederating
with accused Aristela Adaza, did there and then, wilfully, unlawfully and feloniously, falsify a
public document, namely DPB Check No. 0000718668 issued by the DPWH 1st Engineering
District, Dipolog City, by counterfeiting therein the signature of indorsement of Felix Mejorada
when in truth and in fact, as the accused well knew, Felix Mejorada did not affix his signature on
the document and did not authorize the accused to affix Mejoradas signature therein.
CONTRARY TO LAW.[23] (Underscoring supplied)

After petitioner and his co-accused wife Aristela posted their respective bail bonds for
their provisional liberty, Mejorada filed an Affidavit of Confirmation[24] dated October 28,
1998 affirming the truth and veracity of the contents of his Affidavit of Desistance dated
May 22, 1998 and further alleging that he believed that there was no crime of falsification
committed.
Mejorada subsequently filed still another Affidavit of Confirmation[25] dated November
9, 1998 reiterating his allegations in the Affidavit of Confirmation dated October 28, 1998.
Petitioner and Aristela later filed a Motion for Reconsideration[26] dated November 9,
1998 of the July 31, 1998 Resolution of the Office of the Ombudsman finding probable
cause against them, which motion was denied by Resolution[27] of December 10, 1998.
On arraignment, petitioner and Aristela, duly assisted by counsel, pleaded not
guilty[28] to the charges, whereupon trial commenced.
By Decision of June 19, 2002, the Sandiganbayan found petitioner guilty in the first
case, and acquitted him and his wife Aristela in the second case for insufficiency of
evidence.
Petitioner filed on June 28, 2002 a Motion for Reconsideration [29] of the decision
which was denied by Resolution of July 3, 2002, the Sandiganbayan holding that the
same was pro forma as it was not properly set for hearing in accordance with the Rules
of Court.
Petitioner filed an Urgent Motion for Reconsideration[30] of the July 3, 2002
Sandiganbayan Resolution and attached thereto a Notice[31] setting his June 28, 2002
Motion for Reconsideration for hearing.
By Resolution[32] of August 21, 2002, the Sandiganbayan denied petitioners Urgent
Motion for lack of merit.
On August 23, 2002, a Bench Warrant of Arrest[33] was issued by the Sandiganbayan
against petitioner for execution of judgment.
Hence, petitioners present petition for certiorari[34] faulting the Sandiganbayan to
have committed grave abuse of discretion:
1
. . . BY CONSIDERING THE MOTION FOR RECONSIDERATION OF ITS DECISION AS
PRO FORMA
2
. . . BY ALLOWING BALD TECHNICALITY TO PREVAIL OVER THE MERITS OF THE
MOTION FOR RECONSIDERATION THUS IGNORING SECTION 6 OF RULE 1 OF THE
REVISED RULES AND THE APPROPRIATELY APPLICABLE JURISPRUDENCE
3

. . . BY IGNORING THE MERITS OF THE MOTION FOR RECONSIDERATION AND BY


CONVICTING THE ACCUSED/PETITIONER WHEN THERE IS ABSOLUTELY NO
EVIDENCEWHATSOEVER FOR CONVICTING THE ACCUSED/PETITIONER BEYOND
A REASONABLE DOUBT[35] (Underscoring supplied)
On October 29, 2002, the law office of Atty. Felipe Antonio B. Remollo entered its
appearance for petitioner.[36] On even date, petitioner filed a Supplement[37] to the petition
raising the following additional arguments:
I
WITH ALL DUE RESPECT, THE HONORABLE RESPONDENT SANDIGANBAYAN
HAS NO JURISDICTION OVER THE OFFENSE CHARGED OF FALSIFICATION OF
PUBLIC DOCUMENTS UNDER ARTICLE 172 PARAGRAPH 1 IN RELATION TO
ARTICLE 171 PARAGRAPH 1 OF THE REVISED PENAL CODE AGAINST THE
ACCUSED (FORMER) MUNICIPAL MAYOR (WITH SALARY GRADE 27) WHO DID
NOT TAKE ADVANTAGE OF HIS OFFICIAL POSITION IN THE ALLEGED
COMMISSION OF THE CRIME AS RULED BY THE SANDIGANBAYAN. SUCH BEING
THE CASE, THE ALLEGED OFFENSE WAS NOT COMMITTED IN RELATION TO
THE OFFICE OF THE MUNICIPAL MAYOR WHICH IS OUTSIDE THE JURISDICTION
OF THE SANDIGANBAYAN.
II
THE RIGHT OF THE ACCUSED TO A COMPETENT AND INDEPENDENT COUNSEL IS
ENSHRINED IN THE 1987 CONSTITUTION. THIS RIGHT SHOULD BE UPHELD AT ALL
TIMES AND SHOULD NOT BE OUTWEIGHT (sic) OR DISLODGED BY WHATEVER
GROSS PROCEDURAL LAPSES IN SUCCESSION THAT DEFENSE COUNSEL MAY
HAVE COMMITTED TANTAMOUNT TO DENIAL OF DUE PROCESS IN THE INTEREST
OF SUBSTANTIVE JUSTICE.
III
THE PETITION WAS FILED WITH A STRONG SENSE OF URGENCY IN THE LIGHT OF
THE FACT THAT PUBLIC RESPONDENT SANDIGANBAYAN ORDERED THE
IMMEDIATE ARREST OF THE ACCUSED IN ITS AUGUST 21, 2002 RESOLUTION
(SUBJECT OF HEREIN PETITION FOR CERTIORARI) ON THE THEORY THAT THE
ORDER OF CONVICTION OF THE ACCUSED PETITIONER HAS BECOME FINAL BY
SHEER TECHNICALITY THAT ON (sic) THE ACCUSEDS MOTION FOR
RECONSIDERATION DID NOT BEAR A NOTICE OF HEARING.[38] (Emphasis and
underscoring supplied)
Petitioners counsel of record Homobono A. Adaza later withdrew his appearance.[39]
The Office of the Special Prosecutor has filed its Comment[40] on the petition, to which
petitioner filed his Reply[41] reiterating his arguments raised in his Supplement to the
petition.

On the issue of jurisdiction, Section 4 of Republic Act No. 8249 (An Act Further
Defining the Jurisdiction of the Sandiganbayan, Amending for the Purpose Presidential
Decree No. 1606, As Amended, Providing Funds Therefor, and for Other Purposes)
provides:
Sec. 4. Jurisdiction. - The Sandiganbayan shall exercise exclusive original jurisdiction in all
cases involving:
A. Violations of Republic Act No. 3019, as amended, otherwise known as the Anti-Graft and
Corrupt Practices Act, Republic Act No. 1379, and Chapter II, Section 2, Title VII, Book II of
the Revised Penal Code, where one or more of the accused are officials occupying the following
positions in the government, whether in a permanent, acting or interim capacity, at the time of
the commission of the offense:
(1) Officials of the executive branch occupying the positions of regional director and higher,
otherwise classified as Grade 27 and higher, of the Compensation and Position Classification Act
of 1989 (Republic Act No. 6758), specifically including:
(a) Provincial governors, vice-governors, members of the sangguniang panlalawigan, and
provincial treasurers, assessors, engineers, and other city department heads;
(b) City mayor, vice-mayors, members of the sangguniang panlungsod, city treasurers, assessors,
engineers, and other city department heads;
(c) Officials of the diplomatic service occupying the position of consul and higher;
(d) Philippine army and air force colonels, naval captains, and all officers of higher rank;
(e) Officers of the Philippine National Police while occupying the position of provincial director
and those holding the rank of superior superintendent or higher;
(f) City and provincial prosecutors and their assistants, and officials and prosecutors in the Office
of the Ombudsman and special prosecutor;
(g) Presidents, directors or trustees, or managers of government-owned or controlled
corporations, state universities or educational institutions or foundations;
(2) Members of Congress and officials thereof classified as Grade 27 and up under the
Compensation and Position Classification Act of 1989;
(3) Members of the judiciary without prejudice to the provisions of the Constitution;
(4) Chairmen and members of Constitutional Commissions, without prejudice to the provisions
of the Constitution; and

(5) All other national and local officials classified as Grade 27 and higher under the
Compensation and Position Classification Act of 1989.
B. Other offenses or felonies whether simple or complexed with other crimes committed by the
public officials and employees mentioned in subsection a of this section in relation to their
office.
C. Civil and criminal cases filed pursuant to and in connection with Executive Order Nos. 1, 2,
14 and 14-A, issued in 1986.
xxx (Emphasis and underscoring supplied)
For an offense to fall under the exclusive original jurisdiction of the Sandiganbayan,
the following requisites must concur: (1) the offense committed is a violation of (a) R.A.
3019, as amended (the Anti-Graft and Corrupt Practices Act), (b) R.A. 1379 (the law on
ill-gotten wealth), (c) Chapter II, Section 2, Title VII, Book II of the Revised Penal Code
(the law on bribery), (d) Executive Order Nos. 1, 2, 14 and 14-A, issued in 1986
(sequestration cases), or (e) other offenses or felonies whether simple or complexed with
other crimes; (2) the offender committing the offenses in items (a), (b), (c) and (e) is a
public official or employee[42] holding any of the positions enumerated in paragraph A of
Section 4; and (3) the offense committed is in relation to the office.[43]
Discussion shall be limited to the first case, subject of the present petition.
The charge against petitioner falls under above-quoted Section 4, paragraph B of
R.A. 8249. It is undisputed that at the time the alleged crime was committed, he was the
municipal mayor of Jose Dalman, a position corresponding to salary grade 27 under the
Local Government Code of 1991,[44] which fact was properly alleged in the information. It
is thus imperative to determine whether the offense, as charged, may be considered as
having been committed in relation to office as this phrase is employed in the above-quoted
provision of R.A. 8249. For, for the Sandiganbayan to have exclusive jurisdiction, it is
essential that the facts showing the intimate relation between the office of the offender
and the discharge of official duties be alleged in the information.[45]
In Montilla v. Hilario,[46] this Court held that for an offense to be committed in
relation to the office, the relation between the crime and the office must be direct and
not accidental, such that the offense cannot exist without the office.
People v. Montejo,[47] by way of exception, enunciated the principle that although
public office is not an element of the offense charged, as long as the offense charged in
the information is intimately connected with the office of the offender and perpetrated
while he was in the performance, though improper or irregular, of his official functions, the
accused is held to have been indicted for an offense committed in relation to his office.
These rulings were reiterated in Sanchez v. Demetriou,[48] Republic
Asuncion,[49] Cunanan
v.
Arceo,[50] People
v.
Magallanes,[51] Alarilla
Sandiganbayan [52] and Soller v. Sandiganbayan.[53]

v.
v.

That the jurisdiction of a court is determined by the allegations in the complaint or


information, and not by the evidence presented by the parties at the trial,[54] is settled.
As early as 1954, we pronounced that the factor that characterizes the charge is the actual recital
of the facts. The real nature of the criminal charge is determined not from the caption or
preamble of the information nor from the specification of the provision of law alleged to have
been violated, they being conclusions of law, but by the actual recital of facts in the complaint or
information.[55] (Emphasis and underscoring supplied)
It does not thus suffice to merely allege in the information that the crime charged was
committed by the offender in relation to his office or that he took advantage of his position
as these are conclusions of law.[56] The specific factual allegations in the information that
would indicate the close intimacy between the discharge of the offenders official duties
and the commission of the offense charged, in order to qualify the crime as having been
committed in relation to public office,[57] are controlling.
It bears noting that in Montejo,[58] where this Court held that the allegations in the
information for murder were sufficient to bring the case squarely within the meaning of an
offense committed in relation to the accuseds public office, the phrase committed in
relation to public office does not even appear in the information, which only underscores
the fact that said phrase is not what determines the jurisdiction of the Sandiganbayan.
Thus the information in said case read:
Leroy S. Brown, City Mayor of Basilan City, as such, has organized groups of police patrol and
civilian commandoes consisting of regular policemen and xxx special policemen appointed and
provided by him with pistols and high power guns and then established a camp xxx at Tipo-tipo
which is under his command xxx supervision and control where his co-defendants were
stationed, entertained criminal complaints and conducted the corresponding investigations as
well as assumed the authority to arrest and detain persons without due process of law and
without bringing them to the proper court and that in line with this set-up established by said
Mayor of Basilan City as such, and acting upon his orders, his co-defendants arrested and
maltreated Awalin Tebag who died in consequence thereof.
In Alarilla,[59] apart from the phrase in relation to and taking advantage of his official
functions, the information alleged specific factual allegations showing how the therein
petitioner committed the crime of grave threats as a consequence of his office as
municipal mayor, which allegations led this Court to conclude that the crime charged was
intimately connected with the discharge of his official functions. Thus it read:
That on or about October 13, 1982, in Meycauayan, Bulacan, and within the jurisdiction of this
Honorable Court, the above-named accused, a public officer, being then the Municipal Mayor of
Meycauayan, Bulacan, committing the crime herein charged in relation to and taking advantage
of his official functions, did then and there wilfully, unlawfully and feloniously level and aim a
.45 caliber pistol at and threaten to kill one Simeon G. Legaspi, during a public hearing about the
pollution from the operations of the Giant Achievers Enterprises Plastic Factory and after the
said complainant rendered a privilege speech critical of the abuses and excesses of the
administration of said accused.

Although herein petitioner was described in the information as a public officer being
then the Mayor with salary grade 27 of Jose Dalman, Zamboanga del Norte, there was
no allegation showing that the act of falsification of public document attributed to him was
intimately connected to the duties of his office as mayor to bring the case within the
jurisdiction of the Sandiganbayan. Neither was there any allegation to show how he made
use of his position as mayor to facilitate the commission of the crimes charged. The
information merely alleges that petitioner falsified the disbursement voucher by
counterfeiting therein the signature of Mejorada. For the purpose of determining
jurisdiction, it is this allegation that is controlling, not the evidence presented by the
prosecution during the trial.
In Bartolome v. People[60] where the therein accused was charged with falsification of
official document, the information alleged as follows:
That on or about the 12th day of January, 1977, in the City of Manila, Philippines, and within the
jurisdiction of this Honorable Court, accused Rolando Bartolome y Perez, a public officer having
been duly appointed and qualified as Senior Labor Regulation Officer and Chief of the Labor
Regulations Section, Ministry of Labor, National Capital Region, Manila, conspiring and
conniving with the other accused Elino Coronel y Santos, also a public officer having been duly
appointed and qualified as Labor Regulation Officer of the same office, taking advantage of their
official positions, did then and there wilfully, unlawfully and feloniously prepare and falsify an
official document, to wit: the CS Personal Data Sheet (Civil Service Form No. 212) which bears
the Residence Certificate No. A-9086374 issued at Manila on January 12, 1977, by making it
appear in said document that accused Rolando Bartolome y Perez had taken and passed the
Career Service (Professional Qualifying Examination) on May 2, 1976 with a rating of 73.35%
in Manila and that he was a 4th Year AB student at the Far Eastern University (FEU), when in
truth and in fact, as both accused well knew, accused Rolando Bartolome y Perez had not taken
and passed the same nor was he a 4th Year AB student, thereby making untruthful statements in a
narration of facts. (Underscoring supplied)
This Court held:
In the instant case, there is no showing that the alleged falsification was committed by the
accused, if at all, as a consequence of, and while they were discharging, official functions. The
information does not allege that there was an intimate connection between the discharge of
official duties and the commission of the offense. xxx
Clearly therefore, as the alleged falsification was not an offense committed in relation to the
office of the accused, it did not come under the jurisdiction of the Sandiganbayan. It follows that
all its acts in the instant case are null and void ab initio.[61] (Underscoring supplied)
As for petitioners assertion that the Sandiganbayan has no jurisdiction over the
offense of falsification under Article 172 in relation to Article 171 of the Revised Penal
Code, to buttress which he argues that the offender under Article 172, paragraph 1 is not
supposed to be a public official who takes advantage of his position, thus equating the
requirement of taking advantage of ones public position as stated in the aforementioned
provisions of the Revised Penal Code with the prerequisite in relation to ones office for

the acquisition of jurisdiction of the Sandiganbayan as provided for in R.A. 8249, the same
must be discredited.
Article 171 reads:
ART. 171. Falsification by public officer, employee or notary or ecclesiastic minister. The
penalty of prision mayor and a fine not to exceed 5,000 pesos shall be imposed upon any public
officer, employee, or notary who, taking advantage of his official position, shall falsify a
document by committing any of the following acts:
1. Counterfeiting or imitating any handwriting, signature or rubric;
2. Causing it to appear that persons have participated in any act or proceeding when they did not
in fact so participate;
3. Attributing to persons who have participated in an act or proceeding statements other than
those in fact made by them;
4. Making untruthful statements in a narration of facts;
5. Altering true dates;
6. Making any alteration or intercalation in a genuine document which changes its meaning;
7. Issuing in an authenticated form a document purporting to be a copy of an original document
when no such original exists, or including in such copy a statement contrary to, or different from,
that of the genuine original; or
8. Intercalating any instrument or note relative to the issuance thereof in a protocol, registry, or
official book.
xxx
On the other hand, Article 172, paragraph 1 reads:
ART. 172. Falsification by private individuals and use of falsified documents. The penalty
of prision correccional in its medium and maximum periods and a fine of not more than 5,000
pesos shall be imposed upon:
1. Any private individual who shall commit any of the falsifications enumerated in the next
preceding article in any public or official document or letter of exchange or any other kind of
commercial document; xxx
The offender under Article 172 must be a private individual or maybe a public officer,
employee or notary public who does not take advantage of his official position. [62] Under
Article 171, an essential element of the crime is that the act of falsification must be

committed by a public officer, employee or notary who takes advantage of his official
position.
The offender takes advantage of his official position in falsifying a document when (1)
he has the duty to make or to prepare or otherwise intervene in the preparation of the
document; or (2) he has the official custody of the document which he falsifies. [63]
It is thus apparent that for purposes of acquisition of jurisdiction by the
Sandiganbayan, the requirement imposed by R.A. 8249 that the offense be committed in
relation to the offenders office is entirely distinct from the concept of taking advantage of
ones position as provided under Articles 171 and 172 of the Revised Penal Code.
R.A. 8249 mandates that for as long as the offenders public office is intimately
connected with the offense charged or is used to facilitate the commission of said offense
and the same is properly alleged in the information, the Sandiganbayan acquires
jurisdiction.[64] Indeed, the law specifically states that the Sandiganbayan has jurisdiction
over all other offenses or felonies whether simple or complexed with other crimes
committed by the public officials and employees mentioned in subsection a of Section 4
in relation to their office. Public office, it bears reiterating, need not be an element of the
offense charged.
On the other hand, the element of taking advantage of ones position under the
Revised Penal Code becomes relevant only in the present case, not for the purpose of
determining whether the Sandiganbayan has jurisdiction, but for purposes of determining
whether petitioner, if he is held to be liable at all, would be legally responsible under Article
171 or Article 172.
While the Sandiganbayan is declared bereft of jurisdiction over the criminal case filed
against petitioner, the prosecution is not precluded from filing the appropriate charge
against him before the proper court.
In light of the foregoing, further discussion on the other issues raised has become
unnecessary.
WHEREFORE, the petition is GRANTED. The Decision dated June 19, 2002 and
Resolution dated July 3, 2002 of the Sandiganbayan are SET ASIDE and declared NULL
and VOID for lack of jurisdiction.
No pronouncement as to costs.
SO ORDERED.

Anda mungkin juga menyukai